You are on page 1of 1314

C©u hái lÝ thuyÕt

Hãa häc H÷u c¬


OlympiaVN
Lời mở đầu
Thân chào quý độc giả.
Giai đoạn trước 2018, Tạp chí Olympiad Hóa học KEM (tiền thân là box Hóa
học OlympiaVN) có biên soạn một số chuyên đề Hóa học hữu cơ - từ chính các
câu hỏi được thảo luận trên diễn đàn hoặc bài tập do thành viên đề xuất - và
tổng hợp lại thành bộ sách Hóa học "Hữu cơ OlympiaVN".
Bộ sách này tuy có lượng kiến thức khá nhiều, nhưng lại có một số khiếm
khuyết tương đối quan trọng. Điểm đầu tiên là tài liệu chỉ mới dừng ở mức
"tổng hợp", chưa có sự sắp xếp về mặt nội dung và trình độ khó-dễ, gây ra
những khó khăn cho độc giả, đặc biệt là các bạn mới tiếp xúc với Hóa học hữu
cơ. Khiếm khuyết thứ hai là còn thiếu những phần kiến thức quan trọng (như
các chuyên đề hợp chất thiên nhiên) hoặc chưa bổ sung kiến thức mới (như ứng
dụng lí thuyết FMO).
Bởi vậy, trong năm 2019 này, đội ngũ biên tập viên của KEM đã nỗ lực để
hoàn thành bộ sách này - lấy tên là "Câu hỏi lí thuyết Hóa học Hữu cơ
OlympiaVN". Bên cạnh việc giữ lại những tinh túy của bộ sách "Hữu cơ
OlympiaVN" cũ (khoảng 20% các bài tập hay) thì KEM còn bổ sung thêm rất
nhiều kiến thức mới, mà đa số được chọn lọc và sắp xếp lại từ các giáo trình,
bài giảng Hóa hữu cơ ở bậc Đại học hoặc tài liệu bồi dưỡng Olympiad của Nhật
Bản và các quốc gia khác. Tuy vẫn còn nhiều điểm hạn chế như sự thiếu thống
nhất về mặt danh pháp (sử dụng nhiều nguồn tài liệu khác nhau) và hình vẽ
chưa được trau chuốt, nhưng bộ sách này đã mang đến sự cải thiện rất lớn về
mặt nội dung lẫn bố cục - để giúp các bạn học sinh, sinh viên ngành Hóa đón
nhận kiến thức dễ dàng hơn.
Nhằm tránh sự trùng lặp nội dung với các bộ sách "50 chuyên đề Olympiad
Hóa học" và "Tự học Hóa" mà KEM đã biên soạn vào năm 2018, 2019 - trong
bộ sách này, KEM chỉ tập trung vào những vấn đề lớn mà các bộ sách trên còn
thiếu, bao gồm 3 chủ đề chính:
- Cấu trúc hợp chất hữu cơ.
- Cơ chế phản ứng.
- Hợp chất tạp chức.
Nếu có vấn đề gì sai sót hoặc cần góp ý thêm cho KEM, độc giả vui lòng gửi
tới địa chỉ mail: tapchikem@gmai.com. Trân trọng cảm ơn!

1 | Câu hỏi lí thuyết Hóa học hữu cơ OlympiaVN


Các tài liệu tham khảo chính:
1) Jonathan Clayden, Nick Greeves, Stuart Warren (2012) Organic Chemistry,
2nd edition. ISBN: 9780199270293
2) Robert V. Hoffman (2004) Organic Chemistry: An Intermediate Text, 2nd
ddition. ISBN: 9780471450245
3) William Ogilvie, Nathan Ackroyd, Scott Browning, Ghislain
Deslongchamps, Felix Lee, Effie Sauer (2017) Organic Chemistry:
Mechanistic Patterns. ISBN: 9780176500269
4) David R. Klein (2016) Organic Chemistry, 3rd Edition. ISBN:
9781119110477
5) Hellwich, Karl-Heinz, Siebert, Carsten (2006) Stereochemistry – Workbook.
ISBN: 9783540329121
6) James M. Coxon and J.A. Gerrard (1997) Worked Solutions in Organic
Chemistry. ISBN: 9780751404227
7) Tài liệu của Hiệp hội Hóa học Tổng hợp Hữu cơ Nhật Bản. 有機合成化学
協会 (2015) 演習で学ぶ有機反応機構―大学院入試から最先端まで
8) Các bài tập và đề thi của các khóa học Chem 206 (GS. David Evans) và Chem
30 (GS. Andrew Myers) của khoa Hóa học và Hóa sinh, ĐH Harvard.
9) Đề thi Olympiad Hóa học Quốc gia và chọn đội tuyển đại diện Olympiad Hóa
học Thế giới (IChO) của Nhật Bản, Anh, Ấn Độ, Đức.

2 | Câu hỏi lí thuyết Hóa học hữu cơ OlympiaVN


Mục lục
Cấu Trúc Phân Tử Hợp Chất Hữu Cơ 4
Nhiệt Động Hóa Học Và Động Hóa Học 55
Cấu Dạng 74
Liên Hợp, Cộng Hưởng Và Tính Thơm 124
Tính Acid – Base 180
Tương Tác Orbital Biên (Fmo) 260
Hóa Lập Thể 275
Hoạt Tính Của Hợp Chất Hữu Cơ 314
Cơ Bản Về Cơ Chế Phản Ứng 356
Giải Thích Cơ Chế Phản Ứng 374
Giải Thích Cơ Chế Phản Ứng 2 447
Xác Định Cấu Trúc Hợp Chất & Hoàn Thành Sơ Đồ Phản Ứng 661
Luyện Tập Viết Cơ Chế Phản Ứng 881
Monosaccharide 971
Di-, Tri- Và Polysaccharide 1058
Glycoside 1099
Amino Acid 1143
Peptide 1178
Phụ Lục 1: Bảng Thuật Ngữ 1259
Phụ Lục 2: Các Phản Ứng Mang Tên Người 1290

3 | Câu hỏi lí thuyết Hóa học hữu cơ OlympiaVN


Cấu trúc phân tử
hợp chất hữu cơ
Bài 1
Vẽ cấu trúc Lewis của các phân tử sau:

4 | Câu hỏi lí thuyết Hóa học hữu cơ OlympiaVN


Hướng dẫn

5 | Câu hỏi lí thuyết Hóa học hữu cơ OlympiaVN


Bài 2
Bổ sung các cặp electron chưa liên kết cho các nguyên tử phù hợp trong những
cấu trúc Lewis sau:

Hướng dẫn

6 | Câu hỏi lí thuyết Hóa học hữu cơ OlympiaVN


Bài 3
Vẽ cấu trúc một số mạch hydrocarbon no với 7 nguyên tử carbon có: a) mạch
thẳng; b) mạch phân nhánh; c) mạch vòng.
Dựa vào các cấu trúc ở trên, hãy vẽ công thức cấu tạo của một số hợp chất có
chứa cả nhóm chức ketone và carboxylic acid.
Hướng dẫn
CẤU TRÚC MỘT SỐ MẠCH HYDROCARBON
Mạch thẳng:

Mạch phân nhánh:

Mạch vòng:

CẤU TRÚC MỘT SỐ KETO-CACBOXYLIC ACID:


Mạch thẳng:

Mạch phân nhánh:

Mạch vòng:

7 | Câu hỏi lí thuyết Hóa học hữu cơ OlympiaVN


Bài 4
Chuyển các mô tả cấu trúc sau đây thành công thức cấu tạo trực quan hơn:
(a) C6H5CH(OH)(CH2)4COC2H5
(b) O(CH2CH2)2O
(c) (CH3O)2CH=CHCH(OCH3)2
Hướng dẫn

8 | Câu hỏi lí thuyết Hóa học hữu cơ OlympiaVN


Bài 5
Ngoại trừ các nhóm alkyl, hãy chỉ rõ và gọi tên các nhóm chức trong các phân tử
sau:

9 | Câu hỏi lí thuyết Hóa học hữu cơ OlympiaVN


Hướng dẫn

10 | Câu hỏi lí thuyết Hóa học hữu cơ OlympiaVN


11 | Câu hỏi lí thuyết Hóa học hữu cơ OlympiaVN
Bài 6
a) Xác định các nhóm chức có mặt trong mỗi hợp chất dưới đây và chỉ rõ kích
thước vòng.

b) Xác định các nhóm chức trong hai phân tử sau:

12 | Câu hỏi lí thuyết Hóa học hữu cơ OlympiaVN


Hướng dẫn
a) Các nhóm chức trong hợp chất:

Kích thước vòng của amoxycillin: 6-4-5; Tamiflu: 6.


b)

13 | Câu hỏi lí thuyết Hóa học hữu cơ OlympiaVN


Bài 7
Xác định các nhóm chức trong các phân tử sau:

Hướng dẫn

14 | Câu hỏi lí thuyết Hóa học hữu cơ OlympiaVN


Bài 8
Đề xuất ít nhất 6 công thức cấu tạo khác nhau có cùng công thức phân tử C4H7NO.
Xác định các nhóm chức trong mỗi công thức này.
Hướng dẫn
Một số gợi ý

15 | Câu hỏi lí thuyết Hóa học hữu cơ OlympiaVN


Bài 9
Vẽ các orbital biểu diễn sự tạo thành các hợp chất sau: ethylene, aliene, 1,3-
butadiene, nitromethane, acrylonitrile và hydrazine.
Hướng dẫn
Ethylene, CH2=CH2

Orbital 2s và 2 trong số các orbital p của mỗi nguyên tử carbon của ethylene tổ
hợp thành 3 orbital lai hóa sp2. Một orbital sp2 từ mỗi carbon xen phủ với nhau
tạo thành liên kết σ carbon-carbon; mỗi orbital sp2 còn lại xen phủ với orbital 1s
của nguyên tử hydrogen tạo thành các liên kết σ carbon-hydrogen. Orbital p còn
lại của các nguyên tử carbon vuông góc (trực giao) với các liên kết σ và xen phủ
với nhau tạo thành liên kết π carbon-carbon. Hệ quả là ethylene là phân tử phẳng.
Allene, CH2=CH=CH2

Các nguyên tử carbon đầu mạch của allene tương tự như trong ethylene: lai hóa
sp2 và 1 orbital p không lai hóa. Nguyên tử carbon trung tâm lai hóa sp và có 2
orbital p không lai hóa - chúng vuông góc với nhau. Một orbital sp2 từ mỗi nguyên
tử carbon đầu mạch xen phủ với 1 orbital sp từ nguyên tử carbon trung tâm, tạo
thành 2 liên kết σ carbon-carbon. Các orbital p của các nguyên tử carbon đầu
mạch xen phủ với 2 orbital p của nguyên tử carbon trung tâm, tạo thành 2 liên kết
π (các liên kết này trực giao với nhau). Phân tử tạo thành có dạng hình học của
một tứ diện kéo dài. Dạng hình học này gây ra tính quang hoạt trong một số allene
có nhóm thế.

16 | Câu hỏi lí thuyết Hóa học hữu cơ OlympiaVN


1,3-butadiene, CH2=CH-CH=CH2

Có 4 nguyên tử carbon lai hóa sp2, như ethylene. Có 2 orbital phân tử bị chiếm
biểu diễn 2 liên kết π (I và II ở trên). Sự xen phủ mở rộng trong orbital phân tử
có năng lượng thấp hơn (I) dẫn đến liên kết trung tâm từ C-2 đến C-3 có một phần
đặc tính của liên kết đôi (được thể hiện qua độ dài liên kết ngắn hơn liên kết σ
carbon-carbon “thông thường”, 0.146 nm so với 0.154 nm).
Nitromethane, CH3NO2

Nguyên tử nitrogen lai hóa sp2 và tạo thành 3 liên kết σ (1 liên kết carbon-nitrogen
và 2 liên kết nitrogen-oxygen) tương tự như mô hình của các nguyên tử carbon
trong ethylene. Orbital p còn lại xen phủ với các orbital p của mỗi nguyên tử
oxygen, tạo thành 2 orbital phân tử liên kết (I và II ở trên), mỗi orbital có 2
electron. Trong orbital phân tử có năng lượng thấp hơn (I), 2 electron này được
phân bố trên cả 3 nguyên tử, trong khi orbital phân tử có năng lượng cao hơn (II)
thì 2 electron chỉ định xứ trên các nguyên tử oxygen. Có thể được biểu diễn trực
quan bởi 2 dạng cấu trúc cộng hưởng cổ điển theo mô hình liên kết cộng hóa trị
(VB). Do các cấu trúc này cùng biểu diễn phân tử nên chúng được vẽ trong các
dấu ngoặc vuông:

Acrylonitrile, CH2=CH-C≡N

17 | Câu hỏi lí thuyết Hóa học hữu cơ OlympiaVN


Liên kết đôi carbon-carbon của acrylonitrile gần như tương đồng với liên kết
trong ethylene. Carbon và nitrogen của nhóm nitrile đều lai hóa sp và các orbital
lai hóa xen phủ với nhau tạo thành một liên kết σ carbon-nitrogen. 2 orbital p còn
lại của mỗi nguyên tử xen phu tạo thành 2 liên kết π, hoàn thành liên kết ba
carbon-nitrogen.
Hydrazine, H2N-NH2

2 nguyên tử nitrogen trong hydrazine lai hóa sp3. 1 orbital sp3 từ mỗi nguyên tử
nitrogen xen phủ với nhau tạo thành liên kết σ nitrogen-nitrogen. 2 orbital sp3 từ
mỗi nguyên tử nitrogen xen phủ với orbital 1s từ hydrogen tạo thành các liên kết
σ nitrogen-hydrogen. Orbital sp3 còn lại của mỗi nitrogen chứa 1 cặp electron
chưa liên kết (lone pair), tương tác đẩy giữa các cặp lone pair này phần nào giải
thích cho sự kém bền của liên kết đơn nitrogen-nitrogen.

18 | Câu hỏi lí thuyết Hóa học hữu cơ OlympiaVN


Bài 10
1) Xác định trạng thái lai hóa và dạng hình học của mỗi nguyên tử carbon trong
các phân tử sau.

2) Vẽ cấu trúc các phân tử sau và dự đoán dạng hình học của chúng:
CO2, CH2=NCH3, CHF3, CH2=C=CH2, (CH2)2O
3) Vẽ cấu trúc các phân tử sau và dự đoán giá trị các góc liên kết trong đó: (a)
hydrogen peroxide, H2O2 (b) methyl isocyanate CH3NCO (c) hydrazine,
NH2NH2 (d) diimide, N2H2 (e) the azide anion, N3-
4) Vẽ cấu trúc các phân tử sau: (a) nước, (b) acetone (Me2C=O), và (c) nitrogen
(N2)
Hướng dẫn
1) tetrahedral = tứ diện; trigonal = tam giác; linear = thẳng.

2)

19 | Câu hỏi lí thuyết Hóa học hữu cơ OlympiaVN


3)

4)

20 | Câu hỏi lí thuyết Hóa học hữu cơ OlympiaVN


Bài 11
Với mỗi hợp chất sau, hãy bổ sung các cặp electron chưa liên kết vào cấu trúc và
chỉ rõ kiểu lai hóa của orbital chứa các cặp này:

Hướng dẫn

21 | Câu hỏi lí thuyết Hóa học hữu cơ OlympiaVN


Bài 12
Xét phân tử sau:
a) Dự đoán dạng hình học của mỗi nguyên tử
khác-hydrogen.
b) Chỉ rõ sự lai hóa của mỗi nguyên tử khác-
hydrogen.
Hướng dẫn
a)

b)

22 | Câu hỏi lí thuyết Hóa học hữu cơ OlympiaVN


Bài 13
1) Đưa ra sơ đồ tạo thành liên kết (trạng thái lai hóa của nguyên tử, số lượng
mỗi loại liên kết, …) và dạng hình học (góc liên kết) của các nhóm chức sau:
a) alkyl nitrile (kí hiệu R là nhóm alkyl)
b) alkyl azide (kí hiệu R là nhóm alkyl)
c) nitro alkane (kí hiệu R là nhóm alkyl)
2) Dự đoán góc liên kết gần đúng quanh các nguyên tử được chỉ ra bởi các mũi
tên trong mỗi hợp chất sau:

23 | Câu hỏi lí thuyết Hóa học hữu cơ OlympiaVN


Hướng dẫn
1)

2)

24 | Câu hỏi lí thuyết Hóa học hữu cơ OlympiaVN


25 | Câu hỏi lí thuyết Hóa học hữu cơ OlympiaVN
Bài 14
1) Các cấu trúc biểu diễn như sau có gì sai? Hãy đề xuất những cách tốt hơn để
biểu diễn lại chúng.

2) Biểu diễn công thức cấu tạo của các hợp chất sau:
(a) ethyl acetate
(b) chloromethyl methyl ether
(c) pentanenitrile
(d) N-acetyl p-aminophenol
(e) 2,4,6,-tri-(1,1-dimethylethyl)phenylamine

26 | Câu hỏi lí thuyết Hóa học hữu cơ OlympiaVN


Hướng dẫn
1) Cách biểu diễn như trên khiến các góc liên kết trở nên khá “kì dị” – với các
nguyên tử carbon no có dạng vuông phẳng, các alkyne thì bị bẻ cong bởi các
góc 120o, các alkene thì có góc liên kết 90o hoặc 180o, các liên kết đi từ vòng
benzene ra thì sai góc, … (Nếu được vẽ đúng cách, cấu trúc bên trái sẽ trở
nên rõ ràng hơn khi lược bỏ các nguyên tử hydrogen.) Dưới đây là một trong
nhiều cách biểu diễn tốt hơn với mỗi chất:

2) Có nhiều cách để biểu diễn mỗi chất trên:

27 | Câu hỏi lí thuyết Hóa học hữu cơ OlympiaVN


Bài 15
Vẽ công thức cấu tạo của các hợp chất được biểu diễn như sau:
a) AcO(CH2)3NO2
b) MeO2CCH2OCOEt
c) CH2=CHCONH(CH2)2CN
Hướng dẫn
Ở chất đầu tiên, AcO là kí hiệu hợp phần acetate ester, và nitrogen trong nhóm
nitro có thể có 4 (chứ không phải 5) liên kết.
Ở chất thứ hai, Et là kí hiệu nhóm ethyl.

28 | Câu hỏi lí thuyết Hóa học hữu cơ OlympiaVN


Bài 16
Hãy dự đoán trong phổ 13C NMR của mỗi hợp chất sau đây có bao nhiêu tín hiệu?

Hướng dẫn
Hợp chất A có cấu trúc tứ diện và chỉ có 2 loại carbon (CH2
và CH) - do đó nó có 2 tín hiệu. Hợp chất này rất nổi tiếng,
với tên gọi là adamantane - là một chất rắn kết tinh, dù nó chỉ
là hydrocarbon với chỉ 10 nguyên tử carbon. Hợp chất này
xứng đáng là một hình mẫu về tính đối xứng bởi cấu trúc tuyệt
đẹp của nó!
Hợp chất B cũng có tính đối xứng cao: 2 nhóm C=O giống
nhau và các nguyên tử còn lại trong vòng cũng giống nhau.
Chất này là một chất rắn kết tinh màu da cam, gọi là quinone.
Trong phổ 13C NMR có 2 tín hiệu.
Hợp chất C là naphthalene và cũng có tính đối
xứng cao: 2 vòng benzene giống nhau và do
đó chỉ có 3 loại carbon. Do đó, hợp chất này có 3 tín hiệu.
Hợp chất D là triethanolamine - được các
nhà hóa sinh sử dụng nhiều. Nó có tính
đối xứng bậc ba và chỉ có 2 loại nguyên tử carbon. Trong
phổ 13C NMR có 2 tín hiệu.
Hợp chất E là EDTA
(ethylenediaminetetraacetic acid), một tác nhân tạo
phức vòng càng quan trọng với nhiều kim loại. Trong
phân tử này có 3 loại nguyên tử carbon, tương ứng với
3 tín hiệu trong phổ 13C NMR.

29 | Câu hỏi lí thuyết Hóa học hữu cơ OlympiaVN


Bài 17
Các khảo sát cấu trúc cho biết hợp chất 9-metyl-9,10-dihydroantraxen ưu tiên tồn
tại ở cấu trúc không phẳng. Hãy vẽ các cấu trúc không phẳng có thể có của nó và
giải thích sự lựa chọn của mình.
Hướng dẫn
Cấu trúc của 9-metyl-9,10-dehydroantraxen. Trong cấu trúc này thì vòng 6 cạnh
phải ở dạng thuyền (do cấu trúc là không phẳng) để cho vòng benzen được phẳng.
Nhóm metyl trong hợp chất này cũng ưu tiên ở vị trí a thay vì e (tương tác A1,3).

30 | Câu hỏi lí thuyết Hóa học hữu cơ OlympiaVN


Bài 18
Gọi tên các hợp chất sau theo danh pháp IUPAC. Riêng mục 6 bạn có thể làm sau
khi học đến phần Hoá lập thể.
1)

2)

3)

31 | Câu hỏi lí thuyết Hóa học hữu cơ OlympiaVN


4)

5)

32 | Câu hỏi lí thuyết Hóa học hữu cơ OlympiaVN


6)

33 | Câu hỏi lí thuyết Hóa học hữu cơ OlympiaVN


Hướng dẫn
1)

2)

3)

34 | Câu hỏi lí thuyết Hóa học hữu cơ OlympiaVN


4)

5)

5-(1-chloroethyl)-9,10-dimethyl-8-(1,2-dimethyl-1-propyl)undec-2-en-4,6-
dione.
6)

35 | Câu hỏi lí thuyết Hóa học hữu cơ OlympiaVN


36 | Câu hỏi lí thuyết Hóa học hữu cơ OlympiaVN
37 | Câu hỏi lí thuyết Hóa học hữu cơ OlympiaVN
Bài 19
a) Xác định cấu trúc của hợp chất có danh pháp IUPAC như sau:
5,5-difluoro-1-(4-methylheptan-2-yl)cyclohexa-1,3-diene
b) Xác định cấu trúc của các hợp chất sau:
a) propyl 3-oxopentanoate
b) 1-methoxy-4,5-dimethylhex-5-en-3-ol
c) 1-(4-hydroxyphenyl)propan-1-one
d) 3, 6-dimethylcyclohex-4-ene-1,2-diol
e) 4-(2-chloro-4-methylpentan-3-yl)-3-methyloctanal
Hướng dẫn
1) Phần gốc là cyclohexa-1,3-diene, do đó bộ khung phân tử này là:

1-(4-methylheptan-2-yl) có nghĩa là nhóm 7-nguyên tử carbon gắn vào qua


nguyên tử carbon thứ hai. Nhóm này cũng có một nhóm methyl gắn với carbon
thứ tư. Hợp phần này gắn vào vị trí carbon số 1 trong vòng.

5,5-difluoro chỉ hai nguyên tử fluorine, gắn vào carbon số 5 trong vòng:

38 | Câu hỏi lí thuyết Hóa học hữu cơ OlympiaVN


2)

39 | Câu hỏi lí thuyết Hóa học hữu cơ OlympiaVN


Bài 20
a) Liên kết N-H hay B-H phân cực hơn? Khác biệt quan trọng giữa chúng là gì?
b) Hãy vẽ lại các phân tử sau ở
dạng công thức cấu tạo tối giản
và đặt vào các mũi tên lưỡng
cực để chỉ rõ hướng của lưỡng
cực trong mỗi liên kết được
khoanh tròn.
c) Với mỗi phân tử ở ý b, hãy chỉ
ra nguyên tử carbon nào bạn
dự đoán là thiếu hụt electron
nhất.
Hướng dẫn
a) Liên kết N-H sẽ phân cực hơn nhiều do chênh lệch độ âm điện giữa N(3.0)
và H(2.2) lớn hơn giữa B(2.0) và H(2.2). Khác biệt giữa chúng là liên kết N-
H phân cực về phía nitrogen, trong khi đó liên kết B-H phân cực về phía
hydrogen.
b)

c)

40 | Câu hỏi lí thuyết Hóa học hữu cơ OlympiaVN


Bài 21
Giải thích các phát biểu sau:
a) Moment lưỡng cực của 1,2-dichloroethane tăng khi nhiệt độ tăng.
b) Moment lưỡng cực của p-nitroaniline (6.2 D) lớn hơn nhiều so với tổng các
giá trị của nitrobenzene (3.98D) và aniline (1.53 D).
c) Moment lưỡng cực của propenal (CH2=CHCHO; 3.04 D) lớn hơn nhiều so
với propionaldehyde (CH3CH2CHO; 2.73 D).
d) Picric acid (2,4,6-trinitrophenol) giải phóng carbon dioxide khi phản ứng với
sodium carbonate, nhưng phenol thì không.
e) Phản ứng N,N-dimethylation làm tăng gấp 3 lần tính base của aniline nhưng
với 2,4,6-trinitroaniline thì tăng lên 40000 lần.
f) Năng lượng phân li liên kết của PhCH2-H (322 kJ mol-1) nhỏ hơn đáng kể so
với của liên kết CH3-H (426 kJ mol-1).
g) Nhiệt độ sôi của ethanol cao hơn nhiều so với đồng phân dimethyl ether của
nó.
h) Boron trifluoride và aluminium trichloride là các Lewis acid.
Hướng dẫn
a) Moment lưỡng cực của 1,2-dichloroethane tăng khi nhiệt độ tăng.
1,2-Dichloroethane tồn tại ở dạng hỗn hợp nhiều cấu dạng khác nhau. Cấu dạng
bền nhất (năng lượng thấp nhất) là dạng xen kẽ (staggered), trong đó sự quay
quanh liên kết carbon-carbon làm cho các nhóm thế chloro cồng kềnh càng cách
xa nhau càng tốt. Trong cấu dạng này, moment lưỡng cực của hai liên kết carbon-
chlorine có chiều ngược nhau và moment lưỡng cực tổng của phân tử bằng 0.

Các cấu dạng bán lệch (gauche) thì kém bền hơn, trong đó các nguyên tử chlorine
khiến phân tử có moment lưỡng cực khác không. Cấu dạng kém bền nhất (năng
lượng cao nhất) là dạng che khuất (eclipsed), trong đó 2 nguyên tử chlorine gây
ra sự cản trở không gian với nhau. Trong cấu dạng này, moment lưỡng cực của
hai liên kết carbon-chlorine tăng cường lẫn nhau, khiến cho phân tử có moment
lưỡng cực lớn nhất có thể có.
Ở nhiệt độ thấp, các phân tử chủ yếu tồn tại ở cấu dạng xen kẽ năng lượng thấp.
Khi nhiệt độ tăng, càng nhiều phân tử đủ năng lượng để đạt tới các cấu dạng năng
lượng cao hơn - như dạng bán lệch và che khuất. Điều này dẫn đến moment lưỡng
cực trung bình của phân tử sẽ tăng theo nhiệt độ.

41 | Câu hỏi lí thuyết Hóa học hữu cơ OlympiaVN


b) Moment lưỡng cực của p-nitroaniline (6.2 D) lớn hơn nhiều so với tổng các
giá trị của nitrobenzene (3.98D) và aniline (1.53 D).
Khi không có sự liên hợp, moment lưỡng cực dự đoán của p-nitroaniline sẽ là
tổng moment lưỡng cực của nitrobenzene và của aniline (3.98 + 1.53 = 5.51 D).

Thực tế thì moment lưỡng cực đo được lớn hơn giá trị này nhiều (6.2 D) và nó đã
đưa ra bằng chứng về sự giải tỏa của các electron, với sự chuyển toàn bộ điện tích
âm từ nhóm amino đến nhóm nitro qua vòng benzene (được thể hiện qua các cấu
trúc công hưởng ở trên.)
c) Moment lưỡng cực của propenal (CH2=CHCHO; 3.04 D) lớn hơn nhiều so với
propionaldehyde (CH3CH2CHO; 2.73 D).
Moment lưỡng cực của propenal lớn hơn so với dạng no tương ứng của no bởi sự
liên hợp giữa alkene và nhóm carbonyl, dẫn tới sự tách biệt điện tích và làm tăng
moment lưỡng cực.

d) Picric acid (2,4,6-trinitrophenol) giải phóng carbon dioxide khi phản ứng với
sodium carbonate, nhưng phenol thì không.
Khi dung dịch sodium carbonate tương tác với các hợp chất có đủ lực acid thì sẽ
giải phóng carbon dioxide: CO32- + 2H+ → [H2CO3] → CO2 + H2O
Picric acid (pKa = 0.96) có thể giải phóng carbon dioxide từ dung dịch sodium
carbonate nhưng phenol thì không ảnh hưởng gì - điều này phản ánh thực tế là
lực acid của picric acid lớn hơn nhiều so với phenol (pKa = 10.00). Sự tăng lực
acid do anion được bền hóa nhờ sự giải tỏa cực mạnh vào các nhóm nitrogen:

42 | Câu hỏi lí thuyết Hóa học hữu cơ OlympiaVN


e) Phản ứng N,N-dimethylation làm tăng gấp 3 lần tính base của aniline nhưng
với 2,4,6-trinitroaniline thì tăng lên 40000 lần.
Phản ứng N,N-dimethylation làm tăng lực base của aniline bởi hiệu ứng cảm ứng
dương của của 2 nhóm methy làm bền hóa cation tạo thành từ sự proton hóa. Quá
trình proton hóa Sự proton hóa vẫn bị cản trở bởi việc mất năng lượng cộng hưởng
(từ sự giải tỏa của lone pair của nitrogen vào vòng thơm) (xem bài trước).
2,4,6-trinitroaniline (pKa = -9.41) có lực base thấp hơn rất nhiều so với aniline
(pKa = 4.600 do lone pair có thể bị giải tỏa cực mạnh vào cả 3 nhóm nitro. Sự
proton hóa hợp chất này dẫn đến sự giảm đáng kể năng lượng cộng hưởng và nó
rất không thuận tiện. Phản ứng N,N-dimethyl hóa 2,4,6-trinitroaniline gây ra sự
án ngữ không gian giữa các nhóm N-methyl và các nhóm ortho nitro; hiệu ứng
này đủ khiến cho các nhóm N,N-dimethylamino lệch khỏi mặt phẳng vòng
benzene (để làm giảm bớt lực đẩy không gian):

Bởi vậy, sự giải tỏa của lone pair trên amino nitrogen vào vòng benzene được
nitro hóa bị cản trở và lone pair này dễ bị proton hóa.
f) Năng lượng phân li liên kết của PhCH2-H (322 kJ mol-1) nhỏ hơn đáng kể so
với của liên kết CH3-H (426 kJ mol-1).
Liên kết PhCH2-H yếu hơn các liên kết carbon-hydrogen thông thường bởi độ bền
của gốc benzyl tạo thành khi liên kết bị phá vỡ. Tính bền này là hệ quả của sự
giải tỏa cực kì tốt và được phản ánh qua năng lượng của trạng thái chuyển tiếp:

g) Nhiệt độ sôi của ethanol cao hơn nhiều so với đồng phân dimethyl ether của
nó.
Thoạt nhìn qua thì sẽ thật kì lạ khi dimethyl ether và ethanol có nhiệt độ sôi khác
nhau nhiều, do chúng có cùng công thức phân tử (C2H6O). Nhiệt độ sôi cao hơn
của ethanol (78 oC, so với -25 oC của dimethy ether) thể hiện lực tương tác liên
phân tử lớn hơn rất nhiều. Do ethanol có một nguyên tử hydrogen gắn với nguyên
tử oxygen âm điện hơn nên nó có thể tạo thành các liên kết hydrogen; điều này
không xảy ra với dimethyl ether, do đó, ether có nhiệt độ sôi thấp hơn.

43 | Câu hỏi lí thuyết Hóa học hữu cơ OlympiaVN


h) Boron trifluoride và aluminium trichloride là các Lewis acid.
Cả boron và aluminum đều thuộc nhóm 3 của bảng tuần hoàn, do đó chúng tạo
thành các hợp chất chỉ có 6 electron ở lớp vỏ hóa trị bên ngoài. Việc tiếp nhận
thêm một cặp electron từ các hợp chất khác sẽ giúp hoàn thành vỏ bát tử và cho
phép tạo thành các phức chất tứ diện bền với các Lewis base. Vậy nên cả hai chất
này đều là Lewis acid:

44 | Câu hỏi lí thuyết Hóa học hữu cơ OlympiaVN


Bài 22
1) Dự đoán tính tan trong nước và trong hexane của HOCH2(CH2)6COOH với
CH3(CH2)6COOH. Giải thích.
2) Sắp xếp các phân tử sau theo trật tự tăng dần độ tan trong nước:

3) Xác định các vùng kị nước (hydrophobic region) và ưa nước (hydrophilic


region) trong các phân tử sau. Dự đoán chất nào tan tốt trong nước và chất
nào tan tốt trong các dung môi hữu cơ.

4) Sắp xếp các phân tử sau theo trình tự tăng dần độ tan trong nước.

45 | Câu hỏi lí thuyết Hóa học hữu cơ OlympiaVN


Hướng dẫn
1) Dưới đây là công thức cấu tạo hai chất, với các phần ưa nước (hydrophilic) và
kị nước (hydrophobic) được chỉ rõ.

Phân tử chứa alcohol sẽ tan trong nước nhiều hơn, do nó có thêm một tâm
hydrogen (cả phần nhường và phần nhận). Mạch hydrocarbon dài trong phân tử
còn lại sẽ dễ tương tác với các dung môi hydrocarbon mạch dài như hexane và
khiến nó tan nhiều trong hexane hơn.
2)

3) a) Tất cả các nhóm -OH đều là hợp phần nhường và nhận của liên kết hydrogen.
Các nguyên tử carbon trong vòng sẽ là phần kị nước, tuy nhiên tất cả chúng đều
gần với các nhóm phân cực. Điều này khiến cho phân tử rất dễ tan trong nước và
kém tan trong các dung môi hữu cơ.
b) Phần lớn phân tử này là vùng kị nước và nó sẽ tan được trong các dung môi
hữu cơ. Vùng ưa nước nhỏ, do đó dự đoán rằng phân phân tử này tan giới hạn
trong nước.

46 | Câu hỏi lí thuyết Hóa học hữu cơ OlympiaVN


c) Các nhóm amide và carboxylate đều có tính ưa nước.
Khung carbon có tính kị nước. Phân tử này có điện tích
hình thức (nhóm carboxylate) và các hợp phần nhận liên
kết hydrogen (amide và carboxylate). Do đó, nó tan tốt
trong nước và các dung môi hữu cơ phân cực. Nó có thể
tan giới hạn trong các dung môi hữu cơ không-phân cực.
d) Các nhóm amine (-NH2) và -SO2NH2 đều là các hợp
phần nhận và nhường liên kết hydrogen ưa nước. Vòng
thơm là vùng kị nước. Hợp chất này có thể tan tốt trong
nước và các dung môi hữu cơ phân cực nhưng tan giới hạn
trong các dung môi hữu cơ không-phân cực.
4) C < D < B < A. Phân tử mang điện A sẽ tan tốt nhất do các tương tác tĩnh điện
với nước. B sẽ là chất tan tốt thứ hai, với nhóm carboxylic vừa là hợp phần nhận,
vừa là hợp phần nhường liên kết hydrogen. Còn C và D chỉ có các hợp phần nhận
liên kết hydrogen và tan kém nhất. C tan kém hơn D, do nó có vùng kị nước lớn
hơn.

47 | Câu hỏi lí thuyết Hóa học hữu cơ OlympiaVN


Bài 23
Sắp xếp các hợp chất trong mỗi nhóm sau đây theo trình tự giảm dần nhiệt độ sôi
dự đoán của chúng. Giải thích ngắn gọn cách sắp xếp của bạn bằng cách chỉ rõ
các loại lực liên phân tử có thể có.

Hướng dẫn
a) Do tất cả các phân tử có kích thước tương đương nhau, nên loại lực liên kết sẽ
xác định trình tự nhiệt độ sôi. Trong trường hợp này, liên kết hydrogen là mạnh
nhất (tương đương với nhiệt độ sôi cao nhất), sau đó là tương tác lưỡng cực, và
cuối cùng là lực phân tán (nhiệt độ sôi thấp nhất). Alkane mạch thẳng sẽ có nhiệt
độ sôi cao hơn, do cách mạch tiếp xúc hiệu quả với các phân tử kế cận và lực hút
tổng thể sẽ cao hơn.

b) Muối sodium sẽ có nhiệt độ sôi cao hơn, do có lực hút tĩnh điện. Nhóm
carboxylic acid phân cực hơn alcohol nên liên kết hydrogen mạnh hơn.

48 | Câu hỏi lí thuyết Hóa học hữu cơ OlympiaVN


c) Muối ammonium có lực hút tĩnh điện nên nhiệt độ sôi cao nhất. Các phân tử
với các nguyên tử N liên kết với H sẽ có các liên kết hydrogen. Liên kết này mạnh
hơn với nitrogen có hai hydrogen gắn vào. Phân tử cuối cùng không thể tạo liên
kết hydrogen nên chỉ có tương tác lưỡng cực.

49 | Câu hỏi lí thuyết Hóa học hữu cơ OlympiaVN


Bài 24
Cho bảng số liệu sau :
Axit béo : axit stearic axit panmitic axit oleic axit
linoleic
tnc, 0C : 69,6 63,1 13,4 5,2
a) Biết công thức phân tử của axit oleic là C18H34O2 có chứa 1 liên kết đôi ở
dạng cis ở C9-C10 (C cacboxyl là C1), công thức phân tử của axit linoleic
là C18H32O2 có chứa 2 liên kết đôi đều ở dạng cis ở C9-C10 và C12-C13 (C
cacboxyl là C1). Hãy viết công thức cấu trúc của axit oleic và axit linoleic.
b) Hãy nêu nguyên nhân dẫn tới sự giảm dần nhiệt độ nóng chảy của 4 axit đã
cho.
c) Hãy giải tích vì sao chất béo thực vật thường có nhiệt độ đông đặc thấp hơn
chất béo động vật?
Hướng dẫn
Ở thể rắn các phân tử axit stearic có cấu trúc thẳng, gọn gàng, dễ sắp xếp chặt khít nên
lực hút giữa chúng mạnh hơn, trong khi đó các phân tử axit oleic có cấu trúc uốn gập
(ở chỗ cấu hình cis), cồng kềnh, khó sắp xếp chặt khít nên lực hút giữa chúng yếu hơn.
Hình dưới đây cho thấy cùng một thể tích sẽ chứa được nhiều phân tử có cấu trúc thẳng
hơn là phân tử có cấu trúc uốn gập:

Thành phần chính của chất béo thực vật là axit oleic vốn có tonc thấp.

50 | Câu hỏi lí thuyết Hóa học hữu cơ OlympiaVN


Bài 25
Hãy sắp xếp các chất sau theo chiều tăng dần nhiệt độ nóng chảy và gải thích:
CH3CH2CH2CH2NH2 (a), CH3CH2CH2CH2OH (b), CH3CH2OCH2CH3 (c),
CH3CH2COOH (d), H2NCH2COOH (e).
Hướng dẫn
Các hợp chất đã cho có khối lượng phân tử hơn kém nhau không nhiều.
Amino axit H2NCH2COOH (e) ở dạng ion lưỡng cực, H3N+CH2COO-, nên có to nc
cao nhất.
CH3CH2OCH2CH3 (c) chứa các liên kết ít phân cực và không tạo được liên kết H liên
phân tử, nên có to nc thấp nhất.
Nhóm -OH phân cực hơn và tạo liên kết H mạnh hơn nhóm -NH2 nên
CH3CH2CH2CH2OH (b) nóng chảy cao hơn CH3CH2CH2CH2NH2 (a).
CH3CH2COOH (d) có nhóm -COOH vừa phân cực mạnh vừa tạo liên kết H mạnh nên
nóng chảy cao hơn CH3CH2CH2CH2OH (b).
Như vậy to nc tăng theo trật tự sau: (c) < (a) < (b) < (d) < (e)

51 | Câu hỏi lí thuyết Hóa học hữu cơ OlympiaVN


Bài 26
Hãy thử giải thích các hiên tượng sau:
a) Tinh bột và xenlulozơ đều không tan trong nước mặc dù ở mỗi gốc glucozơ tạo ra
chúng đều có 3 nhóm OH là nhóm ưa nước.
b) Đun nóng tinh bột với nước thì tạo thành hồ tinh bột, đun nóng xenlulozơ với nước
thì chẳng được gì.
c) Tinh bột và xenlulozơ không nóng chảy.
d) Ở nhiệt độ phòng glucozơ, fructozơ (M = 180) đều ở thể rắn, trong khi đó axit oleic
có phân tử khối lớn hơn nhiều (M = 282) lại ở thể lỏng.
e) Vì sao lipit không tan trong nước mà tan trong xăng dầu?
Hướng dẫn
a) Tinh bột và xenlulozơ là những hợp chất cao phân tử kích thước phân tử quá lớn
so với các phân tử nước, lại có vô vàn liên kết hiđro liên phân tử làm cho lực hút
giữa các phân tử rất lớn, nên các phân tử nước không thể kéo từng phân tử ra khổi
bó cao phân tử để hòa tan được.
b) Do phân tử amilozơ có cấu trúc xoắn, phân tử amilopectin có cấu trúc phân nhánh,
ở tinh bột số lượng nhóm OH tạo được liên kết hiđro nội phân tử lớn hơn nhiều so
với ở xenlulozơ nên lực hút giữa các phân tử ở tinh bột nhỏ hơn giữa cac phân tử
xenlulozơ. Do đó khi đun nóng với nước, giữa các phân tử trong tinh bột có thể tạo
ra các khoảng trống cho các phân tử nước thâm nhập vào thực hiện sự hyđrát hóa
tạo ra thể keo tụ gọi là hồ tinh bột. Các phân tử xenlulozơ tạo ra các bó cao phân
tử hút nhau rất chặt, đun nóng cũng không rời nhau khiến cho các phân tử nước chỉ
tiếp xúc ở mặt ngoài của bố cao phân tử làm ướt chúng mà không tạo ra thể keo tụ
được.
c) Tinh bột và xenlulozơ là những hợp chất cao phân tử, lại có vô vàn liên kết hiđro
liên phân tử làm cho lực hút giữa các phân tử rất lớn, đến nhiệt độ cao vẫn không
rời nhau ra (không nóng chảy) mà bị cháy nếu có oxi hoặc bị phân hủy nếu không
có mặt oxi.
d) Phân tử glucozơ, fructozơ (C6H12O6) đều có cấu trúc gọn gàng (vòng 6 hoặc 5
cạnh) có 4 nhóm OH và 1 nhóm C=O đều là những nhóm phân cực và tạo được
nhiều liên kết hiddro liên phân tử. Phân tử axit oleic (C18H34O2) có cấu hình cis
cồng kềnh mà chỉ có 1 nhóm OH và 1 nhóm C=O tạo được ít liên kết hiđro liên
phân tử.
e) Phân tử các chất lipit cấu tạo chủ yếu từ các nhóm không phân cực (nhóm kị nước)
và ít phân cực nên chúng tan trong dung môi không phân cực mà không tan trong
nước.

52 | Câu hỏi lí thuyết Hóa học hữu cơ OlympiaVN


Bài 27
Fomon là dung dịch khoảng 40% fomanđehit, trong đó nó tồn tại chủ yếu ở dạng
metanđiol.
a) Sử dụng các giá trị năng lượng liên kết dưới đây, hãy giải thích vì sao metanđiol
là dạng chủ yếu trong dung dich, vì sao không thể tách được nó ra khỏi dung dịch
?
Liên kết C-H =C-H C-O C=O O-H H…O
E (kJ/mol) 410 435 359 736 431 ~30
b) Vì sao fomon thường tạo thành lớp bột trắng ở đáy bình ?
c) Vì sao để bảo quản xác động vật, người ta ngâm chúng vào fomon?
d) Mục tiêu của một số người dùng fomom trong chế biến bánh phở là làm cho
bánh phở ngọt hơn, hay trắng hơn, hay dai hơn ? Giải thích? Vì sao người ta đã
không phát hiện được hàm lượng fomandehit đáng kể trong bánh phở đã dùng
fomon? Tuy vậy nó vẫn có hại cho sức khỏe người tiêu dùng, vì sao ?

53 | Câu hỏi lí thuyết Hóa học hữu cơ OlympiaVN


Hướng dẫn
a) Chưa tính liên kết hidro: CH2(OH)2 → CH2=O + H2O
ΔH hình thành: -2400 - 1606 -862 (kJ/mol)
ΔH phản ứng: -1606 – 862 – (-2400) = -68 (kJ/mol)
Thêm liên kết hidro: [CH2(OH)2...6H2O]; [CH2=O...2H2O]
ΔH hình thành: - 2400 + 8 x (-30 ) = 2580; -1606 + 2 x (-30 ) = 1666
(kJ/mol).
ΔH phản ứng: -2580 – (-1666) = 914 (kJ/mol).
b) n CH2=O → -(-CH2-O-)-n
c) CH2=O không những có tác dụng diệt vi sinh vật mà còn có tác dụng khâu
mạch các protein và các hợp chất hữu cơ khác khiến chúng trở nên bền vững khó
bị phân hủy.
d) CH2=O phản ứng khâu mạch với các nhóm OH của tinh bột làm cho nó dai
hơn. Khi đó nó không còn ở trạng thái tự do dễ phát hiện. Vào cơ thể nó có thể
được giải phóng ra và phản ứng bừa vào các nhóm OH, NH ở các hợp chất có
trong tế bào.

54 | Câu hỏi lí thuyết Hóa học hữu cơ OlympiaVN


Nhiệt động hóa học
và Động hóa học
Bài 1
Năng lượng (ở 25 oC) của các liên kết C-C, C=C, C-H và H-H lần lượt là 347,
610, 414 và 435 kJ mol-1. Tính enthalpy của phản ứng: CH2=CH2 + H-H → CH3-
CH3. Trong thực tế, sự khử ethylene thành ethane có thể được thực hiện nhanh ở
nhiệt độ phòng. Dưới những điều kiện này, có thể thực hiện phản ứng nghịch?
Hướng dẫn
Enthalpy chuẩn của một phản ứng (ΔH0) có thể được tính gần đúng bởi tổng
enthalpy cần để phá vỡ các liên kết cần thiết trong chất phản ứng và enthalpy giải
phóng khi tạo thành các liên kết mới trong sản phẩm. Chúng ta giả định rằng các
enthalpy liên kết không biến đổi nhiều trong các phân tử khác nhau, và ΔH cũng
như ΔS đều không phụ thuộc vào nhiệt độ. Do đó, với phản ứng: CH2=CH2 + H-
H → CH3-CH3
Các liên kết bị phá vỡ:
1 × enthalpy chuẩn của liên kết đôi C=C 610 kJ mol-1
1 × enthalpy chuẩn của liên kết H-H 435 kJ mol-1
Tổng enthalpy cần để phá vỡ các liên kết = +1045 kJ mol-1
Do sự phá vỡ các liên kết cần năng lượng nên phần này của phản ứng thu nhiệt
và năng lượng có giá trị dương.
Các liên kết được tạo thành:
1 × enthalpy chuẩn của liên kết C-C 347 kJ mol-1
2 × enthalpy chuẩn của liên kết C-H 2 × 414 kJ mol
Tổng enthalpy giải phóng khi tạo thành các liên kết -1175 kJ mol-1
Do sự tạo thành liên kết giải phóng năng lượng nên phần này của phản ứng tỏa
nhiệt và năng lượng có giá trị âm.
Enthalpy của phản ứng (ΔH0) = Tổng năng lượng cần để phá vỡ liên kết + Tổng
năng lượng giải phóng khi tạo thành liên kết = (+1045) + (-1175) = -130 kJ mol-
1

ΔH0 có dấu âm, thể hiện đây là quá trình tỏa nhiệt, thuận lợi. Thế nên, không có
gì lạ khi phản ứng này dễ thực hiện ở nhiệt độ phòng. ΔH0 của phản ứng nghịch
cần phải có cùng giá trị nhưng ngược dấu với phản ứng thuận, trong trường hợp
này là ΔH0 = +130 kJ mol-1. Nếu chỉ xét trên phương diện enthalpy thì phản ứng
này không có khả năng diễn ra. Tuy nhiên, việc một phản ứng có thuận lợi về mặt

55 | Câu hỏi lí thuyết Hóa học hữu cơ OlympiaVN


nhiệt động học không thì không chỉ phụ thuộc vào ΔH0 mà còn cả ΔS0 - entropy
chuẩn của phản ứng: nếu entropy của hệ tăng, thì có khả năng quá trình sẽ diễn
ra. Entropy là thước đo mức độ hỗn loạn của hệ. Sự tăng số vi hạt sẽ làm tăng
mức độ hỗn loạn và thường đi kèm với sự tăng entropy. Do đó, một quá trình làm
tăng số vi hạt trong hệ thường thuận lợi về mặt entropy. Độ trật tự cũng tăng nếu
năng lượng của các vi hạt tăng theo nhiệt độ. Các yếu tố này được mô tả dưới
dạng hàm năng lượng tự do Gibbs. Biến đổi năng lượng tự do Gibbs của một quá
trình cho trước được định nghĩa là: G0  H0  TS0 (1.1)
Nếu ΔG0 âm thì quá trình thuận lợi về mặt nhiệt động học. Một quá trình thu nhiệt
(ΔH0 > ) nếu có thể diễn ra thì phải có sự tăng entropy (ΔS0 > 0) và nhiệt độ phải
đủ cao, để TΔS0 > ΔH0. Trong phản ứng tách hydrogen từ ethane, 2 phân tử
(hydrogen và ethylene) được tạo ra từ 1 phân tử (ethane), tương ứng với sự tăng
entropy. Do đó, có thể thực hiện phản ứng nghịch tại nhiệt độ cao, khi mà TΔS0
> ΔH0 (do dó ΔG0 âm)1.

1Khi so sánh giá trị enthalpy và entropy cần lưu ý đến đơn vị, bởi enthalpy thường tính theo
kJ, còn entropy là J K-1.)

56 | Câu hỏi lí thuyết Hóa học hữu cơ OlympiaVN


Bài 2
Biến thiên entropy của quá trình tạo thành ethyl chloride bởi (a) phản ứng chloro
hóa ethane và (b) phản ứng cộng hydrogen chloride vào ethylene lần lượt là +2
và -130 J K-1 mol-1. Hãy bình luận về các dữ liệu này.
Hướng dẫn
Như đã thảo luận ở các bài trước, biến thiên entropy trong phản ứng ΔS0 chịu ảnh
hưởng lớn bửi bất kì sự biến đổi nào về số lượng các phân tử.
Trong phản ứng đầu tiên: H3C-CH3 + Cl-Cl → H3C-CH2Cl + HCl, số phân tử
không thay đổi và do đó ΔS0 tương ứng nhỏ (+2 kJ mol-1).
Trong phản ứng thứ hai: H2C=CH2 + H-Cl → H3C-CH2Cl, số phân tử giảm một
nửa, điều này dẫn đến sự giảm entropy đáng kể (ΔS0 = -130 kJ mol-1).
Bài 3
Dựa vào các dữ kiện trong bảng 1.1 và 1.2, hãy ước tính ΔH0 của các phản ứng
sau. Đánh giá khả năng diễn tiến của các phản ứng.
a) CH4 + Cl2 → CH3Cl + HCl
b) C2H5OH C2H4 + H2O
c) CH3CHO + H2O CH3CH(OH)2
Bảng 1.1: Năng lượng liên kết (kJ mol-1) ở 25 oC.(a trong formaldehyde; b trong
các aldehyde khác; c trong các ketone.)

Bảng 1.2: Năng lượng phân li liên kết (kJ mol-1).

Hướng dẫn
(a) CH4 + Cl—Cl → CH3CI + HCl. Đối với phản ứng thuận:
Các liên kết bị phá vỡ:

57 | Câu hỏi lí thuyết Hóa học hữu cơ OlympiaVN


1 × enthalpy chuẩn của liên kết H3C-H 426 kJ mol-1
1 × enthalpy chuẩn của liên kết Cl-Cl 238 kJ mol-1
Tổng enthalpy cần để phá vỡ các liên kết = +664 kJ mol-1
Các liên kết được tạo thành:
1 × enthalpy chuẩn của liên kết H3C-Cl 339 kJ mol-1
1 × enthalpy chuẩn của liên kết H-Cl 428 kJ mol-1
Tổng enthalpy giải phóng khi tạo thành các liên kết = -767 kJ mol-1
Enthalpy phản ứng (ΔH0) = (+664) + (-767) = -103 kJ mol-1
Do số phân tử không thay đổi trong quá trình phản ứng nên nhiều khả năng ΔS0
không có ảnh hưởng đáng kể đến ΔG0. ΔG0 sẽ gần với giá trị -103 kJ mol-1, cho
thấy nó sẽ có hằng số cân bằng thuận lợi cho phản ứng thuận ở nhiệt độ phòng.
Do ΔG0 = -RTlnK nên chúng ta có thể tính được giá trị gần đúng của K. Nếu ΔG0
= 103000 J mol-1 (nghĩa là chúng ta bỏ qua đại lượng entropy), R = 8.31 J K-1
mol-1 và T = 298 K thì -103000 = -(8.31 × 298 × lnK), vậy nên lnK = 41.6 và K
= 1.3·1018. Vậy nếu ΔS0 không đáng kể so với ΔH0 thì nó dường như chẳng có
đóng góp đáng kể tới ΔG0, kể cả ở nhiệt độ cao.
(b) Sử dụng giả định enthalpy chuẩn của liên kết carbon-oxygen trong CH3CH2-
OH xấp xỉ bằng giá trị trong CH3-OH. Tính toán tương tự ý (a), ΔH0= +23 kJ mol-
1

ΔH0 của phản ứng nhỏ và không thuận lợi. Do số phân tử tăng gấp đôi trong quá
trình phản ứng, nên nhiều khả năng ΔS0 có ảnh hưởng đáng kể đến ΔG0 và vượt
quá enthalpy ở nhiệt độ đủ cao. Do đó, hằng số cân bằng K thấp ở nhiệt độ thường
nhưng sẽ tăng khi nhiệt độ tăng.
(c) Tính toán tương tự ý (a), ΔH0 = -16 kJ mol-1.
Do số phân tử giảm một nửa trong phản ứng này, nên có khả năng ΔS0 có đóng
góp đáng kể vào ΔG0, đặc biệt là khi ΔH0 chỉ bằng -16 kJ mol-1. Trong khi ΔH0
thì thuận lợi với phản ứng thuận thì ΔS0 lại thuận lợi với phản ứng nghịch. Do đó,
ở nhiệt độ thấp thì phản ứng thuận ưu tiên xảy ra, nhưng khi nhiệt độ tăng lên để
đại lượng TΔS0 có đóng góp đáng kể, vượt quá đại lượng enthalpy thì phản ứng
nghịch ưu tiên hơn. Trạng thái của cân bằng này rất nhạy cảm (dễ biến đổi), và
tùy thuộc nhiều vào bản chất cụ thể của aldehyde lẫn các điều kiện phản ứng.

58 | Câu hỏi lí thuyết Hóa học hữu cơ OlympiaVN


Bài 4
Dựa vào các dữ kiện đã cho ở các bảng 1.1 và 1.2, hãy cho biết với phản ứng nào
sau đây, có thể dự đoán K < 1 ở nhiệt độ phòng? (Giả sử rằng biến thiên entropy
trong các phản ứng của ethylene đều bằng -130 kJ mol-1 và bạn có thể đưa ra các
giả định khác.) Tác động nào có thể làm tăng nhiệt độ?
a) C2H4 + Cl2 → C2H4Br2
b) C2H4 + I2 → C2H4I2
c) CH4 + Br2 → CH3Br + HBr
d) CH4 + I2 → CH3I + HI
Hướng dẫn
(a)

Dựa vào dữ kiện đã cho nthalpy phản ứng ΔH0 = -109 kJ mol-1. Cho biết entropy
của phản ứng (ΔS0 = -130 kJ mol-1).

Do ΔG0 âm, nên phản ứng thuận sẽ thuận lợi về mặt nhiệt động học và K > 1 ở
nhiệt độ thường. Khi nhiệt độ tăng, đại lượng entropy (TΔS) có đóng góp đáng
kể hơn, thậm chí cho đến khi ΔG > 0 và phản ứng nghịch trở nên thuận lợi hơn
(K < 1). Nhiệt độ tại đó phản ứng nghịch trở nên thuận lợi hơn phản ứng thuận
có thể được tính dễ dàng bởi phương trình: ΔH0 = TΔS0. Trong trường hợp này,
T = ΔH0/ΔS0 = -109 (kJ mol-1) / -0.130 (kJ K-1 mol-1) = 838 K. Tuy nhiên, ở các
giá trị nhiệt độ cao, các phản ứng khác cũng có thể xảy ra.
(b)

Tính tương tự ý (a):

59 | Câu hỏi lí thuyết Hóa học hữu cơ OlympiaVN


Do ΔG0 = 0 nên K = 1 ở nhiệt độ phòng. Khi nhiệt độ tăng lên, đại lượng entropy
có đóng góp đáng kể hơn và K giảm xuống dưới 1 và cân bằng ưu tiên chuyển
dịch theo chiều nghịch, tạo thành ethylene và iodine.
(c) CH4 + Br2 → CH3Br + HBr ΔH0 = -30 kJ mol-1.
Do không có sự thay đổi về số phân tử trong phản ứng nên ΔS0 nhỏ và dường như
không có đóng góp đáng kể đến K. Giá trị của K được dự đoán lớn hơn 1, do ΔH0
của phản ứng âm. Tuy nhiên, do giá trị ΔH0 cũng không lớn lắm, nên K có thể dễ
chịu tác động bởi các điều kiện phản ứng, bao gồm cả nhiệt độ.
(d) CH4 + I2 → CH3I + HI ΔH0 = +53 kJ mol-1
Tương tự câu (c), phản ứng thuậnkhoong thuận lợi và K được đoán < 1.

60 | Câu hỏi lí thuyết Hóa học hữu cơ OlympiaVN


Bài 5
Năng lượng bền hóa của các hợp chất liên hợp thường tính được khi so sánh nhiệt
hydrogen hóa hoặc nhiệt đốt cháy của các hợp chất này với các hợp chất không
liên hợp phù hợp.
a) Tại sao phương pháp đầu tiên cho giá trị chính xác hơn?
b) Theo bạn thì biphenyl có năng lượng bền hóa lớn hơn 2 lần của benzene
không?
c) Cho biết nhiệt hydrogen hóa của styrene là -326 kJ mol-1. Tính năng
lượng bền hóa của styrene. Sử dụng các dữ kiện cho trong bảng 1.1, 1.2.
Hướng dẫn
a) Do nhiệt độ hydrogen hóa liên quan đến việc phá vỡ các liên kết π nhưng không
ảnh hưởng đến các liên kết σ, nên chúng có khả năng phản ánh chính xác năng
lượng bền hóa tạo ra bởi hệ π. Nhiệt đốt cháy liên quan đến việc phá vỡ tất cả các
liên kết trong phân tử, do đó năng lượng bền hóa được tính sẽ có độ chính xác
thấp hơn.
b) Có thể mong đợi biphenyl có năng lượng bền hóa cao hơn 2 lần của benzene
nếu các electron của hệ π được giải tỏa trên cả hai vòng. Điều này có thể được
mô tả bởi các cấu trúc cộng hưởng:

Mặc dù cấu trúc này tồn tại dạng phẳng ở trạng thái rắn, nhưng trong phase khí
thì biphenyl có 2 vòng lệch nhau 44o. Khi thiếu các lực tương tác liên phân tử,
cấu trúc này bền hơn để tránh tương tác [đẩy] không gian của các ortho hydrogen
tồn tại trong cấu trúc phẳng. Với các biphenyl thế, lực đẩy không gian đủ lớn để
các nhóm thế vuông góc với nhau trong cấu trúc ưu thế.

Vậy năng lượng bền hóa của biphenyl xấp xỉ bằng 2 lần của benzene.
c) Sử dụng các giá trị trong bảng, tính được giá trị ΔH0 của phản ứng hydrogen
hóa styrene thành ethylcyclohexane là -412 kJ mol-1. So sánh với giá trị thực
nghiệm -362 kJ mol-1 thì tính được năng lượng bền hóa là 86 kJ mol-1.

61 | Câu hỏi lí thuyết Hóa học hữu cơ OlympiaVN


Nếu so sánh với năng lượng bền hóa của benzene là 150 kJ mol-1 thì ta thấy con
số ước lượng được ở trên là thấp hơn so với năng lượng bền hóa thực tế, bởi
styrene được bền hóa bởi cả sự cộng hưởng thơm và sự liên hợp nhỏ với liên kết
đôi.

62 | Câu hỏi lí thuyết Hóa học hữu cơ OlympiaVN


Bài 6
Trong mỗi cặp chất sau đây, chất nào có nhiệt hiđro hóa lớn hơn? Giải thích.
a) Penta-1,4-đien và penta-1,3-đien.
b) trans- và cis-4,4-đimetylpent-2-en.
Hướng dẫn
Nhiệt hình thành của hai đồng phân hợp chất không bão hòa chỉ có thể so sánh
khi hiđro hóa chúng cho cùng một sản phẩm và đo nhiệt hiđro hóa. Đồng phân
kém bền có nhiệt hiđro hóa lớn hơn và khi đó tách ra nội năng lớn hơn. Nhiệt
hiđro hóa bằng -H của phản ứng. Vì vậy, nhiệt hiđrohóa của penta-1,4-đien lớn
hơn của penta-1,3-đien; của cis-4,4- đimetylpent-2-en lớn hơn của trans-4,4-
đimetylpet-2-en.

63 | Câu hỏi lí thuyết Hóa học hữu cơ OlympiaVN


Bài 7
Động học của phản ứng dung môi phân Ph2CHCl trong dung dịch acetone tuân
theo biểu thức = d[Ph2CHCl]/dt = k[Ph2CHCl]/(1 + k’[Cl-]). Giải thích.
Hướng dẫn
Trong phản ứng dung môi phân của Ph2CHCl trong dung dịch acetone, nước đóng
vai trò là nucleophile trong phản ứng thế:

Biểu thức động học được cho phù hợp với cơ chế sau đây:
Giai đoạn 1: Một trạng thái tiền-cân bằng, trong đó một lượng nhỏ, ổn định
carbocation được tạo thành:

Giai đoạn 2: Thủy phân nhanh carbocation tạo thành sản phẩm.

Do đó, tốc độ phản ứng chịu sự chi phối của nồng độ ổn định của carbocation,
PhCH2+. Có thể tính được với giả định là nồng độ của cation này là hằng số, nghĩa
là tốc độ tạo thành của Ph2CH+ bằng tốc độ hao hụt của Ph2CH+. Ở trạng thái
dừng:
Tốc độ tạo thành Ph2CH+ = Tốc độ hao hụt Ph2CH+
Do đó:

Do H2O là dung môi, nồng độ của nó sẽ rất cao so với các tác nhân khác và sẽ
biến đổi ít trong tiến trình phản ứng. Chúng ta có thể giả định rằng nó là hằng số
và đặt k2’ = k2[H2O], trong đó k2’ được gọi là hằng số tốc độ giả bậc nhất.

Bây giờ chúng ta có một biểu thức liên hệ nồng độ của carbocation với nồng độ
của các chất phản ứng. Tốc độ là: rate = k2’[Ph2CH+] và thế biểu thức (3.5) vào:

64 | Câu hỏi lí thuyết Hóa học hữu cơ OlympiaVN


Biểu thức này tương đương với biểu thức đã cho khi (k-1/k2) = k’

65 | Câu hỏi lí thuyết Hóa học hữu cơ OlympiaVN


Bài 8
Phenylnitromethane, PhCH2NO2, là chất lỏng tan hòa tan trong dung dịch sodium
hydroxide. Khi acid hóa dung dịch này thì tautomer PhCHN(O)OH bị kết tủa,
nhưng nó từ từ chuyển ngược lại thành chất lỏng ban đầu. Giải thích bản chất hóa
học của sự tạo thành tautomer và vẽ giản đồ năng lượng phù hợp với các quan sát
trên.
Hướng dẫn
Trong môi trường base, phenylnitromethane bị deproton hóa:

Anion tạo thành có thể được biểu diễn ở dạng lai hóa cộng hưởng (các cấu trúc I
và II). Sự proton hóa trên carbon tái tạo thành chất ban đầu (phenylnitromethane).
Đây là sản phẩm bền nhiệt động của phản ứng, do đó tautomer này bền hơn.

Sự proton hóa trên oxgen tạo thành sản phẩm động học của phản ứng. Đây là ví
dụ về hiện tượng tautomer hóa nitro-acid-nitro. Tautomer III tạo thành với tốc độ
nhanh hơn phenylnitromethane nhưng kém bền hơn. Do đó, khi để yên thì III bị
chuyển từ từ thành phenylnitromethane. Quá trình này được minh họa bởi giản
đồ năng lượng:

66 | Câu hỏi lí thuyết Hóa học hữu cơ OlympiaVN


Chú ý rằng do ΔE2 < ΔE1, tautomer III được tạo thành với tốc độ nhanh hơn
phenylnitromethane. Do đó IIII là sản phẩm động học của phản ứng proton hóa.
Tuy nhiên, năng lượng tự do của III cao hơn của phenylnitromethane (sản phẩm
nhiệt động) nên III chuyển trở lại thành phenylnitromethane. Do ΔE3 > ΔE2 nên
quá trình này diễn ra chậm.

67 | Câu hỏi lí thuyết Hóa học hữu cơ OlympiaVN


Bài 9
Phản ứng Cannizzaro của benzaldehyde được cho là diễn ra theo cơ chế như sau.
Dự đoán biểu thức động học của phản ứng.

Hướng dẫn
Trong phản ứng Cannizzaro, giai đoạn 2 (chậm) là giai đoạn tốc định (quyết định
tốc độ phản ứng), dẫn đến biểu thức tốc độ có dạng như sau:

Nồng độ của anion trên có thể được tính theo nồng độ của các chất phản ứng, sử
dụng giả định về trạng thái dừng:
Tốc độ tạo thành anion = Tốc độ hao hụt anion

Thế biểu thức trên vào biểu thức (3.6):

68 | Câu hỏi lí thuyết Hóa học hữu cơ OlympiaVN


Đây là biểu thức động học tổng quát, có thể được đơn giản hóa bằng cách chia
cho k-1:

Tiếp theo, do k2 nhỏ hơn k-1 nhiều nên (k2/k-1) có thể bỏ qua, nghĩa là ta giả định
(k2/k-1) = 0. Do đó:

(ca. nghĩa là “circa” [khoảng, xấp xỉ])


Biểu thức 3.9 được đơn giản hóa thành:

Đặt k2(k1/k-1) = k, ta có:

Thực nghiệm cho thấy phản ứng Cannizzaro phù hợp với biểu thức động học này,
điều này cung cấp bằng chứng cho cơ chế giả định được dẫn ra ở trên.

69 | Câu hỏi lí thuyết Hóa học hữu cơ OlympiaVN


Bài 10
Cơ chế sau được giả định cho phản ứng giữa iodine và một hợp chất hữu cơ HA
khi có mặt hydroxide ion trong dung dịch. Giả sử rằng nồng độ của A- tại mọi
thời điểm nhỏ hơn nhiều so với của HA và IA, hãy dẫn ra biểu thức biến đổi tốc
độ của [HA] theo các hằng số tốc độ và nồng độ của HA, OH-, I2 (OH- và I2 lấy
dư).

Hướng dẫn
Giả định về trạng thái nồng độ dừng của A-:
Tốc độ tạo thành A- = Tốc độ hao hụt A-

Do [H2O] cao hốn với nồng độ của các chất phản ứng và có thể được giả sử là
biến đổi không đáng kể, chúng ta có thể thay k-1[H2O] với hằng số tốc độ giả bậc
nhất k’1. Do đó:

Giả sử rằng:

Thế biểu thức (3.15) vào (3.16), ta có:

Chia cả tử và mẫu cho k2:

70 | Câu hỏi lí thuyết Hóa học hữu cơ OlympiaVN


Trong trường hợp đặc biệt: k-1 << k2, dẫn tới k-1/k2 xấp xỉ bằng 0 thì biểu thức bị
rút gọn thành:

71 | Câu hỏi lí thuyết Hóa học hữu cơ OlympiaVN


Bài 11
Trong một chuỗi phản ứng, C có năng lượng tự do thấp hơn A, và k2 >> k-1 >>
k1. Hãy vẽ giản đồ năng lượng và dẫn ra biểu thức động học phù hợp.

Hướng dẫn
Do k2 >> k-1 >> k1, năng lượng hoạt hóa của chuyển hóa B thành C (ΔE3) thấp
hơn so với của chuyển hóa B thành A (ΔE2), và đều thấp hơn so với của chuyển
hóa A thành B (ΔE1). Giản đồ năng lượng có dạng:

Giả sử rằng B đạt tới trạng thái nồng độ dừng, tốc độ tạo thành B = tốc độ tạo
thành A.

Giả sử rằng

Thì

Chia tử và mẫu cho k2:

72 | Câu hỏi lí thuyết Hóa học hữu cơ OlympiaVN


Biết rằng k2 >> k-1, (k-1/k2) ≈ 0 và (k-1/k2) + 1 ≈ 1. Biểu thức được đơn giản hóa
thành:

73 | Câu hỏi lí thuyết Hóa học hữu cơ OlympiaVN


Cấu dạng
Bài 1
1) Vẽ công thức chiếu Newman cho các cấu trúc sau. Cho biết chúng là cấu dạng
xen kẽ hay che khuất. Hướng nhìn mỗi phân tử là từ bên phải, qua trục liên kết
được chỉ rõ bởi dấu hoa thị (*).

2) Vẽ công thức chiếu Newman cho các cấu dạng xen kẽ và che khuất của 2,2,2-
trifluoroethan-1- ol.
3) Chuyển công thức chiếu Newman sau thành công thức dạng
đường kẻ mà không làm thay đổi cấu dạng phân tử.
Hướng dẫn
a) Cấu dạng che khuất:

b) Cấu dạng xen kẽ:

c) Cấu dạng che khuất:

d) Cấu dạng xen kẽ:

74 | Câu hỏi lí thuyết Hóa học hữu cơ OlympiaVN


2)

3)

75 | Câu hỏi lí thuyết Hóa học hữu cơ OlympiaVN


Bài 2
1) Vẽ cấu dạng ghế cho các hợp chất sau:
i) 1, 1,3,3-tetramethylcyclohexane
ii) 4-bromo-1, 1-dimethylcyclohexane (đặt Br ở vị trí trục)
iii)

2) Vẽ cấu dạng từ sự nghịch đảo vòng của các cấu dạng ghế sau. Chỉ ra cấu dạng
bền nhất trong mỗi trường hợp.

3) Biểu diễn các cấu trúc dưới đây ở cả hai cấu dạng ghế:

4)
a) Nhìn dọc liên kết C-N, vẽ công thức chiếu Newman cho cấu dạng của
dimethylamine được cho bên dưới.

76 | Câu hỏi lí thuyết Hóa học hữu cơ OlympiaVN


b) Vẽ định tính đồ thị của năng lượng biến dạng với góc nhị
diện cho sự quay 360o quanh liên kết C-N này. Xác định
góc nhị diện theo hai nguyên tử C, một nguyên tử N và
một nguyên tử H của dimethyl như được chỉ ra ở bên.
c) Có bao nhiêu cấu dạng của dimethylamine có cùng năng
lượng?
d) Có bao nhiêu hàng rào quay trong dimethylamine có cùng độ cao?
e) Bạn dự đoán các hàng rào quay có độ cao bằng, lớn hơn hay nhỏ hơn so
với trường hợp methylamine (CH3NH2)? Tại sao?
5)
a) Vẽ các công thức phối cảnh cho hai cấu dạng của cis-1,3-
dimethylcyclobutane. Cấu dạng nào bền nhất? Giải thích dựa vào sức
căng nội phân tử.
b) Tương tự, vẽ hai cấu dạng của trans-1,3-dimethylcyclobutane và cho biết
cấu dạng nào chiếm ưu thế hơn.
6) a-Glucopyranose, một trong các đồng phân vòng của glucose, có công thức
dưới đây. Hãy vẽ các cấu dạng ghế của nó.

77 | Câu hỏi lí thuyết Hóa học hữu cơ OlympiaVN


Hướng dẫn
1) i)

ii)

iii)

78 | Câu hỏi lí thuyết Hóa học hữu cơ OlympiaVN


2)

3)

79 | Câu hỏi lí thuyết Hóa học hữu cơ OlympiaVN


4) a)

b) Tất cả các cấu dạng che khuất có cùng tương tác và cùng năng lượng biến dạng.

c) Tất cả các cấu dạng che khuất có cùng năng lượng. Tất cả các cấu dạng xen kẽ
có cùng năng lượng.
d) Tất cả các hàng rào có độ cao bằng nhau.
e) Các hàng rào sẽ cao hơn CH3NH2, do nhóm methyl cồng kềnh hơn (trong
dimethylamine) đã bị thay thế bởi các nguyên tử hydrogen nhỏ hơn (trong
methylamine).

80 | Câu hỏi lí thuyết Hóa học hữu cơ OlympiaVN


5) a) Dạng biên (equatorial) của cis-1,3-dimethylcyclobutane sẽ bền hơn, do nó
tối đa hóa khoảng cách của các nhóm lớn methyl với nhau.

b) Cả hai cấu dạng của trans-1,3-dimethylcyclobutane đều có các tương tác như
nhau.

6)

81 | Câu hỏi lí thuyết Hóa học hữu cơ OlympiaVN


Bài 3
1. Cho biết cấu dạng bền nhất của chất sau đây:

2. Hãy biểu diễn các cấu dạng có thể tạo vòng lacton của chất dưới đây. Từ đó vẽ cấu
dạng của lacton tương ứng.

3. Chất nào trong số 2 chất sau đây phản ứng nhanh hơn với NaBH4? Giải thích.
(Biết rằng NaBH4 khử nhóm C=O thành CHOH)

4. Giải thích hướng của cân bằng sau đây:

Hướng dẫn
1. Do có khả năng tạo liên kết hydrogen nội phân tử giữa các nhóm COOH nên cấu
dạng bền nhất của triacid sẽ là:

2. Theo đề bài, 2 vòng 6 ở cấu hình trans, vậy cấu trúc lập thể của hợp chất có khả năng
tạo vòng lactone như sau:

82 | Câu hỏi lí thuyết Hóa học hữu cơ OlympiaVN


Và vòng lactone tương ứng là :

3. Cấu trúc decalin phản ứng nhanh hơn do sự chuyển dạng lai hóa từ Csp2 thành
Csp3 dẫn đến sự giải tỏa tương tác che khuất (xem hình)

4. Hai lưỡng cực C – X và C = O bù trừ lẫn nhau khi X ở axial. Như vậy đồng
phân phân cực sẽ ưu thế hơn trong dung môi phân cực.

83 | Câu hỏi lí thuyết Hóa học hữu cơ OlympiaVN


Bài 4
Hợp chất có cấu trúc vòng 3 cạnh, tạo nên từ 2 nguyên tử carbon và 1 nguyên tử
oxygen thường được gọi là epoxy. Dưới đây là mô tả quy trình tổng hợp hợp chất
epoxy 1.

“Cho hydrocarbon X vào dung dịch chlorine ở nhiệt độ khoảng 35-50 oC, sau khi
phản ứng xảy ra thu được sản phẩm chính là hợp chất 2. Hợp chất 2 được xử lí
với một base mạnh, như calcium hydroxide, thì xảy ra phản ứng tách hydrogen
chloride và hợp chất 1 được tạo thành.”
1) Xác định cấu trúc hydrocarbon X.
Hợp chất có bộ khung Cl-C-C-OH, như hợp chất 2 ở trên, được gọi là
chlorohydrin. Khi cho chlorohydrin phản ứng với base mạnh, thu được một hợp
chất epoxy.
2 nguyên tử carbon liên kết với các nhóm hydroxyl và chloro trong hợp chất 2
được nối liên với nhau bởi một liên kết đơn và có thể quay tự do quanh trục liên
kết. Lúc này, khi một nguyên tử carbon được cố định và nguyên tử còn lại quay
thì cấu trúc mà trong đó các nhóm thế (nguyên tử/nhóm nguyên tử) xen kẽ (2a và
2’’a) và che khuất (2’a) lẫn nhau khi sự quay diễn ra được cho dưới đây. Trong
hình này, các đường nét đậm biểu diễn các liên kết hướng lên trên mặt phẳng giấy
(mặt phẳng chứa liên kết C-C trung tâm), còn đường nét đứt biểu diễn các liên
kết hướng xuống dưới.

“Công thức chiếu Newman” là phương pháp đơn giản và thuận tiện để biểu diễn
sự quay giống như sự quay quanh trục C-C trên. Như mô tả trong hình 1, công
thức chiếu Newman là cách biểu diễn phân tử được nhìn theo hướng xuyên
quatrục quay và nguyên tử carbon đặt xa hơn điểm quan sát được biểu diễn ở
dạng quả cầu đặc.

Hình 1: Cách vẽ công thức chiếu Newman.

84 | Câu hỏi lí thuyết Hóa học hữu cơ OlympiaVN


Chlorohydrin 2a, 2’a,2’’a và hợp chất epoxy 1a ở trên được biểu diễn bởi các
công thức chiếu Newman tương ứng 2b, 2’b, 2’’b và 1b dưới đây. (Với các dạng
che khuất, để thuận tiện [dễ nhìn] thì sẽ được biểu diễn theo kiểu hơi lệch một
chút.)

Còn nếu xử lí hợp chất 3 - một chlorohydrin khác với hợp chất 2 - với base mạnh
thì chỉ có duy nhất epoxy 4 được tạo thành.

2) Với các hợp chất 5 và 6, mỗi trường hợp tạo ra một hợp chất epoxy. Nếu bạn
biểu diễn các cấu trúc đó như dạng liên kết nét đậm-nét đứt kiểu hợp chất 7,
thì hãy cho biết các nhóm (a), (b), (c) là gì?

3) 8 có 2 nguyên tử chlorine. Khi phản ứng với base mạnh thì chỉ có duy nhất
một nguyên tử chlorine phản ứng, do đó epichlorohydrin - là hợp chất epoxy
có 1 nguyên tử chlorine - có thể được tạo thành. Liệu các epochlorohydrin
được tạo thành bởi phản ứng của mỗi nguyên tử chlorine trong 8 sẽ giống hay
khác nhau? Giải thích. (Ở đây chúng ta không nói đến sự khác biệt gây ra bởi
các đồng vị của các nguyên tử chlorine.)

Hợp chất epoxy có sự biến dạng không gian (ba chiều) bởi kích thước nhỏ của
vòng 3 cạnh và nó có xu hướng gây ra phản ứng mở vòng để loại bỏ sự biến dạng
đó. Ví dụ, hợp chất epoxy 9 phản ứng với phenol khi có mặt base mạnh, tạo thành
hợp chất 10.

85 | Câu hỏi lí thuyết Hóa học hữu cơ OlympiaVN


Nghiên cứu kĩ về phản ứng giữa phenol và hợp chất epoxy 9 thì thấy rằng hiện
tượng trong đó phenoxide ion (C6H5O-) đã tiếp cận với vòng 3 cạnh từ hướng bên
kia (so với oxygen).

Hình 2: Phản ứng của hợp chất epoxy 9 với phenol.


Còn nếu phản ứng với phenol, có mặt base mạnh, được tiến hành với hợp chất 4
thì tạo thành hợp chất 11.

4) Biểu diễn cấu trúc sản phẩm của phản ứng giữa hợp chất epoxy 12 với phenol
khi có mặt base mạnh theo công thức chiếu Newman (kiểu như hợp chất 13).
Xác định các nhóm (a), (b), (c), (d), (e).

Epichlorohydrin có hoạt tính độc đáo và phản ứng lần lượt với 2 phân tử phenol
khi có mặt base như sau:

86 | Câu hỏi lí thuyết Hóa học hữu cơ OlympiaVN


Khi phản ứng tương tự được tiến hành với các phenol có 2 nhóm hydroxyl thay
vì phenol đơn giản thì phản ứng trùng hợp diễn ra và một hợp chất epoxy gọi là
nhựa resin được tạo thành. Nhựa resin được dùng làm sơn, vật liệu bán dẫn, keo
dính, …. và các hợp chất polymer quen thuộc trong đời sống thường này. Một
trong các phương pháp tổng hợp được cho dưới đây. (NaCl và H2O tạo thành bởi
các phản ứng được loại bỏ [không biểu diễn] trong sơ đồ.)

5) Hoàn thành phản ứng tổng hợp nhựa resin ở trên. Chỉ cần biểu diễn phần cấu
trúc trong đường gợn sóng.

87 | Câu hỏi lí thuyết Hóa học hữu cơ OlympiaVN


Hướng dẫn
1) CH3-CH=CH2
2)

3) Khác nhau. Bởi các sản phẩm là ảnh gương của nhau và không thể chồng khít
(giống nhau) được.
4)

5)

88 | Câu hỏi lí thuyết Hóa học hữu cơ OlympiaVN


Bài 5
Hãy vẽ cấu dạng bền nhất của các chất sau

Hướng dẫn

89 | Câu hỏi lí thuyết Hóa học hữu cơ OlympiaVN


Bài 6
Nateglinide được dùng để điều trị bệnh tiểu đường, có công thức cấu tạo như
sau :
CH3

H CH3
N

O
O OH
Nateglinide

Trong đó, nguyên tử carbon bất đối ở cấu hình (R), hai nhóm thế trên xiclohexan
ở cấu hình trans. Hãy biểu diễn cấu dạng của nateglinide.
Hướng dẫn

CH3
CH3

H Nateglinide
(R)
N

HOOC H O

90 | Câu hỏi lí thuyết Hóa học hữu cơ OlympiaVN


Bài 7
1. Vẽ cấu dạng bền nhất của cis-1,1,3,4-tetrametylxiclohexan
2. Cho biết chất nào trong số 2 chất sau đây có momen lưỡng cực lớn hơn (Gợi
ý: Xét tất cả các cấu dạng có thể có của mỗi cấu trúc phẳng)

Hướng dẫn
1. Cấu dạng bền nhất:

2. Các cấu dạng có thể có:

Xét hợp chất A thì cấu dạng trong đó toàn bộ các liên kết e là cấu dạng chiếm
100% ưu thế, cấu dạng toàn a gần như không tồn tại trong cân bằng. Vì thế momen
lưỡng cực chủ yếu xét cho dạng toàn e này. Đối với cấu trúc B thì hai dạng trên
nằm cân bằng lẫn nhau. Thành ra mặc dù cấu trúc có 2 Cl ở e là cấu trúc có momen
lưỡng cực lớn nhưng do có một khoảng thời gian nó chuyển dạng cấu trúc sang 2
Cl ở a vốn có momen lưỡng cực gần như triệt tiêu nên chính cân bằng này vô hình
trung làm giảm momen lưỡng cực trung bình của cấu trúc B so với cấu trúc A.
Hệ quả là cấu trúc A có momen lưỡng cực lớn hơn.

91 | Câu hỏi lí thuyết Hóa học hữu cơ OlympiaVN


Bài 8
1. Nhiệt hydro hóa của bixiclo[2.2.2]octa-2-en và bixiclo[2.2.2]octa-2,5-dien
lần lượt là 28,2 và 56,2 kcal/mol. Tuy nhiên điều bất ngờ là nhiệt hydro hóa
của bixiclo[2.2.2]octa-2-en lại cao hơn xiclohexen (27,1 kcal/mol). Hãy giải
thích tại sao đó là điều bất thường và thử đề nghị một phương án giải thích
điều bất thường này.
2. Đối với vòng xiclohexan một nhóm thế sẽ có sự chuyển đổi cấu dạng sau đây:

Với A = -∆G = -RTlnK là giá trị năng lượng của sự cân bằng cấu dạng.
Hãy giải thích một số vấn đề sau đây
a) Trong dung môi phân cực giá trị A của OH là 0,87, còn không phân cực
là 0,52. Điều tương tự cũng xảy ra với nhóm NH2 (phân cực 1,60, không
phân cực 1,20)
b) Giá trị A của F (0,26), Cl (0,53), Br (0,50) và I (0,47).
Hướng dẫn
1. Do sự tạo thành bixiclo[2.2.2]octan làm gia tăng số tương tác van der
Waals nên bình thường nhiệt hydro hóa bixiclo[2.2.2]octa-2-en sẽ phải
thấp hơn. Lý do của sự bất thường này là bixiclo[2.2.2]octa-2-en có thể
tồn tại được ở cấu dạng xoắn và chính sức căng tạo ra trong liên kết đôi
đã làm tăng giá trị nhiệt hydro hóa
2. Trong dung môi phân cực, sự solvat hóa ion sinh ra sẽ làm tăng sự cồng
kềnh của nhóm thế nên sẽ làm tăng giá trị A. Còn trong dãy các halogen
thì việc F có giá trị A nhỏ nhất không làm chúng ta ngạc nhiên do bán
kính van der Waals của nó là nhỏ nhất, nhưng riêng với các halogen còn
lại thì không tuân theo điều đó. Vấn đề này được giải thích dựa trên độ
dài và tính phân cực của liên kết. Khi tăng kích thước của halogen thì độ
dài liên kết tăng và độ phân cực liên kết giảm (giảm mật độ e quanh
halogen) sẽ dẫn đến sự giảm tương tác không gian (thực chất là tương tác
đẩy giữa các mây e với nhau) nên giá trị A giảm.

92 | Câu hỏi lí thuyết Hóa học hữu cơ OlympiaVN


Bài 9
1. Xét các cấu dạng có thể có của 1- clobutan và chỉ rõ các cấu dạng bền
hơn [chỉ xét cấu dạng xen kẽ]
2. Campho là dẫn xuất bixicloxeton quan trọng nhất của nhóm camphan.
Hãy xác định các cấu trúc có thể có của hợp chất này

Hướng dẫn
1. Công thức cấu tạo của 1-clobutan:

Ta phải xét cấu dạng ở cả 2 liên kết a và b. Trong đó cấu dạng bền khi nhìn theo
liên kết a là cấu dạng ở đó hai nhóm Cl và Et ở vị trí anti để tránh sự đẩy nhau
giữa hai nhóm thế. Còn cấu dạng bền khi nhìn theo liên kết b là cấu dạng ở đó hai
nhóm Me và CH2Cl ở vị trí anti.

2. Các cấu trúc có thể có: về nguyên tắc có thể có 4 đồng phân quang học như
dưới đây

93 | Câu hỏi lí thuyết Hóa học hữu cơ OlympiaVN


Hai cấu trúc sau tuy vòng 6 cạnh ở dạng ghế nhưng cả hai vòng năm cạnh đều ở
dạng nửa ghế nên sức căng quá lớn và thực tế không tồn tại. Thực tế chỉ tồn tại
hai cấu trúc đầu.

94 | Câu hỏi lí thuyết Hóa học hữu cơ OlympiaVN


Bài 9
Xicloocta-1,3,5,7-tetraen (COT) được tổng hợp lần đầu tiên vào năm 1911. Khi
tương tác với kali, COT nhận thêm 2 electron theo kiểu cộng 1,4 và trở thành
đianion với 2 nguyên tử cacbon mang điện tích âm, kí hiệu là [COT]2-. Phương
pháp nhiễu xạ tia X cho thấy COT có cấu trúc không phẳng, khoảng cách giữa
các nguyên tử cacbon cạnh nhau lần lượt là 1,33 và 1,46 Å.Trong khi đó, đianion
[COT]2- có cấu trúc phẳng, khoảng cách giữa các nguyên tử cacbon cạnh nhau
đều bằng 1,41 Å.
1.Hãy vẽ cấu trúc dạng ghế và cấu trúc dạng thuyền của COT. So sánh độ bền
của hai cấu trúc này và giải thích.
2.Vẽ công thức cấu tạo của [COT]2-. Vì sao nó có cấu trúc phẳng và có các cạnh
dài như nhau?
Hướng dẫn
1.
H H H
H
8 7
H H
6
H H H H H
H H 1 5
H 2 3 4
H

Dạng thuyền Dạng ghế


Dạng thuyền bền hơn dạng ghế, vì ở dạng thuyền, tất cả các liên kết đôi đều có
cấu tạo phẳng. Trong khi đó, ở dạng ghế, hai liên kết đôi giữa C3 với C4, C7 với
C8 không đồng phẳng, tạo ra sức căng, kém bền.
2. Với cấu trúc phẳng, [COT]2- bền vì trở thành hệ thơm (đó là một hệ liên hợp
vòng khép kín, số electron π thỏa mãn qui tắc Huckel 4n+2), các cạnh có độ dài
như nhau.

... 2

95 | Câu hỏi lí thuyết Hóa học hữu cơ OlympiaVN


Bài 10
1) Vẽ hai cấu dạng 1-methylphosphetane oxide. Dự đoán hai cấu dạng này có
năng lượng bằng nhau không. Giải thích tại sao.

2) Dựa vào sự xen phủ của các orbital nguyên tử, hãy giải thích tại sao azetane
(trái) có sức căng góc nhỏ hơn aziridine (phải).

Hướng dẫn
1) Do có các tương tác 1,3-diaxial khác nhau (=O, H hoặc CH3, H) nên sức căng
trong mỗi cấu dạng khác nhau.

2) Các nguyên tử trong cả hai vòng đều lai hóa sp3. Góc liên kết 109o gần với 90o
trong vòng bốn cạnh hơn là góc 60o trong vòng ba cạnh. Do đó, sự xen phủ trong
azetane hiệu quả hơn và phân tử này bền hơn.

96 | Câu hỏi lí thuyết Hóa học hữu cơ OlympiaVN


Bài 11
Xét cân bằng cấu dạng của hai xeton sau đây:

Trong dung môi DMSO thì cấu dạng A là cấu dạng ưu thế (100%), còn nếu sử
dụng dung môi là isooctan thì cấu dạng B lại là cấu dạng chủ yếu (22% A và 78%
B). Giải thích lý do.
Hướng dẫn
Ở cấu dạng A do tương tác lưỡng cực – lưỡng cực định hướng ngược nhau nên A
là cấu dạng chủ yếu trong dung môi có tính phân cực như DMSO. Còn cấu dạng
B mặc dù có tương tác không gian lớn (nhóm Me và i-Pr ở vị trí axial) nhưng do
có liên kết hydro trong dung môi không phân cực có thể bù trừ cho tương tác
lưỡng cực và tương tác không gian bất lợi, kết quả là cấu dạng B ưu thế hơn trong
dung môi không cực.

97 | Câu hỏi lí thuyết Hóa học hữu cơ OlympiaVN


Bài 12
Decalin, C10H18, là hydrocarbon chứa hai vòng cyclohexane ngưng tụ, có chung
một liên kết C-C. Decalin tồn tại ở hai dạng: một dạng là đồng phân cis, có hai
nguyên tử carbon đánh dấu (hình tròn) của vòng bên phải ở vị trí các nhóm thế
cis so với vòng bên trái; dạng thứ hai là đồng phân trans, trong đó hai nguyên tử
carbon đánh dấu của vòng bên phải nằm ở vị trí nhóm thế trans so với vòng bên
trái.

a) Đồng phân nào ở trên là cis, đồng phân nào là trans?


b) Vẽ lại các đồng phân trên ở dạng công thức chiếu phẳng, chú ý vào sự khác
nhau giữa các đồng phân trong mối quan hệ cis-trans của chúng.
c) Xác định đồng phân nào bền hơn.
d) Một trong hai đồng phân decalin không thể xảy ra sự nghịch đảo vòng. Đó là
đồng phân nào? Giải thích nguyên nhân.
Hướng dẫn
a)

b)

c) Dạng trans không có sức căng không


gian đáng kể. Dạng cis có hai nhóm CH2,
tương tác với các nguyên tử hydrogen ở vị
trí axial và gây ra sức căng không gian. Do
đó dạng trans bền hơn.

98 | Câu hỏi lí thuyết Hóa học hữu cơ OlympiaVN


d) Dạng cis có thể xảy ra sự lật vòng (nghịch đảo).

Với dạng trans, nếu xảy ra sự nghịch đảo thì vòng mới sẽ có sức căng rất lớn.

99 | Câu hỏi lí thuyết Hóa học hữu cơ OlympiaVN


Bài 13
Trong cơ thể người và các động vật có xương sống khác,
γ-aminobutyric acid (GABA, hình bên) đóng vai trò như
chất dẫn truyền thần kinh bằng cách điều chỉnh dòng
chloride ion (Cl-) vào các tế bào của não và hệ thần kinh
trung ương. Để hiểu hơn về mối quan hệ giữa vai trò hóa sinh và các cấu dạng
của nó, các nhà nghiên cứu đã khảo sát một số hợp chất có cấu trúc rất tương
đồng với GABA nhưng về mặt cấu dạng thì cứng nhắc [kém linh động] hơn.
Một trong các hợp chất đó là (+)-CAMP, có cấu trúc như sau:

a) Vẽ công thức chiếu Newman của GABA theo cấu dạng có góc nhìn tương tự
như (+)-CAMP (được chỉ rõ theo hướng mũi tên như trên). Bạn có nghĩ rằng
cấu dạng này thuận lợi về mặt năng lượng với GABA không? Giải thích.
Một cấu trúc tương đồng khác của GABA là piperidinium carboxylate. Trong
công thức cấu tạo dưới đây của piperidinium
carboxylate, có thể thấy được phần cấu trúc GABA.
b) Vẽ cấu dạng ghế bền nhất của piperidinium
carboxylate. Nếu biểu diễn GABA theo cấu
dạng này thì có khác so với cấu dạng GABA đã
biểu diễn ở ý a không?
Có ý kiến cho rằng muscimol, hợp chất được phân lập từ nấm thuộc
chi Amanita, cũng đóng vai trò như một chất tương đồng GABA.
Vẽ cấu trúc của GABA theo cấu dạng trong đó 4 nguyên tử carbon
và nhóm ammonium ở vị trí giống như 4 nguyên tử carbon và nhóm
ammonium của muscimol (xem hình bên). Mô tả cấu dạng này của
GABA bằng cách xác định xem liệu bốn liên kết giữa hai nhóm
chức nằm ở cấu dạng xen kẽ hay che khuất? Cấu dạng này có khác
với hai cấu dạng đã vẽ ở phần a, b không?

100 | Câu hỏi lí thuyết Hóa học hữu cơ OlympiaVN


Hướng dẫn
a)

Việc cho phép các nhóm chức mang điện tích trái dấu gần nhau
nhất có thể là một tác động bền hóa rất mạnh. Nhiều khả năng
GABA, không bị giới hạn quay, sẽ thuận theo cấu dạng tối đa
hóa tương tác này.
b) Đây là một cấu dạng khác. Vòng ngăn cản các nhóm chức mang điện khỏi việc
tạo thành cấu dạng trong đó chúng bị xen kẽ.

c) Hai nhóm chức ở vị trí trans. Tương tự cấu dạng ở ý b.

101 | Câu hỏi lí thuyết Hóa học hữu cơ OlympiaVN


Bài 14
Sự tương tác của các nhóm thế trong các hợp chất mạch hở có thể được mô tả
bằng việc sử dụng mô hình mạng lưới kim cương:

1. Trong cấu trúc của gauche butan (cấu dạng xen kẽ kề), có hai hydro tương
tác đẩy lẫn nhau tương đối mạnh hơn so với các cặp hydro bất kỳ khác (năng
lượng tương tác ~ 0.9 kcal/mol), được gọi là tương tác butan bán lệch, hay
1,6-H,H. Dựa vào mô hình trên, hãy chỉ ra cặp hydrogen đó.
2. Pentan cũng có các cấu dạng như sau:

a. Sử dụng mô hình mạng kim cương, hãy mô tả các cấu dạng trên của
pentan.
b. Khoanh tròn các cặp hydro có tương tác 1,6-H,H (nếu có).
c. Trong cấu trạng syn-pentan, có một cặp hydro có tương tác mạch hơn rất
nhiều so với tương tác 1,6-H,H. Cặp hydrog này có quan hệ là 1,7 (năng
lượng tương tác ≥ 1.5 kcal/mol). Khoanh tròn cặp hydro nói trên.
3. trans-1-(metyl)etyl-2-metylxiclohexan có hai cấu dạng chủ yếu là dieq và
diax.
a) Vẽ hai cấu dạng nói trên sao cho không tồn tại tương tác 1,7-H,H.
b) Đánh dấu các cặp hydrog có tương tác 1,6-H,H.
c) Với giả thiết mỗi tương tác 1,6-H,H đóng góp khoảng 0.9 kcal/mol vào
năng lượng tự do của mỗi cấu dạng. Hãy tính tỷ số nồng độ giữa hai cấu
dạng trên.

102 | Câu hỏi lí thuyết Hóa học hữu cơ OlympiaVN


4. Phản ứng tách loại Hoffmann có thể sử dụng lý luận của tương tác 1,7-H,H
để giải thích sự chọn lọc của sản phẩm anken ít nhóm thế. Giải thích sự chọn
lọc trong phản ứng dưới đây:

Hướng dẫn
1.

2.

103 | Câu hỏi lí thuyết Hóa học hữu cơ OlympiaVN


3.

∆G0 = 6×(0.9 kcal/mol) – 4×(0.9 kcal/mol) = 1.8 kcal/mol


[𝑑𝑖𝑎𝑥] 1.8 𝑘𝑐𝑎𝑙/𝑚𝑜𝑙
= (− ) = 0.05
[𝑑𝑖𝑒𝑞] (1.987 × 10 𝑘𝑐𝑎𝑙. 𝑚𝑜𝑙 −1 . 𝐾 −1 ) × 298 𝐾
−3

4. Cấu dạng chính của muối amoni tham gia phản ứng tách loại Hoffmann:

Trong đó liên kết C-H ở nhóm ít nhóm thế đã sẵn anti-periplanar với liên kết C-
N, sẵn sàng tham gia phản ứng E2.

104 | Câu hỏi lí thuyết Hóa học hữu cơ OlympiaVN


Bài 15
Sơ đồ dưới đây biểu diễn sự tạo thành một tâm sp3 mới từ một tâm sp2. Sử dụng
các công thức chiếu Newman đã được đưa ra, hãy dự đoán cấu trúc sản phẩm
chính và giải thích ngắn gọn.

Hướng dẫn

Sản phẩm chính tạo thành từ cấu dạng bền hơn.

105 | Câu hỏi lí thuyết Hóa học hữu cơ OlympiaVN


Bài 16
1) Những giá trị năng lượng biến dạng sau đây gợi ý gì về nguồn gốc sức căng
trong các vòng ba cạnh?

2) Mặc dù cyclobutane là phân tử bị gấp khúc (khoảng 25o), tuy nhiên hợp chất
tương đồng chứa oxygen của nó là oxetane thì gần như là phẳng. Hãy đưa ra
lời giải thích.

3) Năng lượng biến dạng của spiropentane (62.5 kcal/mol) lớn hơn cả 2 lần năng
lượng biến dạng của cyclopropane (27.3 kcal/mol). Hãy đưa ra lời giải thích.

Hướng dẫn
1) Nguồn gốc của biến dạng trong các vòng ba cạnh có thể là bởi cả biến dạng
góc và biến dạng xoắn. Do các cặp chưa liên kết nhỏ hơn nhiều so với các
liên kết tới hydrogen, nên sự thay thế các liên kết C-H trong cyclopropane
bởi một cặp chưa liên kết trong aziridine hoặc hai cặp trng oxirane sẽ làm
giảm biến dạng xoắn. Do những thay đổi này không làm biến đổi năng lượng
biến dạng nhiều nên rõ ràng biến dạng trong vòng ba cạnh chủ yếu là do biến
dạng góc.
2) Sự gấp khúc trong vòng bốn cạnh là do phân tử phải làm giảm biến dạng
xoắn. Khi làm như vậy, biến dạng góc tăng thêm một chút nhưng kết quả tổng
thể thì phân tử vẫn bền hơn. Khi thay thế các liên kết C-H bởi các cặp chưa
liên kết nhỏ hơn liên kết C-H nhiều thì sẽ làm giảm biến dạng xoắn, do đó
phân tử sẽ trở nên phẳng để giảm biến dạng góc.
3) Thường thì các liên kết ngoài vòng (nghĩa là các liên kết C-H) của
cyclopropane lớn hơn 109o và thực tế là tiếp cận giá trị 120o. Đó là bởi các
liên kết vòng có đặc trưng p lớn hơn để điều tiết góc nhỏ hơn, còn các liên
kết ngoài vòng thì có đặc trưng s lớn hơn. Đặc trưng s lớn khiến cho các góc
liên kết 109o. Thêm một vòng spiro vào sẽ buộc spiro carbon phải có các góc
liên kết ngoài vòng nhỏ hơn 109o. Dẫn đến làm tăng biến dạng của hệ và biến

106 | Câu hỏi lí thuyết Hóa học hữu cơ OlympiaVN


dạng này lớn hơn nhiều so với chỉ các vòng đơn cyclopropyl (2 × 27.3
kcal/mol = 54.6 kcal/mol).

107 | Câu hỏi lí thuyết Hóa học hữu cơ OlympiaVN


Bài 17
Các hợp chất propellan là các hợp chất có khả năng phản ứng rất cao do sức căng
rất lớn, và các phản ứng này luôn xảy ra theo hướng bẻ gãy liên kết cacbon –
cacbon trung tâm. Các kết quả thực nghiệm đã cho thấy [1.1.1]propellan có khả
năng phản ứng thấp hơn [2.2.1]propellan và [2.1.1]propellan như kết quả được
trình bày ở hình dưới. Tính năng lượng liên kết C – C trung tâm trong các
propellan này và cho biết kết quả này có thể được dùng để giải thích mối quan hệ
về khả năng phản ứng của các propellan được không ? Biết năng lượng liên kết
C – H trong tất cả các trường hợp là -104 kcal/mol.

Hướng dẫn
Giá trị năng lượng liên kết lần lượt là 5, 31 và 65 kcal/mol. Đối với các phản ứng
nhiệt phân những propellane này thì người ta cho rằng chúng đi qua trạng thái
chuyển tiếp là một gốc tự do kép (diradical). Trong đó với cấu trúc của mình thì
[1.1.1]propellane khó tạo thành gốc kép nhất nên khả năng phản ứng của nó là
kém nhất. Điều này được minh họa như ở hình sau

108 | Câu hỏi lí thuyết Hóa học hữu cơ OlympiaVN


Bài 18
Năm 1985, khi nghiên cứu cẩn thận các cấu trúc nhiễu xạ tia X của một số lactone,
Burgi đã nhận thấy rằng các góc liên kết O-C-C (a) và O-C-O (b) không bằng
nhau. Trong các trường hợp này, bản chất của sự bẻ cong góc liên kết thú vị ở
chỗ chiều hướng biến dạng (a – b > 0) làm dịch chuyển các oxygen giàu electron
gần nhau hơn, thay vì ở xa nhau (a – b < 0) như dự đoán bởi các quan điểm của
sự đẩy electron-electron.

a) Tại sao bạn có thể dự đoán góc liên kết b, đặc biệt là trong lactone vòng 4
cạnh, nhỏ hơn góc liên kết a?
b) Giải thích tại sao giá trị a - b sẽ tăng khi kích thước vòng giảm?
Hướng dẫn
a) Góc O-C-O sẽ bị biến dạng để căn chỉnh tốt hơn cho tương tác giữa cặp electron
chưa liên kết của oxygen với orbital phản liên kết C-O như hình dưới đây:

b) Tăng sức căng vòng sẽ làm tăng năng lượng của orbital σ C-O và hạ năng lượng
của orbital phản liên kết σ* C-O tương ứng.

109 | Câu hỏi lí thuyết Hóa học hữu cơ OlympiaVN


Bài 19
1) Dựa vào các tính chất của vòng cyclohexane, hãy dự đoán đồng phân nào sau
đây có moment lưỡng cực lớn hơn.

2) Vẽ các đồng phân cấu dạng của cis-1,2-dimethylcyclohexane và cis-3,4-


dimethylcyclohexanone. Trong khi các cấu dạng của cyclohexane có năng
lượng bằng nhau thì các cấu dạng của cyclohexanone thì không. Chỉ ra cấu
dạng nào được ưu đãi và giải thích tại sao.
Hướng dẫn
1) Xem xét kĩ hai bộ cấu dạng ghế thì thấy trong một hợp chất cấu dạng toàn
biên (tất cả đều equatorial) được ưu đãi quá mức. Trong hợp chất còn lại, cả
hai cấu dạng ghế đều có năng lượng tương đương, do đó đều chiếm ưu thế
đáng kể. Trong cấu dạng toàn biên, các moment lưỡng cực carbon-chlorine
cũng cố lẫn nhau dẫn đến một moment phân tử lớn. Trong hợp chất còn lại,
các nguyên tử chlorine có lúc ở vị trí lưỡng biên (diequatorial), nhưng có lúc
lại ở vị trí lưỡng trục (diaxial) – tại đó các moment lưỡng cực carbon-chlorine
có xu hướng triệt tiêu lẫn nhau. Do đó, moment lưỡng cực trung bình của hai
cấu dạng này sẽ nhỏ hơn của hợp chất thứ nhất (chủ yếu tồn tại ở cấu dạng
toàn biên).

2) Trong cyclohexane, mỗi cấu dạng có một methyl trục và một methyl biên; do
đó hai cấu dạng này có năng lượng bằng nhau và đều chiếm ưu thế như nhau.
Với cyclohexanone thì cấu dạng 1 có một tương tác 1,3 diaxial methyl-proton
và một tương tác 1,3 giữa nhóm methyl và carbonyl. Còn trong cấu dạng 2,
có hai tương tác 1,3 diaxial methyl-proton. Do nhóm carbonyl nhỏ hơn đôi
chút so với nhóm CH2 nên cấu dạng 1 có các tương tác 1,3 diaxial nhỏ hơn
một chút và do đo sẽ được ưu đãi hơn một chút.

110 | Câu hỏi lí thuyết Hóa học hữu cơ OlympiaVN


111 | Câu hỏi lí thuyết Hóa học hữu cơ OlympiaVN
Bài 20
Với mỗi cân bằng dưới đây, hãy:

a) Xác định đồng phân bền hơn.


b) Xác định nguồn gốc của sư bền hóa [orbital] của đồng phân ưu thế.

112 | Câu hỏi lí thuyết Hóa học hữu cơ OlympiaVN


Hướng dẫn

Định hướng hợp phần nhường-nhận (donor-acceptor) dạng anti tốt hơn định
hướng syn.

113 | Câu hỏi lí thuyết Hóa học hữu cơ OlympiaVN


Bài 21
Giải thích tại sao phân tử H3C-S-S-CH3 khi nhìn dọc theo trục liên kết S-S thì ưu
tiên cấu dạng bán lệch. (Gợi ý: Nên vẽ công thức chiếu Newman ra để dễ nhìn.)
Hướng dẫn

Được bền hóa bởi hai tương tác ns → Không được bền hóa bởi bất kì
σ*CS thuận lợi. tương tác ns → σ*CS thuận lợi nào.

114 | Câu hỏi lí thuyết Hóa học hữu cơ OlympiaVN


Bài 22
Giải thích tại sao đối với axetandehit cấu dạng bền nhất là cấu dạng mà ở đó nhóm
C = O nằm ở vị trí che khuất với liên kết C – H trong nhóm CH3 mà không phải
là cấu dạng xen kẽ như trong hình vẽ sau:

Hướng dẫn
Lý do ở đây là ở cấu dạng che khuất có sự xen phủ giữa MO σ C – H và MO *
C = O làm tăng sự bền vững của cấu dạng che khuất (Hiệu ứng siêu liên hợp +H),
còn với cấu dạng xen kẽ thì tồn tại tương tác H/H che khuất làm giảm tính bền.

115 | Câu hỏi lí thuyết Hóa học hữu cơ OlympiaVN


Bài 23
Hệ hai vòng dung hợp cấu hình trans 2 (X = CH2) bền hơn đồng phân cis 1 (X =
CH2) với một giá trị định lượng tương ứng với 3 tương tác butane xen kẽ kề
[gauche] (3 x 0.87 kcal/mol). Tuy nhiên, acetal hai vòng dung hợp cis 1 (X = O)
lại bền hơn đồng phân trans 2 một giá trị định lượng là 0.17 kcal/mol.

Giải thích tại sao 1 (X = O) bền hơn, dựa vào những kiến thức về các hiệu ứng
điện tử.
Hướng dẫn

Tương tác này không có ở đồng phân dung hợp-trans.

116 | Câu hỏi lí thuyết Hóa học hữu cơ OlympiaVN


Bài 24
Trong ví dụ dưới đây, cân bằng được thiết lập giữa hai đồng phân cấu dạng để
tạo thành đồng phân dường như kém bền hơn nếu phân tích các hiệu ứng không
gian. Dựa vào hiểu biết của bạn về thuyết orbital phân tử biên để trả lời các câu
hỏi dưới đây.
a) Hãy xác định tương tác HOMO-LUMO chính.

b) Vẽ công thức chiếu Newman (từ góc nhìn qua liên kết O-
C) của cấu dạng ưu đãi của dimethoxymethane.
c) Biểu diễn các orbital tương tác trong cấu trúc dưới đây:

Hướng dẫn
a) Tương tác HOMO-LUMO chính:

b) Công thức chiếu Newman:

c)

117 | Câu hỏi lí thuyết Hóa học hữu cơ OlympiaVN


Bài 25
a) Ở cấu dạng ghế năng lượng thấp của tetraza-cyclohexane 1, hai nhóm thế ở
các nguyên tử nitrogen ở vị trí axial, còn hai nhóm khác lại ở vị trí equatorial,
nói cách khác là 1b bền hơn 1a. Hãy giải thích tại sao.

b) Cấu dạng 2b hơn hơn cấu dạng 2a, mặc dù nhóm CH2Ph gây ra hiệu ứng
không gian lớn hơn Me. Giải thích tại sao.

Hướng dẫn
a) Cấu dạng 1b bền hơn do các cặp electron chưa liên kết của nitrogen (NLP)
tham gia vào siêu liên hợp:

b) 2b bền hơn do sự bền hóa gây ra bởi NLP → σ*N-C(benzylic) tốt hơn NLP → σ*N-
C(allylic). σ*N-C(benzylic) có năng lượng thấp hơn σ*N-C(allylic) là bởi độ âm điện.

Bài 26
Trong dung dịch, 1,7-dioxa-spiro[5.5]undecane tồn tại ở 3 đồng phân cấu dạng
và chúng có thể chuyển hóa qua lại dễ dàng khi có mặt acid. Hãy cho biết đồng
phân cấu dạng nào chiếm ưu thế trong cân bằng này?

118 | Câu hỏi lí thuyết Hóa học hữu cơ OlympiaVN


Hướng dẫn
Đây là một ví dụ về hiệu ứng anomer. Trong dung
dịch B, cả hai nguyên tử oxygen đều ở vị trí tối ưu
cho sự xen phủ nO → σ*C-O. Chú ý rằng cả hai hợp
phần nhận oxygen đều ở vị trí trục so với vòng 6
cạnh có chứa hợp phần nhường. Từ quan điểm này,
có thể thấy rằng trong cân bằng của A, B, C thì B
chiếm ưu thế, sau đó đến C và cuối cùng là A.

119 | Câu hỏi lí thuyết Hóa học hữu cơ OlympiaVN


Bài 27
Giải thích những vấn đề sau đây
1. Người ta tìm thấy được một số gốc tự do A và B có cấu trúc như hình dưới.
Cho biết cấu trúc hình học của nguyên tử cacbon trung tâm trong hai gốc này và
giải thích tại sao nó có dạng hình học đó.

2. Cấu dạng bên phải ưu thế hơn bên trái

3. Khi khảo sát hợp chất sau thì thấy rằng với nhóm OH nằm ở e thì sẽ thể hiện
tính axit cao hơn so với lúc nhóm OH nằm ở a

Hướng dẫn
1. Do không thể đạt được cấu trúc phẳng vì sự cứng nhắc của vòng nên cả hai
gốc tự do A và B đều có dạng hình tháp với nguyên tử cacbon trung tâm lai hóa
sp3
2.
- Ở cân bằng đầu tiên cấu dạng bên trái chịu sức căng 1,3-allyl bất lợi

120 | Câu hỏi lí thuyết Hóa học hữu cơ OlympiaVN


- Ở cân bằng thứ hai liên kết hydro tạo thành sẽ làm ổn định cấu dạng bên phải

- Ở cân bằng thứ ba cấu dạng bên trái chịu tương tác bất lợi giữa các nhóm
metyl và hydro

3. Có thể giải thích theo hai hướng


- Nguyên tử hydro của nhóm OH đã bị ràng buộc bởi liên kết hydro nên khả
năng phân ly giảm
- Anion sinh ra chịu đựng tương tác bất lợi với cặp e tự do trên nguyên tử O
trong vòng làm giảm tính bền dẫn đến sự giảm tính axit của chất đầu.

121 | Câu hỏi lí thuyết Hóa học hữu cơ OlympiaVN


Bài 28

a) Dựa vào cấu dạng hãy giải thích tại sao chất cuối cùng có giá trị momen lưỡng
cực rất cao hơn phần còn lại.
b) Tổng hợp chất cuối từ pyridin. Biết rằng các hóa chất và điều kiện cần thiết
coi như có đủ
Hướng dẫn
a) Định hướng momen lưỡng cực trong ba chất đầu như sau:

Trong 1-metyl-4-piperidon thì momen lưỡng cực sẽ giảm đi so với xiclohexan do


hai momen lưỡng cực định hướng ngược chiều nhau. Tuy nhiên momen lưỡng
cực của chất cuối cùng lại lớn hơn, chứng tỏ trong cấu trúc hai momen đó phải
định hướng cùng chiều. Nhưng mặc dù hai momen có hỗ trợ cho nhau thì giá trị
cực đại cũng chỉ có thể đạt đến 4D. Như vậy rõ ràng trong cấu trúc của chất cuối
phải tồn tại sự phân chia điện tích. Tức cấu dạng thực của chất này chỉ có thể như
hình dưới:

122 | Câu hỏi lí thuyết Hóa học hữu cơ OlympiaVN


b) Sơ đồ tổng hợp:

123 | Câu hỏi lí thuyết Hóa học hữu cơ OlympiaVN


Liên hợp, cộng hưởng và tính thơm
Bài 1
Những phân tử nào sau đây có hệ liên hợp? Biểu diễn sự liên hợp trong các phân
tử.

Hướng dẫn
Trường hợp đầu tiên rất rõ ràng với một hệ liên hợp enone và một liên kết đôi
C=C không liên hợp. Bạn có thể cùng mũi tên cong để biểu diễn sự liên hợp hoặc
đưa ra một giản đồ biểu diễn sự phân bố các electron.

3 hợp chất còn lại đều được tạo ra từ một bộ khung carbon và chỉ có vị trí liên kết
đôi di chuyển. Nhóm chức ester dĩ nhiên có sự liên hợp, và đây là hệ liên hợp duy
nhất trong phân tử cuối.

Phân tử thứ ba còn có sự liên hợp giữa liên kết đôi và nhóm carbonyl.

124 | Câu hỏi lí thuyết Hóa học hữu cơ OlympiaVN


Còn phân tử thứ ba có hệ liên hợp mở rộng, từ cặp electron chưa liên kết của
nitrogen tới nhóm carbonyl.

125 | Câu hỏi lí thuyết Hóa học hữu cơ OlympiaVN


Bài 2
Xác định hệ liên hợp trong các hợp chất sau:

Hướng dẫn
Cả hai hợp chất đều là những hệ liên hợp hoàn toàn: thậm chí cả nguyên tử
nitrogen trong chất đầu và nhóm carbonyl trong chất thứ hai.

126 | Câu hỏi lí thuyết Hóa học hữu cơ OlympiaVN


Bài 3
Vẽ hai loại giản đồ biểu diễn sự liên hợp trong các phân tử sau:
a) Sử dụng mũi tên cong biểu diễn sự chuyển dịch electron trong phân tử.
b) Sử dụng các đường nét đứt và điện tích từng phần để biểu diễn sự phân bố các
liên kết bội và điện tích trong phân tử.

Hướng dẫn

127 | Câu hỏi lí thuyết Hóa học hữu cơ OlympiaVN


Bài 4
Giải Nobel Hóa học năm 2000
được tao cho giáo sư Hideki
Shirakawa. Năm 1976, ngài
Shirakawa đã thành công trong
việc dẫn dòng điện qua vật liệu
hữu cơ polyacetylene (1). Nghiên
cứu này đã được phát triển theo
nhiều hướng và các hợp chất hữu
cơ dẫn điện đã được áp dụng rộng rãi trong công nghệ pin, màn hình, … nhờ ưu
thế cực nhẹ và có thể thu nhỏ kích cỡ.
Có thể xem polyacetylene là phân tử trong đó nhiều ethylene (2) được liên kết
với nhau. Nguyên tử carboncuar ethylene có các electron liên kết yếu với mỗi hạt
nhân nhưng các electron này chịu trách nhiệm cho tính dẫn điện của
polyacetylene. Như biểu diễn trong hình 1, các electron này phân tán và tồn tại
theo hướng trực giao [vuông góc] với mặt phẳng phân tử của ethylene. Vùng xuất
hiện các electron gọi là “orbital”.

Hình 1: Giản đồ biểu diễn sự xuất hiện của electron của nguyên tử carbon trong
ethylene và các electron tồn tại thế nào trong các orbital.
Chú thích: エチレンの分子平面 = Mặt phẳng phân tử của ethylene; 核による
束縛が弱い電子 が存在する領域(軌道)= Vùng mà các electron yếu được
liên kết với các hạt nhân (orbital)
Kí hiệu màu sắc ở phần trên và phần dưới của hình biểu diễn sự khác nhau về
trạng thái của các electron. Sự bền hóa xảy ra khi các phần cùng màu tương tác
với nhau, và ngược lại sự kém bền hóa xảy ra khi những phần khác màu cạnh
nhau. Như trong hình 2, các electron của mỗi nguyên tử carbon thuộc ethyelen
đi vào các orbital tạo thành bởi sự bền hóa, và một liên kết được tạo thành (đây
là liên kết π).

128 | Câu hỏi lí thuyết Hóa học hữu cơ OlympiaVN


Hình 2: Sự tạo thành orbital π của ethylene (2).
Chú thích: エネルギー = Năng lượng; 化によって 形成された軌道 = Orbital
tạo thành bởi sự kém bền hóa; 化によって 形成された軌道 (π軌道)=
Orbital π tạo thành bởi sự bền hóa.
Liên kết π được tạo thành theo cách này. Để xem xét tính chất dẫn điện của
polyacetylene, chỉ cần hiểu về các electron có trong liên kết này.
Khi hai đơn vị ethylene được nối với nhau thì tạo thành butadiene (3). Orbital π
của butadiene được tạo thành từ 4 electron (mỗi electron từ 1 nguyên tử carbon).
Theo tính toán lí thuyết, có 4 kiểu sắp xếp màu sắc orbital như trong hình 3.
Trong đó, năng lượng của các orbital càng thấp (bền) khi sự nghịch đảo màu sắc
trên-dưới là nhỏ nhất. Khi mỗi quỹ đạo được xem là một sóng biểu diễn bởi đường
nét đứt trong mỗi orbital π trong hình 3 thì nó sẽ được xem là càng bền khi số
giao điểm với đường thẳng nối các nguyên tử carbon càng nhỏ.

Hình 3: Orbital π của butadiene.

129 | Câu hỏi lí thuyết Hóa học hữu cơ OlympiaVN


Số nút sóng được biểu diễn bởi các điểm giao cắt giữa đường nét đứt với đường
thẳng nối các nguyên tử carbon.
Mỗi orbital là một quỹ đạo trải rộng khắp phân tử và các electron có thể phân bố
khắp phân tử qua các liên kết π theo cách này. Như trong hình 3, orbital với năng
lượng cao nhất trong số các orbital có chứa electron được gọi là “orbital bị chiếm
cao nhất” (HOMO), và orbital với năng lượng
thấp nhất trong số các orbital không có electron
gọi là “orbital chưa bị chiếm thấp nhất”
(LUMO). Hãy xét polyacetylene trong đó có N
đơn vị ethylene được nối với nhau.

Hình 4: Orbital π của decapentaene.


1) Hình 4 biểu diễn 10 orbital π có thể hình dung được với phân tử decapentaene
có 5 đơn vị ethylene. Hãy chọn ra orbital bền nhất trong số các phương án từ
(ア) đến (コ).
2) Chọn ra HOMO và LUMO của decapentaene trong số các phương án từ (ア)
đến (コ).
3) Biết rằng chênh lệch năng lượng ΔE giữa HOMO và LUMO và số đơn vị
ethylene (N) của polyacetylene có mối liên hệ như trong hình 5. Trong đó eV
(electronvolt) là đơn vị năng lượng, và 1 eV = 1.6·10-19 J. Xác định ΔE của
decapentaene theo đơn vị J.

130 | Câu hỏi lí thuyết Hóa học hữu cơ OlympiaVN


Hình 5: Liên hệ giữa số đơn vị ethylene N với chênh lệch năng lượng ΔE.
4) Ethylene và butadiene không có màu, nhưng các polyacetylene có màu khi N
tăng cao. Hợp chất có màu bởi cơ chế sau: Khi bị chiếu xạ bởi ánh sáng, 1
electron của HOMO chuyển lên LUMO và năng lượng ΔE được hấp thụ. Có
mối liên hệ ΔE = hc/λ giữa bước sóng của ánh sáng bị hấp thụ và ΔE, trong
đó h là hằng số Planck và c là tốc độ ánh sáng (h = 6.6·10-34 J s, c = 3.0·108
m s-1.) Khoảng bước sóng của ánh sáng mà con người có thể nhìn được [khả
kiến] là 400 đến 800 nm (1 nm = 10-9m), và chất có thể hấp thụ ánh sáng trong
khoảng này thì có màu. Mối liên hệ giữa bước sóng và màu sắc của ánh sáng
mà chất hấp thụ được biểu diễn như trong hình 6.

Màu sắc xuất Xanh Xanh Xanh


Tím Vàng Cam Đỏ
hiện khi hấp thụ dương chàm lục

Hình 6: Mối liên hệ giữa bước sóng và màu sắc.


Chú thích: 吸収すると見える色 = màu sắc xuất hiện khi hấp thụ.
a) Xác định bước sóng của ánh sáng bị hấp thụ bởi decapentaene theo nm và
đánh giá xem liệu decapentane có màu hay không?
b) Từ hình 6 có thể thấy rằng các chất hấp thụ ánh sáng ở chủ yếu ở 500 nm thì
có màu đỏ. Polyacetylene có bao nhiêu đơn vị ethylene thì hấp thụ ánh sáng
ở bước sóng này? Giải thích.

131 | Câu hỏi lí thuyết Hóa học hữu cơ OlympiaVN


5) Khi N tăng, số orbital cũng tăng. Chênh lệch năng lượng giữa các orbital giảm
và thậm chí có thể được xem như là liên tục. Đây được gọi là “cấu trúc vùng”,
vùng chứa các electron được gọi là “vùng hóa trị”, còn vùng không chứa
electron được gọi là “vùng trống” (hình 7).

Hình 7: Số đơn vị ethylene của polyacetylene (N) và năng lượng orbital π.


Với các chất có vùng hóa trị được lấp đầy electron, các electron không thể dịch
chuyển thậm chí là khi có điện trường được áp vào, do đó dòng điện không thể
chạy qua [dẫn qua] các chất này. Tuy nhiên, nếu có một khoảng trống trong vùng
hóa trị hoặc một chất có các electron trong vùng trống thì các electron có thể di
chuyển tự do khi đeiẹn trường được áp vào và dòng điện có thể dẫn qua chất.
Giáo sư Shirakawa đã chế tạo polyacetylene với độ dẫn điện cao bằng cách thêm
vào các tạp chất ở dạng vi lượng. Ví dụ, khi trộn một lượng nhỏ lithium kim loại
với polyacetylene thì độ dẫn điện tăng lên đáng kể. Hãy giải thích hiện tượng này.
Hướng dẫn
1) ク
2) HOMO (エ), LUMO (カ)
3) 5.8·10-19 J
4) a) 340 nm. Không có màu.
b) Do λ = 500·10-9 nên ΔE = (6.6·10-34) · (3.0 · 108) / (500 · 10-9) = 4.0·10-19 (J)
hay 2.5 eV. Từ hình 5, có thể thấy rằng khi ΔE = 2.5 eV thì N khoảng 14 đến 15.
5) Khi trộn thêm lithium, một chất dễ nhường electron vào polyacetylene thì các
electron của lithium sẽ di chuyển đến polyacetylene. Do các polyacetylene có các
electron trong vùng trống nên nó sẽ có độ dẫn điện cao hơn.

132 | Câu hỏi lí thuyết Hóa học hữu cơ OlympiaVN


Bài 5
Xác định các cấu trúc nào sau đây có thể tham gia vào sự cộng hưởng. Với mỗi
trường hợp, hãy vẽ các dạng cộng hưởng có thể có.

Hướng dẫn
a) Phân tử này có một cặp electron chưa liên kết cạnh một carbocation, dẫn đến
các dạng cộng hưởng như sau:

b-d) Không thể có cộng hưởng vì điện tích bị cô lập với liên kết đôi bởi một
nguyên tử carbon no.
e) Điện tích kề với hai hệ π, dẫn đến các dạng cộng hưởng sau:

f) Điện tích kề với hai hệ π, dẫn đến các dạng cộng hưởng sau:

133 | Câu hỏi lí thuyết Hóa học hữu cơ OlympiaVN


Bài 6
Vẽ các dạng cộng hưởng cho mỗi cấu trúc sau:

Hướng dẫn
a)

b)

c)

134 | Câu hỏi lí thuyết Hóa học hữu cơ OlympiaVN


d)

135 | Câu hỏi lí thuyết Hóa học hữu cơ OlympiaVN


Bài 7
Trong formamide, CH3NO, nguyên tử N, O và một nguyên tử H được gắn với
nguyên tử carbon, còn hai nguyên tử H khác thì được gắn với nguyên tử N.
a) Vẽ cấu trúc Lewis của formamide, trong đó tất cả các orbital nguyên tử hóa
trị được lấp đầy và không có nguyên tử nào mang điện tích hình thức.
b) Formamide có thể được biểu diễn theo hai cấu trúc Lewis khả thi khác, là các
dạng cộng hưởng của phân tử. Hãy vẽ các cấu trúc Lewis này.
Hướng dẫn

136 | Câu hỏi lí thuyết Hóa học hữu cơ OlympiaVN


Bài 8
Phân tử bên biểu diễn một nhóm quan trọng, đóng vai trò như
một hợp phần nhường (donor) cặp electron trong nhiều loại
phản ứng hóa học. Cấu trúc này có ba dạng cộng hưởng (gồm
một dạng đã cho). Hãy vẽ cấu trúc các dạng này và dự đoán các
tâm trong phân tử có thể đóng vai trò như những hợp phần nhường cặp electron.
Hướng dẫn

Các cấu trúc trên cho thấy hai nguyên tử oxygen và nguyên tử carbon trung tâm
mang điện tích âm trong một cộng hưởng. Chúng sẽ là các tâm đóng vai trò hợp
phần nhường electron trong các phản ứng.

137 | Câu hỏi lí thuyết Hóa học hữu cơ OlympiaVN


Bài 9
Hãy dự đoán các cấu trúc cộng hưởng nào trong mỗi trường hợp sau đây chiếm
ưu thế nhất. Giải thích ngắn gọn:

Hướng dẫn
a) Khác biệt chính trong hai dạng này là vị trí của điện tích âm. Dạng cộng hưởng
sẽ bền hơn nếu điện tích âm nằm ở vị trí nguyên tử âm điện hơn, trong trường
hợp này là N.

b) Sự tồn tại và tách biệt của các điện tích là yếu tố chính phân biệt các dạng cộng
hưởng này.

138 | Câu hỏi lí thuyết Hóa học hữu cơ OlympiaVN


c) Dạng kém ưu tiên nhất có điện tích âm trên nguyên tử kém âm điện nhất, C.
Hai cấu trúc khác có hệ π tương tự nhau, và điện tích đều trên O nên ưu tiên như
nhau.

d) Cấu trúc bên trái có một orbital hóa trị trống trên nguyên tử carbon, sẽ kém
quan trọng hơn hai cấu trúc tương đương ở bên phải.

139 | Câu hỏi lí thuyết Hóa học hữu cơ OlympiaVN


e)

140 | Câu hỏi lí thuyết Hóa học hữu cơ OlympiaVN


Bài 10
Xác định các dạng cộng hưởng chính trong mỗi trường hợp sau:

Hướng dẫn

141 | Câu hỏi lí thuyết Hóa học hữu cơ OlympiaVN


142 | Câu hỏi lí thuyết Hóa học hữu cơ OlympiaVN
Bài 11
Vẽ tất cả các dạng cộng hưởng chính của mỗi phân tử sau và sắp xếp chúng theo
mức độ đóng góp đến lai hóa cộng hưởng tổng thể (1 = đóng góp nhiều nhất).
Nếu hai cấu trúc có mức độ đóng góp như nhau thì xếp chúng cùng mức độ [rank]
với nhau.

Hướng dẫn
a)

b)

c)

143 | Câu hỏi lí thuyết Hóa học hữu cơ OlympiaVN


d)

e)

f)

g)

144 | Câu hỏi lí thuyết Hóa học hữu cơ OlympiaVN


145 | Câu hỏi lí thuyết Hóa học hữu cơ OlympiaVN
Bài 12
Kết quả đo NMR cho thấy rằng các nhóm α-methylene (đánh dấu *) của N-
acetylpyrrolidine 1 không tương đương nhau, trong khi các nhóm α-methylene
trong N-(2-propenyl)-pyrrolidine 2 thì tương đương nhau. Giải thích dựa vào sự
cộng hưởng.

Hướng dẫn
Nếu các nhóm CH2 trong 1 không tương đương thì tốc độ quay của nhóm acetyl
là chậm để một nhóm methylene ở vị trí lân cận với nhóm C=O, còn nhóm còn
lại thì lân cận nhóm CH3. Điều này không xảy đến trong trường hợp chất 2. Tốc
độ quay của nhóm isopropenyl nhanh và các nhóm methylene thuộc một môi
trường được bình quân hóa [averaged environment]. Do độ âm điện của oxygen,
nên sự đóng góp của cấu trúc cộng hưởng 1a so với 1 quan trọng hơn nhiều sự
đóng góp của 2a với 2. Do đó, 1 có liên kết C-N mang đặc trưng liên kết đôi và
khó để thực hiện sự quay tự do. Trong 2, đặc trưng liên kết đôi của C-N nhỏ, nên
sự quay thuận lợi.

146 | Câu hỏi lí thuyết Hóa học hữu cơ OlympiaVN


Bài 13
Dùng mũi tên cong để xác định mối liên hệ giữa các cấu trúc cộng hưởng để biểu
diễn sự giải toả trong
a) diazomethane CH2N2
b) nitrous oxide, N2O
c) dinitrogen tetroxide, N2O4
Hướng dẫn

147 | Câu hỏi lí thuyết Hóa học hữu cơ OlympiaVN


Bài 14
Trong dung dịch nước, acetaldehyde (ethanal) bị hydrate khoảng 50 %. Vẽ cấu
trúc hdyrate của acetaldehyde. Trong cùng điều kiện, lại không thể phát hiện dạng
hydrate của N,N-dimethylformamide. Giải thích sự khác nhau.

Hướng dẫn
Các aldehyde bị hydrate hoá nhanh. Tuy nhiên, với các amide quá trình này sẽ
làm biến mất khả năng giải toả electron (làm bền cấu trúc) . Do vậy,
dimethylformamide không bị hydrate hoá trong dung dịch nước.

148 | Câu hỏi lí thuyết Hóa học hữu cơ OlympiaVN


Bài 14A
Hãy chú ý đến các olefinic proton (nguyên tử hydrogen gắn trực tiếp vào liên kết
đôi) và xác định ảnh hưởng của các nhóm chức khác trong phân tử đến mật độ
electron ở nguyên tử carbon gắn với proton này.
1) Trước tiên hãy vẽ tất cả các cấu trúc cộng hưởng quan trọng của mỗi phân tử.

2) Mỗi olefinic proton trên có một độ chuyển dịch hóa học khác nhau, được xác
định bởi môi trường điện tử xung quanh. Hãy gán các proton với độ dịch
chuyển hóa học lí tưởng hóa (bỏ qua tương tác spin-spin) dưới đây.

149 | Câu hỏi lí thuyết Hóa học hữu cơ OlympiaVN


Hướng dẫn
1)

2)

150 | Câu hỏi lí thuyết Hóa học hữu cơ OlympiaVN


Bài 15
Các phản ứng được đưa ra dưới đây đều bị tác động bởi các hiệu ứng cộng hưởng.
a) Xét phản ứng thế SN1 của các dẫn xuất benzyl halide với formic acid:

Sử dụng hiểu biết của bạn về sự cộng hưởng và hiệu ứng cảm ứng, hãy dự
đoán trật tự giảm dần hoạt tính của bộ ba chất nền (chất phản ứng) sau đây:

b) Đối với chất nền có hoạt tính mạnh nhất ở phần a, hãy vẽ các cấu trúc cộng
hưởng có đóng góp chính đến độ bền của tiểu phân carbonium ion trung gian.
c) Xét phản ứng acid-base của các phenol có nhóm thế sau với dung dịch
hydroxide:

Hãy dự đoán trật tự giảm dần tính acid của bộ ba phenol dưới đây:

d) Đối với phenol có tính acid mạnh nhất ở phần c, hãy vẽ các cấu trúc cộng
hưởng có đóng góp chính đến độ bền của base liên hợp.

151 | Câu hỏi lí thuyết Hóa học hữu cơ OlympiaVN


Hướng dẫn
a) Trật tự giảm dần hoạt tính (1 = mạnh nhất; 3 = yếu nhất)

b)

c) Trật tự giảm dần tính acid (1 = mạnh nhất; 3 = yếu nhất)

d)

152 | Câu hỏi lí thuyết Hóa học hữu cơ OlympiaVN


Bài 16
Giải thích các phát biểu đưới đây (Gợi ý: Hãy nghĩ về định hướng của các electron
trong không gian ba chiều).
a) B là một cấu trúc cộng hưởng hợp thức của A, tuy nhiên, D lại không phải
cấu trúc cộng hưởng hợp thức của C.

b) Mặc dù nguyên tử nitrogen trong pyridine có


một cặp electron chưa liên kết nhưng nó
không tham gia cộng hưởng với các liên kết
π xung quanh.
Hướng dẫn
a) Không thể có dạng cộng
hưởng D. Các orbital p bị ràng
buộc bởi cấu trúc vòng cứng
nhắc và không thể định hướng để
xen phủ và tạo thành liên kết π
với nguyên tử carbon kế cận.

b) Tương tự như ý a. Cặp electron chưa liên


kết của nguyên tử N đặt trong một orbital lai
hóa sp2 nằm vuông góc với các orbital p của
vòng, do đó không thể xen phủ với chúng để
tạo thành liên kết π.

153 | Câu hỏi lí thuyết Hóa học hữu cơ OlympiaVN


Bài 17
1) Hợp chất hoặc ion nào sau đây có tính thơm? Hãy vẽ giản đồ orbital để chứng
minh.

2) Xét các anion tropanyl T và cyclopentadienyl C. Tiểu phân nào bền hơn?
Giải thích tại sao.

3) Hợp chất K được phát hiện thấy gần như tồn tại ở dạng enol E. Giải thích tại
sao.

4) Giải thích tại sao hợp chất B có thể xem là phân tử có tính thơm kép?

154 | Câu hỏi lí thuyết Hóa học hữu cơ OlympiaVN


5) Giải thích tại sao bromodiazirine A bị mất bromide tạo thành cation dễ hơn
bromocyclopropane C.

6) Phản ứng của pyrrole với acid mạnh dẫn đến sự proton hóa ở vị trí C2 chứ
không phải trên nitrogen. Giải thích tại sao.

155 | Câu hỏi lí thuyết Hóa học hữu cơ OlympiaVN


Hướng dẫn
1)

156 | Câu hỏi lí thuyết Hóa học hữu cơ OlympiaVN


2)

3) Dạng tautomer K là cấu trúc được bền hóa cộng hưởng như dưới đây, tuy
nhiên dạng tautomer E mới là cấu trúc đóng góp cộng hưởng chính, bởi nó có
tính thơm. Việc tách biệt các điện tích làm giảm tính bền một chút nhưng hệ thơm
lại là một đặc trưng bền hóa rất lớn. Do đó, E có năng lượng thấp hơn K và cân
bằng giữa K và E dịch chuyển về phía K.

4) Cấu trúc cộng hưởng B’ dưới đây cho thấy mỗi vòng có 6 electron π và do đó
đều thỏa mãn quy tắc Huckel. Mỗi vòng đều là vòng thơm và phân tử có thể xem
là hệ thơm kép. Việc phải tách riêng các điện tích đi kèm với sự giảm tính bền
một chút, tuy nhiên tính bền hóa thơm làm cho B’ là cấu trúc đóng góp cộng
hưởng đáng kể. Hệ quả là hợp chất này có sự tách điện tích lớn và thực tế là có
moment lưỡng cực lớn (9.6 D).

5) Cả hai bromide đều tạo thành các benzylic cation được bền hóa cộng hưởng
bởi các vòng benzene. Các nguyên tử nitogen trong A có độ âm điện cao hơn các
nguyên tử carbon trong C, sẽ làm kém bền hóa ion đôi chút. Tác động bền hóa
lớn nhất đến từ thực tế rằng cation vòng ba cạnh A là một hệ thơm 2π. Sự bền hóa
thơm này khiến cho cation được tạo thành từ A nhanh hơn từ C rất nhiều.

157 | Câu hỏi lí thuyết Hóa học hữu cơ OlympiaVN


6) Sự proton hóa ở C2 tạo thành một cation được bền hóa cộng hưởng (dù không
thơm). Sự proton hóa ở nitrogen phá hủy tính thơm của vòng pyrrole bằng cách
lấy đi cặp electron chưa liên kết khỏi hệ thơm. Đồng thời, cation tạo thành không
được bền hóa cộng hưởng. Do đó, sự proton hóa ở C2 thuận lợi hơn.

158 | Câu hỏi lí thuyết Hóa học hữu cơ OlympiaVN


Bài 18
Phần nào trong các phân tử sau có tính thơm?

Hướng dẫn
Các con số biểu diễn số ỗi liên kết hoặc mỗi nguyên tử.
Hợp chất đầu tiên có một cặp electron chưa liên kết trên nguyên tử nitrogen, chia
sẻ giữa hai vòng. Mỗ ệ có 10 electron. Mỗi vòng
và toàn hệ đều có tính thơm. Hợp chất thứ ần
nào có tính thơm. Hợp chất thứ ồm cả cặp
electron chưa liên kết trên oxygen (nhưng không tính đến nhóm carbonyl ngoài
vòng). Hợp chất này cũng có tính thơm.

Với hai chất còn lại, chúng ta đặt số ỗi vòng và sẽ thấy


có 2 vòng thơm trong mỗi hợp chất. Chúng ta cũng không tính đến các electron
của nhóm carbonyl ở ngoài vòng. Do đó một vòng trong aklavinone chỉ có 4
h thơm.

159 | Câu hỏi lí thuyết Hóa học hữu cơ OlympiaVN


Bài 19
Các hợp chất sau được xem là có tính thơm. Giải thích bằng các xác định số
electron giải toả.

Hướng dẫn
Indone, như hình vẽ dưới đây, có 4 liên kết đôi, tương ứng với 8 electron giải toả.
Ngoài ra, còn có cặp electron chưa liên kết của nguyên tử nitrogen, nên tổng số
ạng 2n + 2). Do vậy, nó có tính thơm.
Azulene là đồng phân với naphthalene, và khá dễ để thấy nó cũng có 10

ừ các liên kết đôi trong vòng và 2 từ nguyên tử


nitrogen. Nhóm carbonyl ở ngoài vòng không đóng góp vào hệ electron này.

Adenine là một trong 4 base thực hiện mã hoá gene trong DNA. Nó có 10
ừ các liên kết đôi và 2 từ nguyên tử nitrogen trong vòng 5 cạnh.
Còn 3 nguyên tử nitrogen khác không đóng góp cặp electron bởi chúng không
được giải toả (tương tự như cặp electron trong pyridine).

160 | Câu hỏi lí thuyết Hóa học hữu cơ OlympiaVN


Bài 20
a) Trong mỗi tiểu phân sau có bao nhiêu electron π?

b) Dự đoán những cấu trúc nào ở trên thể hiện tính thơm.

161 | Câu hỏi lí thuyết Hóa học hữu cơ OlympiaVN


Hướng dẫn
1) Chú thích: BP = electron trong liên kết π (bond pair); LP = electron chưa liên
kết trong một orbital p (lone pair).

162 | Câu hỏi lí thuyết Hóa học hữu cơ OlympiaVN


2)

163 | Câu hỏi lí thuyết Hóa học hữu cơ OlympiaVN


Bài 21
1) Giả sử các tiểu phân sau đều phẳng, xác định xem liệu chúng có tính thơm,
phản-thơm hay không-thơm?

2) Vẽ các cấu trúc cộng hưởng để chỉ rõ tính thơm trong mỗi vòng của caffeine.

3) Dự đoán tính thơm của: i) [18]-annulene; ii) [22]-annulene


4) Các hợp chất sau thể hiện tính thơm, nhưng không xuất hiện trong các cấu
trúc cộng hưởng dưới đây. Với mỗi chất, hãy vẽ cấu trúc cộng hưởng để giải
thích tính thơm quan sát được.

5) Các hợp chất sau đây đều có chứa nhiều vòng. Với mỗi cấu trúc, hãy xác định
bất kì hệ thơm nào có thể tồn tại trong phân tử.

164 | Câu hỏi lí thuyết Hóa học hữu cơ OlympiaVN


6) Napthalene không có màu và không phân
cực (moment lưỡng cực = 0 D). Còn
azulene thì có màu xanh dương thẫm và
phân cực (moment lưỡng cực = 1.08 D, độ
phân cực giống như H-Cl). Giải thích quan
sát này.
7) Cyclooctatetraene không phẳng. Hãy cho biết cấu dạng ưu tiên của dianion
cyclooctatetraene.

165 | Câu hỏi lí thuyết Hóa học hữu cơ OlympiaVN


Hướng dẫn
1)
Chất a b c d e f
Số electron π 2 8 6 10 6 8
Tính thơm Thơm Phản Thơm Thơm Thơm Phản
thơm thơm
2)

3) Không có tương tác không gian nào cản trở các phân tử này có tính phẳng.

166 | Câu hỏi lí thuyết Hóa học hữu cơ OlympiaVN


4)

5)

6) Sự khác biệt giữa napthlene và azulene xuất hiện bởi các dạng cộng hưởng
khác nhau của mỗi phân tử. Napthalene không có dạng cộng hưởng mang điện
tích nào, nhưng azulene thì có nhiều dạng với sự tách biệt điện tích (một trong số
đó được biểu diễn dưới đây).

167 | Câu hỏi lí thuyết Hóa học hữu cơ OlympiaVN


Cả hai vòng azulene đều là hệ thơm sáu electron ở dạng các tiểu phân mang điện,
do đó nó có một lưỡng cực như minh họa dưới đây:

7) Dianion sẽ là một hệ thơm phẳng.

168 | Câu hỏi lí thuyết Hóa học hữu cơ OlympiaVN


Bài 22
1) Sự proton hóa các dạng 1- hoặc 5-methylcyclopenta1,3-diene đều tạo ra cùng
anion. Giải thích và vẽ cấu trúc sản phẩm.

2) Trong các điều kiện khử hóa, 1,2,3-tri(9H-fluroen9-ylidine)cyclopropane


nhận 2 electron. Hãy vẽ cấu trúc cộng hưởng bền nhất của sản phẩm dianion.

169 | Câu hỏi lí thuyết Hóa học hữu cơ OlympiaVN


Hướng dẫn
1) Sự tách proton của 1-methylcyclopenta-1,3-diene hoặc 5-methylcyclopenta-
1,3-diene sẽ tạo thành các anion có hệ 6 electron π với 5 orbital p. Do chúng đều
có tính thơm nên không thể phân biệt được với nhau.

2) Dạng cộng hưởng bền nhất sẽ cực đại hóa tính thơm. Dianion được biểu diễn
bên phải có các hệ thơm với cả ba vòng năm cạnh (5 orbital p, 6 electron) và với
vòng ba cạnh (3 orbital p, 2 electron). Tất cả các vòng 6 cạnh cũng có tính thơm.

170 | Câu hỏi lí thuyết Hóa học hữu cơ OlympiaVN


Bài 23
a) 4-Isopropyltropolon C là ví dụ đầu tiên của một hợp chất thơm không giống
benzen. Vẽ cấu trúc cộng hưởng để minh họa tính thơm của hợp chất C.

b) Proton của nhóm OH trong tropolon có tính axit. Ba mol tropolon C có thể
tác dụng với một mol tris(2,4-pentanedionato) sắt(III) [Fe(acac)3] để hình
thành một phức có màu đỏ D. Cho biết cấu trúc của D?
Hướng dẫn
a)
O- O- O- O-

OH OH OH OH
O-
OH
hay O- O- O-

OH OH OH

b) Cấu trúc của D

O
Fe
O
3

171 | Câu hỏi lí thuyết Hóa học hữu cơ OlympiaVN


Bài 24
Trong một số trường hợp khác thì các cấu trúc thơm lại chuyển thành các cấu trúc
không có vòng thơm. Hãy tìm các cấu trúc không thơm ứng với các hệ thống
thơm đã cho và giải thích tại sao các cấu trúc này lại ưu thế hơn. Cho các ví dụ
khác về sự tautome hóa mà trong đó phân tử ưu tiên tồn tại ở cấu trúc không thơm.
OH OH OCH3

N N N N N N

NH2 OH OH

Hướng dẫn
Chất đầu tiên là xitozin, chất thứ hai là uraxin. Vai trò quan trọng của sự tautomer
hóa này chính là sự tham gia của các liên kết hydro của các mạch axit nucleic.
Cấu trúc thơm sẽ làm giảm số liên kết hydro đối với các cặp ADN. Ta có thể xem
thêm guanin và thymin (còn adenin vẫn giữ cấu trúc thơm).

172 | Câu hỏi lí thuyết Hóa học hữu cơ OlympiaVN


Bài 25
Cyclooctatetraene phản ứng nhanh với potassium, tạo thành muối
K2[cyclooctatetraene]. Phản ứng tương tự của hexa(trimethylsilyl)benzene với
lithium cũng tạo thành một muối. Dự đoán hình dạng vòng trong các hợp chất
này.

Hướng dẫn
Cyclooctatetraene có cấu trúc dạng hình chậu và không phẳng, bở
của nó không tạo thành dạng 4n+2. Tuy nhiên, hai nguyên tử potassium có thể
khử cyclooctatetraene thành một dianion bằng cách chuyển cho nó 2 electron, tạo
thành hệ 10 electron – có tính thơm, và phẳng. Dưới đây là một trong các cấu trúc
cộng hưởng:

Khi cho lithium khử hexa(trimethylsilyl)benzene, hệ thơm sextet (6e) tăng lên
thành 8e, do vậy hợp chất không còn tính thơm. Vòng 6 cạnh trong muối không
còn dạng phẳng.

173 | Câu hỏi lí thuyết Hóa học hữu cơ OlympiaVN


174 | Câu hỏi lí thuyết Hóa học hữu cơ OlympiaVN
Bài 26
Dự đoán sản phẩm của phản ứng sau:

Hướng dẫn
Phản ứng đặc trưng của các vòng thơm là phản ứng thế để giữ nguyên hệ thơm
sextet (6e). Còn các alkene thì đặc trưng bởi phản ứng cộng. Trong hydrocarbon
đã cho (được gọi là indene) có chứa 1 vòng thơm và 1 vòng 5 cạnh thì không
thơm. Do hoạt tính của alkene mạnh hơn benzene nên phản ứng dễ xảy ra tại vị
trí alkene hơn:

175 | Câu hỏi lí thuyết Hóa học hữu cơ OlympiaVN


Bài 27
1) Dự đoán giản đồ năng lượng orbital phân tử cho các cấu trúc sau và giải thích
tính thơm hoặc phản-thơm của chúng.

2) Hệ MO cyclopentadineyl là một hệ vòng thơm liên hợp hoàn toàn, phẳng. Vẽ


giản đồ năng lượng MO với số electron phù hợp cho cation, anion, và gốc tự
do cyclopentadienyl. Sử dụng giản đồ để dự đoán tính bền tương đối của các
tiểu phân này.
3) Sử dụng giản đồ Frost, hãy ước lượng các mức năng lượng tương đối của các
orbital phân tử π của các anion cyclopentadienide và cycloheptatrienide. Giải
thích sự xuất hiện hoặc vắng mặt của tính thơm trong mỗi phân tử.

Hướng dẫn
1)
a) Hệ 2 electron π và 3 orbital p.

b) Hệ 6 electron π và 7 orbital p.

2) Sự khác nhau giữa ba hệ là số electron π trong các MO:

176 | Câu hỏi lí thuyết Hóa học hữu cơ OlympiaVN


3)

177 | Câu hỏi lí thuyết Hóa học hữu cơ OlympiaVN


Bài 28
1) Một trong những vấn đề thách thức nhất trong việc tạo nên protein tổng hợp
là duy trì được hóa lập thể khi các amino acid được gắn vào với nhau. Thách
thức này nảy sinh bởi các amino acid có xu hướng epimer hóa khi “được hoạt
hóa” để ghép cặp. Sự epimer hóa có thể diễn ra khi phần đầu chuỗi amino
acid tạo thành trung gian vòng như dưới đây. Hãy giải thích tại sao cấu trúc
này có thể epimer hóa dễ dàng.

2) Một nhóm bảo vệ được sử dụng để giữ cho một amine không đóng vai trò
của một nucleophile là nhóm fluorenylmethyloxycarbonyl (fmoc). Khi đến
lúc cần loại nhóm fmoc, xử lí hydroxide để loại một proton có tính acid. Điều
này gây ra sự phân mảnh phân tử, tái tạo amine. Hãy đề xuất cơ chế cho giai
đoạn loại nhóm bảo vệ và giải thích tại sao proton bị loại bỏ có tính acid.

178 | Câu hỏi lí thuyết Hóa học hữu cơ OlympiaVN


Hướng dẫn
1) Nguyên nhân sự epimer hóa diễn ra dễ dàng là bởi anion tạo thành bởi sự mất
một proton được bền hóa cộng hưởng. Sự cộng hợp proton vào anion không đặc
trưng lập thể nên xảy ra sự racemic hóa.

2) Tính acid được tăng cường của proton bị tách loại là bởi sự bền hóa thơm của
anion tạo thành. Cơ chế của quá trình tách loại được mô tả dưới đây:

179 | Câu hỏi lí thuyết Hóa học hữu cơ OlympiaVN


Tính acid – base
Bài 1
Đánh giá lực acid của mỗi nhóm hợp chất sau và sắp xếp các chất trong mỗi nhóm
theo trật tự giảm dần lực acid.
a) HCO2H, CH3CO2H, CICH2CO2H, FCH2CO2H
b) Phenol, m- và p-chlorophenol, m- và p-cresol
c) Benzoic acid, m- và p-nitrobenzoic acid, m- và p-methoxybenzoic acid.
d) 1,4-Pentadiene and cyclopentadiene

e)

f)
g) CHF3 và CHCI3

h)
Hướng dẫn
Khi xét đến lực acid, yếu tố quan trọng nhất cần xét đến là độ bền của base liên
hợp so với acid. Nhìn chung, bất kì yếu tố nào làm tăng độ bền của base liên hợp
so với acid thì sẽ làm tăng tính acid. Các thuật ngữ hiệu ứng cảm ứng và hiệu ứng
cộng hưởng (mesomeric effect) được sử dụng xuyên suốt trong các thảo luận về
tính acid và tính base. Hiệu ứng đầu tiên tác động qua khung σ của phân tử, còn
hiệu ứng sau thì qua hệ π.
Lực acid tương đối được đo bởi các giá trị pKa. Lực acid của một phân tử là phép
đo liệu nó dễ mất proton tới mức nào; phân tử có lực acid càng mạnh thì mất
proton càng nhanh. Với một acid dạng HA, chúng ta có thể viết cân bằng sau:
HA H  A 
Trong một hợp chất có lực acid rất mạnh, đa số các proton có tính acid sẽ được
giải phóng và cân bằng chuyển dịch sang bên phải. Nồng độ của H+ và A- cao,
còn nồng độ của HA thấp. Do đó, hằng số cân bằng Ka (Ka = [H+][A-]/[HA]) lớn.
Ngược lại, nếu một hợp có lực acid yếu, chỉ có một vài proton được tách ra và
cân bằng chuyển dịch sang bên trái. Do đó, hăng số cân bằng nhỏ.
Do khoảng hằng số cân bằng rất lớn nên Ka thường không được sử dụng để đo
trực tiếp tính acid, thay vào đó các giá trị pKa thường được dẫn ra. pKa của một
hợp chất là logarithm âm của giá trị Ka (pKa = -log10Ka). pKa nhỏ thì ứng với lực

180 | Câu hỏi lí thuyết Hóa học hữu cơ OlympiaVN


acid mạnh, pKa lớn thì lực acid yếu. Cần lưu ý là do sử dụng thang logarith, nên
một thay đổi nhỏ cũng thể hiện biến đổi hằng số cân bằng lớn (bởi pKa thay đổi
1 đơn vị là hằng số cân bằng đã biến đổi 10 lần.)
a) HCO2H, CH3CO2H, ClCH2CO2H, FCH2CO2H
Xu hướng biến đổi tính acid trong dãy các hợp chất X-CO2H thể hiện hiệu ứng
cảm ứng của nhóm thế X. Anion tạo thành bởi sự mất 1 proton được bền hóa bởi
các nhóm hút electron như fluorine và chlorine nhưng sẽ bị làm kém bền bởi các
nhóm nhường electron như methyl. Do đó, FCH2CO2H có lực acid mạnh nhất
trong dãy (pKa = 2.58), sau đó là ClCH2CO2H (pKa = 2.86), HCO2H (pKa = 3.75)
và CH3CO2H (pKa = 4.76).
b) Phenol, m- và p-chlorophenol, m- và p-cresol
Phenol (pKa = 10.0) có lực acid mạnh hơn các alcohol không thơm do anion tạo
thành bởi sự mất 1 proton được bền hóa qua sự giải tỏa điện tích âm khắp vòng
benzene. Hiện tượng này được minh họa bởi các cấu trúc cộng hưởng sau đây:

Lực acid của các phenol thế có thể được giải thích dựa vào ảnh hưởng của nhóm
thế trên mỗi cấu trúc cộng hưởng. Anion được tạo thành bởi sự mất 1 proton từ
chlorophenol được bền hóa bởi hiệu ứng cảm ứng âm (hút electron) từ nguyên tử
chlorine, nên các chlorophenol có lực acid mạnh hơn phenol. Tuy nhiên, tình
huống này phức tạp hơn thế: bởi nguyên tử chlorine với cặp lone pair (cặp electron
chưa liên kết) có hiệu ứng liên hợp dương (đẩy electron) với vòng benzene và
làm giảm độ bền anion2. Tác động này trở nên rõ ràng với p-chlorophenol (pKa =
9.38) hơn m-chlorophenol (pKa = 9.02).

Trong cấu trúc cộng hưởng thứ 3, lực đẩy điện tử giữa lone pair trên chlorine và
nguyên tử carbon mang điện tích âm rõ ràng không thuận lợi, và đóng góp của
cấu trúc này rõ ràng làm năng lượng bền hóa cộng hưởng giảm đi.

2 Các tác động ngược nhau của hiệu ứng cảm ứng âm và sự liên hợp nhường electron
trong các benzene thế halo- cũng được quan sát thấy trong các phản ứng thế electrophile
nhân thơm của chúng.

181 | Câu hỏi lí thuyết Hóa học hữu cơ OlympiaVN


Trong m-chlorophenol, hiệu ứng liên hợp dương có tác động làm bền hóa nhỏ
hơn do không có cấu trúc cộng hưởng tương đương nào với điện tích âm trên
meta carbon.

Trong các crezol (methylphenol), hiệu ứng cảm ứng dương của nhóm methyl làm
giảm độ bèn của anion, nên các cresol có lực acid tháp hơn phenol. Tương tự như
hiệu ứng liên hợp trong chlorophenol, hiệu ứng cảm ứng dương này trở nên rõ
ràng với p-crezol (pKa = 10.27) hơn m-cresol (pKa = 10.9).
c) Benzoic acid, m- và p-nitrobenzoic acid, m- và p-methoxybenzoic acid.
Trong benzoic acid (pKa = 4.20), anion được tạo thành bởi sự mất 1 proton được
bền hóa bởi sự giải tỏa điện tích âm trên 2 nguyên tử oxygen tương đương - được
thể hiện qua 2 cấu trúc cộng hưởng dưới đây. Trái ngược với phenol, điện tích
âm không thể được bền hóa bằng cách giải tỏa vào vòng benzene. Do lực acid
của benzoic acid lớn hơn phenol nhiều, nên có thể kết luận rằng sự giải tỏa trong
hợp phần carboxylic acid hiệu quả hơn sự giải tỏa cộng hưởng của phenoxide
anion vào vòng thơm.

Nhóm nitro là nhóm hút electron, và đo đó cũng làm bền hóa anion nên các
nitrobenzoic acid có lực acid mạnh hơn benzoic acid. p-Nitrobenzoic acid (pKa =
3.44) có lực acid mạnh hơn một chút so với m-nitrobenzoic (pKa = 3.45). Điều
này không có gì bất ngờ bởi sự tồn tại của cấu trúc cộng hưởng thứ ba dưới đây,
với điện tích dương liền kề nhóm carboxylate giàu electron - bền hơn bất kì cấu
trúc cộng hưởng tương tự nào của đồng phân meta. Chú ý rằng với mỗi cấu trúc
cộng hưởng dưới đây, đều tồn tại một cấu trúc cộng hưởng tương đương của nhóm
carboxylate.

182 | Câu hỏi lí thuyết Hóa học hữu cơ OlympiaVN


Nhóm methoxy, cũng như nhóm thế chloro, có hiệu ứng cảm ứng âm (hút
electron) nhưng lại có hiệu ứng liên hợp dương (nhường electron). Trái ngược
với nhóm thế chloro, hiệu ứng liên hợp dương của nguyên tử oxygen mạnh hơn
hiệu ứng cảm ứng âm nhiều. (Mặc dù độ âm điện của 2 nguyên tử tương đương
nhau, nhưng bán kính nguyên tử nhỏ hơn của oxygen giúp sự xen phủ orbital với
nguyên tử carbon tốt hơn, do đó hiệu ứng liên hợp mạnh hơn.) Các
methoxybenzoic acid có lực acid yếu hơn benzoic acid. Lực acid của p-
methoxybenzoic acid (pKa = 4.47) yếu hơn một chút so với m-methoxybenzoic
acid (pKa = 4.09) do cấu trúc cộng hưởng thứ ba (dưới đây), có nguyên tử carbon
mang điện tích âm cạnh nhóm carboxylate anion - kém bền hơn bất kì cấu trúc
tương đương nào của đồng phân meta.

d) 1,4-Pentadiene và cyclopentadiene.
1,4-Pentadiene (pKa = 35) có lực acid mạnh hơn hydrocarbon no tương ứng, vì
anion tạo thành bởi sự mất 1 proton được bền hóa cộng hưởng:

183 | Câu hỏi lí thuyết Hóa học hữu cơ OlympiaVN


(Việc đo pKa của 1,4-pentadiene không dễ thực hiện, do anion này tồn tại trong
cân bằng với cả 1,4-pentadiene lẫn 1,3-pentadiene [bền về mặt nhiệt động học
hơn]. Do đó, giá trị pKa trên là tính toán được chứ không phải đo trực tiếp.)
Phân tử phẳng cyclopentadiene (pKa = 15.0) thậm chí còn có lực acid mạnh hơn
bởi sự mất 1 proton sẽ tạo thành hệ vòng 6 electron (thơm), được bền hóa cộng
hưởng tương tự như benzene:

e) Trong mỗi hợp chất trên, proton có lực acid mạnh nhất đều thuộc về nguyên tử
carbon đánh dấu hoa thị (*) và anion tương ứng được bền hóa bởi sự tạo thành
enolate ion, có thể được biểu diễn bởi các cấu trúc cộng hưởng sau đây:

Cả 3 nhóm thế R đều khiến cho anion bền hơn so với dạng enolate không liên
hợp, bởi điện tích âm có thể được giải tỏa tiếp. 2 nhóm keto làm cho anion có độ
bền lớn nhất (pKa = 8.9) bởi sự giải tỏa trên 5 nguyên tử, gồm 2 nguyên tử oxygen
âm điện.

Hợp chất cyano có lực acid nhỏ hơn (pKa = 10.0) do nitrogen có độ âm điện thấp
hơn oxygen:

Nhóm thế ester làm bền hóa kém nhất (pKa = 11.0) do sự giải tỏa điện tích âm
vào nhóm carbonyl (ester) lại được bị giảm chút bởi hiệu ứng liên hợp dương của
lone pair từ oxygen:

184 | Câu hỏi lí thuyết Hóa học hữu cơ OlympiaVN


f) Trong phân tử đầu tiên (pKa > 40), sự tạo thành enolate ion rất không thuận lợi:
khi loại bỏ 1 proton, orbital có điện tích âm nằm vuông góc với các carbonyl π
orbital. Dạng hình học này ngăn cản sự xen phủ orbital để tạo thành một hệ π mở
rộng:

Cấu trúc cộng hưởng II (dưới đây) - vi phạm quy tắc Bredt - có đóng góp rất nhỏ
bởi dạng hình học của phân tử không thuận lợi cho sự xen phủ carbon-carbon π.

Với trường hợp thứ hai, enolate ion có thể được tạo thành và hợp chất này có lực
acid mạnh hơn (pKa = 20):

g) CHF3 và CHCI3
Do độ âm điện của fluorine lớn hơn nhiều so với chlorne nên F3C- được dự đoán
là bền hơn nhiều so với dẫn xuất chlorine tương ứng, và CHF3 có thể có lực acid
mạnh hơn CHCl3 rát nhiều. Tuy nhiên, thực tế thì CHCl3 có lực acid lớn hơn, do
Cl3C- được bền hóa bởi sự giải tỏa điện tích âm trong các d orbital của chlorine.
Trái lại, nguyên tử fluorine không có d orbital nên F3C- kém bền hơn nhiều. Do
đó, lực acid của CHF3 (pKa = 32) thấp hơn CHCl3 (pKa = 24) khá nhiều.
h) Dù có độ âm điện thấp hơn nhưng hợp chất của sulfur (pKa = 35) lại có lực
acid mạnh hơn chất tương ứng chứa oxygen (pKa > 40). Điều này được giải thích
bởi khả năng mở rộng vỏ bát tử của sulfur (sử dụng các orbital d), tạo thành các
cấu trúc cộng hưởng sau:

185 | Câu hỏi lí thuyết Hóa học hữu cơ OlympiaVN


186 | Câu hỏi lí thuyết Hóa học hữu cơ OlympiaVN
Bài 2
Sắp xếp các chất trong mỗi nhóm sau đây theo trình tự giảm dần lực base:
a) Ammonia, aniline, m- và p-nitroaniline.
b) Các ion: ethoxide, t-butoxide, acetate và phenoxide.
c) Pyrrole và pyrrolidine (tetrahydropyrrole).
d) Pyridine và piperidine (hexahydropyridine).
Hướng dẫn
Khi xét đến lực base, yếu tố quan trọng nhất là độ bền của acid liên hợp so với độ
bền của base. Nhìn chung, bất kì yếu tố nào làm tăng độ bền của acid liên hợp so
với của base sẽ dẫn đến làm tăng lực base. Khi đo “độ mạnh” của một base, B, ta
thường sử dụng pKa của acid liên hợp BH: B  H BH . Với base mạnh thì
acid liên hợp yếu và giá trị pKa của BH+ cao. Ngược lại, với base yếu, acid liên
hợp mạnh và giá trị pKa thấp.
a) Ammonia, aniline, m- và p-nitroaniline.
Ammonia (pKa = 9.12) là base mạnh nhất trong số các hợp chất, do có có cặp
electron chưa liên kết (lone pair) dễ tạo ra liên kết cho-nhận với proton, tạo thành
ammonium ion tứ diện:

Các aniline có lực base yếu hơn ammonia do năng lượng nhận được từ sự tạo
thành liên kết nitrogen-hydrogen bị giảm một phần bởi sự mất năng lượng cộng
khi do quá trình proton hóa. Trong phân tử trung hòa, lone pair của nitrogen liên
hợp với các electron của vòng benzene:

Hệ liên hợp này mất đi trong quá trình proton hóa bởi lone pair của nitrogen không
còn sẵn có để tham gia cộng hưởng với vòng thơm.

Vậy nên aniline (pKa = 4.60) có lực base yếu hơn ammonia. Do nhóm thế nitro là
nhóm hút electron, nó làm giảm độ bền của acid liên hợp mang điện tích dương,

187 | Câu hỏi lí thuyết Hóa học hữu cơ OlympiaVN


dẫn đến các nitroaniline có lực base yếu hơn aniline. Tác động này thể hiện ở p-
nitroaniline (pKa = 1.00) rõ hơn m-nitroaniline (pKa = 2.47). Nguyên nhân này
được làm sáng tỏ khi giải thích bằng cấu trúc cộng hưởng của các cation:

Cấu trúc cộng hưởng thứ 3 làm kém bền cation bởi sự có mặt của các điện tích
dương ở 2 nguyên tử cạnh nhau. Trường hợp này không xuất hiện với đồng phân
meta.
b) Các ion: ethoxide, t-butoxide, acetate và phenoxide
Bởi hiệu ứng nhường electron của 3 nhóm methyl, nên t-butoxide là anion kém
bền nhất được liệt kê, do đó có lực base mạnh nhất (pKa = 20.0). Ethoxide bền
hơn do chỉ có 1 nhóm alkyl nhường electron nên có lực base kém hơn t-butoxide
(pKa = 16.5). Phenoxide có lực base kém hơn các alcohol không thơm bởi anion
này được bền hóa bởi sự giải tỏa điện tích âm trong vòng benzene:

Acetate có lực base kém nhất (pKa = 4.76) do anion được bền hóa bởi sự giải tỏa
điện tích âm trên 2 nguyên tử oxygen tương đương. (Ở các bài trước chúng ta đã
biết sự giải tỏa vào nhóm -COO hiệu quả hơn so với giải tỏa vào vòng thơm.)

c) Pyrrole và pyrrolidine (tetrahydropyrrole)


Pyrrolidine (pKa = 11.3) có sẵn một cặp electron để tham gia proton hóa, do đó
có pKa điển hình của một amine bậc hai:

188 | Câu hỏi lí thuyết Hóa học hữu cơ OlympiaVN


Còn trong pyrrole thì các electron tương ứng không còn là lone pair (cặp chưa
liên kết) mà đã tham gia vào một hệ thơm 6 electron π giải tỏa, có thể được biểu
diễn bởi các cấu trúc cộng hưởng phẳng sau đây:

Sự proton hóa làm mất tính thơm (và kèm theo cả năng lượng cộng hưởng), do
đó pyrrole (pKa = -0.27) có tính base yếu hơn pyrrolidine rất nhiều. Thực tế thì
khi quá trình proton được thực hiện thì sẽ diễn ra trên carbon thay vì trên nitrogen
do nó cho phép điện tích dương được giải tỏa một phần:

Chú ý rằng nếu sự proton hóa diễn ra trên nitrogen thì không có sự giải tỏa điện
tích nào:

d) Pyridine và piperidine (hexahydropyridine)


Tương tự như pyrrolidine, piperidine (pKa = 11.1) có sẵn một lone pair cho quá
trình proton hóa, do đó có pKa điển hình của một amine bậc hai.

Không như pyrrole, pyridine (pKa = 5.25) có vòng phẳng và có hệ electron 6π


hoàn chỉnh, cộng thêm 1 lone pair trên nguyên tử (vuông góc với hệ π). Do đó,
sự proton hóa có thể diễn ra trên nitrogen mà không làm mất tính thơm:

Kết quả là giá trị pKa của nó cao hơn pyrrole nhiều nhưng vẫn thấp hơn đáng kể
so với hợp chất thơm tương đồng (piperidine). Khác biệt này thể hiện thực tế rằng
trong pyridine, lone pair của nguyên tử nitrogen thuộc orbital sp2 - về bản chất là

189 | Câu hỏi lí thuyết Hóa học hữu cơ OlympiaVN


có tính base kém hơn lone pair của piperidine (thuộc orbital sp3). Nhìn chung, bản
chất s càng lớn thì tính base của orbital đó càng thấp3. Do đó, trong các nitrile,
lone pair của nguyên tử nitrogen thuộc orbital sp và gần như không có tính base.

3Khi bản chất (%) s tăng thì electron càng bị hút gần các hạt nhân nguyên tử và càng khó
sẵn sàng tham gia liên kết.

190 | Câu hỏi lí thuyết Hóa học hữu cơ OlympiaVN


Bài 3
Vẽ đồ thị tọa độ phản ứng của phản ứng acid-base trong đó acid HA phản ứng
với Y- và
a) Cân bằng ưu tiên chiều tái tạo chất phản ứng.
b) Cân bằng không ưu tiên tạo thành sản phẩm hoặc tái tạo chất phản ứng.
c) Base liên hợp kém bền hơn base ban đầu.
d) Base liên hợp bền hơn base ban đầu.
Hướng dẫn
a) Do cân bằng ưu tiên tái tạo chất phản ứng, nên chúng phải có năng lượng
thấp hơn sản phẩm.

b) Chất phản ứng và sản phẩm phải ở cùng mức năng lượng.

c) Do base liên hợp A- kém bền hơn base Y- nên chất phản ứng phải ở mức
năng lượng thấp hơn và được ưu tiên hơn.

191 | Câu hỏi lí thuyết Hóa học hữu cơ OlympiaVN


d) Do base liên hợp A- bền hơn base Y- nên sản phẩm phải có năng lượng thấp
hơn và được ưu tiên tạo thành hơn.

192 | Câu hỏi lí thuyết Hóa học hữu cơ OlympiaVN


Bài 4
Aniline và cyclohexylamine đều là các
acid rất yếu, tuy nhiên aniline vẫn có
tính acid mạnh hơn cyclohexylamine
một chút. Sử dụng thông tin này, hãy vẽ
đồ thị tọa độ phản ứng biểu diễn sự phân
li của cả hai acid liên hợp bởi base tổng quát B-.
Hướng dẫn
Do cả hai acid đều rất yếu nên cân bằng của chúng sẽ ưu tiên tạo thành chất phản
ứng thay vì sản phẩm. Đồ thị tọa độ phản ứng sẽ được vẽ sao cho sản phẩm nằm
ở mức năng lượng cao hơn chất phản ứng – đối với cả hai amine. Do aniline có
tính acid cao hơn một chút so với cyclohexylamine nên nó sẽ ưu tiên tạo thành
sản phẩm hơn một chút. Do đó, đường cong năng lượng của nó cần kết thúc với
sản phẩm nằm ở mức năng lượng thấp hơn một chút (bền hơn) so với
cyclohexylamine.

193 | Câu hỏi lí thuyết Hóa học hữu cơ OlympiaVN


Bài 5
Với các cặp chất sau, hãy dự đoán chất có lực acid mạnh hơn:

Hướng dẫn
a)

Cả hai acid đều mang điện, do đó, chúng có thể được so sánh trực tiếp. Điện tích
dương trên nguyên tử O âm điện hơn thì sẽ kém bền hơn trên nguyên tử N kém
âm điện hơn, do đó, ether được proton hóa sẽ có lực acid mạnh hơn.
b)

Điện tích dương trên nhóm carbonyl được proton hóa trong phân tử đầu tiên được
bền hóa bởi sự cộng hưởng. Còn phân tử thứ có oxygen mang proton nên không
có dạng cộng hưởng. Do phân tử đầu là tiểu phân bền hơn, nên phân tử thứ hai có
tính acid mạnh hơn (dễ phân li mất H+ hơn).

c)

194 | Câu hỏi lí thuyết Hóa học hữu cơ OlympiaVN


d)

e)

f)

Để xác định tính acid tương đối, chúng ta cần so sánh các base liên hợp mang
điện tích. Việc tách bất kì hydrogen nào khỏi alkene đều để lại một điện tích âm
cô lập (ví dụ CB1), trong đó điện tích trên một nguyên tử carbon lai hóa sp3. Còn
việc tách hydrogen khỏi carbon của alkene (ví dụ CB2) để lại điện tích trên carbon
lai hóa sp2.

Điện tích âm trên CB2 bền hơn trong CB1 do đặc tính s lớn hơn của orbital sp2.
Do đó alkene có lực acid mạnh hơn.
Thậm chí, việc tách một hydrogen bậc ba trong alkene còn tại ra một base liên
hợp khác (CB3), trong đó điện tích âm được giải tỏa bởi sự cộng hưởng.

195 | Câu hỏi lí thuyết Hóa học hữu cơ OlympiaVN


g)

h)

Base liên hợp của nhóm có tính acid mạnh nhất trong mỗi trường hợp sẽ là:

Điện tích âm trên base liên hợp CB1 được bền hóa hơn bởi kích thước nguyên tử
lớn hơn của nguyên tử S so với nguyên tử O trong CB2.

196 | Câu hỏi lí thuyết Hóa học hữu cơ OlympiaVN


Bài 6
Sắp xếp các nhóm hợp chất sau đây theo thứ tự tăng dần tính acid:

197 | Câu hỏi lí thuyết Hóa học hữu cơ OlympiaVN


Hướng dẫn
a) Do các phân tử đều trung hòa điện, nên cần so sánh tính bền của các base liên
hợp (CB - conjugate base). Ba base liên hợp là:

CB2 là kém bền nhất do nó không có dạng cộng hưởng. CB1 và CB3 có các dạng
cộng hưởng nhưng điện tích trong CB3 được giải tỏa hơn. Do đó CB3 bền hơn
CB1, và bền hơn CB2. Do đó tính acid tương đối sẽ là:

b) Tương tự như trên, so sánh các base liên hợp:

CB4 không có các nhóm âm điện và các nguyên tử carbon nhường electron sẽ hơi
làm kém bền hóa điện tích âm trên nguyên tử O bởi hiệu ứng cảm ứng. Tất cả các
phân tử khác đều có nhóm âm điện, chúng có thể làm giảm mật độ electron trên
nguyên tử O mang điện và làm bền hóa base liên hợp. Do F hút electron mạnh
hơn Cl nên CB2 bền hơn CB3. Cả CB1 và CB3 đều có ba nguyên tử chlorine,
nhưng nhóm CCl3 của CB1 ở xa oxygen hơn và sẽ có hiệu ứng bền hóa kém hơn.
Do đó, trật tự tăng dần tính bền của các base liên hợp là CB4, CB1, CB3, CB2.
Trật tự tăng dần tính acid:

198 | Câu hỏi lí thuyết Hóa học hữu cơ OlympiaVN


c) So sánh các base liên hợp:

CB1 kém bền nhất do nó không có các nhóm hút electron để bền hóa (nhờ giải
tỏa) điện tích âm trên nguyên tử oxygen. CB2 và CB3 đều có nhóm nitro hút
electron, làm bền hóa điện tích âm. Trong CB3, hiệu ứng này được tăng cường
do điện tích âm được giải tỏa trên nhóm nitro nhờ sự cộng hưởng. Trong CB2,
không thể có sự giải tỏa điện tích âm nhờ cộng hưởng.

Do đó tính bền của các base liên hợp là: CB1 (kém bền nhất) < CB2 < CB3 (bền
nhất) và trật tự tăng dần tính acid là:

199 | Câu hỏi lí thuyết Hóa học hữu cơ OlympiaVN


d) Các base liên hợp:

CB2 không có cộng hưởng và không có các nhóm hút electron gần nguyên tử
oxygen mang điện tích âm, trong khi đó CB1 và CB3 đều có những đặc tính này.
Do đó, CB2 sẽ là base liên hợp mạnh nhất (kém bền nhất).

Các dạng cộng hưởng trong cả CB1 và CB3 đều giải tỏa điện tích vào các nguyên
tử oxygen kế cận, nhưng CB1 thực hiện giải tỏa trên ba nguyên tử oxygen, còn
CB3 chỉ có sự tham gia của hai oxygen. Do đó, trật tự tăng dần tính bền của các
base liên hợp là CB2 (kém bền nhất), CB3, CB1 (bền nhất). Trật tự tăng dần tính
acid là:

e) So sánh các base liên hợp:

Các base này đều được bền hóa bởi sự cộng hưởng, nhưng sự khác biệt chính là
độ âm điện của nguyên tử mang điện âm. Nguyên tử càng âm điện thì base liên
hợp tương đối càng bền.
Trật tự tăng dần tính acid:

200 | Câu hỏi lí thuyết Hóa học hữu cơ OlympiaVN


f) Trong trường hợp này, khác biệt chính là dạng lai hóa của nguyên tử carbon
mang điện âm. Bản chất s càng lớn thì base liên hợp càng bền.

Do đó, tính bền của CB2 (carbon sp3) < CB1 (sp2) < CB3 (sp). Trật tự tăng dần
tính acid:

201 | Câu hỏi lí thuyết Hóa học hữu cơ OlympiaVN


Bài 7
Sắp xếp các nhóm hợp chất sau đây theo thứ tự tăng dần tính base:

Hướng dẫn
a) Các tiểu phân này mang điện và độ bền tương đối của chúng có thể được so
sánh trực tiếp:

Hợp chất B2 không có bền hóa cộng hưởng, còn B1 và B3 đều có các dạng cộng
hưởng.

202 | Câu hỏi lí thuyết Hóa học hữu cơ OlympiaVN


B1 có điện tích âm được giải tỏa trên ba nguyên tử oxygen, trong khi đó B3 chỉ
sử dụng hai nguyên tử. Do đó, B1 bền hơn B3. Tính bền giảm khi tính base càng
mạnh, do đó trật tự tăng dần tính base là:

b)

c) Cả ba tiểu phân đều có bền hóa cộng hưởng:

203 | Câu hỏi lí thuyết Hóa học hữu cơ OlympiaVN


CB3 bền nhất do điện tích âm được giải tỏa nhất. B2 và B1 được giải tỏa qua hai
nguyên tử nhưng trong B1 thì một trong số đó là oxygen, thay vì là carbon như
trong B2. Do đó B1 bền hơn B2. Trật tự tăng dần tính bền của các tiểu phân này
là B2, B1, B3 – dẫn đến trật tự tăng dần tính base là:

d)

e)

f) Các hợp chất này không mang điện và cần so sánh acid liên hợp (CA - conjugate
acid) mang điện của chúng:

Các nhóm thế có hiệu ứng cảm ứng khác nhau với điện tích âm trong mỗi acid
liên hợp. Nhóm methyl trong CA1 là nhóm nhường electron và làm bền hóa điện
tích dương. Các nhóm hydroxyl và amino trong CA2 và CA3 là các nhóm hút
electron và làm kém bền hóa điện tích dương. Do oxygen âm điện hơn nitrogen
nên hiệu ứng hút electron mạnh hơn, do đó CA2 kém bền hơn CA3. Trật tự tăng
dần tính bền của các acid liên hợp là: CA2 < CA3 < CA1 và trật tự tăng dần tính
base là:

204 | Câu hỏi lí thuyết Hóa học hữu cơ OlympiaVN


205 | Câu hỏi lí thuyết Hóa học hữu cơ OlympiaVN
Bài 8
Sắp xếp tính acid tương đối của các proton được chỉ rõ trong mỗi hợp chất sau.

Hướng dẫn

206 | Câu hỏi lí thuyết Hóa học hữu cơ OlympiaVN


a) Để ước lượng tính acid của các loại proton này, hãy so sánh base liên hợp
tương ứng của chúng.

CB-B bền nhất do nó là tiểu phân duy nhất được bền hóa cộng hưởng. Giữa hai
tiểu phân còn lại thì CB-A bền hơn do nó có điện tích âm trên orbital sp2, có đặc
trưng s lớn hơn sp3 trong CB-C. Do đó, độ bền của các base liên hợp là: CB-C,
CB-A, CB-B và trật tự tăng dần tính acid của các proton trong phân tử ban đầu là
C < A < B.
b) Các base liên hợp được tạo ra bằng cách loại các proton này là:

CB-A không có bền hóa cộng hưởng, còn CB-B và CB-C thì đều có. CB-B có thể
giải tỏa điện tích âm trên cả hai nguyên tử oxygen của nhóm carbonyl, còn CB-C
thì chỉ giải tỏa được trên một nguyên tử như vậy. Do đó tính bền CB-A < CB-C
< CB-B. Tính acid: A < C < B.
c)

Tính acid: B < A < C

207 | Câu hỏi lí thuyết Hóa học hữu cơ OlympiaVN


d)

Tính acid: A < B


e)

Tính acid: A < B


f)

Tính acid: A < B < C


g) Các base liên hợp:

CB-A rất kém bền, với một cặp electron chưa liên kết cũng như một orbital trống
trên nguyên tử carbon. CB-B có thể tạo thành hệ thơm rất bền (xem ở trên). Do
đó tính acid: A < B.

208 | Câu hỏi lí thuyết Hóa học hữu cơ OlympiaVN


h) Vẽ các dạng cộng hưởng của acid:

Có thể thấy rằng tất cả các nguyên tử nitrogen trong phân tử tương đương nhau.
Do đó, cả ba tâm A, B, C sẽ có tính acid như nhau.

209 | Câu hỏi lí thuyết Hóa học hữu cơ OlympiaVN


Bài 9
So sánh các giá trị pKa2 và đọc tên IUPAC của 3 axit: Fumaric (F), Maleic (M)
và Squaric (S) có công thức và trị pKa cho như sau:

Hướng dẫn
pKa2 của axit squaric lớn hơn pKa2 của hai axit fumaric và maleic do dianion
của axit squaric tạo được hệ thống liên hợp bền vững

210 | Câu hỏi lí thuyết Hóa học hữu cơ OlympiaVN


Bài 10
Các hợp chất A và B dưới đây có cùng giá trị pKa. Ngược lại, hợp chất C có pKa
thấp hơn nhiều so với hợp chất D. Giải thích các quan sát này.

Hướng dẫn

Để đánh giá tính acid của các hợp chất trung hòa này, cần nhìn vào base liên hợp
của chúng. Cả bốn base liên hợp đều được hưởng lợi từ sự bền hóa cộng hưởng;
tuy nhiên, một điểm khác biệt quan trọng sẽ được nhìn thấy khi xem xét các đóng
góp cộng hưởng, làm dịch chuyển điện tích vào các nhóm thế trên vòng thơm.
Mặc dù có thể vẽ ra cấu trúc như vậy cho cả bốn base liên hợp, nhưng đóng góp
cộng hưởng được vẽ cho CB-D (base liên hợp của D) không phải là cấu trúc cộng
hưởng hợp thức.

211 | Câu hỏi lí thuyết Hóa học hữu cơ OlympiaVN


Để một cộng hưởng như vậy tồn tại, các orbital p liên quan cần phải song song
với nhau để xen phủ hiệu quả. Do các nhóm isopropyl cồng kềnh trong CB-D nên
nhóm nitro phẳng không thể nằm trong mặt phẳng của vòng, thay vào đó, nó cần
quay để tránh các tương tác không gian với các nhóm isopropyl. Điều này có
nghĩa là các orbital p không được xếp cùng với hệ π của vòng để đạt được xen
phủ, và không thể cộng hưởng với nhóm nitro.

212 | Câu hỏi lí thuyết Hóa học hữu cơ OlympiaVN


Bài 11
Tiểu phân nào của hydroxy-acid sau đây có thể tồn tại trong: a) nước ở pH 7; b)
dung dịch kiềm ở pH 12; c) dung dịch acid vô cơ đặc

Hướng dẫn

213 | Câu hỏi lí thuyết Hóa học hữu cơ OlympiaVN


Bài 12
Các phenol sau có giá trị pKa xấp xỉ 4, 7, 9, 10, và 11. Hãy cho biết mỗi giá trị
pKa ứng với phenol nào. Giải thích.

Hướng dẫn
Các hiệu ứng hút electron làm cho phenol có tính acid mạnh hơn, còn hiệu ứng
đẩy electron thì ngược lại. Phenol (chất thứ 4) có pKa 10. Chất duy nhất có tính
acid yếu hơn phenol phải là chất thứ ba với 3 nhóm methyl đẩy electron yếu. Một
nguyên tử chloride có hiệu ứng cảm ứng âm (hút electron), do vậy hợp chất cuối
cùng có pKa 9. Hai hợp chất còn lại có nhóm nitro hút electron mạnh. Hợp chất
đầu tiên có 2 nhóm NO2 là chất có pKa 4 (tính acid xấp xỉ acetic acid), còn chất
thứ hai, có 1 nhóm NO2, có pKa 7.

214 | Câu hỏi lí thuyết Hóa học hữu cơ OlympiaVN


Bài 13
Giải thích các quan sát sau:
a) Các amide bị proton hóa trên oxygen thay vì nitrogen.
b) Các ether là Lewis base tốt hơn ketone.
c) Tetramethyl guanidine (pKa ≈ 12) có tính base mạnh hơn nhiều so với
N,Ndimethylacetamide (pKa ≈ −0.5).

d) Boron trifluoride (BF3) là Lewis acid mạnh hơn nhiều so với trimethyl borate
[(CH3O)3B].
e) Piperidine là Lewis base mạnh hơn nhiều so với pyridine.

f) p-nitrophenol có pKa ≈ 8 trong khi đó phenol có pKa ≈ 10.

g) o-chloroaniline là base yếu hơn p-chloroaniline.

h) Sodium borohydride trong alcohol không khử được các imine hiệu quả. Tuy
nhiên, nếu thêm BF3 vào hỗn hợp thì quá trình khử diễn ra rất nhanh và hiệu
quả.

215 | Câu hỏi lí thuyết Hóa học hữu cơ OlympiaVN


i) Giải thích tại sao hydrogen sulfide (H2S) lại có tính acid mạnh hơn nước
(H2O)?
Hướng dẫn
a) Các amide bị proton hóa trên oxygen bởi tạo ra cation được bền hóa cộng
hưởng. Sự proton hóa trên nitrogen chỉ tạo ra cation định xứ điện tích.

b) Cặp electron chưa liên kết trên oxygen của ether nằm ở orbital lai hóa sp3,
trong khi đó của ketone là orbital sp2. Do đó cặp electron của ketone bị giữ
chặt hơn và khó nhường hơn, dẫn đến ketone là base yếu hơn.
c) Nguyên nhận là bởi acid liên hợp của tetramethylguanidine có sự bền hóa
cộng hưởng lớn hơn N,N-dimethyl amide (nhiều dạng hơn và các dạng tương
đương nhau).

d) Sự nhường electron vào boron bởi sự cộng hưởng đối với oxygen lớn hơn
nhiều đối với fluorine bởi fluorine có độ âm điện lớn hơn. Do đó, boron trong
trimethyl borate giàu electron hơn và là Lewis acid yếu hơn.

216 | Câu hỏi lí thuyết Hóa học hữu cơ OlympiaVN


e) Cặp electron chưa liên kết của piperidine thuộc orbital sp3 trong khi của
pyridine là orbital sp2. Đặc trưng s lớn hơn nhiều cho cặp electron trong
pyridine bị giữ chặt hơn và khó nhường cho H+ hơn.
f) Điện tích âm của base liên hợp của p-nitrophenol được giải tỏa cộng hưởng
vào nhóm nitro. Do đó base liên hợp bền hơn và p-nitrophenol sẽ có tính acid
cao hơn.

g) Hiệu ứng cảm ứng của chlorine ở vị trí ortho lớn hơn (do gần nhóm amino
hơn) của chlorine ở vị trí para. Hiệu ứng cảm ứng càng lớn thì càng làm giảm
mật độ electron trên nitrogen là khiến cho đồng phân ortho là base yếu hơn.
h) BF3 tạo phức với cặp electron chưa liên kết của imine và khiến nó thiếu hụt
electron hơn, tăng tính electrophile. Ở dạng phức hóa, nó là electrophile tốt
hơn nên phản ứng được với NaBH4.
i) Oxygen âm điện hơn lưu huỳnh, do đó hydroxide ion sẽ bền hơn hydrosulfide
anion. Như vậy lẽ ra nước phải có tính acid mạnh hơn. Tuy nhiên, do bán
kính lưu huỳnh lớn hơn oxygen nhiều nên liên kết hydrogen-lưu huỳnh yếu
hơn khoảng 40 kcal/mol so với liên kết hydrogen-oxygen. Do đó, cần ít năng
lượng để phá vỡ liên kết của hydrogen sulfide hơn so với nước. Yếu tố này
che mờ tác động của độ âm điện và thực tế thì hydrogen sulfide mới có tính
acid mạnh hơn.

217 | Câu hỏi lí thuyết Hóa học hữu cơ OlympiaVN


Bài 14
Xác định mối liên hệ giữa hai phân tử sau. Hãy thảo luận về cấu trúc anion có thể
tạo thành từ phản ứng deproton hoá mỗi chất.

Hướng dẫn
Chúng là đồng phân hỗ biến (tautomer): chỉ khác nhau về vị trí của một nguyên
tử hydrogen. Ở cấu trúc thứ nhất, hydrogen nằm trên nguyên tử nitrogen; còn ở
cấu trúc thứ hai thì trên nguyên tử oxygen. Cả hai cấu trúc đều có tính thơm,
nhưng cấu trúc thứ nhất có nhóm carbonyl bền hơn, còn cấu trúc thứ hai lại là
nhóm imine kém bền hơn. Phản ứng deproton hoá có thể xảy ra, tạo thành hai
anion khác nhau nhưng thực chất lại giống nhau bởi sự giải toả.

218 | Câu hỏi lí thuyết Hóa học hữu cơ OlympiaVN


Bài 15
Cho các chất sau đây và các giá trị pKa tương ứng. Hãy giải thích tại sao thêm
một nhóm NO2 tính axit tăng mạnh, nhưng khi thêm 2 nhóm, rồi 3 nhóm thì tính
axit lại tăng không nhiều?

Hướng dẫn
Anion triphenylmetan không thể có cấu trúc phẳng do sự đẩy nhau của các H
ortho nên chỉ có một vòng thực sự đồng phẳng với carbon trung tâm. Chính vì thế
việc thêm một nhóm NO2 sẽ khiến tính axit tăng mạnh do lúc này nhóm NO2 vừa
thể hiện -C và -I, nhưng khi thêm 2 nhóm, rồi 3 nhóm thì nhóm NO2 chỉ thể hiện
-I (do hai trong số ba vòng benzen không đồng phẳng) làm cho tính axit tăng
không nhiều.

219 | Câu hỏi lí thuyết Hóa học hữu cơ OlympiaVN


Bài 16
Tính acid của 1a và 2a xấp xỉ nhau nhưng 1b lại là acid mạnh hơn nhiều so với
2b. Hãy giải thích các quan sát này.

Hướng dẫn
Đây là cấu trúc cộng hưởng quan trọng đối với sự bền hóa phenoxide.

Khi R = H, sẽ rất dễ đạt được tương tác cộng hưởng này. Khi R = Me, các tương
tác không liên kết giữa R và nhóm chức nitro sẽ làm giảm tính bền của cấu trúc
cộng hưởng này, do đó làm giảm tính acid của 2b (cản trở không gian đối với sự
cộng hưởng).

Cản trở không gian đối với nhóm cyano nhỏ hơn so với NO2. Do đó, các nhóm
thế methyl như trong 2a không tác động nhiều đến pKa của nó.

220 | Câu hỏi lí thuyết Hóa học hữu cơ OlympiaVN


Bài 17
Một axit cacboxylic có thể tồn tại dưới hai cấu dạng E và Z khác nhau về góc
giữa hai mặt phẳng xung quanh liên kết C–O. Các tính toán lý thuyết gần đây cho
thấy rằng cấu dạng Z bền hơn 4.04 kcal/mol. Giả sử rằng pKa của axit fomic bằng
3.77, tính pKa của cấu dạng E của axit này và dựa vào tính toán ở trên, cho biết
cặp e không liên kết nào trong anion cacboxylat có tính bazơ cao hơn? Gần đúng
∆G = 1,4 pK kcal/mol.

Hướng dẫn
Ta có:

(1) + (2)  (3) do đó Ka(E) = K.Ka(Z)


Ta có: ∆G = 1,4 pK do đó K = 10∆G/1,4 = 769.
Ka(E) = K.Ka(Z) = 769.10-3,77 = 1,31.10-1  pKa(E) = 0,883
Do pKa(E) < pKa(Z) nên cặp e ở vị trí E có tính bazơ kém hơn cặp e chiếm vị trí Z

221 | Câu hỏi lí thuyết Hóa học hữu cơ OlympiaVN


Bài 18
Dưới đây là giá trị pKa của hai amino acid:
(a) cysteine: 1.8, 8.3, và 10.8
(b) arginine: 2.2, 9.0, và 13.2.
Gắn các giá trị pKa vào các nhóm chức tương ứng trong mỗi amino acid và vẽ
cấu trúc chiếm ưu thế của mỗi chất ở pH 1, 7, 10 và 14.

Hướng dẫn
Ở pH cao, cysteine tồn tại ở dạng dianion (cả thiol và carboxylic acid đều chuyển
thành anion). Nếu thêm acid, ở pH khoảng 10 (thực tế là 10.8) thì nhóm amine sẽ
bị proton hoá, sau đó là anion của thiol bị proton hoá ở khoảng pH 8 (thực tế là
8.3) và cuối cùng là anion của carboxylic acid sẽ bị proton hoá ở pH thấp (thấp
hơn MeCO2H do nhóm NH3+ có hiệu ứng hút electron làm tăng tính acid).

Arginine cũng tương tự:

222 | Câu hỏi lí thuyết Hóa học hữu cơ OlympiaVN


Bài 19
Đề xuất cách tách riêng hỗn hợp 3 chất này:

Hướng dẫn
Pyridine là một base yếu (pKa của pyridinium ion là khoảng 5.5) và có thể hoà
tan trong dung dịch acid. Naphthalene thì không có tính acid hoặc base và không
tan trong nước tại mọi giá trị pH. p-toluic acid là một acid yếu (pKa khoảng 4.5)
và có thể tan trong dung dịch base. Do vậy, có thể hoà tan hỗn hợp này vào một
dung môi hữu cơ không trộn lẫn được với nước (như ether Et2O hoặc
dichloromethane CH2Cl2) và chiết bởi dung dịch acid. Pyridine sẽ bị hoà tan ở
dạng cation. Sau đó chiết lớp hữu cơ còn lại với dung dịch base như NaHCO3, thì
sẽ loại được toluic acid ở dạng anion tan trong nước. Lúc này ta có 3 dung dịch.
Làm bay hơi dung dịch hữu cơ, thu được naphthalene dạng tinh thể. Acid hoá
dung dịch base của p-toluic acid thu được acid tự do tách ra dưới dạng kết tủa.
Thêm base vào dung dịch pyridine, rồi chiết pyridine bởi dung môi hữu cơ và
chưng cất pyrdine.

223 | Câu hỏi lí thuyết Hóa học hữu cơ OlympiaVN


Bài 20
1,8-Bis(dimetylamino)naphtalen (pKa = 12,8) là một bazơ hữu cơ rất mạnh. Thực
nghiệm cho thấy nó dễ dàng deproton hóa phenol, nhưng mặt khác nó không thể
deproton hóa axit mạnh hơn là trinitrometan. Hãy giải thích tính bazơ mạnh bất
thường của nó và cho biết lý do tại sao nó không thể deproton hóa được
trinitrometan.
Hướng dẫn
Tính bazơ mạnh bất thường của hợp chất này được giải thích do cation sinh ra
được ổn định nhờ liên kết hydro. Tuy nhiên do cấu trúc cồng kềnh nên nó không
thể deproton hóa được trinitrometan.

224 | Câu hỏi lí thuyết Hóa học hữu cơ OlympiaVN


Bài 21
Với mỗi nhóm ba hợp chất được cho dưới đây, hãy chỉ ra hợp chất nào có tính
base mạnh nhất và yếu nhất. Giải thích ngắn gọn.

Hướng dẫn

Các amide bị proton hóa trên oxygen (do các hiệu ứng cộng hưởng). Lone pair
của oxygen có tính base kém hơn lone pair của nitrogen, do đó amide là tiểu phân
có tính base kém nhất. Lone pair N trong ammonia thuộc orbital sp3, có tính base
mạnh hơn lone pair sp trong nitrile (nitrile bị proton hóa trên nguyên tử N do các
hiệu ứng cộng hưởng.)

Cấu trúc đầu tiên là của amine bậc ba. Cấu trúc ở giữa thoạt nhìn qua thì giống
các amide thông thường, nhưng lone pair N không được định hướng để có thể
nhường vào π*CO của carbonyl. Do đó, cấu trúc ở giữa thực tế là một keto-amine
(với nguyên tử nitrogen lai hóa sp3); hiệu ứng cảm ứng âm [hút electron] của
nhóm carboyl làm giảm tính base của nitrogen so với trong cấu trúc đầu tiên. Cấu

225 | Câu hỏi lí thuyết Hóa học hữu cơ OlympiaVN


trúc cuối cùng là của một amide thật sự, với sự proton hóa diễn ra trên oxygen
(lone pair có năng lượng thấp nhất), do đó có tính base kém nhất.

Cấu túc ở giữa có tính base mạnh nhất do không có các nhóm chức hút electron.
Trong cấu trúc đầu tiên, nhóm nitro làm giảm mật độ electron từ nhóm amine do
hiệu ứng cộng hưởng, làm cho cấu trúc này có tính base kém nhất. Trong cấu trúc
cuối, nhóm nitro chỉ làm giảm mật độ electron của amine bởi hiệu ứng cảm ứng
âm mà không có hiệu ứng cộng hưởng bởi 2 nhóm tert-butyl làm cho hệ π của
nhóm nitrogen phải vuông góc với hệ π thơm. Đây là một ví dụ về sự ức chế
không gian đối với hiệu ứng cộng hưởng.

226 | Câu hỏi lí thuyết Hóa học hữu cơ OlympiaVN


Bài 22
Đọc và trả lời các câu hỏi từ (105)-(131).
Khi đề cập đến acid và base, nhiều người sẽ nghĩ đến các hợp chất vô cơ, ví dụ
như hydrogen chloride - một acid mạnh, và sodium hydroxide - một base mạnh.
Mặt khác, cũng có những chất được phân loại hữu cơ nhưng cũng thể hiện tính
acid hoặc base - chúng thường được gọi là các acid và base hữu cơ. Nhiều phản
ứng có sự tham gia của các hợp chất hưu cơ này cũng được coi là phản ứng acid-
base. Trong bài này, chúng ta hãy tìm hiểu về acid và base trong hóa hữu cơ.
S. A. Arrhenius là người đầu tiên đề ra khái niệm acid-base. Ông nói một chất
giải phóng hydrogen ion (H+, proton - nhưng thực chất phải là oxonium, H3O+,
trong dung dịch nước) trong dung dịch nước là acid, còn chất giải phóng
hydroxide ion (OH-) là base. Ví dụ, khi hydrogen chloride và sodium hydroxide
tan trong nước thì xảy ra các quá trình:

Lần lượt, tạo thành H+ và OH-. Định nghĩa của Arrhenius chỉ giới hạn với các
phản ứng trong dung dịch nước và không thể áp dụng với các hệ phi-nước như
dung môi hữu cơ. Mặt khác, hydrogen chloride và ammonia phản ứng với nhau
trong benzene thì có ammonium chloride (NH4Cl) kết tủa. Đây rõ ràng là một
phản ứng acid-base. Do đó, J. N. Brønsted và T. M. Lowry đã mở rộng định nghĩa
của Arrhenius thành “acid có thể nhường H+ còn base có thể nhận H+.” Ví dụ như
trong phản ứng của hydrogen chloride và ammonia trong benzene, thì hydrogen
chloride giải phóng H+ và ammonia nhận H+. Khoảng áp dụng của định nghĩa này
rộng hơn và có thể bao trùm không chỉ phase lỏng mà còn cả phản ứng trong
phase khí và phase rắn. Trong định nghĩa của Brønsted và Lowry, H+ đóng vai
trò chủ động, nhưng G. N. Lewis đã mở rộng định nghĩa này với các phản ứng
thậm chí không có H+. Ông định nghĩa base là tác nhân nhường (donor) một cặp
electron, còn acid là tác nhân nhận (acceptor) cặp electron. Định nghĩa của Lewis
bao hàm cả định nghĩa của Brønsted và Lowry nhưng vượt quá những giới hạn.
Ví dụ, BH3 và BF3 là các Lewis acid điển hình - cách phân loại có thể không phù
hợp với những chất này nếu xét theo các định nghĩa cũ, bởi chúng trung hòa điện
và không tạo thành các cation. Hãy tìm hiểu về Lewis acid với ví dụ về BH3.
BH3 kết hợp với ammonia tạo thành sản phẩm cộng bền (cặp Lewis) - theo định
nghĩa của Lewis, đây là một phản ứng acid-base.

Lớp vỏ ngoài cùng của nguyên tử boron có 3 electron, và phân tử được tạo thành
bởi sự chia sẻ các electron này với 3 nguyên tử hydrogen. Do lớp vỏ ngoài cùng
của boron có thể chứa tối đa 8 electron nên nó có thể tiếp nhận thêm 1 cặp electron
nữa. Bằng cách tiếp nhận cặp electron chưa liên kết của nguyên tử nitrogen trong

227 | Câu hỏi lí thuyết Hóa học hữu cơ OlympiaVN


ammonia, cấu hình electron của nguyên tử boron được xem là giống như của khí
hiếm neon - một trong những trạng thái bền vững nhất.
Khi xét các phản ứng acid-base, thì “lực” [độ mạnh] của mỗi tác nhân là vấn đề
quan trọng. Một thước đo là mức độ điện li. Khi một chất điện li như acid hoặc
base tan trong nước, nó bị điện li tạo thành các ion. Acid mạnh như hydrogen
chloride gần như bị điện li hoàn toàn, còn acid yếu như acetic acid thì xảy ra theo
cân bằng sau:

Hằng số cân bằng của phản ứng này được kí hiệu là K. Trong dung dịch loãng,
đặt K·[H2O] là Ka (hằng số ion hóa của acid) và [H3O+] là [H+], và viết lại biểu
thức hằng số cân bằng:

Ka có thể xem là không đổi nếu nhiệt độ không đổi. Trong trường hợp acid yếu,
giá trị của hằng số ion hóa rất nhỏ. Ví dụ với acetic acid thì Ka = 1.7·10-5 mol L-
1
. Ngoài ra, giá trị hằng số ion hóa biến đổi nhiều bậc [theo số mũ], tùy thuộc vào
bản chất của chất điện li. Do đó, thay vì mô tả cách hằng số ion hóa trên một
khoảng rất rộng như vậy thì chúng ta thường sử dụng thang logarithm, với đại
lượng pKa = -log10Ka. Ví dụ đối với acetic acid thì pKa = 4.8.

228 | Câu hỏi lí thuyết Hóa học hữu cơ OlympiaVN


Như đã thấy rõ từ định nghĩa, giá trị pKa càng nhỏ thì lực acid càng mạnh. Bảng
1 cho biết giá trị pKa của một số carboxylic acid. So với acetic acid thì
chloroacetic acid và fluoroacetic acid là những acid mạnh hơn. Do các nguyên tử
chlorine và fluorine hút electron mạnh nên chúng có vai trò phân tán điện tích âm
của carboxylate anion (-COO-), dẫn đến tăng độ bền của anion và tăng khả năng
ion hóa. Hiệu ứng này có tác động rất lớn, thậm chí các acid hữu cơ này còn mạnh
hơn những acid vô cơ như HF (pKa = 3.2). Các dẫn xuất với nhóm nitro (-NO2),
cyano (-CN), acetyl (CH3CO-) và hydroxyl (-OH) cũng có lực acid mạnh hơn
acetic acid, đó là bởi tính chất hút electron. Các nhóm chức như vậy được gọi là
nhóm hút electron. Mặt khác, pKa của propionic acid (CH3CH2COOH) lớn hơn
một chút so với acetic acid, điều này dường như ngược với tác động của nhóm
thế hút electron như trên. Đó là do khi thay thế hydrogen bởi nhóm methyl thì
electron sẽ bị đẩy về điện tích âm, làm cho anion trở nên kém bền và quá trình
ion hóa khó xảy ra hơn. Các nhóm alkyl như methyl được gọi là nhóm đẩy
electron.
Bảng 1
Chú thích: カルボン酸 = carboxylic acid

Câu hỏi ア: Dựa vào các diễn giải ở trên và dữ kiện trong bảng 1, hãy chọn ra
một phát biểu đúng dưới đây. (105)
① Formic acid (HCOOH) mạnh hơn acetic acid.
② CH3CH2CH2COOH là acid mạnh hơn acetic acid.
③ CH3CH2CHClCOOH là acid yếu hơn acetic acid.
④ BrCH2COOH là acid mạnh hơn FCH2COOH.
Từ các thảo luận tới thời điểm hiện tại, chúng ta thấy rằng lực acid của carboxylic
acid bị ảnh hưởng nhiều bởi hiệu ứng hút electron của nhóm thế có trong cấu trúc.
Tiếp theo, hãy cùng xem các nhóm thế ảnh hưởng như thế nào đến hoạt tính và
độ chọn lọc trong các phản ứng acid-base có sự tham gia của hợp chất hữu cơ.
Theo định nghĩa của Lewis thì có rất nhiều phản ứng hóa học có thể được xem là
phản ứng acid-base, như phản ứng thế của benzene dưới đây. Khi benzene phản
ứng với một tác nhân electrophile [ái điện tử] - là chất dễ tiếp nhận electron - thì
phản ứng thế với hydrogen diễn ra tương đối dễ dàng. Phản ứng này được gọi là
phản ứng thế electrophile. Ví dụ, xét phản ứng nitro hóa benzene. Khi trộn nitric
acid đặc với sulfuric acid đặc thì nitronium ion (NO2+) được tạo thành và đóng

229 | Câu hỏi lí thuyết Hóa học hữu cơ OlympiaVN


vai trò là tác nhân electrophile. Nitronium ion nhận cặp electron từ liên kết π của
vòng benzene và tạo thành liên kết carbon-nitrogen trong hợp chất trung gian
a
carbocation I. Tiếp đó, H+ bị tách khỏi I, tạo thành nitrobenzene. Do đó, trong
phản ứng này, tác nhân electrophile (nitronium ion) là Lewis acid và benzene là
Lewis base.
Câu hỏi イ: Chọn ra hai phát biểu không đúng khi mô tả tính chất của benzene.
(106) (107)
① Nó là chất độc và tác nhân gây ung thư.
② Sodium chloride khó tan trong chất này.
③ Trong không khí, nó bị đốt cháy không hoàn toàn và tạo ra nhiều muội.
④ 6π electron được giải tỏa.
⑤ Tất cả các nguyên tử carbon và hydrogen nằm trên cùng mặt phẳng.
⑥ Hàm lượng hydrogen cao hơn các hydrocarbon no.
⑦ Tất cả các liên kết carbon-carbon có độ dài bằng nhau.
⑧ Nhiệt độ sôi thấp hơn toluene.
Câu hỏi ウ: Chọn ra 2 trong số các cấu trúc dưới đây không phải của carbocation
I (ở phần gạch chân a ở trên). (108) (109)

Câu hỏi エ: Chọn ra 3 phản ứng có thể tạo thành electrophile:

Chú thích: 紫外線 = ánh sáng tử ngoại (2); 鉄触媒 = xúc tác sắt (3); 濃硫酸 =
sulfuric acid đặc (4); ニッケル触媒 = xúc tác nickel (5)

230 | Câu hỏi lí thuyết Hóa học hữu cơ OlympiaVN


Trong benzene, nếu 1 nguyên tử hydrogen bị thay thế bởi 1 tác nhân electrophile
thì sản phẩm là benzene 1 lần thế (thế mono). Kí hiệu nhóm thế này là G. Nếu
phản ứng thế với tác nhân electrophile được thực hiện với dẫn xuất mono C6H5G
thì nhóm thế G đã tồn tại có 2 kiểu tác động với vòng benzene.
1) Nhóm thế G tác động đến hoạt tính của vòng benzene, một số nhóm làm tăng
tốc độ phản ứng [hoạt hóa], một số khác thì lại làm chậm [phản hoạt hóa].
- Các nhóm hoạt hóa: -NH2, -CH3, -OH, -OCH3, …
- Các nhóm phản hoạt hóa: -F, -Cl, -NO2, -COOH, …
2) Nhóm thế G xác định vị trí của nhóm thế tiếp theo (đây gọi là sự định hướng
của nhóm thế.)
Như đã mô tả ở trên, tác nhân electrophile được xem là Lewis acid và vòng
benzene là Lewis base. Vòng benzene sẽ nhường cặp electron. Khi mật độ
electron trong vòng benzene tăng lên hoạt tính tăng, và ngược lại, hoạt tính sẽ
giảm khi mật độ giảm. Do fluorine và chlorine có độ âm điện cao hơn hydrogen,
có thể hiểu rằng nó đóng vai trò nhóm giảm hoạt tính. Tuy nhiên, với trường hợp
oxygen, tuy có độ âm điện tương đối cao nhưng các nhóm thế có liên kết qua
oxygen lại được xếp vào nhóm làm tăng hoạt tính. Điều này không thể được giải
thích chỉ bởi bằng độ âm điện. Trong trường hợp này, mật độ electron tăng lên
bởi cặp electron của oxygen đã giải tỏa vào vòng benzene. Nguyên tắc này cũng
được áp dụng với NH2. Có thể thấy được điều này từ thực tế rằng ammonia đóng
vai trò như một base, nguyên tử nitrogen có khả năng nhường cặp electron chưa
liên kết, dẫn đến tăng khả năng phản ứng. Còn nguyên nhân tại sao nhóm nitro (-
NO2) thể hiện tính chất trái ngược hoàn toàn là bởi 2 nguyên tử oxygen với độ
âm điện lớn có liên kết với nguyên tử nitrogen, dẫn đến khả năng hút electron cực
mạnh và làm giảm khả năng phản ứng của vòng benzene.
Câu hỏi オ: Chọn ra 4 nhóm thế làm giảm khả năng phản ứng thế electrophile
của vòng benzene. (113) (114) (115) (116)

Tiếp theo, liên quan đến sự định hướng của nhóm thế G, hãy xét phản ứng của
phenol với tác nhân electrophile.

Cặp electron chưa liên kết của oxygen trong phenol có thể được giải tỏa khi tham
gia vào hệ electron π liên hợp của vòng benzene (như biểu diễn trong các cấu trúc
cộng hưởng trên). Sự giải tỏa này đóng vai trò then chốt trong việc làm bền hóa
hệ (“bền hóa cộng hưởng”). Khi nitronium ion phản ứng với phenol, nếu phản

231 | Câu hỏi lí thuyết Hóa học hữu cơ OlympiaVN


ứng thế diễn ra ở (117) và (118) của nhóm hydroxyl, cấu trúc cộng hưởng có thể
giải tỏa điện tích dương của trung gian và có đóng góp lớn vào sự bền hóa cấu
trúc này. Còn nếu phản ứng thế ở vị trí (119) thì không có hiệu ứng như vậy. Thực
tế, phản ứng này gần như chỉ xảy ra ở các vị trí (117) và (118). Sản phẩm mà tất
cả vị trí (117) và (118) được nitro hóa gọi là (120) và được sử dụng làm nguyên
liệu thô cho thuốc nổ. Ngoài ra, sodium phenoxide - được tạo thành khi xử lí
phenol với sodium hydroxide - có hoạt tính cao hơn phenol, bởi điện tích âm có
xu hướng được giải tỏa vào vòng benzene, và có thể phản ứng với cả electrophile
kém hoạt động như carbon dioxide ở vị trí (117). Sản phẩm sau khi xử lí với
sulfuric acid loãng được gọi là (121) và được dùng làm nguyên liệu cho thuốc
giảm đau, chống viêm.
Tiếp theo, chúng ta hãy xét đến sự phản hoạt hóa gây ra bởi
nhóm nitro. Hợp chất này (nitrobenzene) có các cấu trúc cộng
hưởng mà điện tích dương xuất hiện ở các vị trí (117) và (118)
nên rất khó để thực hiện phản ứng thế ở đây. Do đó phản ứng
ở vị trí (119) chiếm ưu thế hơn. Tiếp theo, hãy xét mối quan
hệ giữa hoạt tính của nhóm thế và sự định hướng vị trí. Dựa vào các thảo luận
vừa qua, có thể dự đoán rằng nhóm thế hoạt hóa định hướng vị trí (117), (118)
còn các nhóm phản hoạt hóa định hướng vị trí (119). Xu hướng này gần như luôn
đúng, tuy nhiên vẫn có những ngoại lệ, đó là fluorine và chlorine. Chúng đóng
vai trò các nhóm thế phản hoạt hóa, nhưng mặt khác, cũng có (Q122) cặp electron
chưa liên kết quanh nguyên tử gắn trực tiếp với vòng benzene, do đó, tác nhân
electrophile dễ phản ứng vào vị trí (117) và (118).
Câu hỏi カ: Xác định từ/cụm từ phù hợp với (117)-(119).
① orth ② meta ③ para
Câu hỏi キ: Chọn tên hợp chất phù hợp với (120), (121).
① Salicylic acid ② Phthalic acid
③ Picric acid ④ Benzoic acid
Câu hỏi ク: Chọn số phù hợp với (112).
Cũng như benzene, các alkane cũng có thể được xem là base. Từ quan điểm của
sự nhường và nhận cặp electron, phản ứng cộng với hydrogen halide (HX) có thể
được xem là phản ứng acid-base. Trong phản ứng thế của benzene thì các nhóm
thế có ảnh hưởng đến sự chọn lọc sản phẩm, ảnh hưởng tương tự cũng xảy ra với
phản ứng của alkene. Trong phần tiếp đây, hãy xét hiệu ứng điện tử của nhóm thế
đến độ chọn lọc của sản phẩm cộng. Trong phản ứng của 2-methylpropene và
hydrogen chloride, có thể dự doán 2 sản phẩm sẽ được tạo thành, nhưng thực tế
chỉ thu được A. Sự chọn lọc vùng như vậy cũng được quan sát thấy với các alkene
bất đối và được gọi là quy tắc Markovnikov (đặt theo tên nhà hóa học đầu tiên
phát hiện ra, V. V. Markovnikov). Nguyên nhân tại sao sự chọn lọc vùng như vậy

232 | Câu hỏi lí thuyết Hóa học hữu cơ OlympiaVN


xảy ra có thể được hiểu khi xét đến ảnh hưởng điện tử của nhóm alkyl và xét trung
gian bền hơn trong tiến trình phản ứng.
Câu hỏi ケ: Chọn tiểu phân trung gian phù hợp để tạo thành các sản phẩm A và
B trong phản ứng sau đây.

A: (123), B: (124)

Câu hỏi コ: Sự chọn lọc tương tự cũng được quan sát thấy trong phản ứng cộng
HBr. Xác định các sản phẩm của phản ứng (1)-(3). (Q125)

233 | Câu hỏi lí thuyết Hóa học hữu cơ OlympiaVN


Từ phản ứng trung hòa của hydrogen chloride với sodium hydroxide, tạo thành
sodium chloride trung tính và nước, có thể suy đoán phản ứng acid-base tạo thành
sản phẩm bền với hoạt tính kém. Tuy nhiên, với sự kết hợp của Lewis acid và
base, có những trường hợp mà chất tạo thành lại có hoạt tính hơn nhiều. Hãy xét
những ví dụ như vậy. Cặp Lewis ammonia borane (HN3-BH3) là chất rắn bền ở
nhiệt độ và áp suất thường, có hàm lượng hydrogen cao, và được nghiên cứu sử
dụng làm môi trường lưu trữ hydrogen cho pin nhiên liệu. Nếu thay thế hydrogen
của ammonia borane bởi một nhóm alkyl thì độ bền của cặp Lewis sẽ thay đổi.
Ví dụ, trong cặp Lewis của tripropylamine và triethylborane, mỗi nhóm alkyl
chiếm một vùng không gian đáng kể, do đó các tương tác đẩy không gian có thể
diễn ra (xem hình bên dưới) và độ bền sẽ giảm. Hiện tượng này được gọi là “sức
căng trước “ (front strain, hay F-strain).

Một trong những ví dụ kinh điển nhất là sự kết hợp của B(C6F5)3 và [(CH)3C]3P
được thực hiện bởi D. W. Stephan.

Phosphorus đóng vai trò như một Lewis base tương tự nitrogen bởi trong bảng
tuần hoàn nó nằm ngay dưới nitrogen và cũng có một cặp electron có thể nhường
như nitrogen. Do nhóm [(CH)3C]- là nhóm nhường electron nên tính base tăng
lên. Mặt khác, nhóm pentafluorophenyl của B(C6F5)3 là nhóm hút electron mạnh
và làm tặng lực acid. Do đó, hai chất này có xu hướng kết hợp tạo thành cặp
Lewis. Tuy nhiên, do lực đẩy không gian của các nhóm C6F5- với [(CH)3C]- là
rất lớn nên không thể tạo thành sản phẩm cộng như ammonia borane. Chính vì
vậy mà các hợp chất kiểu như vậy được gọi là “Frustrated Lewis pair” hay FLP
(“frustrated” có nghĩa là “thất bại, mất tác dụng”). FLP có khả năng phản ứng
mạnh, nó hoạt hóa các phân tử hydrogen trung hòa bền và phân hủy thành acid
(H+) và base (H-). H+ liên kết với phosphorus và H- nhường cặp electron cho
boron, tạo thành một cặp ion [hợp chất ion]. Cặp ion này bền và không phân hủy
giải phóng hydrogen, thậm chí là khi đun nóng tới 100 oC.

234 | Câu hỏi lí thuyết Hóa học hữu cơ OlympiaVN


Câu hỏi サ: FLP cũng phản ứng với nhiều chất. Hãy dự đoán sản phẩm chính
của phản ứng dưới đây. (Trong các công thức cấu tạo, t-Bu là kí hiệu của nhóm
[(CH3)3C-].
(126) Khi xảy ra phản ứng cộng:

(127) Khi liên kết C-H bị hoạt hóa và phân cắt:

(128) Khi xảy ra phản ứng cộng:

Stephan đã tiếp tục thiết kế các FLP có các tâm Lewis acid và Lewis base trong
cùng phân tử. FLP này có thể đóng vai trò xúc tác hydrogen hóa cho imine (xem
hình dưới).

Phản ứng hydrogen hóa xúc tác của imine đã được thực hiện sử dụng các kim loại
chuyển tiếp như platiunum. Phương pháp sử dụng FLP mới này đã thu hút nhiều
sự quan tâm, gọi là phản ứng xúc tác không cần kim loại (metal-free), và hiện nay
vẫn được nghiên cứu tích cực.

235 | Câu hỏi lí thuyết Hóa học hữu cơ OlympiaVN


Câu hỏi シ: Lựa chọn nhóm gồm tất cả các chất đều phù hợp để trở thành FLP
như trong phần gạch chân. (129)

236 | Câu hỏi lí thuyết Hóa học hữu cơ OlympiaVN


Hướng dẫn

Gợi ý cho một số câu hỏi


Câu hỏi カ:

Câu hỏi コ:

237 | Câu hỏi lí thuyết Hóa học hữu cơ OlympiaVN


Các cation tạo ra ban đầu được biểu diễn như hình trên. Các trung gian mang điện
tích dương trên nguyên tử carbon phân nhánh hơn dễ dược tạo thành. Điện tích
được giải tỏa bởi tính chất hút electron của các nhóm alkyl kế cận, do đó càng
nhiều nhánh thì càng bền.
Câu hỏi サ:

238 | Câu hỏi lí thuyết Hóa học hữu cơ OlympiaVN


Bài 23
1) Imidazole là một base, tạo thành acid liên hợp với pKa 7.05. Nguyên tử
nitrogen nào bị proton hóa trong phản ứng acid-base sau?

2) pKa của cyclopentane cao hơn nhiều pKa của imidazolidine, thể hiện rằng độ
âm điện của nitrogen có tác động đáng kể. Còn pKa của imidazole chỉ cao
hơn một chút so với pKa của cyclopentadiene. Tại sao cyclopentadiene lại có
tính acid cao hơn nhiều so với dự kiến?

3) Thường thì phản ứng tạo thành enol ngay lập tức sẽ bị hỗ biến thành dạng
keto, như trong chuyển hóa giữa cyclohexenol thành cyclohexanone. Tuy
nhiên, một phản ứng như vậy sẽ ưu tiên chuyển thành dạng enol của pyridine-
2-ol hơn là dạng keto của pyridine-2-one. Giải thích tại sao.

4) Ketone nào phân cực hơn: cyclopropanone hay cycloprop-2-enone? Giải


thích.

5) Các DNA base đều có tính thơm. Hãy vẽ các cấu trúc cộng hưởng để chỉ rõ
đặc tính thơm của chúng.

239 | Câu hỏi lí thuyết Hóa học hữu cơ OlympiaVN


Hướng dẫn
1) Để nguyên tử nitrogen thể hiện tính chất như một base, nó cần phải có một cặp
electron để tạo liên kết với nguyên tử hydrogen. Cặp electron này phải không
thuộc hệ thơm do việc tạo thành liên kết N-H sẽ phá vỡ tính thơm.

2) Các anion được tạo thành bằng cách tách proton của cyclopentadiene hoặc
imidazolidine có điện tích âm định xứ.

Cả imidazole và cyclopentadiene đều có tính acid mạnh hơn nhiều imidazolidine


hoặc cyclopentane do các base liên hợp của chúng có tính thơm. Tính chất này
làm bền hóa tốt các anion và khiến chúng rất dễ được tạo thành.

240 | Câu hỏi lí thuyết Hóa học hữu cơ OlympiaVN


Khác biệt duy nhất giữa hai anion này là trong imizadole đã tách proton thì điện
tích âm ở trên nguyên tử N âm điện hơn trong hai cấu trúc cộng hưởng. Giá trị
pKa gần nhau là bởi sự giải tỏa cộng hưởng điện tích có tác động bền hóa lớn hơn
độ âm điện của nguyên tử nitrogen.
3) Dạng enol pyridine-2-ol thuận lợi hơn bởi nó có tính thơm, còn dạng keto
pyridin-2-one thì không.

4) Cycloprop-2-enone sẽ phân cực hơn. Dạng cộng hưởng ion lưỡng cực của nó
có một vòng thơm. Đặc tính này tạo ra một hợp phần đóng góp to lớn cho cấu
trúc cộng hưởng tổng thể. Dạng tương ứng của cyclopropanone không có các yếu
tố bền hóa và có đóng góp rất nhỏ.

241 | Câu hỏi lí thuyết Hóa học hữu cơ OlympiaVN


Sự tách biệt điện tích sẽ làm tăng tính phân cực, làm cho cycloprop-2-enone là
phân tử phân cực hơn.
5) Adenine rõ ràng đã có vòng thơm 6 cạnh. Các cấu trúc cộng hưởng dưới đây
của ba base còn lại sẽ thể hiện đặc tính thơm của chúng.

242 | Câu hỏi lí thuyết Hóa học hữu cơ OlympiaVN


Bài 24
1. So sánh tính axit của những hợp chất sau đây, lần lượt trong môi trường axit
và môi trường bazơ: (CH3CH2)2NCH2CH2COOH, H2NCH2COOH,
O2NCH2CH2COOH.
2. Axit axetic có pKa = 4,8 trong nước và 10,3 trong etanol, trong khi đó ion
anilinium (C6H5NH3+) có pKa = 4,6 trong nước và 5,7 trong etanol. Giải thích
sự khác biệt lớn về pKa trong trường hợp của acid axetic.
3. Xiclopropyl phenyl xeton (pK = 28,2) lại có tính axit kém hơn axetophenon
(pK = 24,7).
4. Việc gắn thêm vòng benzen làm giảm pK của inden, fluoren so với
xiclopentadien.

5. Các ion kim loại như Zn2+, Ni2+ và Cu2+ làm tăng tốc độ enol hóa 2-
axetylpyridin trong bazơ. Giải thích.
6. So sánh tính axit của mỗi dãy sau
a) Axit bixyclo [1.1.1] pentan-1-cacboxylic và axit 2,2-đimetylpropanoic
b) CH3COOH, CH3CO3H, CH3SO3H (C)
Hướng dẫn
1. Trong môi trường axit, các nguyên tử nitơ của nhóm amin sẽ bị proton hoá
(mang điện tích dương), góp phần làm tăng tính axit của hợp chất tương ứng;
trong khi nguyên tử nitơ của nhóm nitro không bị proton hoá. Do đó tính acid
được xếp như sau: H2NCH2COOH > (CH3CH2)2NCH2CH2COOH >
O2NCH2CH2COOH.
Trong môi trường bazơ, sự xác định tính axit dựa trên những lý luận thông
thường của hiệu ứng cảm ứng nên tính axit được sắp xếp như sau:
O2NCH2CH2COOH > H2NCH2COOH > (CH3CH2)2NCH2CH2COOH.
2. Axit axetic trong nước tạo ion từ phân tử trung hòa, còn phân ly ion anilinium
không làm thay đổi ion :

CH3COOH + H2O CH3COO + H3O

C6H5NH3 + H2O C6H5NH2 + H3O

CH3COOH + C2H5OH CH3COO + C2H5OH2

243 | Câu hỏi lí thuyết Hóa học hữu cơ OlympiaVN


C6H5NH3 + C2H5OH C6H5NH2 + C2H5OH2

Ảnh hưởng dung môi trong acid axetic lớn hơn ion anilinium. Nước có hằng
số điện môi lớn hơn và solvat hóa anion tốt hơn nên khi chuyển từ nước sang
etanol, trạng thái cuối của phân ly acid axetic mất ổn định mạnh hơn do giảm
solvat hóa (solvat ion thay đổi mạnh hơn phân tử trung hòa khi thay đổi dung
môi).
3. Sự tạo thành enolate làm tăng mạnh sức căng của hệ xiclopropan.
4. Càng nhiều hệ thống vòng gắn vào tính bền của hệ thơm càng giảm. Có thể
thấy rõ điều này trong hệ thống benzen – naphtalen – antraxen.
5. Sự tạo phức với ion kim loại giúp tăng độ âm điện của nguyên tử oxy carbonyl
giúp tăng tính axit của Hα

6.

(Vòng cứng nhắc dễ sonvat hóa hơn)


CH3CO3H < CH3COOH < CH3SO3H (dựa vào sự giải tỏa của anion)

244 | Câu hỏi lí thuyết Hóa học hữu cơ OlympiaVN


Bài 25
Dự đoán nguyên tử bị a) proton hoá và b) deproton hoá khi các hợp chất sau được
xử lí với acid hoặc base phù hợp. Trong mỗi trường hợp, cho biết nên dùng acid
hoặc base nào và xác định cấu trúc sản phẩm.

Hướng dẫn
Amine đơn giản (piperidine) có thể dễ dàng bị proton hoá, thậm chí là bởi các
acid yếu. Mọi acid vô cơ (như HCl) hoặc các acid hữu cơ (ROOH) đều có thể
được sử dụng. Còn phản ứng deproton hoá sẽ loại proton N-H do nitrogen có độ
âm điện cao hơn carbon; tuy nhiên cũng cần phải dùng các base rất mạnh như
BuLi do pKa cũng chỉ khoảng 30-35.

Hợp chất thứ hai phức tạp hơn một chút. Tuy nhiên, phản ứng proton hoá vẫn dễ
dự đoán do chứa hợp phần amine bậc ba, dễ phản ứng với đa số acid. Amine bậc
ba không thể bị deproton hoá và trong trường hợp này thì phản ứng xảy ra với
hợp phần alcohol; cần dùng base mạnh như NaH.

Với chất thứ ba, có một nhóm OH (pKa khoảng 16) và một nhóm alkyne với tính
acid yếu (pKa khoảng 32). Nhóm có tính base không đơn thuần là amine mà là
dạng amidine giải toả. Phản ứng proton hoá diễn ra ở nguyên tử nitrogen phía trên
(imine) do điện tích dương sau đó được giải toả đều trên cả hai nguyên tử nitrogen.
Phản ứng proton hoá không diễn ra ở nguyên tử nitrogen khác.

245 | Câu hỏi lí thuyết Hóa học hữu cơ OlympiaVN


Proton đầu tiên bị loại bởi base là từ hợp phần alcohol và sẽ cần base mạnh như
NaH. Còn để tách proton khỏi alkyne sẽ cần base mạnh hơn như BuLi.

246 | Câu hỏi lí thuyết Hóa học hữu cơ OlympiaVN


Bài 26
1) Giải thích tại sao squaric acid bị ion hóa hoàn toàn (phân li cả hai proton)
trong nước.

2) Giải thích tại sao guanidine là một trong những base hữu cơ không mang điện
mạnh nhất từng được biết tới.

3) Giải thích tại sao tính acid của cyclopentadiene mạnh hơn nhiều so với
indene.

4) Theo các kết quả đo pKa có thể thấy rằng 1,1,1-bicyclopentane carboxylic
acid có lực acid mạnh gấp 10 lần 2,2-dimethylpropanoic acid, dù cả hai đều
có carbon bậc bốn gắn với nhóm carboxylic acid. Giải thích sự khác biệt pKa
này.

247 | Câu hỏi lí thuyết Hóa học hữu cơ OlympiaVN


Hướng dẫn
1) Bis anion của squaric acid được bền hóa cộng hưởng bởi bốn cấu trúc đóng
góp cộng hưởng tương đương và bởi một cấu trúc cộng hưởng thơm. Do đó,
việc tách cả hai proton rất dễ dàng. Thực tế, lực acid của squaric acid mạnh
gần bằng sulfuric acid loãng.

2) Sự proton hóa guanidine tạo thành một anion có bốn dạng cộng hưởng (ba
trong số đó tương đương nhau), dẫn đến sự bền hóa cộng hưởng rất mạnh.
Do đó, guanidine rất dễ được proton hóa và là một base rất tốt.

3) Anion cyclopentadiene được bền hóa bởi năm cấu trúc cộng hưởng tương
đương. Anion này có tính thơm bởi nó có hệ 6 electron π. Anion indenyl thì
được bền hóa bởi bảy cấu trúc cộng hưởng, tuy nhiên chúng không tương
đương và năm trong số các cấu trúc này phá vỡ vòng thơm benzene. Do đó,
dù điện tích âm được giải tỏa tốt hơn nhưng sự bền hóa cộng hưởng lại kém
hơn hệ cyclopentadiene. Do đó, khó tách proton khỏi indene hơn, dẫn đến nó
có tính acid yếu hơn.

248 | Câu hỏi lí thuyết Hóa học hữu cơ OlympiaVN


4)

249 | Câu hỏi lí thuyết Hóa học hữu cơ OlympiaVN


Bài 27
Dự đoán tiểu phân nào có thể tạo thành bởi sự kết hợp của các cặp tác nhân sau:

Hướng dẫn
Trong mỗi trường hợp, một trong các tác nhân sẽ lấy proton từ chất còn lại. Ở cặp
a) phenolate có thể lấy proton từ acetic acid, lý do là bởi acid acetic có tính acid
mạnh hơn phenol. Sự khác biệt là khoảng 5 đơn vị pH, do vậy hằng số cân bằng
là khoảng 105 và cân bằng sẽ chuyển dịch theo chiều thuận:

Ở cặp b) cũng xảy ra phản ứng tương tự nhưng lần này khác biệt pKa nhỏ hơn
nhiều và cân bằng lại ưu tiên chuyển dịch theo chiều nghịch:

Còn cặp c) thì khá là khác. Lần này tạo ra acid carboxylic mạnh hơn nhiều, và
cân bằng chủ yếu chuyển dịch theo chiều nghịch.

Bài 28
Những base nào bên dưới có thể được chọn để deproton hoá các phân tử sau?

250 | Câu hỏi lí thuyết Hóa học hữu cơ OlympiaVN


Hướng dẫn
Hãy ước lượng giá trị pKa của các proton có tính acid mạnh nhất trong mỗi phân
tử cần deproton hoá, đồng thời ước lượng cả pKa của các acid liên hợp của những
base đã đề xuất.

Các base có thể deproton hoá các hợp chất phía trên, bên trái chúng. Để deproton
hoá chất có tính acid yếu nhất, amine, bạn có thể chọn butyllithium. Để deproton
hoá alkyne (một phản ứng thường dùng để tạo liên kết C-C), có thể dùng BuLi
hoặc NaH. BuLi cần bảo quản trong khí quyển trơ, trong khi NaH tuy có phản
ứng với nước, nhưng có thể bảo quản ở dạng huyền phù trong dầu.
Alcohol có pKa gần với nước, do vậy hydroxide không phải là lựa chọn tốt cho
quá trình deproton hoá, và NaH thường được dùng trong trường hợp này. Tuy
nhiên, hydroxide có thể deproton hoá cả carboxylic acid (tạo thành muối
carboxylate) hoặc ammonium ion (tạo thành amine tự do). Bircarbonate cũng
thường được dùng để deproton hoá acid.

251 | Câu hỏi lí thuyết Hóa học hữu cơ OlympiaVN


Bài 29
So sánh momen lưỡng cực của từng cặp chất cho sau đây và giải thích.

a)
COOH vµ HOOC

b) O O NO2

NO2

c) O O
Br vµ
OH OH

Hướng dẫn
a) Dựa vào cấu trúc cộng hưởng của hệ vòng để giải thích:

Cấu trúc cộng hưởng A bền nhất do tạo được hệ 2 vòng thơm ngưng tụ. Cấu trúc
này có vectơ momen lưỡng cực hướng từ vòng 7 cạnh sang vòng 5 cạnh. Nhóm
COOH hút electron, thế vào vòng 5 cạnh sẽ làm tăng momen lưỡng cực, thế vào
vòng 7 cạnh sẽ làm giảm momen lưỡng cực của phân tử.

COOH > HOOC

b) Cả NO2 và O đều hút electron, vì vậy momen lưỡng cực của cả phân tử phụ
thuộc vào cách định hướng của chúng: cấu trúc endo làm giảm momen lưỡng cực,
trong khi cấu trúc exo làm tăng momen lưỡng cực.

252 | Câu hỏi lí thuyết Hóa học hữu cơ OlympiaVN


O NO2
O
<
NO2
exo
c) Hợp chất 2-hiđroxixiclohepta-2,4,6- trienon có một số dạng cấu trúc cộng
hưởng, trong đó cấu trúc B là bền nhất do tạo được hệ vòng thơm, với vectơ
momen lưỡng cực hướng từ vòng 7 cạnh đến O-.

O O O O
...
OH OH OH OH
B
Việc thế H bằng Br là nguyên tố hút electron sẽ làm giảm momen lưỡng cực, vì
vậy:

O O
Br <
OH OH
d) Thay O trong cấu trúc của 2-hiđroxixiclohepta-2,4,6-trienon bằng nitơ làm thay
đổi hướng của momen lưỡng cực (hướng từ nitơ vào vòng 7 cạnh). Thế H bằng
Br làm tăng momen lưỡng cực của hợp chất do Br hút electron, tạo ra momen
lưỡng cực cùng hướng với momen lưỡng cực của phân tử. Do đó:

NCH3 NCH3
Br >
NHCH3 NHCH3

253 | Câu hỏi lí thuyết Hóa học hữu cơ OlympiaVN


Bài 30
Hợp chất L1 và L2 đều là lactone. Giải thích tại sao việc tách α proton từ L1 khó
hơn từ L2 rất nhiều.

Hướng dẫn
Bởi dạng hình học cứng nhắc của hệ bicyclic nên việc tách một α proton từ L1
tạo thành một anion có cặp electron chưa liên kết vuông góc với liên kết π của
nhóm carbonyl. Hệ quả là cặp electron không thể xen phủ với liên kết π carbonyl
và không thể giải tỏa qua cộng cưởng – có nghĩa, đây đơn giản là một anion định
xứ. Còn việc tách một α proton từ L2 sẽ tạo thành một cặp electron trong orbital
p có thể xen phủ với liên kết π carbonyl và có thể xảy ra sự giải tỏa cộng hưởng.
Do đó, anion từ L2 được bền hóa cộng hưởng và dễ tạo thành hơn.

254 | Câu hỏi lí thuyết Hóa học hữu cơ OlympiaVN


Bài 14
Hãy xếp các hợp chất sau đây theo thứ tự tăng dần khả năng enol hóa: axeton,
etyl axetoaxetat, dietyl malonat, axetylaxeton, biacetyl và xiclopentan-1,2-dion.
Hướng dẫn
Thứ tự sẽ là: axeton < biaxetyl < dietyl malonat < etyl axetoaxetat <
axetylaxeton < xiclopentan-1,2-dion
Dạng enol trong cân bằng tautomer xeto - enol các hợp chất monocarbonyl chiếm
một lượng rất nhỏ, nên axeton sẽ là chất có hàm lượng enol bé nhất.
Đối với các hợp chất có hai nhóm C=O kế nhau (biaxetyl và xiclopentan-1,2-
dion) thì hai nhóm C=O trong biaxetyl sẽ nằm ở vị trí trans với nhau để giảm tối
đa tương tác lưỡng cực bất lợi nên khả năng enol hóa không cao. Tuy nhiên khi
ở trong vòng thì do yếu tố cấu dạng buộc hai nhóm C=O phải cis, dẫn đến việc
hai nhóm C=O phải tìm cách khác để tránh tương tác đẩy bất lợi. Kết quả là hợp
chất xiclopentan-1,2-dion chủ yếu nằm ở dạng enol

Các hợp chất 1,3-dicarbonyl có xu hướng tạo thành dạng enol tương đối đáng kể.
Khả năng enol hóa tăng dần phụ thuộc bản chất nhóm thế, trong đó nhóm este có
xu hướng enol hóa thấp nhất, và cao nhất là dixeton. Ở đây có thể cho rằng sự
cộng hưởng của nhóm OR este vào nhóm C=O làm giảm khả năng rút e của nhóm
C=O este, dẫn đến việc giảm tính axit của H alpha kế cận. Như vậy thứ tự sẽ là:

Từ đó dẫn ra kết quả như trên.

255 | Câu hỏi lí thuyết Hóa học hữu cơ OlympiaVN


Bài 31
1. Tính axit của cuban cao hơn xiclobutan, và còn hơn cả xiclopropan.

Cuban

2. Các ion kim loại như Zn2+, Ni2+ và Cu2+ làm tăng tốc độ enol hóa 2-
axetylpyridin trong bazơ. Giải thích.
3. Trong nước phenol có pK = 10, còn axit axetic có pK = 4. Tuy nhiên trong
pha khí tính axit của hai chất này lại gần như nhau.
Hướng dẫn
1. Việc chuyển cuban thành anion tương ứng làm tăng tính s của liên kết C – H.
Tính toán cho thấy anion cuban có 32% s, cao hơn xiclopropan 28% s. Như
vậy tính axit cao của cuban là kết quả của hai yếu tố, sự tăng tính s của anion
tạo thành và ảnh hưởng của sức căng vòng.
2. Sự tạo phức với ion kim loại giúp tăng độ âm điện của nguyên tử oxy carbonyl
giúp tăng tính axit của Hα

3. Cả hai anion phenoxide và axetat đều có sự giải tỏa điện tích mạnh. Tuy nhiên
trong nước thì ion axetat được solvat hóa hiệu quả hơn do nhóm Me kích
thước bé, và điện tích không được giải tỏa giãn rộng như trong PhO-. Trong
pha khí yếu tố solvat hóa bị mất đi nên tính axit của hai chất không khác nhau
nhiều.

256 | Câu hỏi lí thuyết Hóa học hữu cơ OlympiaVN


Bài 32
1. Hợp chất X (C10H19N3) không phân nhánh, có tính đối xứng cao có khả năng
phản ứng được với axit để giải phóng H2. Khả năng phản ứng đặc biệt này
của X bắt nguồn từ cấu trúc ba chiều của nó. Hãy lập luận xác định X và vẽ
cấu trúc không gian của nó để giải thích cho tính chất thú vị này.
2. Trong dung dịch loãng axit axetic thì (-)-menton (trans-2-isopropyl-5-
metylxiclohexanon) sẽ bị epime hóa ở 20oC để thu được một hỗn hợp cân bằng
gồm (-)-menton và (+)-isomenton. Hỗn hợp này có góc quay cực đo được là
-3o. Quá trình chuyển hóa (-)-menton thành (+)-isomenton đi qua sự tạo thành
một trung gian kém bền X vốn là đồng phân của cả menton và isomenton.
a) Tính % (-)-menton (góc quay cực riêng -30o) và (+)-isomenton (góc quay cực
riêng 92o) trong hỗn hợp cân bằng và từ đó xác định hằng số cân bằng K.
b) Vẽ cấu dạng bền nhất của X, (-)-menton và (+)-isomenton.

3. Hợp chất A (C12H4Cl4O2) có tâm đối xứng và ba mặt phẳng đối xứng. A bền
nhiệt, không làm mất màu dung dịch brom và kali permanganat.
a) Lập luận xác định cấu trúc A.
b) Dự đoán trạng thái tồn tại của A ở nhiệt độ thường và tính tan của nó.
c) Từ cấu tạo, hãy cho biết liệu A có bền với ánh sáng, kiềm và axit hay không.
4. Giải thích ảnh hưởng của nhóm thế đến tốc độ phản ứng sau:

R k (106.mol-1.s-1)
p – NMe2 338
p - OMe 102
H 73
p - Cl 78
m – NO2 79
p – NO2 88

5. Thực nghiệm cho thấy độ dài liên kết C-C trong hệ thống antraxen không
bằng nhau mà lần lượt có các giá trị 144pm, 140pm và 137pm. Hãy cho biết
giá trị độ dài ấy ứng với những liên kết nào trong số các nguyên tử cacbon
được xét dưới đây (có đánh số) và giải thích lựa chọn của mình:

257 | Câu hỏi lí thuyết Hóa học hữu cơ OlympiaVN


Hướng dẫn
1. Dựa trên các đặc điểm đã nêu thì hợp chất X sẽ có cấu trúc như sau:

Lúc này carbocation hình thành ở carbon trung tâm sẽ được bền hóa mạnh do
tương tác với ba cặp e của ba nguyên tử N kế cận khiến cho liên kết C – H trung
tâm rất dễ phân ly nên có thể tương tác được với axit để sinh ra H2.
2. Gọi x là phần mol của (-)-menton trong hỗn hợp. Như vậy phần mol của (+)-
isomenton là (1-x). Từ đó ta có -30x + 92(1 – x) = -3. Giải được x = 0,78 tức
(-)-menton chiếm 78% trong hỗn hợp cân bằng, còn (+)-isomenton là 22%.
Hằng số cân bằng K = 0,22 / 0,78 = 0,28
Sự đồng phân hóa (-)-menton thành (+)-isomenton qua trung gian X được
biểu thị như sau:

Menton X Isomenton
Cấu dạng bền nhất của chúng được biểu diễn như sau:

3. a) A là hợp chất thơm vì không làm mất màu dung dịch brom và dung dịch
KMnO4. A có độ không no là 9. A thơm, bền nhiệt nên dựa trên yếu tố đối
xứng có thể cho rằng A gồm hai vòng benzen nối với nhau bằng hai cầu ete
ở vị trí ortho. Vậy công thức của A là:

258 | Câu hỏi lí thuyết Hóa học hữu cơ OlympiaVN


b. A có phân tử khối lớn, có nhiều liên kết phân cực nên nó là chất rắn. Dựa trên
cấu trúc thì có thể suy ra A kém tan trong nước, tan tốt trong dung môi hữu
cơ.
c. A tương đối bền khi chiếu sáng bởi hệ thơm không có liên kết nào dễ bị đứt
khi chiếu sáng. A có liên kết C sp2 – Cl nên bền vững với kiềm. A cũng bền
với axit do cặp e trên oxy đã bị liên hợp với hệ vòng nên khó phản ứng trong
axit.
4. Phản ứng được khảo sát là Diels – Alder ‘ngược’. Phản ứng DA thường yêu
cầu anken mang nhóm rút và dien mang nhóm đẩy. Tuy nhiên dien được đề
cập đến trong phản ứng là tetraphenylxiclopentadienon lại mang nhóm rút
nên để đẩy nhanh tốc độ phản ứng thì hợp phần anken phải mang nhóm đẩy.
Từ đó dẫn ra kết quả như đã khảo sát

5. Antraxen có các cộng hưởng sau:

Các liên kết C2 – C7, C2 – C3 và C5 – C6 đều có độ dài 144pm do trong bốn


cộng hưởng đưa ra thì có đến ba cấu trúc các liên kết này đều là liên kết đơn. Liên
kết C1 – C2, C3 – C4 có độ dài 140pm do có hai cộng hưởng liên kết đôi và hai
cộng hưởng liên kết đơn. Còn liên kết C4 – C5 và C6 – C7 có độ dài 137pm do
có ba cộng hưởng nó mang liên kết đôi và một cộng hưởng liên kết đơn.

259 | Câu hỏi lí thuyết Hóa học hữu cơ OlympiaVN


Tương tác orbital biên (FMO)
Bài 1
a) Vẽ giản đồ năng lượng biểu diễn các orbital phân tử của các hợp chất sau.
Xác định LUMO của mỗi chất.

b) Dựa vào đóng góp của mỗi nguyên tử với LUMO, dự đoán tâm phản ứng với
nucleophile.
c) Dựa vào đóng góp của mỗi nguyên tử với LUMO, dự đoán tâm phản ứng với
electrophile.
Hướng dẫn
a) Trong cả hai trường hợp, LUMO đều là orbital π*.

b) Cả hai LUMO đều có năng lượng gần với năng lượng orbital p của carbon. Do
đó, các nucleophile (phân tử nhường electron) nhiều khả năng sẽ phản ứng ở
carbon.
c) Cả hai HOMO đều là các orbital  đã điền đủ electron (bão hòa). Chúng có
năng lượng lần lượt gần với N hoặc S. Do đó, các electrophile sẽ phản ứng ở
nguyên tử N hoặc S.

260 | Câu hỏi lí thuyết Hóa học hữu cơ OlympiaVN


Bài 2
a) Xét hai benzylic bromide 1 và 2. Chất nền nào có hoạt tính mạnh hơn trong
phản ứng SN1? Giải thích.

b) Xét hai Lewis acid 1 và 2 (hình dưới). Chất nào là Lewis acid mạnh hơn?

c) Carbonium ion 2 bền hơn nhiều so với carbonium ion 1. Hãy giải thích kết
quả này dựa vào những hiểu biết về các khái niệm liên kết.

d) Vẽ cấu dạng ưu đãi của 1,2-difluoroethane và giải thích cho câu trả lời của
bạn.
Hướng dẫn
a) Chất 1. Bởi nhóm thế -CO2Me làm carbocation trung gian từ 2 bị kém bền
hóa hơn.
b) Chất 2. Fluorine có liên kết π back mạnh hơn chlorine. Liên kết “π-π back”
mạnh hơn làm giảm tính Lewis acid.

c) Liên kết C-C ngắn hơn liên kết C-Si nên trong carbonium ion 2 sẽ có sự xen
phủ của σC-Si bền hóa và p tốt hơn.

261 | Câu hỏi lí thuyết Hóa học hữu cơ OlympiaVN


d) Hiệu ứng siêu liên hợp của σC-H vào σ*C-F được cực đại hóa ở cấu dạng xen
kẽ kề, hay cấu dạng gauche (gọi là hiệu ứng gauche).

262 | Câu hỏi lí thuyết Hóa học hữu cơ OlympiaVN


Bài 3
Khoanh tròn hợp chất có nguyên tử H có tính acid mạnh hơn và giải thích lựa
chọn của bạn.

Hướng dẫn
- Ketone bên trái có một liên kết C-H vuông góc với nhóm carbonyl (được định
hướng với π*CO). Proton này có thể bị tách ra, tạo thành một enolate.
- Ketone bên phải không có bất kì liên kết C-H nào định hướng được với π*CO
và do đó không thể enol hóa và không dễ để tách proton.

263 | Câu hỏi lí thuyết Hóa học hữu cơ OlympiaVN


Bài 4
1) Giải thích tại sao độ chuyển dịch hóa học 13C NMR của nguyên tử carbonyl
carbon trong butylrolactone lại xuất hiện ở trường thấp hơn (178 ppm) so với
hợp chất không vòng tương tự là ethyl propionate (174 ppm).

2) Mỗi dẫn xuất benzene dưới đây có độ chuyển dịch hóa học của proton thơm
ở vị trí 2 (ortho) khác nhau. Hãy vận dụng những hiểu biết của bạn về cấu
trúc cộng hưởng để trả lời các câu hỏi sau:

a) Hợp chất nào có ortho proton ít bị chắn nhất?


b) Hợp chất nào có ortho proton bị chắn mạnh nhất?
Hướng dẫn
1) Ester ở cấu dạng (Z) có tương tác siêu liên hợp quan
trọng, làm tăng mật độ electron ở carbonyl carbon. Mật
độ electron tăng cường này được thể hiện ở một cộng
hưởng 13C trường cao. Trong lactone, tương tác siêu
liên hợp này không được phép.
2)
a) Hợp chất 2. Nguyên nhân: Nhóm -NO2 sẽ làm giảm [rút] mật độ electron ở
các vị trí 2, 4, 6 trong vòng thơm. Nhóm -CHO cũng có hiệu ứng hút electron
nhưng bởi các lí do liên quan đến độ âm điện nên khả năng hút của nó kém
hơn.

b) Hợp chất 1. Nguyên nhân: Nhóm -NH2 làm tăng mật độ electron ở các vị trí
2, 4, 6 trong vòng thơm. Nhóm -OH trong 4 cũng có hiệu ứng nhường electron

264 | Câu hỏi lí thuyết Hóa học hữu cơ OlympiaVN


nhưng bởi các lí do liên quan đến độ âm điện nên khả năng đẩy của nó kém
hơn -NH2.

265 | Câu hỏi lí thuyết Hóa học hữu cơ OlympiaVN


Bài 5
a) Tại sao thioester lại có ái lực với nucleophile mạnh hơn ester thông thường?

b) Khi xử lí thioacetic acid với base tạo thành các thioacetate ion – một “chất
tương đồng chứa lưu huỳnh” của carboxylate. Cũng như carboxylate,
thioacetate phản ứng với alkyl halide tạo thành “ester” tương ứng. Về mặt
nguyên tắc, có thể có 2 sản phẩm bởi sự alkyl hóa có thể diễn ra trên oxygen
hoặc lưu huỳnh. Thường thì sự alkyl hóa chỉ quan sát thấy ở vị trí lưu huỳnh.
Dựa vào luận điểm thuyết FMO, hãy giải thích tại sao chỉ có sự alkyl hóa
thioacetate trên lưu huỳnh thay vì oxygen?

Hướng dẫn
a) Cặp lone pair của S thuộc orbital 3p, trong khi đó lone pair của O là trên orbital
2p. Sự xen phủ giữa π*CO với orbital 2p (O) tốt hơn (do có mức năng lượng gần
nhau hơn) là với orbital 3p (S).
b) Sự alkyl hóa diễn ra trên lưu huỳnh bởi lone pair của S có năng lượng cao hơn
các lone pair của O. Cụ thể, các lone pair liên quan của O và S đều là các sp2 lone
pair (các lone pair khác bị “trói” bởi cộng hưởng). Các lone pair S có năng lượng
cao hơn bởi 2 lí do:
- Oxygen có độ âm điện cao hơn lưu huỳnh, làm hạ năng lượng của lone
pair O so với lone pair tương ứng của S.
- Các lone pair O thuộc các orbital 2sp2, trong khi đó các lone pair của lưu
huỳnh thuộc các orbital 3sp2. Nhìn chung, các orbital 3sp2 có năng lượng
cao hơn orbital 2sp2.

266 | Câu hỏi lí thuyết Hóa học hữu cơ OlympiaVN


Bài 6
Các yếu tố ảnh hưởng đến xu hướng biến đổi tính acid của các carboxylic acid
(phương trình 1, X = O) cũng có tác động tương tự đến các acid yếu nền amide
(phương trình 1, X = NR).

Dựa vào hiểu biết của bạn về các hiệu ứng cảm ứng và sự tạo liên kết, hãy giải
thích tại sao chất 1 lại có pKa nhỏ hơn 4 đơn vị so với chất 2.

Hướng dẫn
Chú ý đến sự định hướng của các cặp electron không liên kết được xét đến dưới
đây trong hai urethan:

Trong cấu dạng này, cặp electron chưa liên kết của
oxygen được định hướng để xen phủ với orbital
sigma C-O phản liên kết, tương tác này làm giảm
lưỡng cực C=O và do đó giảm tính acid.

267 | Câu hỏi lí thuyết Hóa học hữu cơ OlympiaVN


Bài 7
Trong mỗi trường hợp dưới đây, cân bằng được thiết lập giữa hai đồng phân cấu
dạng hoặc đồng phân hình học để tạo thành đồng phân dường như kém bền hơn
nếu xét trên các quan điểm về hiệu ứng không gian hoặc lưỡng cực. Dựa vào hiểu
biết của bạn về thuyết orbital phân tử biên (FMO), hãy xác định tương tác
HOMO-LUMO (siêu liên hợp) có ảnh hưởng nhất, làm bền hóa đồng phân chính
trong mỗi cân bằng.
a)

b)

c)

Hướng dẫn
a) Tương tác chính là orbital chứa cặp electron chưa liên kết của nitrogen (N
lone pair) với orbital phản liên kết C-Cl.

b) O lone pair với orbital phản liên kết C-O.

268 | Câu hỏi lí thuyết Hóa học hữu cơ OlympiaVN


c) N lone pair với orbital phản liên kết C-N.

269 | Câu hỏi lí thuyết Hóa học hữu cơ OlympiaVN


Bài 8
a) Đề xuất cơ chế cho phản ứng thủy phân anomeric acetate A dưới đây:

b) Sử dụng các quan điểm về orbital phân tử, giải thích sự khác biệt về tốc độ
biểu kiến trong quá trình thủy phân hai anomer A và B.

Hướng dẫn
a)

270 | Câu hỏi lí thuyết Hóa học hữu cơ OlympiaVN


b) Sự nhường electron của một cặp electron chưa liên kết được định hướng vào
một orbital phản liên kết dẫn đến sự suy yếu [giảm độ bền] của liên kết C-O ngoài
vòng. Trong trường hợp α-anomer A, liên kết σC-OAc bị suy yếu bởi tương tác nO
→ σ*C-O. Do đó, sự phân cắt nhóm acetate diễn ra nhanh hơn trong anomer A hơn
trong B (không có tương tác giữa nO và σ*C-O do cản trở về mặt hình học.)

271 | Câu hỏi lí thuyết Hóa học hữu cơ OlympiaVN


Bài 9
a) Phản ứng ester chéo hóa dưới đây không thuận lợi. Giải thích dựa vào cấu
trúc phân tử, các số liệu định lượng (tra cứu từ internet) và lập luận về mặt cơ
chế.

b) Lactone A dưới đây có tính electrophile mạnh hơn ester không vòng tương
ứng B của nó. Giải thích hiện tượng này dựa theo quan điểm thuyết orbital
phân tử biên (FMO).

c) Sự deproton hóa của thioacetal A, sau đó cho carbanion tạo thành bắt
deuterium từ nước nặng thì phản ứng diễn ra với độ chọn lọc cao ở vị trí biên
(equatorial). Giải thích kết quả này theo luận điểm củ thuyết orbital phân tử
biên (FMO).

d) Phản ứng epoxid hóa chất B có độ chọn lọc cao. Khi xử lí với m-CPBA, chỉ
có một trong các alkene bị chuyển thành epoxide, như dưới đây. Giải thích
kết quả này theo thuyết orbital phân tử biên (FMO).

272 | Câu hỏi lí thuyết Hóa học hữu cơ OlympiaVN


Hướng dẫn
a) Trung gian tứ diện của phản ứng này là:
Phenol có pKa 10, trong khi đó cyclohexanol có pKa xấp xỉ
16. Do đó, nhóm rời đi bền nhất trong trung gian tứ diện này
là phenoxide anion (do được bền hóa cộng hưởng), do đó,
trung gian tứ diện này chỉ đơn giản là bị “sụp đổ”, tạo trở lại
các chất đầu - và phản ứng thất bại.
b)
Cặp electron chưa liên kết sp2 của Cặp electron chưa liên kết sp2 của
oxygen không thể nhường electron oxygen có thể nhường electron vào
vào σ*CO. Do đó, lactone có tính σ*CO. Điều này làm giảm tính
electrophile mạnh hơn. electrophile của carbonyl

Chú ý rằng không xét đến việc cặp electron chưa liên kết p của oxygen nhường
electron vào π*CO do sự bền hóa này tồn tại trong cả hai phân tử.
c) Equatorial anion được bền hóa bởi hai tương tác nhường-nhận (donor-
acceptor). Ngược lại, không có các tương tác bền hóa như vậy với axial anion.
Do đó, sự deproton hóa ban đầu sẽ diễn ra ưu tiên ở vị trí biên (equatorial), có
tính acid cao hơn, sau đó deuterium hóa equatorial carbanion.

273 | Câu hỏi lí thuyết Hóa học hữu cơ OlympiaVN


d) Một trong các nhóm chức alkene trong B có
định hướng tốt để nhường electron vào orbital
phản liên kết σ*O-Cl. Kết quả là, alkene này có
tính nucleophile kém hơn và do đó hoạt tính
trong phản ứng epoxid hóa kém hơn. Dẫn dến,
phản ứng epoxide hóa chọn lọc ở nhóm alkene
đối diện.

274 | Câu hỏi lí thuyết Hóa học hữu cơ OlympiaVN


Hóa lập thể
Bài 1
Dự đoán các hợp chất sau có thể tồn tại ở dạng đồng phân lập thể nào?

a)
b) (CH3)2C=C=C=C(CH3)2
c) (C2H5)(CH3)C=C=C(CH3)(C2H5)
d) PhCH(OH)-C≡C-CH(OH)CH3

e)

f)

g)

h)

i)

j)

k)

l)

275 | Câu hỏi lí thuyết Hóa học hữu cơ OlympiaVN


Hướng dẫn
a) Nếu một phân tử có n tâm chiral thì số đòng phân lập thể tối đa mà nó có thể
có là 2n. Phân tử này có 2 tâm chiral, do đó số đồng phân lập thể tối đa là 22. Vẽ
4 đồng phân có thể có thì thấy các đồng phân (RS)- và (SR)- tương đương nhau
– đây là dạng meso và có một phẳng gương bên trong. Các đồng phân (RR)- và
(SS)- là các ảnh gương không thể chồng khít và là các đối quang.

b) Phân tử này có dạng hình học tương tự một alkene đơn giản. Nếu các nhóm
thế không đối xứng thì nó có thể có các đồng phân hình học. Tuy nhiên, tất cả các
nhóm thế đều là methyl nên chỉ có duy nhất một cấu trúc:

c) Các orbital π của allene này được định hướng theo cách mà các nhóm thế ở
mỗi đầu của phân tử vuông góc với nhau.

Dạng hình học này làm xuất hiện tính không trùng vật-ảnh. Các allene như vậy
tồn tại ở 2 dạng đối quang:

d) Phân tử này có 2 tâm chiral nên có 22 đồng phân lập thể.

276 | Câu hỏi lí thuyết Hóa học hữu cơ OlympiaVN


Chú ý rằng liên kết ba carbon-carbon được xét ưu tiên hơn nhóm phenyl (theo
quy tắc Cahn-Ingold-Prelog.
e) Hợp chất spiro có dạng hình học tương tự allene và tồn tại hai dạng đối quang:

f) Phân tử này có 4 tâm chiral, nên số đồng phân lập thể tối đa có thể có là 24 =
16. Xem xét kĩ 16 khả năng (bên dưới) thì thấy có 6 cặp có cấu trúc giống nhau.
Chúng tương ứng với 2 hợp chất meso và 4 cấu trúc quang hoạt trong đó sự nghịch
đảo cấu hình của 2 tâm chiral với các nhóm thế giống nhau tạo thành cấu trúc
tương đương. Vậy có tổng cộng 10 đồng phân quang học:

g) Sự có mặt của 2 nhóm amide phẳng trong vòng 6 cạnh dẫn đến cấu trúc vòng
gần như phẳng. Có 2 tâm chiral, cho thấy số đồng phân lập thể tối đa có thể có là
22 = 4. Xét mỗi khả năng (bên dưới) thì thấy các đồng phân lập thể (RS)- và (SR)-
giống nhau (có một tâm đối xứng trong phân tử). Các đồng phân (RR)- và (SS)-
là các ảnh gương không thể chồng khít và là các đối quang:

277 | Câu hỏi lí thuyết Hóa học hữu cơ OlympiaVN


h) Trong phân tử này có 4 tâm chiral, cho thấy số đồng phân lập thể tối đa có thể
có là 24 = 16. Tuy nhiên, dạng hình học của phân tử này bị ràng buộc bởi hệ vòng.
Số đồng phân lập thể có thể có giảm một nửa, do một nguyên tử carbon đầu cầu
riêng biệt không thể nghịch đảo (vì nguyên nhân hình học). Có 8 đồng phân lập
thể, tương ứng với 4 cặp đối quang:

i) Các biphenyl có nhóm thế có dạng hình học trong đó 2 vòng phenyl vuông góc
với nhau. Khi hai vòng này đều có nhóm thế, phân tử có thể có tính quang hoạt.
Tuy nhiên, trong trường hợp trên thì phân tử trùng vật-ảnh với ảnh gương của nó
và chỉ có một cấu trúc tồn tại:

j) Hai nhóm methyl không thể nằm trong cùng một mặt phẳng. Do đó, phân tử
này có cấu trúc kiểu xoắn ốc, dẫn đến sự tồn tại của hai dạng đối quang:

278 | Câu hỏi lí thuyết Hóa học hữu cơ OlympiaVN


k) Phân tử này có 2 tâm chiral, do đó có thể có tối đa 22 = 4 đồng phân lập thể
(xem bên dưới). Tuy nhiên, do phân tử có 1 tâm đối xứng nên các đồng phân
(RS)- và (SR)- tương đương nhau; do đó có 3 đồng phân lập thể.

Cần lưu ý rằng các cấu trúc này không phẳng và mỗi cấu trúc ở trên biểu diễn một
hỗn hợp cân bằng nhanh của các cấu dạng, ví dụ:

l) Hệ decalin có các đồng phân lập thể bởi có các đường giao vòng cis và trans.
Trong trường hợp này, hai vòng không tương đường, nên có 4 đồng phân lập thể.

279 | Câu hỏi lí thuyết Hóa học hữu cơ OlympiaVN


Bài 2
Chỉ ra các tâm quang hoạt trong mỗi phân tử sau và xác định cấu hình R/S của
chúng.

280 | Câu hỏi lí thuyết Hóa học hữu cơ OlympiaVN


Hướng dẫn

281 | Câu hỏi lí thuyết Hóa học hữu cơ OlympiaVN


Bài 3
1) Xác định cấu hình tuyệt đối của các hợp chất sau:

2) Xác định mối liên hệ giữa các cặp chất sau:

3) Kháng sinh linezolid là đối quang S tinh khiết. Hãy vẽ cấu trúc phân tử với
cấu hình này.

282 | Câu hỏi lí thuyết Hóa học hữu cơ OlympiaVN


4) Gọi tên chính xác của hợp chất sau:

5) Xác định cấu trúc của các hợp chất sau:


a) l-erythro-2-amino-3-hydroxybutanoic acid (ở dạng công thức chiếu
Newman nhìn dọc qua trục liên kết C2-C3)
b) d-glyceraldehyde (công thức chiếu hình giá cưa [sawhorse])
c) (Z)-4-bromo-3-(methoxymethyl)but-2-enoyl chloride
d) u-3-bromopentan-2-ol (công thức chiếu zigzag)
e) (R,R)-tartaric acid (2,3-dihydroxybutanedioic acid) (công thức chiếu
Fischer)
f) (S)-(1-2H1)ethanol.
6) Điền các cụm từ mô tả lập thể endo, exo, syn hoặc anti vào tên gọi hệ thống:
7-.....-ethyl-5-.....-isopropyl-6-.....-methyl-7-.....-propylbicyclo [2.2.1]hept-2-
ene của hợp chất dưới đây. Giải thích tại sao tên gọi này không thể mô tả
hoàn toàn hợp chất này.

7) Xác định cấu hình các tâm thủ tính trong thuốc chống sốt rét cinchonine có
cấu trúc như dưới đây:

283 | Câu hỏi lí thuyết Hóa học hữu cơ OlympiaVN


284 | Câu hỏi lí thuyết Hóa học hữu cơ OlympiaVN
Hướng dẫn
1)

2)
a) Hai hợp chất là đối quang của nhau:

b) Hai hợp chất là đối quang của nhau:

285 | Câu hỏi lí thuyết Hóa học hữu cơ OlympiaVN


c) Hai hợp chất này là đồng phân dia:

d) Hai hợp chất phối trí khác nhau về vị trí các nguyên tử bromine và các phối tử
carbonyl, do đó chúng là đồng phân dia.
3)

4)
a) (Z)-1,2-dibromo-1-chloro-2-iodoethene
b) (Ra)-1,3-dichloroallene [(R)-1,3-Dichloroallene cũng là một phương án
nhưng nó không được khuyến nghị bởi bỏ qua chỉ mục a là mô tả lập thể của
cấu hình của một trục thủ tính.]
c) (4S)-4-chloro-2-methyloctane
d) (3S,4R)-4-hydroxy-3-methylpentanoic acid.

286 | Câu hỏi lí thuyết Hóa học hữu cơ OlympiaVN


5)

287 | Câu hỏi lí thuyết Hóa học hữu cơ OlympiaVN


6) 7-syn-Ethyl-5-endo-isopropyl-6-exo-methyl-7-anti-propylbicyclo[2.2.1] -
hept-2-ene)
Vì không thể dẫn ra cấu hình tuyệt đối từ các mô tả lập thể endo, exo, syn và anti
nên tên gọi này có thể sử dụng cho cả các đối quang tinh khiết lẫn các racemate.

7)

288 | Câu hỏi lí thuyết Hóa học hữu cơ OlympiaVN


Bài 4
1) Xác định cấu hình tuyệt đối của các hợp chất biểu diễn bởi các cấu trúc sau:

2) Latanoprost là một tiền dược được sử dụng trong điều trị bệnh tăng nhãn áp.
Đồng phân 15S của acid này chỉ có hoạt tính bằng khoảng 10% so với đồng
phân 15R. Vẽ cấu trúc của epimer hoạt tính hơn và xác định cấu hình của các
tâm thủ tính còn lại.

3) Vẽ cấu trúc của biotin (vitamin H), biết tên hệ thống của nó là 5-
[(3aS,4S,6aR)-2-oxohexahydrothieno[3,4-d]-imidazol-4-yl]pentanoic acid
và cấu trúc phẳng được cho dưới đây.

4) Vẽ cấu trúc của cis-1-[(R)-sec-butyl]-2-methylcyclohexane ở cấu dạng ghế


có năng lượng thấp nhất. Hợp chất này có bao nhiêu đồng phân?
5) Vẽ công thức chiếu Newman dọc theo các trục liên kết C1-C2 và C5-C4 của
2,2-dichloro-1,1-difluoro-4,4-dimethylcyclohexane. Giả sử rằng hợp chất có
cấu dạng ghế.
6) Có những tâm thủ tính nào trong 1-(bromomethyl)-4-[chloro (methoxy)-
methylidene]cyclohexanol? Xác định cấu hình tuyệt đối của đồng phân dưới
đây.

289 | Câu hỏi lí thuyết Hóa học hữu cơ OlympiaVN


7) Có những tâm thủ tính nào trong dẫn xuất adamantane dưới đây. Xác định
cấu hình của nó.

8) Hợp chất dưới đây có cấu hình threo hay erythro?

290 | Câu hỏi lí thuyết Hóa học hữu cơ OlympiaVN


Hướng dẫn
1) a) Trong phosphane này, cặp electron chưa liên kết trên phosphorus có độ hơn
cấp thấp nhất. Do đó hợp chất có cấu hình S:

b) Hợp chất này có cấu hình S.

c) Hợp chất này có cấu hình S.

2)

3) Thứ tự độ hơn cấp ở cả hai vị trí vòng ngưng tụ là N > C(S,...,H) > C(N,C,H)
> H và ở vị trí 4 là S > C(N,C,H) > C(C,H,H) > H.

291 | Câu hỏi lí thuyết Hóa học hữu cơ OlympiaVN


4) Do kí hiệu mô tả lập thể cis chỉ chỉ ra cấu hình tương đối của hai tâm thủ tính
trong vòng, nên sẽ có hai đồng phân của cis-1- [(R)-sec-butyl]-2-
methylcycohexane. Do phía sec-butyl của hai đồng phân phải có cấu hình R nên
hai hợp chất này sẽ là đồng phân dia. Mỗi đồng phân dia có thể tồn tại ở hai cấu
dạng ghế. Cấu dạng thuận lợi về mặt năng lượng hơn sẽ có nhóm sec-butyl chiếm
vị trí biên (equatorial).

5)

6) Hợp chất này có một trục thủ tính qua liên kết đôi và carbon 1. Để xác định
cấu hình tuyệt đối, hãy quan sát dọc theo trục này, ví dụ từ C1, các nhóm hydroxyl
và bromomethyl được sắp xếp theo chiều dọc. Các nhóm thế ở nhóm methylidene
trên C4 được định hướng theo chiều ngang. Độ hơn cấp của bốn nhóm này có thể
được xác định là cấu hình Sa. Do đó, hợp chất này là (Sa)-1-(bromomethyl)-4-
[chloro(methoxy)methylidene] cyclohexanol.

7)

292 | Câu hỏi lí thuyết Hóa học hữu cơ OlympiaVN


8)

293 | Câu hỏi lí thuyết Hóa học hữu cơ OlympiaVN


Bài 5
1) Các cấu trúc sau đây là của 3-fluoro-2-phenyl-2- pentanol. Xác định mối quan
hệ lập thể của mỗi cấu trúc với (2R,3R)-3-fluoro-2-phenyl-2-pentanol:

2)
a) Vẽ bốn đồng phân lập thể của 4-methyl-2-hexanol và nêu rõ mối quan hệ
của chúng với nhau.
b) Vẽ tất cả các đồng phân lập thể của 3-bromo-4-methylhexane. Gán kí
hiệu R, S cho mỗi tâm quang hoạt và nêu rõ mối quan hệ giữa các đồng
phân với nhau.
c) Sử dụng công thức chiếu Fischer, vẽ tất cả các đồng phân lập thể của 2-
fluoro-3-methyl-1,4-pentanediol và nêu rõ mối quan hệ giữa các đồng
phân với nhau.
d) Vẽ tất cả các đồng phân lập thể của 1,4-diphenyl-1,4-dibromobutane.
Gán kí hiệu R, S cho mỗi tâm quang hoạt và nêu rõ mối quan hệ giữa các
đồng phân với nhau.
3) Xác định mối liên hệ giữa các cặp chất sau:

294 | Câu hỏi lí thuyết Hóa học hữu cơ OlympiaVN


295 | Câu hỏi lí thuyết Hóa học hữu cơ OlympiaVN
296 | Câu hỏi lí thuyết Hóa học hữu cơ OlympiaVN
Hướng dẫn
1) (a) 2R, 3R (b) đồng phân dia 2S, 3R (c) 2R, 3R (d) đối quang 2S, 3S (e) đồng
phân dia 2R, 3S.
2)

1,4 là đối quang; 2,3 là đối quang; các trường hợp khác là đồng phân dia với nhau.

1,4 là đối quang; 2,3 là đối quang; các trường hợp khác là đồng phân dia với nhau.

297 | Câu hỏi lí thuyết Hóa học hữu cơ OlympiaVN


1,4 ; 2,7; 3,6; 4,5 là đối quang; các trường hợp khác là đồng phân dia với nhau.

3) (a) Cả hai đều là đồng phân cấu dạng 1S, 2R (b) Syn-anti, đồng phân dia (c)
Đối quang (d) Đối quang (e) Giống nhau (f) Đồng phân dia (g) Đồng phân dia
(h) Đối quang (i) Đối quang (j) Đồng phân cấu dạng

298 | Câu hỏi lí thuyết Hóa học hữu cơ OlympiaVN


Bài 6
Vẽ công thức cấu tạo và cấu dạng bền nhất có chú thích lập thể của đồng phân
quang học (4aS, 5R, 8aR) của chất C:

Hướng dẫn

299 | Câu hỏi lí thuyết Hóa học hữu cơ OlympiaVN


Bài 7
Cặp đồng phân dia nào sau đây là epimer và cặp epimer nào có thể dễ dàng chuyển
hóa qua lại?

Hướng dẫn
a) Hai phân tử này chỉ khác nhau ở 1 tâm chiral (*) và do đó chúng là các epimer.
2 epimer này có thể chuyển hóa qua lại dễ dàng qua dạng enol của aldehyde:

b) Hai phân tử này khác nhau ở 2 tâm chiral (*), do đó không phải là các epimer.
c) Hai phân tử này chỉ khác nhau ở 1 tâm chiral (*), do đó chúng là các epimer.
Tâm chiral thứ hai (#) dễ chuyển hóa qua lại qua dạng enol (xem bên dưới); tuy
nhiên, điều này không làm biến đổi tính thủ tính (chirality) của tâm chiral chính
(*), do đó 2 epimer không dễ chuyển hóa qua lại. Sự epimer chuyển đồng phân
dia thứ nhất (RS) thành đối quang (RR) của đồng phân dia thứ hai.

300 | Câu hỏi lí thuyết Hóa học hữu cơ OlympiaVN


301 | Câu hỏi lí thuyết Hóa học hữu cơ OlympiaVN
Bài 8
Vẽ công thức chiếu Newman của các cấu trúc sau (được biểu diễn ở dạng công
thức chiếu Fischer). Hợp chất nào không quang hoạt?

Hướng dẫn
2 cấu trúc sau không phải là công thức chiếu Fischer “chuẩn”, bởi quy ước của
công thức chiếu này là mạch carbon phải nằm trên trục dọc.
Các công thức chiếu trên biểu diễn các cấu trúc sau:

Hãy nhìn qua liên kết carbon-carbon trung tâm để thu được công thức chiếu
Newman:

Cấu trúc (II) có mặt phẳng gương bên trong, do đó không quang hoạt:

302 | Câu hỏi lí thuyết Hóa học hữu cơ OlympiaVN


Các cấu trúc I và III là đối quang.

303 | Câu hỏi lí thuyết Hóa học hữu cơ OlympiaVN


Bài 9
Chuyển các cấu trúc sau thành công thức chiếu Fischer:

Hướng dẫn

304 | Câu hỏi lí thuyết Hóa học hữu cơ OlympiaVN


305 | Câu hỏi lí thuyết Hóa học hữu cơ OlympiaVN
306 | Câu hỏi lí thuyết Hóa học hữu cơ OlympiaVN
Bài 10
Chuyển các công thức chiều Fischer sau thành công thức zigzag:

Hướng dẫn

307 | Câu hỏi lí thuyết Hóa học hữu cơ OlympiaVN


308 | Câu hỏi lí thuyết Hóa học hữu cơ OlympiaVN
Bài 11
Cấu trúc của axit cordyceic (hình dưới) [α]D = +40,3o được cho là không chính
xác. Khẳng định này đúng hay sai?

Hướng dẫn
Phân tử có mặt phẳng đối xứng đi qua C1 và C4 nên không thể quang hoạt được,
trái ngược với đề bài là đo được giá trị góc quay cực (chứng tỏ axit cordyceic
buộc phải quang hoạt). Chính vì thế cấu trúc đề nghị là sai, không khớp với thực
nghiệm.

309 | Câu hỏi lí thuyết Hóa học hữu cơ OlympiaVN


Bài 12
Gán cấu hình tuyệt đối của mỗi tâm quang hoạt trong các phân tử sau đây:

Hướng dẫn

310 | Câu hỏi lí thuyết Hóa học hữu cơ OlympiaVN


311 | Câu hỏi lí thuyết Hóa học hữu cơ OlympiaVN
312 | Câu hỏi lí thuyết Hóa học hữu cơ OlympiaVN
Bài 13
Một mẫu trans-cyclooctene được
phát hiện thấy có khả năng làm
quay ánh sáng phân cực, trong
khi đó mẫu trans-cyclodecene
thì không. Giải thích quan sát
này.
Hướng dẫn
Sử dụng các đường nét đậm và nét đứt để
biểu diễn tốt hơn cấu trúc ba chiều của các
chất trên thì thấy cả trans-cyclooctene và
trans-cyclodecene đều có tính quang hoạt -
không trùng với ảnh gương của chính chúng.
Mỗi cặp chất và ảnh gương là các đối quang.
Sự khác nhau giữa các đối quang này, như đối
với trường hợp trans-cyclodecene là phần
còn lại của vòng nằm trên phía nào của liên
kết đôi (xem hình bên).
Trans-cyclodecene không làm quay mặt
phẳng ánh sáng phân cực bởi hai đối quang
tồn tại đẳng mol trong một cân bằng. Thậm
chí, một mẫu tinh khiết của một đối quang
của trans-cyclodecene cũng sẽ bị racemic hóa (nghĩa là nó sẽ chuyển hóa thành
các lượng bằng nhau của các đối quang). Có thể xảy ra sự racemic hóa bởi trong
vòng trans-cyclodecene có đủ carbon để một phía của liên kết đôi dễ dàng lật sang
bên kia bằng cách xoay quanh các liên kết đơn, như biểu diễn dưới đây.

Tuy nhiên, sự quay này chỉ có thể với các dòng lớn hơn cyclodecene, còn vòng
trong trans-cyclooctene quá nhỏ để chuyển phía này của liên kết đôi sang phía
kia. Sự quay bị cản trở nghĩa là có thể phân lập riêng các mẫu đối quang tinh khiết
trans-cyclooctene và chúng có thể làm quay mặt phẳng ánh sáng phân cực. Ngược
lại, không thể phân lập các mẫu đối quang tinh khiết của trans-cyclodecene do sự
quay quanh các liên kết đôi sẽ dẫn đến sự racemic hóa nhanh chóng. Do đó, mẫu
trans-cyclodecene sẽ không làm quay mặt phẳng ánh sáng phân cực.

313 | Câu hỏi lí thuyết Hóa học hữu cơ OlympiaVN


Hoạt tính của hợp chất hữu cơ
Bài 1
Hãy gán các điện tích hình thức (khác 0) cho các nguyên tử trong những phân tử
sau:

Hướng dẫn

314 | Câu hỏi lí thuyết Hóa học hữu cơ OlympiaVN


Bài 2
Xác định số oxid hóa của các nguyên tử carbon được chỉ rõ trong mỗi hợp chất
sau:

315 | Câu hỏi lí thuyết Hóa học hữu cơ OlympiaVN


Hướng dẫn

316 | Câu hỏi lí thuyết Hóa học hữu cơ OlympiaVN


Bài 3
Sử dụng các cấu trúc cộng hưởng để dự đoán tâm phản ứng nào trên các phân tử
sau sẽ phản ứng với tiểu phân nhường (donor) electron. Gợi ý: Hãy chú ý vào các
nguyên tử không thỏa mãn bát tử.

Hướng dẫn
a) Một electron donor sẽ phản ứng ở orbital trống của carbon được thấy ở dạng
cộng hưởng thứ hai của nó.

b)

c)

317 | Câu hỏi lí thuyết Hóa học hữu cơ OlympiaVN


d) Các dạng cộng hưởng của phân tử này cho thấy rằng có thể có đến ba tâm phản
ứng, tất cả chúng đều là các nguyên tử carbon với một orbital trống. Do chúng
đều là carbon bậc hai, nên khả năng phản ứng với donor electron gần như bằng
nhau.

e)

f)

318 | Câu hỏi lí thuyết Hóa học hữu cơ OlympiaVN


Bài 4
Sử dụng các cấu trúc cộng hưởng để dự đoán tâm phản ứng nào trên các phân tử
sau sẽ phản ứng với tiểu phân nhận (acceptor) electron. Gợi ý: Hãy chú ý vào các
cặp electron chưa liên kết.

Hướng dẫn

319 | Câu hỏi lí thuyết Hóa học hữu cơ OlympiaVN


320 | Câu hỏi lí thuyết Hóa học hữu cơ OlympiaVN
Bài 5
Với các phản ứng sau, hãy: (1) viết phương trình được cân bằng; (2) xác định chất
đầu đã bị oxid hóa, khử hoặc không biến đổi; (3) nếu đã diễn ra quá trình oxid
hóa-khử, hãy chỉ ra các sản phẩm bị oxid hóa và khử; (4) chỉ rõ tác nhân đóng vai
trò chất oxid hóa hay chất khử; (5) cho biết tên gọi nhóm chức trong chất phản
ứng trước và sau khi bị biến đổi (nếu có).

321 | Câu hỏi lí thuyết Hóa học hữu cơ OlympiaVN


322 | Câu hỏi lí thuyết Hóa học hữu cơ OlympiaVN
Hướng dẫn

323 | Câu hỏi lí thuyết Hóa học hữu cơ OlympiaVN


324 | Câu hỏi lí thuyết Hóa học hữu cơ OlympiaVN
Bài 6
Với các phản ứng sau, hãy biểu diễn cấu trúc hoàn chỉnh của các chất phản ứng
và sản phẩm của giai đoạn dưới đây, chỉ rõ liên kết nào được tạo thành hoặc phá
vỡ, xác định các hợp phần nhường (donor) electron và nhận (acceptor). Sử dụng
kí hiệu mũi tên cong để chỉ sự chuyển dời electron.

325 | Câu hỏi lí thuyết Hóa học hữu cơ OlympiaVN


Hướng dẫn

326 | Câu hỏi lí thuyết Hóa học hữu cơ OlympiaVN


327 | Câu hỏi lí thuyết Hóa học hữu cơ OlympiaVN
Bài 7
Bổ sung thêm các mũi tên cơ chế để chỉ rõ chiều chuyển dịch electron trong mỗi
chuyển hóa sau:

328 | Câu hỏi lí thuyết Hóa học hữu cơ OlympiaVN


Hướng dẫn

329 | Câu hỏi lí thuyết Hóa học hữu cơ OlympiaVN


330 | Câu hỏi lí thuyết Hóa học hữu cơ OlympiaVN
Bài 8
Sử dụng các mũi tên cong, hãy biểu diễn cách các hợp chất lưỡng cực 1,3 này
được tạo thành như thế nào từ các chất đầu được chỉ định sẵn:

331 | Câu hỏi lí thuyết Hóa học hữu cơ OlympiaVN


Hướng dẫn

332 | Câu hỏi lí thuyết Hóa học hữu cơ OlympiaVN


Bài 9
Sắp xếp các dẫn xuất carboxylic acid sau theo thứ tự tính nucleophile giảm dần.
Giải thích ngắn gọn.

Hướng dẫn
Hoạt tính nucleophile có liên hệ với tính acid
của acid liên hợp của nhóm rời đi. Thang hoạt
tính cũng phản ánh khả năng của nhóm rời đi
chứa dị tố với việc nhường electron vào π*CO
của nhóm carbonyl.

333 | Câu hỏi lí thuyết Hóa học hữu cơ OlympiaVN


Bài 10
Sử dụng thuyết orbital phân tử, hãy lí giải về hoạt tính của các hợp chất tương
đồng với carbonyl sau đây đối với sự tấn công nucleophile.

Hướng dẫn
Hoạt tính của mỗi chất tương đồng carbonyl phụ thuộc vào mức năng lượng của
LUMO của nó. Năng lượng LUMO càng thấp, hoạt tính càng mạnh. Các mức
năng lượng LUMO tương đối được biểu diễn dưới đây:

π*CO ở mức năng lượng thấp hơn π*CN bởi O có độ âm điện lớn hơn N. Ngoài ra,
do có nguyên tử điện tích dương, có độ âm điện cao hơn (khả năng hút giữ
electron mạnh hơn) nên π*CN+ và π*CO+ thấp hơn π*CN và π*CO.

334 | Câu hỏi lí thuyết Hóa học hữu cơ OlympiaVN


Bài 11
Bạn đã biết rằng các nhóm alkyl làm tăng tính nucleophile của các liên kết đôi
trong phản ứng với các electrophile.
a) Tương tác orbital chính nào gây ra sự tăng tính nucleophile của olefin bởi các
nhóm thế methyl.
b) Vẽ cấu trúc cộng hưởng siêu liên hợp để mô tả tương tác
được nói đến ở ý a.
c) Tại sao tương tác được chỉ ra ở trên gây ra sự tăng tính
nucleophile của C=C? Vẽ giản đồ năng lượng orbital để
trả lời.
Hướng dẫn
a) Tương tác HOMO-LUMO chính:

b)

c) Bởi σ C-H giải tỏa vào orbital π phản liên kết. Năng lượng liên kết π tăng lên
do sự đẩy electron-electron.

335 | Câu hỏi lí thuyết Hóa học hữu cơ OlympiaVN


Bài 12
Trong mỗi phản ứng dưới đây, các tương tác HOMO-LUMO có tác động đến cả
dạng hình học của trạng thái chuyển tiếp và tốc độ phản ứng. Hãy xác định tương
tác HOMO-LUMO chủ đạo trong giai đoạn tốc định của mỗi chuyển hóa sau.

336 | Câu hỏi lí thuyết Hóa học hữu cơ OlympiaVN


Hướng dẫn

337 | Câu hỏi lí thuyết Hóa học hữu cơ OlympiaVN


Bài 13
1) Mỗi tiểu phân sau đây đều có tính ái điện tử (electrophile). Hãy xác định
nguyên tử có tính electrophile (tâm electrophile) và vẽ cơ chế phản ứng của
mỗi tiểu phân với một tác nhân nucleophile dạng tổng quát Nu-.

2) Mỗi electrophile sau có thể phản ứng với một nucleophile ở tại (ít nhất) một
nguyên tử. Xác định các nguyên tử đó và trình bày cơ chế của mỗi phản ứng.

Hướng dẫn
1) Chúng ta có 2 cation, 2 hợp chất carbonyl và 2 hợp chất chỉ có các liên kết σ.
Một trong các cation có 3 liên kết với nguyên tử carbon mang điện tích dương,
do đó đây là tâm electron bởi nó có 1 orbital trống. Nucleophile sẽ tấn công vào
vị trí này.

Cation còn lại có nguyên tử oxygen hóa trị ba, không thể là tâm electrophile. Thay
vào đó, tác nhân nucleophile cần phải tấn công vào proton. Một số nucleophile
có thể tấn công vào nguyên tử carbon liên kết với nguyên tử oxygen có tính
cationic.

Tác nhân nucleophile sẽ tấn công vào 2 hợp chất carbonyl ở vị trí nhóm carbonyl.
Thường thì các liên kết π dễ bị bẻ gãy hơn các liên kết σ và điện tích âm chuyển

338 | Câu hỏi lí thuyết Hóa học hữu cơ OlympiaVN


đến nguyên tử oxygen âm điện. Trong thực tế, các anion này không phải là sản
phẩm cuối của phản ứng, tùy vào từng trường hợp mà chúng sẽ có hướng chuyển
hóa khác nhau. Ví dụ như anion đầu tiên sẽ lấy 1 proton để tạo thành alcohol, còn
anion thứ hai có thể phân hủy tạo thành carboxylate anion bền.

Các electrophile còn lại chỉ có liên kết σ và một trong số chúng phải bị bẻ gãy.
Chlorine có tính đối xứng, do đó đầu nào bị tấn công cũng như nhau. Với MeSCl
thì sẽ có nhiều phương án tấn công hơn, nhưng trong trường hợp này thì tính bền
của chloride ion tạo thành quyết định và sự tấn công diễn ra ở nguyên tử lưu
huỳnh.

2) Trong mỗi trường hợp, một trong các tâm electrophile là proton có tính acid.
Ngoài ra con có liên kết π có tính electron (C=N+ hoặc C=O). Với chất đầu tiên,
chúng ta có hai phản ứng riêng biệt:

Trong trường hợp bạn chú ý đến


nguyên tử nitrogen mang điện tích
dương thì cũng cần nhớ rằng hướng
phản ứng này (tấn công trực tiếp vào
N) tuyệt đối không thể xảy ra, bởi sản
phẩm sẽ có tới 5 liên kết gắn vào
nitrogen!

339 | Câu hỏi lí thuyết Hóa học hữu cơ OlympiaVN


Với hợp chất thứ hai, có 3 hướng phản ứng có thể xảy ra. Ngoài proton của
carboxylic acid và liên kết C=O có tính electrophile thì còn có cả nguyên tử
phosphorus mang điện tích dương.

340 | Câu hỏi lí thuyết Hóa học hữu cơ OlympiaVN


Bài 14
Mỗi tiểu phân sau đây đều có tính ái nhân (nucleophile). Hãy xác định nguyên tử
có tính nucleophile (tâm nucleophile) và vẽ cơ chế phản ứng của mỗi tiểu phân
với một tác nhân electrophile dạng tổng quát E+.

Hướng dẫn
Trong các tiểu phân trên, có 3 anion và 2 trong số chúng (alkyne và sulfur anion)
có các electron chưa liên kết. Chúng ta sẽ bắt đầu những mũi tên chuyển dịch từ
các điện tích âm và chúng là các vị trí sẽ liên kết với electrophile tạo thành sản
phẩm.

Anion thứ ba là aluminium anion. Trong trường hợp này, điện tích âm không thể
hiện có một cặp electron trên Al, mà tất cả các electron đều thuộc và các liên kết
Al-H và mũi tên chuyển dịch sẽ phải bắt đầu từ một trong số chúng. Tâm
nucleophile trong trường hợp này là một nguyên tử hydrogen.

Các nuclephile còn lại đều có cặp electron chưa liên kết. Phân tử chứa nitrogen
là hydrazine, cả 2 nguyên tử nitrogen đều giống nhau. Trong trường hợp này, tiểu
phân tạo thành mang điện dương và sẽ mất 1 proton để trở nên bền hơn.

Hợp chất phosphorus có 4 nguyên tử có cặp electron chưa liên kết: 1P và 3O. Các
cặp electron trên oxygen thuộc về những orbital ở mức năng lượng thấp hơn trên
phosphoruss (P có độ âm điện nhỏ hơn O), do đó cặp electron trên P sẽ phản ứng.
Tiểu phân tạo thành mang điện tích dương, nhưng trong trường hợp này nó không
tách proton.

341 | Câu hỏi lí thuyết Hóa học hữu cơ OlympiaVN


342 | Câu hỏi lí thuyết Hóa học hữu cơ OlympiaVN
Bài 15
Sắp xếp các chất sau đây theo trình tự giảm dần hoạt tính với tác nhân electrophile
(ví dụ H+): CH2=CHN+(CH3)3, CH2=CH2,CH2=CHCH3, CH2=CHOCH3,
CH2=CHBr, CH2=CHNO2.
Hướng dẫn
Trật tự được dự đoán là:

Các alkene phản ứng với các electrophile như H+ tạo thành carbocation, có thể
phản ứng tiếp với một nucleophile (X-) tạo thành sản phẩm cộng:

Một phản ứng điển hình như vậy có giản đồ năng lượng như sau, với sự tạo thành
carbocation là giai đoạn tốc định:

Nếu trạng thái chuyển tiếp của giai đoạn thứ nhất của phản ứng có thể được mô
hình hóa theo carbocation trung gian thì có thể giả định rằng mọi yếu tố làm giảm
năng lượng của carbocation, liên quan đến các chất đầu, sẽ làm giảm năng lượng
của trạng thái chuyển tiếp và do đó là cả năng lượng hoạt hóa của phản ứng (ΔE)4.
Nhìn chung, các alkene với nhóm thế đẩy electron (như CH2=CHCH3,
CH2=CHOCH3) sẽ có hoạt tính cao hơn các alkene với nhóm thế hút electron
(như CH2=CHN+(CH3)3, CH2=CHNO2).

4Đây là phát biểu của định đề Hammond, giả định rằng cấu trúc trạng thái chuyển tiếp gần
với của carbocation trung gian hơn là alkene ban đầu. Do đó, mọi yếu tố làm bền hóa
carbocation hơn các chất đầu sẽ làm tăng tốc độ phản ứng.

343 | Câu hỏi lí thuyết Hóa học hữu cơ OlympiaVN


Với các alkene bất đối xứng, vấn đề phức tạp hơn bởi có thể có 2 sản phẩm được
tạo thành (đồng phân vùng [đồng phân vị trí]):

Đồ bền tương đối của 2 carbocation trung gian sẽ khống chế tỉ lệ của sản phảm
tạo thành (điều này được giải thích bởi quy tắc kinh nghiệm của Markovnikov).

Để dự đoán hoạt tính tương đối của các alkene trên với H+, cần xét đến độ bền
tương đối của các carbocation trung gian. Alkene hoạt động nhất là
CH2=CHOCH3, do carbocation tương ứng được bền hóa bởi hiệu ứng liên hợp
dương của lone pair trên oxygen. Hiệu ứng này mạnh hơn hiệu ứng cảm ứng âm
qua khung σ.

Nhóm thế bromo của CH2=CHBr cũng có một lone pair tham gia vào bền hóa
liên hợp của carbocation:

Tuy nhiên, trong trường hợp này, các orbital của nguyên tử bromine lớn xen phủ
kém hiệu quả hơn với các orbital p trống của carbon, dẫn đến sự bền hóa kém hơn
so với trường hợp methoxy carbocation. Sự bền hóa này cũng bị giảm đi bởi hiệu
ứng cảm ứng âm của nguyên tử bromine âm điện qua khung σ.
Nhóm methyl có hiệu ứng cảm ứng dương nên cũng làm bền hóa carbocation. Do
đó CH2=CHCH3 có hoạt tính cao hơn CH2=CH2.

Cả CH2=CHN+(CH3)3 và CH2=CHNO2 đều có các nhóm hút electron làm kém


bền hóa các cation có thể tạo thành. Do đó, các hợpchatas này có hoạt tính kém

344 | Câu hỏi lí thuyết Hóa học hữu cơ OlympiaVN


hơn ethylene và thể hiện sự chọn lọc vùng trái ngược với các chất tương đồng có
nhóm thế nhường electron (ở trên). Do CH2=CHN+(CH3)3 có điện tích dương toàn
phần trên nitrogen nên hoạt tính còn kém hơn CH2=CHNO2.

345 | Câu hỏi lí thuyết Hóa học hữu cơ OlympiaVN


Bài 16
Dự đoán những khác biệt về hoạt tính giữa các cặp chất sau:

a)
b) EtOEt (ether) và CH2=CHOEt (enol ether)

c)

d)

e)

f)

g)
Hướng dẫn

a)
Mỗi hợp chất trên đều có thể phản ứng với các electrophile theo cách tương tự
như các alkene (ở bài tập trước). Liên kết đôi không liên hợp dễ bị electrophile
tấn công hơn do phần ester là nhóm hút electron, dẫn đến liên kết đôi carbon-
carbon của hợp chất đầu tiên thiếu hụt electron hơn so với chất thứ hai:

e
Các nucleophile, như ethoxide, hyroxide hoặc amine, tấn công vào nhóm
carbonyl của alkene không liên hợp tạo thành chất trung gian tứ diện, rồi tách
ethoxide (hoặc ethanol, trong điều kiện acid) tạo thành sản phẩm thế:

346 | Câu hỏi lí thuyết Hóa học hữu cơ OlympiaVN


Alkene liên hợp cũng có thể phản ứng theo hướng này:

Tuy nhiên, một hướng phản ứng thay thế khác với các nucleophile cũng có thể:

Phản ứng thứ nhất tương tự như phản ứng của ester không liên hợp (ở trên). Tuy
nhiên, trong phản ứng thứ hai, anion tạo thành của phản ứng cộng nucleophile
không thể tách ethoxide, sẽdẫn đến sự tạo thành một ketene năng lượng cao.
Thường thì sẽ xảy ra sự proton hóa trên carbon để tạo lại một ester (đây là ví dụ
của phản ứng cộng Michael). Hướng phản ứng của alkene liên hợp như vậy phụ
thuộc vào bản chất nucleophile và điều kiện phản ứng.
b) EtOEt (ether) và CH2=CHOEt (enol ether)
Các ether có thể phản ứng với tác nhân electrophile như H+.

Tiểu phân proton hóa dễ bị tấn công nucleophile hơn tiểu phân trung hòa, nhưng
các ether thường kém hoạt động, trừ khi được xử lí với acid mạnh, trong đó base
liên hợp là một nucleophile tốt (ví dụ như hydroiodic acid):

Các enol ehter có liên kết đôi giàu electron và do đó dễ bị tấn công bởi các tiểu
phân electrophile (như đã thảo luận ở bài trước, CH2=CHOMe). Sự proton hóa
có thể diễn ra thuận nghịch trên oxygen - theo cách tương tự như ether ở trên,
hoặc trên carbon - trong trường hợp đó phản ứng có thể diễn ra tiếp (xem bên
dưới). Sự xen phủ giữa các liên kết π C=C và lone pair của oxygen làm bền hóa
liên kết C-Oet và dẫn đến sự giảm hoạt tính của phân tử trung hòa với các
nucleophile:

347 | Câu hỏi lí thuyết Hóa học hữu cơ OlympiaVN


Tuy nhiên nếu enol ether proton hóa trên carbon thì sự tấn công của các
nucleophile lại trở nên thuận lợi:

Nếu nước là tác nhân nucleophile thì enol ether bị thủy phân:

c)
Liên kết đôi thế chloro dễ bị tấn công bởi tác nhân electrophile hơn (so với khi
không nhóm thế) do có hiệu ứng liên hợp dương của lone pair trên nguyên tử
chlorine (xem bài trước, CH2=CHBr). Allylic chloride có thể phản ứng dễ dàng
với các tác nhân nucleophile theo các cơ chế SN2 lẫn SN1. Trong điều kiện SN2,
sự cản trở không gian theo hướng tấn công phái sau của nucleophile ít và phản
ứng diễn ra dễ dàng. Trong điều kiện SN1, allylic cation tạo thành khi mất chloride
ion được bền hóa bởi sự giải tỏa và có thể biểu diễn bởi 2 cấu trúc cộng hưởng.
(Các hệ allylic cũng có thể phản ứng theo cơ chế SN2’). Nucleophile có thể tấn
công ở một trong hai phía. Trong trường hợp này, do phân tử đối xứng nên cả hai
phía đều tạo thành cùng sản phẩm:

348 | Câu hỏi lí thuyết Hóa học hữu cơ OlympiaVN


Liên kết carbon-chlorine của vinyl chloride được bền hóa theo các tương tự như
liên kết carbon-oxygen trong vinyl ethyl ether (xem ý trước). Sự xen phủ giữa
liên kết π C=C và lone pair trên chlorine làm bền hóa liên kết carbon-chlorine và
làm giảm hoạt tính với các nucleophile.

d)
Các acid chloride có phản ứng thuận nghịch với các electrophile, như H+, trên
oxygen của carbonyl. Ethyl chloride thì không dễ phản ứng với các electrophile.
Cả hai hợp chất đều có phản ứng với các nucleophile. Ethyl chloride phản ứng
theo cơ chế SN2, tạo thành các sản phẩm thế:

Với các nucleophile có tính base thì các phản ứng thế cạnh tranh với các phản
ứng thế.
Các acid chloride cũng xảy ra phản ứng thế với các nucleophile, nhưng phản ứng
thế trực tiếp không xảy ra do hướng cộng-tách ưu tiên hơn. Nucleophile phản ứng
với acid chloride tạo thành trung gian tứ diện, sau đó nhóm chloride rời đi. Hướng
này có năng lượng hoạt hóa thấp hơn so với hướng thế trực tiếp, do liên kết
carbon-nucleophile tạo thành trước khi liên kết carbon-chloride bị phá vỡ.

Phản ứng tách từ các acid chloride cần các điều kiện cụ thể và tạo thành sản phẩm
ketene hoạt động mạnh.

349 | Câu hỏi lí thuyết Hóa học hữu cơ OlympiaVN


e)
Cả hai hợp chất đều có chứa một nhóm carbonyl và do đó có thể được dự đoán là
có hoạt tính tương tự nhau. Tuy nhiên, trong khi phản ứng cộng electrophile thuận
nghịch (với một proton) có thể diễn ra với tốc độ tương tự nhau thì các phản ứng
với nucleophile và phản ứng trong môi trường base thường bị ảnh hưởng nhiều
bởi bản chất của các nhóm thế. Trong hợp chất thứ hai, nhóm carbonyl bị án ngữ
không gian bởi các nhóm t-butyl và do đó có hoạt tính kém hơn đáng kể so với
hợp chất thứ nhất – trừ phản ứng với các tác nhân nucleophile nhỏ nhất. Trong
môi trường base, hợp chất thứ nhất tạo thành một enolate ion có thể tham gia
nhiều ngưng tụ carbonyl khác nhau.

Tính chất hóa học tương tự không xuất hiện trong hợp chất thứ hai do nhóm
carbonul không có các nguyên tử hydrogen α và không tạo được các enolate ion.

f)
Bromoethane có cách thức phản ứng tương tự chloroethane (xem các ý trước),
theo cơ chế SN2 tạo thành các sản phẩm thế, hoặc theo cơ chế E2 tạo thành
ethylene. Neopentyl bromide không thể phản ứng theo cơ chế E2 (do không có
hydrogen β), cũng không thể theo cơ chế SN2 do sự án ngữ không gian lớn của
nhóm t-butyl (cản trở tấn công từ phía sau). Dưới các điều kiện thuận lợi, phản
ứng kiểu SN1 có thể diễn ra. Tuy nhiên, thay vì một phản ứng thế đơn giản,
neopentyl cation chuyển vị tạo thành một carbocation bậc 3 bền hơn, tiểu phân
này có thể tách loại proton tạo thành alkene hoặc phản ứng với nucleophile tạo
thành sản phẩm cộng.

350 | Câu hỏi lí thuyết Hóa học hữu cơ OlympiaVN


g)
Cả hai phân tử đều có chứa một nguyên tử sulfur (lưu huỳnh) và thể hiện hoạt
tính tương tự nhau với các electrophie:

Do nguyên tử chlorine có độ âm điện lớn hơn nguyên tử sulfur nên lone pair của
nó có tính nucleophile thấp hơn. Sự tấn công bởi các tác nhân nucleophile bên
ngoài có thể được dự đoán là sẽ thay thế nguyên tử chlorine và tạo thành các sản
phẩm thế theo cách tương tự như phản ứng của ethyl chloride (xem các bài trước).
Tuy nhiên, do lone pair của nguyên tử sulfur gần đó có tính nucleophile nên nó
có thể thay thế chlorine trong phản ứng nội phân tử. Phản ứng này diễn ra nhanh
hơn phản ứng nucleophile liên phân tử do một nguyên tử bên trong phân tử sẽ có
nhiều khả năng đạt được vị trí phù hợp hơn nhằm thúc đẩy phản ứng và ít bị cản
trở bởi dung môi hơn:

Tác nhân nucleophile bên ngoài có thể tấn công vào carbon của vòng trung gian
3 cạnh tạo thành với hợp chất đầu tiên. Trong trường hợp này, sản phẩm giống
với trường hợp thế trực tiếp, nhưng phản ứng diễn ra nhanh hơn.
Với hợp chất thứ hai, phản ứng thế nội phân tử của nguyên tử chlorine đòi hỏi
phải có sự tạo thành vòng 4 cạnh, chậm hơn đáng kể so với sự tạo thành vòng 3

351 | Câu hỏi lí thuyết Hóa học hữu cơ OlympiaVN


cạnh (hiện tượng này được mô tả bởi các quy tắc của Baldwin). Do đó, tốc độ
phản ứng giảm.

352 | Câu hỏi lí thuyết Hóa học hữu cơ OlympiaVN


Bài 17
1) Tốc độ thủy phân chất A tăng mạnh khi có mặt LiCl, còn B gần như không
đổi. Giải thích điều này:

2) Giải thích sự giảm momen lưỡng cực của chất sau khi thay H bằng brom:

3) Giải thích sự tăng dần khả năng decarboxyl hóa trong dãy CX3COOH (F <
Cl < Br).
4) Khi hoàn tan đồng phân cis và trans của hợp chất dưới đây vào axit sunfuric
60% thì chỉ mỗi đồng phân cis tạo dung dịch màu vàng đậm, còn đồng phân
trans cho dung dịch trong suốt. Hãy giải thích điều này.

5) Hãy cho biết đồng phân nào tạo sản phẩm nhanh hơn, giải thích

353 | Câu hỏi lí thuyết Hóa học hữu cơ OlympiaVN


Hướng dẫn
1) Khi H2O tấn công vào axyl clorua sẽ tạo thành trung gian tứ diện có dạng:

Ở A sự tạo thành trung gian tứ diện không hề thuận lợi do trung gian tạo thành sẽ
tương tác mạnh với hai nhóm CH3 ở ortho. Thế nên ở A sẽ xảy ra sự phân ly trực
tiếp tạo thành RCO+ , thành ra việc thêm LiCl sẽ thúc đẩy mạnh hơn sự phân chia
điện tích, dẫn đến sự thúc đẩy phản ứng.

Ở B không xảy ra hiện tượng này, một phần do nhóm NO2 làm giảm tính bền của
carbocation tạo thành.
2) Cấu trúc trong hệ thống có tính thơm, chiều momen lưỡng cực hướng về phía
nguyên tử oxy C=O. Chính vì vậy việc gắn thêm brom sẽ làm giảm momen lưỡng
cực tổng của phân tử (liên kết C-Br ngược phía C-O làm giảm đi momen lưỡng
cực phân tử).

3) Phản ứng xảy ra do tạo thành anion bền CX3COOH  CX3- + CO2 + H+. Tuy
nhiên nếu như vậy tốc độ của CF3 phải lớn nhất, nhưng anion CBr3- tạo thành
nhanh nhất vì sự tạo thành anion có cấu trúc phẳng sẽ giảm tương tác đẩy giữa
các nguyên tử brom có kích thước lớn.
4) Ở đồng phân cis tạo thành carbocation được ổn định bằng obitan pi của liên
kết đôi, cộng thêm việc nhóm phenyl trợ màu dẫn đến tạo thành dung dịch có
màu. Đồng phân trans không có sự hỗ trợ từ liên kết đôi bên vòng xiclohexen nên
không có màu:

354 | Câu hỏi lí thuyết Hóa học hữu cơ OlympiaVN


5) Phản ứng sẽ xảy ra nhanh nếu Br ở vị trí a, là vị trí thuận lợi cho việc di chuyển
nối đôi. Như vậy A sẽ cho phản ứng chậm hơn B.

355 | Câu hỏi lí thuyết Hóa học hữu cơ OlympiaVN


Cơ bản về cơ chế phản ứng
Bài 1
1) Bổ sung các mũi tên cong để biểu diễn sự tạo thành mỗi sản phẩm sau:

2) Sử dụng các mũi tên cong đã cho để bổ sung các điện tích còn thiếu:

356 | Câu hỏi lí thuyết Hóa học hữu cơ OlympiaVN


3) Xác định sản phẩm của các phản ứng sau. Chỉ rõ điện tích.

357 | Câu hỏi lí thuyết Hóa học hữu cơ OlympiaVN


Hướng dẫn
1)

2)

358 | Câu hỏi lí thuyết Hóa học hữu cơ OlympiaVN


359 | Câu hỏi lí thuyết Hóa học hữu cơ OlympiaVN
3)

360 | Câu hỏi lí thuyết Hóa học hữu cơ OlympiaVN


Bài 2
Với mỗi phản ứng sau, hãy thêm mũi tên cơ chế để biểu diễn dòng chuyển dời
electron. Nếu cần thiết, hãy chỉ rõ các cặp electron chưa liên kết hoặc hydrogen.

361 | Câu hỏi lí thuyết Hóa học hữu cơ OlympiaVN


Hướng dẫn

362 | Câu hỏi lí thuyết Hóa học hữu cơ OlympiaVN


Bài 3
Với mỗi phản ứng sau, hãy thêm mũi tên cơ chế để biểu diễn dòng chuyển dời
electron. Nếu cần thiết, hãy chỉ rõ các cặp electron chưa liên kết hoặc bất kì điện
tích còn thiếu nào.

Hướng dẫn

363 | Câu hỏi lí thuyết Hóa học hữu cơ OlympiaVN


Bài 4
Với mỗi phản ứng sau, hãy thêm mũi tên cơ chế để biểu diễn dòng chuyển dời
electron. Nếu cần thiết, hãy chỉ rõ các cặp electron chưa liên kết hoặc bất kì điện
tích còn thiếu nào.

Hướng dẫn

364 | Câu hỏi lí thuyết Hóa học hữu cơ OlympiaVN


365 | Câu hỏi lí thuyết Hóa học hữu cơ OlympiaVN
Bài 5
Viết cơ chế quá trình ngược với mỗi phản ứng sau:

366 | Câu hỏi lí thuyết Hóa học hữu cơ OlympiaVN


Hướng dẫn

367 | Câu hỏi lí thuyết Hóa học hữu cơ OlympiaVN


Bài 6
Hoàn thành các cơ chế phản ứng sau

Hướng dẫn

368 | Câu hỏi lí thuyết Hóa học hữu cơ OlympiaVN


Bài 7
Đặt các mũi tên thích hợp vào các chất đầu để biểu diễn sự tạo thành các sản
phẩm.

Hướng dẫn

369 | Câu hỏi lí thuyết Hóa học hữu cơ OlympiaVN


Bài 8
Trình bày cơ chế các phản ứng trong chuỗi chuyển hoá sau đây:

Hướng dẫn
Trong giai đoạn đầu tiên, liên kết C-I yếu đã bị phá vỡ, và các liên kết C-O được
tạo thành. Nhóm OH mới phải có nguồn gốc từ hydroxide, so vậy tác nhân
nucleophile sẽ là OH-; electrophile là alkyl iodide.

Ở giai đoạn thứ hai, 1H (hay chính xác hơn là 1H+) từ nhóm hydroxyl bị mất. Tác
nhân phản ứng sodium hydride chứa ion hydride là base mạnh, đóng vai trò tác
nhân nucleophile tấn công H, phá vỡ liên kết O-H, đẩy các electron về phía
oxygen tạo thành anion.

Trong giai đoạn cuối, oxyanion đóng vai trò tác nhân nucleophile và electrophile
là benzyl bromide. Mũi tên bắt đầu từ nguyên tử mang điện tích âm, biểu diễn sự
tạo thành liên kết C-O mới và phá vỡ liên kết C-Br cũ.

370 | Câu hỏi lí thuyết Hóa học hữu cơ OlympiaVN


Bài 9
Mỗi electrophile sau đây có thể phản ứng với một nucleophile ở (ít nhất) 1 trong
2 vị trí. Xác định các vị trí (nguyên tử) đó và vẽ cơ chế, sản phẩm tạo thành.

Hướng dẫn
Trong mỗi trường hợp, một trong các tâm electrophile là một nguyên tử H có tính
+
acid. Ngoài ra còn có các liên kế và C=O).
Trong trường hợp đầu tiên, có hai hướng phản ứng riêng biệt:

Đừng để bị nguyên tử nitrogen mang điện tích dương đánh lừa. Phản ứng này
không thể xảy ra được bởi sẽ tạo thành hợp chất chứa nitrogen hoá trị V (vô lí!).

Trong hợp chất thứ hai, có 3 hướng khả thi. Nguyên tử hydrogen có tính acid của
nhóm carboxylic acid và liên kết C=O có tính electrophile đều có thể là tâm phản
ứng. Ngoài ra cũng cần chú ý đến nguyên tử phosphorus mang điện dương. Tuy
cùng nhóm với nitrogen trong bảng tuần hoàn, nhưng P có thể tạo ra 5 liên kết.

371 | Câu hỏi lí thuyết Hóa học hữu cơ OlympiaVN


372 | Câu hỏi lí thuyết Hóa học hữu cơ OlympiaVN
Bài 10
Phản ứng của 3 cặp chất với nhau tạo thành các sản phẩm như sau, nhưng cơ chế
phản ứng chưa đúng. Giải thích và viết cơ chế hợp lí hơn.

Hướng dẫn
Cơ chế 1:

Cơ chế 2:

Cơ chế 3:

373 | Câu hỏi lí thuyết Hóa học hữu cơ OlympiaVN


Giải thích cơ chế phản ứng
Bài 1
Dự đoán sản phẩm tạo thành từ các phản ứng sau:

Hướng dẫn
Khi có mặt aluminium chloride, t-butyl chloride phản ứng tạo thành carbocation,
t-butyl cation, rồi tiểu phân này phản ứng như một electrophile với propene:

Carbocation bậc hai tạo thành được giữ lại bởi chloride ion. (Khi nồng độ propene
cao, phản ứng tiếp diễn với propene sẽ tạo thành vật liệu polymer). Sự chọn lọc
của phản ứng như vậy là bởi t-butyl cation cồng kềnh phản ứng ở carbon đầu
mạch của propene, do đó tạo thành điện tích trên carbon bậc hai. Nếu electrophile
phản ưng ở C2 của propene thì điện tích sinh ra ở carbon bậc nhất, kém thuận lợi
hơn.

Hướng dẫn
Xử lí propene với formaldehyde khi có mặt acid tạo thành butane-1,3-diol (phản
ứng Prins) cùng với một acetal vòng được dẫn ra từ phản ứng của nó với phân tử
formaldehyde thứ hai. Acid làm tăng tính electrophile của formaldehyde và khả
năng phản ứng của nó với alkene.

374 | Câu hỏi lí thuyết Hóa học hữu cơ OlympiaVN


Phản ứng với alkene hai nhóm thế chủ yếu tạo thành các 1,3-diol với hiệu suất
thấp, trong khi đó các alkene một lần thế chủ yếu tạo thành acetal (1,3-dioxan).

Hướng dẫn
Các aldehyde và ketone có khả năng enol hóa thì có thể trải qua phản ứng tự
ngưng tụ, hoặc phản ứng với hợp chất carbonyl khác. Formaldehyde không có
hydrogen có thể enol hóa. Khi có mặt acid, acetone tham gia vào một cân bằng
động, tạo thành một lượng nhỏ enol:

Khi có mặt formaldehyde được proton hóa, enol của acetone phản ứng tạo thành
β-hydroxy ketone:

Các β-hydroxy ketone kém bền trong acid và bị tách nước xúc tác bởi acid:

375 | Câu hỏi lí thuyết Hóa học hữu cơ OlympiaVN


Tuy nhiên, hiệu suất thường thấp và các carbonyl không no α,β có thể được điều
chế tốt hơn qua phản ứng Mannich.

Hướng dẫn
Boron trifluoride, một Lewis acid, tạo phối trí với carbonyl oxygen, xúc tác cho
sự enol hóa. Enol nhiều lần thế hơn sẽ chiếm ưu thế trong hỗn hợp các đồng phân
E và Z.

Phản ứng của butanone, đã phối trí với BF3, với đồng phân Z đã được biểu diễn.
Trong môi trường acid, sự mất nước xảy ra dễ dàng và tạo thành hỗn hợp các
enone liên hợp.

376 | Câu hỏi lí thuyết Hóa học hữu cơ OlympiaVN


Đồng phân enol phụ tạo thành một enone đồng phân:

Phản ứng trong chuỗi có tính thuận nghịch và trong điều kiện acid, thường tạo
thành một hỗn hợp sản phẩm. Các ketone không đối xứng thường xảy ra sự tự
ngưng tụ bởi sự tấn công của một α-carbonyl ít bị án ngữ không gian vào carbonyl.
Do đó, butanone, một phân tử tương đối ít bị án ngữ, là một ngoại lệ.

Hướng dẫn
Butanone phản ứng với hỗn hợp formaldehyde và một amine bậc hai, khi có mặt
acid, sẽ tạo thành một dẫn xuất N,Ndimethylaminomethyl - sau khi kiềm hóa.
Đây là phản ứng Mannich. Dimethylamine ưu tiên phản ứng với formaldehyde
hơn là butanone do carbonyl carbon của formaldehyde dương điện hơn. Giai đoạn
đầu của phản ứng bao gồm sự cộng hợp dimethylamine vào formaldehyde tạo
thành một sản phẩm cộng, chất này khi có mặt acid bị tách nước tạo thành một
electrophile:

Butanone sẽ enol hóa thành hỗn hợp ba enol, với các enol nhiều lần thế hơn chiếm
ưu thế.

377 | Câu hỏi lí thuyết Hóa học hữu cơ OlympiaVN


Giai đoạn cuối được biểu diễn dưới đây là cho phản ứng của (Z)-enol với iminium
cation.

Hướng dẫn
Trong số hai hợp chất chứa carbonyl, chỉ có acetophenone có hydrogen có thể
enol hóa. Sự ngưng tụ acetophenone diễn ra với aldehyde, sau đó tạo thành enone
liên hợp bởi sự tách nước. Phản ứng này thuận lợi hơn sự tự ngưng tụ của
acetophenone bởi aldehyde carbonyl hoạt động hơn. Trong acid, enone tạo thành
tồn tại trong hỗn hợp cân bằng của các đồng phân (Z) và (E). Sự tạo thành
hemiacetal nội phân tử từ (Z)-enone, sau đó tách nước xúc tác acid, tạo thành
muối oxonium là tiền thân của hệ anthocyanidin.

378 | Câu hỏi lí thuyết Hóa học hữu cơ OlympiaVN


379 | Câu hỏi lí thuyết Hóa học hữu cơ OlympiaVN
Bài 2
Đề xuất cơ chế cho các phản ứng sau:

Hướng dẫn
Phản ứng này bao gồm quá trình vòng hóa xúc tác acid. Cân bằng công thức phân
tử cho thấy phản ứng có sự cộng hợp nước. Sự proton hóa có thể diễn ra trên
carbonyl, ban đầu tạo thành vòng sáu cạnh (phản ứng Prins), và carbocation bậc
ba sẽ phản ứng với nước. Cuối cùng, quá trình kết thúc bởi sự tách proton.

Một hướng vòng hóa thay thế tạo thành diol vòng bảy cạnh thì kém thuận lợi hơn
do đòi hỏi sự tạo thành trạng thái chuyển tiếp vòng bảy cạnh và trung gian cation
bậc hai.

380 | Câu hỏi lí thuyết Hóa học hữu cơ OlympiaVN


Hướng dẫn
Tương tự như bài trước, phản ứng này cũng dẫn đến sự vòng hóa. Cân bằng công
thức phân tử cho thấy có sự mất nước, chỉ ra rằng sự vòng hóa diễn ra với sự tách
nước. Sự proton hóa alcohol và mất nước tạo thành cation bậc ba, tiểu phân này
có thể trải qua sự vòng hóa để tạo thành vòng bốn hoặc năm cạnh, rồi cuối cùng
tách proton để hoàn thành chuyển hóa. Vòng năm cạnh thuận lợi hơn bởi các
nguyên nhân về hiệu ứng điện tử lập thể, mặc dù thực tế rằng nó dẫn đến sự tạo
thành cation bậc hai.

Hướng dẫn
Đã xảy ra sự vòng hóa và hai sản phẩm tạo thành là đồng phân của epoxide ban
đầu. Với sản phẩm thứ hai, là một phân tử bicyclic, thì không chỉ một, mà cần
đến hai quá trình vòng hóa. Để hiểu phản ứng này, cần xét đến các phân tử trong
không gian ba chiều. Boron trifluoride xúc tác cho phản ứng bằng cách tạo phối
trí với epoxide oxygen, làm suy yếu các liên kết carbon-oxygen, đặc biệt là với
tâm bậc ba khi sự gia tăng điện tích thuận lợi hơn. Sự vòng hóa với sự tạo thành
vòng sáu cạnh thuận lợi hơn các hương khác.
Sự chọn lọc của phản ứng được quyết định bởi sự ưu tiên của alkene nucleophile
nội phân tử tấn công và tạo thành liên kết với carbon đầu mạch, do đó phát sinh
điện tích ở carbon bậc ba thay vì các vị trí khác.

381 | Câu hỏi lí thuyết Hóa học hữu cơ OlympiaVN


382 | Câu hỏi lí thuyết Hóa học hữu cơ OlympiaVN
Bài 3
Giải thích các phát biểu sau:
a) Trong khi t-butyl chloride gần như kết tủa tức thời với dung dịch silver
nitrate-alcohol thì hợp chất chloride (I) lại trơ, thậm chí là khi đun sôi kéo
dài.

b) Các acetal bền với base nhưng bị thủy phân dễ dàng bởi acid.
c) Khi độ phân cực dung môi tăng lên, tốc độ phản ứng SN2 trong phương trình
4.1 giảm một chút nhưng trong phản ứng 4.2 thì lại tăng mạnh.

d) Sự thủy phân kiềm của ethyl bromide được xúc tác bởi iodide ion.
e) Phản ứng tách nước của CH3CH(OH)C(CH3)3 với sulfuric acid đặc tạo thành
tetramethylethylene.
f) Các β-keto-acid đơn giản, RCOCH2CO2H, dễ decarboxyl khi đun nóng,
nhưng hợp chất (II) thì lại bền.

g) Các bromo-ether đồng phân (III) và (IV) bị thủy phân trong acetic acid tạo
thành cùng một hỗn hợp sản phẩm.

Hướng dẫn

383 | Câu hỏi lí thuyết Hóa học hữu cơ OlympiaVN


a) Trong khi t-butyl chloride gần như kết tủa tứ thời với dung dịch silver nitrate
trong alcohol thì hợp chất chloride (I) lại trơ, thậm chí là khi đun sôi kéo dài.
Trong dung dịch silver nitrate-alcohol thì t-butyl chloride giải phóng chloride ion
kết tủa ở dạng silver chloride. Carbocation t-butyl phẳng, tương đối bền sẽ mất
proton và tạo thành sản phẩm tách:

Chloride (I) lại rất trơ trong những điều kiện này do sự mất chloride ion sẽ tạo
thành một carbocation không đạt được cấu trúc phẳng - điều kiện cần thiết cho
tính bền.

Các phản ứng SN2 và E2 cũng không thể xảy ra với cả hai hợp chất này do chúng
có sự án ngữ không gian lớn đối với sự tấn công phía sau.
b) Các acetal bền với base nhưng bị thủy phân dễ dàng bởi acid.
Cơ chế thủy phân acetal được cho dưới đây. Giai đoạn đầu tiên là proton hóa
nhóm alkoxy (OR). ROH là nhóm rời đi tốt, do đó sẽ dễ bị thế:

384 | Câu hỏi lí thuyết Hóa học hữu cơ OlympiaVN


Trong môi trường base, nhóm rời đi của phản ứng này phải là alkoxide ion (RO-
). Đây là nhóm rời đi tương đối kém, do đó các acetal bền trong môi trường base:

Vì tính bền với base nhưng dễ thủy phân bởi acid nên các acetal thường được sử
dụng làm nhóm bảo vệ cho các nhóm carbonyl.
c) Khi độ phân cực dung môi tăng lên, tốc độ phản ứng SN2 trong phương trình
4.1 giảm một chút nhưng trong phản ứng 4.2 thì lại tăng mạnh.
Tốc độ của một phản ứng phụ thuộc vào tính bền tương đối của các chất đầu và
trạng thái chuyển tiếp. Nếu điều kiện phản ứng thay đổi sao cho độ bền của trạng
thái chuyển tiếp tăng lên so với chất đầu thì tốc độ phản ứng sẽ giảm. Tương tự,
nếu một biến đổi trong điều kiện phản ứng dẫn đến tính bền của trạng thái chuyển
tiếp giảm so với chất đầu thì tốc độ sẽ tăng lên.
Các tiểu phân mang điện bền hơn trong các dung môi phân cực hơn. Trong
phương trình 4.1, cả các chất đầu lẫn sản phẩm đều có 1 phân tử trung hòa và 1
anion; số tiểu phân mang điện không thay đổi trong toàn phản ứng, cũng như ở
trạng thái chuyển tiếp (xem bên dưới). Do đó, không có gì bất ngờ khi độ phân
cực dung môi thay đổi chỉ ảnh hưởng rất nhỏ đến tốc độ phản ứng5.

Trong phương trình 4.2, 2 ion được tạo thành từ 2 phân tử trung hòa. Cấu trúc
trạng thái chuyển tiếp của phản ứng là một tiểu phân lưỡng cực mạnh:

5 Trong các chất phản ứng, điện tích âm định xứ trên 1 nguyên tử oxygen; trong trạng thái
chuyển tiếp, điện tích được phân tán nhiều hơn. Do đó, các chất đầu bền hơn khá nhiều
trong dung môi phân cực, và tốc độ phản ứng giảm một chút khi độ phân cực dung môi
tăng lên.

385 | Câu hỏi lí thuyết Hóa học hữu cơ OlympiaVN


Tiểu phân như vậy sẽ bền hơn trong dung môi phân cực hơn là trong dung môi
không phân cực. Do đó, sự tăng độ phân cực dung môi sẽ hạ thấp đáng kể năng
lượng của trạng thái chuyển tiếp so với các chất đầu, nhờ vậy tốc độ phản ứng sẽ
tăng lên.
d) Sự thủy phân kiềm của ethyl bromide được xúc tác bởi iodide ion.
Phản ứng thủy phân kiềm của ethyl bromide diễn ra theo cơ chế SN2 một giai
đoạn:

Khi thêm iodide ion sẽ làm tăng tốc độ phản ứng - giờ đây diễn ra theo 2 giai
đoạn, mỗi giai đoạn là 1 phản ứng thế SN2:

Giai đoạn thứ nhất nhanh hơn phản ứng thế trực tiếp bromide bởi hydroxide bởi
iodide ion là nucleophile tốt hơn hydroxide ion. Giai đoạn thứ hai cũng nhanh
hơn phản ứng thế trực tiếp do iodide là nhóm rời đi tốt hơn so với bromide ion.
Phản ứng toàn phần nhanh hơn và iodide đóng vai trò là xúc tác nucleophile.
Lưu ý là nếu bromide ban đầu quang hoạt thì do sự nghịch đảo cấu hình ở mỗi
giai đoạn của phản ứng nên hóa lập thể của alcohol sẽ giống như bromide ban
đầu.
e) Phản ứng tách nước của CH3CH(OH)C(CH3)3 với sulfuric acid đặc tạo thành
tetramethylethylene.
Phản ứng diễn ra theo cơ chế sau:

386 | Câu hỏi lí thuyết Hóa học hữu cơ OlympiaVN


Nước tách ra dẫn đến sự tạo thành carbocation bậc hai kém bền (I), tiểu phân này
nhanh chóng chuyển vị tạo thành carbocation bậc ba bền hơn (II). Cation này sau
đó mất proton tạo thành tetramethylethylene.
f) Các β-keto-acid đơn giản, RCOCH2CO2H, dễ decarboxyl khi đun nóng, nhưng
hợp chất (II) thì lại bền.
Các β-keto-acid đơn giản decarboxyl theo cơ chế sau:

Phân tử (II) bền do dạng hình học của trạng thái chuyển tiếp của quá trình
decarboxyl hóa nội phân tử không cho phép tạo thành liên kết đôi carbon-carbon
và do đó có năng lượng cao đến mức không thể đạt tới.

Sản phẩm dự đoán tạo thành có liên kết đôi cầu không thỏa mãn quy tắc Bredt và
không thể được tạo thành.
g) Các bromo-ether đồng phân (III) và (IV) bị thủy phân trong acetic acid tạo
thành cùng một hỗn hợp sản phẩm.
2 bromo-ether tạo thành cùng sản phẩm với lượng giống nhau, cho thấy các phản
ứng này có cùng hợp chất trung gian. Do các phản ứng diễn ra trong môi trường
phân cực và acetic acid là nucleophile kém, nên phản ứng kiểu SN1 thuận lợi. Do
đó, mỗi hợp chất đều phản ứng tạo thành cùng oxonium ion:

387 | Câu hỏi lí thuyết Hóa học hữu cơ OlympiaVN


Hợp chất trung gian này sau đó có thể bị tấn công bởi acetic acid ở một trong hai
vị trí (a và b) tạo thành các sản phẩm đã biết. Phản ứng tấn công nucleophile, ở
carbon ít nhóm thế hơn (hướng a), là thuận lợi hơn về mặt không gian.

388 | Câu hỏi lí thuyết Hóa học hữu cơ OlympiaVN


Bài 4
Dự đoán sản phẩm chính của các phản ứng sau:

a)

b)

c)

d)

e)

f)
g)

h)

i)

j)

k)
Hướng dẫn

a)
Phản ứng cộng của alkene đã được thảo luận ở các bài trước.

Giai đoạn đầu tiên của phản ứng liên quan đến sự tạo thành các cation sau:

389 | Câu hỏi lí thuyết Hóa học hữu cơ OlympiaVN


Carbocation I được giải tỏa điện tích bởi nhóm aryl nên bền hơn và chiếm ưu thế,
do đó sản phẩm tạo thành từ cation này là sản phẩm chính.

b)
Halide bậc ba dễ tạo thành sản phẩm khử hơn là sản phẩm thế để giảm bớt sự án
ngữ không gian, trừ khi nucleophile không có tính base (ví dụ bromide hoặc
iodide ion).

c)
Đây là một ví dụ về phản ứng tách Hofmann bất thường: nó tạo thành sản phẩm
chính là alkene ít nhóm thế hơn - cho thấy đây là một quá trình khống chế động
học.

Điện tích dương của nhóm N(CH3)3 làm tăng lực acid của các proton ở các nguyên
tử carbon kế cận, nên phản ứng diễn ra qua một trạng thái chuyển tiếp tựa-
carbanion, có thể biểu diễn bởi cấu trúc sau:

Carbanion bậc nhất bền hơn carbanion bậc hai (tiểu phân này bị kém bền hóa bởi
hiệu ứng cảm ứng dương của hai nhóm alkyl). Do đó, alkene ít nhóm thế hơn
được tạo thành.

d)

390 | Câu hỏi lí thuyết Hóa học hữu cơ OlympiaVN


Trong môi trường kiềm ở 55 oC, phản ứng thế SN2 - cạnh tranh với phản ứng tách
- chiếm ưu thế. Khi nhiệt độ tăng lên thì lượng sản phẩm phụ alkene cũng tăng.

e)
Với một haloalkane bậc hai, phản ứng tách thuận lợi hơn phản ứng thế. Quá trình
tách E2 diễn ra qua một trạng thái chuyển tiếp tựa-alkene. Do độ bền của các
alkene tăng theo mức độ thế quanh liên kết đôi, dẫn đến sản phẩm có nhiều nhóm
thế hơn chiếm ưu thế.

f)
Phản ứng cộng vào hệ chứa 2 hoặc nhiều liên kết đôi tạo thành hỗn hợp sản phẩm
bởi điện tích trong carbocation trung gian được giải tỏa trên 2 hoặc nhiều nguyên
tử carbon, và bất kì vị trí nào cũng có thể bị tấn công ở giai đoạn thứ hai. Trong
điều kiện phù hợp, liên kết đôi của dibromide có thể cộng bromine tiếp.

g)
Mercury(II) ion thường được sử dụng làm xúc tác cho phản ứng cộng các
nucleophile vào alkyne, do nó tạo phức với liên kết ba và làm cho việc tấn công
của nucleophile thuận tiện hơn.

391 | Câu hỏi lí thuyết Hóa học hữu cơ OlympiaVN


h)
Phản ứng cộng bromine này có đặc trưng lập thể: chỉ sản phẩm anti được tạo
thành.

i)
Cation tạo thành khi cộng proton vào liên kết đôi nhanh chóng chuyển vị tạo
thành carbocation bậc ba bền hơn, sau đó cation mới này phản ứng với chloride
ion tạo thành sản phẩm chính (II).

392 | Câu hỏi lí thuyết Hóa học hữu cơ OlympiaVN


j)
Hydroxide ion tách một proton từ nhóm alcohol. Phản ứng nội phân tử tạo thành
epoxide nhanh hơn nhiều so với phản ứng thế liên phân tử của chlorine bởi
hydroxide ion. Dưới những điều kiện nhất định, epoxide bị tấn công tiếp bởi
hydroxide tạo thành diol.

k)
Đây là phản ứng decarboxyl nội phân tử điển hình của các β-keto-acid.

393 | Câu hỏi lí thuyết Hóa học hữu cơ OlympiaVN


Bài 5
Hãy cho biết anken sau đây là anken nào ? Giải thích bằng cách viết sơ đồ phản
ứng. Gọi tên anken ban đầu và viết chính xác lập thể sản phẩm
1. Tác dụng với Br2/CCl4 cho (2R, 3R)-2,3-dibrom-3-metylpentan
2. Tác dụng với KMnO4 loãng lạnh cho (1S, 2R)-1-phenylpropan-1,2-diol.
3. Cho 2,7-dimetyl-2,6-octadien tác dụng với axit p-toluensunfonic (TsOH).
Hướng dẫn

394 | Câu hỏi lí thuyết Hóa học hữu cơ OlympiaVN


Bài 6
Giải thích các hiện tượng sau đây
1. Khi một ankyl halogenua phản ứng với anion isoxianat SCN- thì phản ứng
thế xảy ra ở nguyên tử lưu huỳnh cho sản phẩm RSCN. Tuy nhiên nếu thay
anion thioxianat bằng anion xianat OCN- thì phản ứng thế xảy ra trên nguyên
tử N cho sản phẩm RNCO.
2. Dung môi phân các dẫn xuất bromua sau đây theo cơ chế SN1 sẽ cho các kết
quả khác nhau như sơ đồ dưới.

3. Khi tiến hành phản ứng giữa axit (2R) – bromaxetic với NaOH 1% thu được
sản phẩm axit (2R) – hydroxiaxetic, thế nhưng khi tăng nồng độ NaOH lên
50% lại thu được axit (2S) – hydroxiaxetic.
4. Các hợp chất có dạng ROCH2X (X: Cl, Br, I) rất nhạy với phản ứng SN1, nhất
là khi R là gốc phenyl thì vận tốc tương đối của phản ứng luôn vào khoảng
1014. Giải thích
Hướng dẫn
1. Với các ion thioxianat và xianat thì các công thức cộng hưởng được biểu diễn
như sau:

Theo các công thức cộng hưởng thì điện tích âm được san cho O, N, và
S. Đầu đóng vai trò nuclephin sẽ là đầu S trong thioxianat và đầu N trong
xianat do tính nucleophin của nguyên tố tăng từ trên xuống dưới trong
một nhóm và từ phái sang trái trong một chu kỳ.
2. Phản ứng xảy ra do sự tạo cacbocation norbornyl không cổ điển nên sự tấn
công sau bị hạn chế, vì thế nên chỉ có thể xảy ra sự tấn công trước với sự bảo
toàn cấu hình.

395 | Câu hỏi lí thuyết Hóa học hữu cơ OlympiaVN


3. Trong NaOH 1% có sự tham gia của nhóm kề este. Phản ứng xảy ra theo cơ
chế sau:

Phản ứng xảy ra qua hai lần SN2 liên tiếp nên bảo toàn cấu hình
Tuy nhiên nếu phản ứng được tiến hành trong môi trường OH- đặc thì lúc
này do nồng độ lớn của OH- nên nó có thể tranh chấp được với sự tấn
công nội phân tử của anion carboxylat nên có thể thu được sản phẩm quay
cấu hình
4. Do cacbocation tạo thành được bền vững bởi hiệu ứng liên hợp n – p của cặp
e trên oxy với obitan p trống của cacbocation

396 | Câu hỏi lí thuyết Hóa học hữu cơ OlympiaVN


Bài 7
Đề nghị cơ chế cho các phản ứng sau:

Hướng dẫn

397 | Câu hỏi lí thuyết Hóa học hữu cơ OlympiaVN


398 | Câu hỏi lí thuyết Hóa học hữu cơ OlympiaVN
Bài 8
Terpineol có thể bị chuyển hóa thành cineole (được tìm thấy tự nhiên trong gia vị
thảo quả) khi cho vào dung dịch acid loãng. Trình bày cơ chế phản ứng để giải
thích chuyển hóa này.
Hướng dẫn
Nhóm OH đóng vai trò như một nucleophile. Giai đoạn đầu tiên là tạo thành
carbocation bằng cách cộng hợp proton vào alkene. Sau đó, nhóm -OH phản ứng
với carbocation bậc ba, tạo thành một hệ vòng bicyclic. Rồi chloride ion tách
proton khỏi trung gian để tạo thành sản phẩm cuối.

399 | Câu hỏi lí thuyết Hóa học hữu cơ OlympiaVN


Bài 9
1. Hãy giải thích tại sao khi dung môi phân (t-Bu)3CBr thì tốc độ xảy ra nhanh
hơn gấp 10000 lần so với khi dung môi phân Me3CBr, mặc dù cacbocation Me3C+
bền hơn rất nhiều.
2. Hãy đề nghị một cơ chế chi tiết cho sự chuyển vị cacbocation sau đây:

3. Giải thích tốc độ thủy phân tương đối của các dẫn xuất halogen sau đây

Hướng dẫn
1. Cacbocation (t-Bu)3C+ được tạo thành nhanh hơn do lúc này sự cản trở không
gian được giải phóng vì ba nhóm thế lớn có góc hóa trị 109,5o chuyển thành 120o.
2. Cơ chế

3. Tốc độ thủy phân càng tăng khi sự cứng nhắc của vòng càng giảm (tăng số C
trong vòng), vì như vậy việc tạo thành cacbocation phẳng sẽ tốt hơn.

400 | Câu hỏi lí thuyết Hóa học hữu cơ OlympiaVN


Bài 10
1. Giải thích trị số pKa của các hydrocarbon sau đây:

2. Hợp chất A1 là dẫn xuất của eugenol có khả năng kích thích sinh trưởng thực
vật. Từ eugenol có thể tổng hợp A1 theo hai cách sau:
OH

a. OCH3
NaOH/EtOH 1. ClCH2COONa, 90OC A1
A
140OC - 180OC 2. HCl

CH2-CH=CH2
OH

b. OCH3
1. NaOH O
B 1. NaOH/EtOH, 100 C A1
2. ClCH2COONa, 90OC 2. HCl

CH2-CH=CH2

Hãy giải thích và so sánh khả năng phản ứng của hai cách tổng hợp trên.
Hướng dẫn
1. Việc cộng hưởng của hai cacbanion còn lại làm phá vỡ tính thơm của vòng
benzen nên độ bền của cacbanion tương ứng của các vòng ghép benzen giảm đi,
tức pKa tăng lên.
2. Phản ứng tạo thành A là phản ứng đồng phân hoá eugenol thành isoeugenol
trong môi trường kiềm rượu theo cơ chế tạo cacbanion. Ở đây ArOH chuyển thành
ArO- , sự liên hợp của O- với nhân thơm làm giảm độ bền của cacbanion.
Trong quá trình b. O- của ion ArOCH2COO- không liên hợp với nhân thơm nên
cacbanion bền hơn làm cho phản ứng đồng phân hoá dễ dàng hơn. Do đó việc
thực hiện theo quá trình b. thuận lợi hơn, ở nhiệt độ thấp hơn và cho hiệu suất cao
hơn.

401 | Câu hỏi lí thuyết Hóa học hữu cơ OlympiaVN


Bài 11
1. Cho biết sản phẩm của các phản ứng sang và giải thích bằng cơ chế (sản phẩm
cũng là tiểu phân trung gian)

2. Người ta tìm thấy được một số gốc tự do A và B có cấu trúc như hình dưới.
Cho biết cấu trúc hình học của nguyên tử cacbon trung tâm trong hai gốc này và
giải thích tại sao nó có dạng hình học đó.

Hướng dẫn
1. Cơ chế

2. Do không thể đạt được cấu trúc phẳng vì sự cứng nhắc của vòng nên cả hai gốc
tự do A và B đều có dạng hình tháp với nguyên tử cacbon trung tâm lai hóa sp3

402 | Câu hỏi lí thuyết Hóa học hữu cơ OlympiaVN


Bài 12
Hai alkyl bromide 1 và 2 khi tham gia phản ứng dung môi phân SN1 bị chuyển
hóa thành cùng một alcohol bậc hai 3 thay vì tạo thành hỗn hợp các sản phẩm
dung môi phân 3 và 4. Trong chuyển hóa này, 1 có tốc độ chuyển hóa nhanh gấp
1 triệu lần so với 2.

Giải thích tại sao 1 lại có hoạt tính mạnh hơn nhiều so với 2. Và tại sao 1 chỉ tạo
thành alcohol 3 mà không phải hỗn hợp 3 và 4.
Hướng dẫn
Trong phân tử 1, liên kết π có thể hỗ trợ cho sự tách ra của Br- bằng cách nhường
[electron] vào σ*(C-Br). Tương tác này cũng giải thích cho hoạt tính lớn hơn
nhiều so với 2. Sự chọn lọc vùng [tạo thành đồng phân 3] quan sát thấy được hiểu
là kết quả của tiểu phân trung gian chung dưới đây.

403 | Câu hỏi lí thuyết Hóa học hữu cơ OlympiaVN


Bài 13
Trong phản ứng thế lưỡng phân tử SN2 của các cycloalkyl halide (và các nhóm
rời đi hữu hiệu khác) thì tốc độ phản ứng giảm khi kích thước vòng giảm:
cyclopentyl > cyclobutyl >> cyclopropyl.

Giải thích cho xu hướng biến đổi khả năng phản ứng trên.
Hướng dẫn
Trong phản ứng SN2, nguyên tử carbon xảy ra phản ứng thế sẽ lai hóa lại từ dạng
tứ diện thành dạng tam giác trong trạng thái chuyển tiếp. Theo đó, khung vòng
phải điều tiết theo sự tăng kích thước góc liên kết ở carbon này từ ~104 lên 120o.
Khi giảm kích thước vòng, sự điều tiết góc tăng lên trở nên khó khăn hơn, dẫn tới
năng lượng hoạt hóa của quá trình sẽ tăng lên khi kích thước vòng giảm.

404 | Câu hỏi lí thuyết Hóa học hữu cơ OlympiaVN


Bài 14
1) Các bromohydrin 1 và 2, dưới tác động của base bị vòng hóa tạo thành dẫn
xuất epoxide với tốc độ rất khác nhau. 2 phản ứng nhanh hơn 1 nhiều. Từ
hiểu biết của bạn về phản ứng theo cơ chế SN2, hãy giải thích quan sát này.

2) Cyclohexene epoxide dưới đây phản ứng với nhiều nucleophile tạo thành các
sản phẩm với cấu trúc tổng quát 2 với độ chọn lọc cao, bất chấp thực tế là sự
tấn công ở C4 tạo thành đồng phân dia 1 bền hơn.

405 | Câu hỏi lí thuyết Hóa học hữu cơ OlympiaVN


Hướng dẫn
1) Phản ứng SN2 có sự khống chế lập thể, đòi hỏi có sự tấn
công từ phía sau: Nu → σ*C-Br. Trong 2, điều kiện này đạt
được ở cấu dạng ghế, diaxial.

Trong 1, phải có một quá trình lật vòng, tiêu tốn năng lượng để đạt được xen phủ
như yêu cầu.
2) Như đã nói ở trên, phản ứng SN2 có sự khống chế lập thể, đòi hỏi sự tấn công
từ phía sau: Nu → σ*C-O.
Sự tấn công diaxial (lưỡng trục) diễn Sự tấn công equatorial (biên) đòi hỏi
ra với sự xen phủ qua trạng thái phải có sự lật vòng tiêu tốn năng
chuyển tiếp dạng ghế. lượng và qua một trạng thái chuyển
tiếp không phải dạng ghế.

406 | Câu hỏi lí thuyết Hóa học hữu cơ OlympiaVN


Bài 15
a) Giải thích tại sao 4 – tert – butylxiclohexen được hình thành từ cis – 4 –
brom - tert – butylxiclohexan nhanh hơn nhiều so với đồng phân trans
của nó ?
b) Giải thích tại sao khi tách 2-flobutan bằng EtO- thì sản phẩm thu được
đến 81% là but-1-en ?
c) MTBE (metyl – tert – butylete) là chất được sử dụng để thay thế tetraetyl
chì làm chất chống kích nổ cho xăng (xăng không pha chì). Người ta điều
chế nó theo hai cách sau: Cho t-BuOK tác dụng với CH3Br và cho
CH3ONa tác dụng với t-BuCl. Trong hai cách đó thì cách nào dẫn đến
sản phẩm ? Giải thích.
d) Sắp xếp các chất sau đây theo khả năng tăng dần sự tách E2, giải thích.

Hướng dẫn
1. Do nhóm t-Bu đã ở vị trí e nên trong đồng phân cis thì nhóm brom sẽ ở a,
thêm vào đó với sự xuất hiện H axial ở C2 (và C6) nên sự tách trở nên dễ
dàng, còn trong đồng phân trans thì nhóm brom ở e nên khó tách.
2. Ở đây phải giải thích bằng yếu tố tính axit của nguyên tử H kế cận, nguyên
tử H của nhóm CH3 có tính axit hơn nguyên tử H của nhóm CH2 nên cho sản
phẩm Hofmann là chủ yếu.
3. Con đường cho muối của ancol bậc 3 tác dụng với dẫn xuất bậc một, vì con
đường còn lại sẽ dẫn đến phản ứng tách E.
4. Đầu tiên sẽ phải xác định sự tách ở đây là quá trình tách syn, do trong hệ
thống bixiclo vòng nhỏ cứng nhắc sẽ không có chỗ cho sự tách anti. Do cơ
chế là tách syn nên thứ tự phản ứng sẽ như sau: 2 > 1 > 3. Sự tách ở 1 ít ưu
tiên hơn 2 do ở 1 có tương tác không gian cao giữa hai nhóm Cl làm cho góc
tách syn không thể là 0o lý tưởng như 2, mặc dù cả 1 lẫn 2 đều có H syn

407 | Câu hỏi lí thuyết Hóa học hữu cơ OlympiaVN


Bài 16
1. Norlupinan có cấu trúc như sau:

Thực hiện sự tách Hofmann hợp chất này ba lần liên tiếp, và cứ sau mỗi
lần tách lại hydro hóa sản phẩm thu được.
Cho biết cấu trúc sản phẩm cuối
2.
a) Khi đun ancol neopentylic trong axit thu được hai anken A và B có cùng
công thức C5H10 theo tỉ lệ 85/15. Cho biết anken nào là sản phẩm chính
và giải thích bằng cơ chế.
b) Neopentyl bromua có thể tách E2 được không ?
c) Khi đun neopentyl bromua trong ancol loãng thì thu được hỗn hợp anken
tương tự ở câu a. Hãy đề nghị một cơ chế phù hợp cho trường hợp này và
giải thích tại sao phản ứng không cần có sự xúc tác axit?
Hướng dẫn
1. Cấu trúc sản phẩm cuối: n-nonan

2.
a) Cơ chế giải thích sự hình thành sản phẩm cuối như sau, sự tách tuân theo
Zaitsev

408 | Câu hỏi lí thuyết Hóa học hữu cơ OlympiaVN


b) Neopentyl bromua không tách E2 được do không còn H beta bên cạnh để
tách.
c) Phản ứng xảy ra với bước đầu là sự phân cắt dị ly liên kết C-Br rồi
cacbocation bậc 1 trung gian chuyển vị thành cacbocation bền hơn. Mà như đã
biết cacbocation bền trong dung môi có tính phân cực cao (như etanol), mặt khác
Br là nhóm đi ra tốt hơn nhiều so với OH (phải dùng axit để chuyển OH thành
OH2+ đi ra dễ) nên cơ chế tạo cacbocation này xảy ra được mà không cần đền axit
như ancol neopentylic.

409 | Câu hỏi lí thuyết Hóa học hữu cơ OlympiaVN


Bài 17
Xác định sản phẩm hữu cơ của các phản ứng sau và giải thích bằng cơ chế phản
ứng

410 | Câu hỏi lí thuyết Hóa học hữu cơ OlympiaVN


Hướng dẫn

411 | Câu hỏi lí thuyết Hóa học hữu cơ OlympiaVN


Bài 18
Giải thích những vấn đề sau đây
1. Benzendiazoni clorua cho phản ứng tạo azo với phenol nhưng không cho
phản ứng với anisol có hoạt tính kém hơn. Trái lại 2,4-
dinitrobenzendiazoni clorua lại cho phản ứng được với cả phenol và
anisol.
2. Các giá trị tốc độ phần xác định được khi nitro hóa t-butylbenzen là fo =
4,5 ; fm = 3,0 và fp = 75. Xác định xem t-butylbenzen phản ứng nhanh
hơn benzen bao nhiêu lần và tính % từng sản phẩm nitro hóa.
3. N,N-dimetylanilin là hợp chất thơm có hoạt tính cao và dễ bị tấn công
bởi tác nhân electrophin yếu như cation diazoni, ion nitrosyl. Giải thích
tại sao khả năng phản ứng lại giảm mạnh khi đưa thêm nhóm ankyl vào
ortho.
4. Khi tiến hành phản ứng Friedel – Crafts giữa naphtalen và axetyl clorua
trong CS2 thu được1-axetylnaphtalen còn nếu dùng dung môi là
nitrobenzen sản phẩm lại là 2-axetylnaphtalen
Hướng dẫn
Giải thích đúng mỗi vấn đề được
1. Cation diazoni là tác nhân electrophin yếu do điện tích dương trên N được
giải tỏa bởi vòng thơm nên làm giảm hoạt tính, vì thế nó chỉ có thể phản ứng
SE với các vòng đã được hoạt hóa mạnh.
Việc gắn các nhóm hút electron vào nhân thơm (đặc biệt ở o và p) làm tăng
điện tích dương trên nguyên tử nitơ và chính điều đó làm tăng hoạt tính với
tác nhân. electrophin nên khả năng thế sẽ tăng lên
2. 15 lần, 10% o , 6,67% m và 83,33 % p
3. Tương tác đẩy giữa nhóm metyl và nhóm NMe2 làm mất đi sự liên hợp giữa
cặp e trên N vào vòng thơm nên khả năng thế giảm.
4. Dung môi nitrobenzen có kích thước lớn nên khi solvat hóa sẽ tạo tương tác
không gian với H ở vị trí 8 (peri) làm giảm tính bền của phức sigma ở 1.

412 | Câu hỏi lí thuyết Hóa học hữu cơ OlympiaVN


Bài 19
Đề nghị sản phẩm các phản ứng sau và giải thích bằng cơ chế phản ứng

413 | Câu hỏi lí thuyết Hóa học hữu cơ OlympiaVN


Hướng dẫn

Chấp nhận sản phẩm hai lần thế

414 | Câu hỏi lí thuyết Hóa học hữu cơ OlympiaVN


Bài 20
Giải thích các quan sát sau:
a) Iodine là một xúc tác cho phản ứng bromine hóa nhân thơm.
Hướng dẫn
Iodine được sử dụng làm xúc tác cho phản ứng bromine. Nó hoạt động bằng cách
tạo thành iodine bromide, tiểu phân này tạo điều kiện cho sự bromine hóa bằng
cách loại bromide ion ra ở dạng IBr2-, do đó làm tăng tốc độ giai đoạn tốc định
trong phản ứng.

b) Sản phẩm sulfonic hóa naphthalene phụ thuộc vào nhiệt độ phản ứng.
Hướng dẫn
(Các) sản phẩm sulfonic hóa phụ thuộc vào nhiệt độ bởi phản ứng có tính thuận
nghịch. Sản phẩm chịu sự khống chế động học là naphthalene-l-sulfonic acid,
nhưng sản phẩm ưu tiên nhiệt động hơn là naphthalene-2-sulfonic acid.
Trong mỗi trạng thái chuyển tiếp, điện tích dương đều được giải tỏa mở rộng hơn
so với trong phản ứng của benzene, dẫn đến năng lượng hoạt hóa thấp hơn. Do
sự thế ở vị trí 1 có hai cấu trúc đóng góp năng lượng thấp benzenoid (có vòng
benzene) còn vị trí 2 chỉ có một cấu trúc, nên có thể hiểu được rằng vị trí 1 dễ
phản ứng hơn và là sản phẩm khống chế động học. Ở nhiệt độ thấp, sản phẩm này
chiếm ưu thế:

Sự va chạm không gian giữa nhóm sulfonic acid với peri-hydrogen khiến sản
phẩm khống chế động học kém bền nhiệt động hơn 2-sulfonic acid:

415 | Câu hỏi lí thuyết Hóa học hữu cơ OlympiaVN


Sự sulfonic hóa khác biệt so với các phản ứng thế electrophile khác ở chỗ có tính
thuận nghịch; khi nhiệt độ hoặc thời gian tăng lên thì sản phẩm 2-sulfonic acid
bền nhiệt động hơn sẽ chiếm ưu thế. Cơ chế của phản ứng desulfonic hóa được
biểu diễn dưới đây cho benzene sulfonic acid:

c) 2,6-dimethylcetanilide chủ yếu bị nitro hóa ở vị số 3.


Hướng dẫn
Thoạt nhìn qua thì có thể dự đoán rằng sự nitro hóa 2,6-dimethylcetanilide sẽ diễn
ra ở vị trí 4, do trung gian Wheland có khả năng được bền hóa nhờ sự đóng góp
từ cấu trúc chính tắc:

Tuy nhiên, có hai yếu tố ngăn trở hướng phản ứng này. Thứ nhất là cặp electron
chưa liên kết trên amide nitrogen liên hợp với carbonyl oxygen. Thứ hai là sự va
chạm của các nhóm thế trên nitrogen với các nhóm ortho methyl khiến cho phân
tử bị bẻ cong nên cặp electron chưa liên kết trên nitrogen không xen phủ với hệ π
của vòng thơm và không thể bền hóa trạng thái chuyển tiếp cho sự thế vị trí 4. Do
đó, sự thế diễn ra ở vị trí ortho so với methyl và meta so với amide:

416 | Câu hỏi lí thuyết Hóa học hữu cơ OlympiaVN


d) Pyrrole dễ phản ứng với các electrophile ở vị trí 2 hơn là vị trí 3, trong khi đó
với indole thì ngược lại.
Hướng dẫn
Trung gian Wheland cho sự thế vị trí 2 của pyrrole có ba cấu trúc cộng hưởng
chính tắc hợp lí, trong khi đó sự thế vị trí 3 chỉ có hai. Do đó, hướng đầu tiên ưu
tiên hơn. Phản ứng này chịu sự khống chế động học.

Ngược lại, đóng góp bền nhất cho trung gian Wheland của phản ứng thế
electrophile của indole ở C3 vẫn giữ được tính thơm của vòng aryl, trong khi đó
sự thế C2 thì không:

e) Sự nitro hóa dimetylanilne chủ yếu tạo thành dẫn xuất m-nitro khi sử dụng
nitric và sulfuric acid đặc, trong khi với các điều kiện ít tính acid hơn thì chủ
yếu tạo thành các dẫn xuất o- và p-nitro.
Hướng dẫn
Khi có acid đặc, nitrogen của dimethylaniline bị proton hóa và trở thành nhóm
hút electron, định hướng nitro hóa vị trí meta. Khi không có acid đặc, thì lại định
hướng ortho va para.

417 | Câu hỏi lí thuyết Hóa học hữu cơ OlympiaVN


418 | Câu hỏi lí thuyết Hóa học hữu cơ OlympiaVN
Bài 21
Dự đoán các phản ứng thế nucleophile của nhóm carbonyl sau có xảy ra được hay
không:

Hướng dẫn
a, c, d) Phản ứng thất bại bởi nhóm mới đưa vào là nhóm rời đi tốt hơn, do đó
trung gian tứ diện tạo thành sẽ tách nhóm thế mới ra, tái tạo lại thành chất đầu.

419 | Câu hỏi lí thuyết Hóa học hữu cơ OlympiaVN


b) Phản ứng thực hiện được bởi nhóm mới đưa vào không phải nhóm rời đi tốt
hơn, do đó trung gian tứ diện sẽ tách nhóm thế cũ ra, tạo thành sản phẩm mới.

420 | Câu hỏi lí thuyết Hóa học hữu cơ OlympiaVN


Bài 22
1) Trình bày cơ chế các phản ứng sau

2) Trong nước, cyclopropanone tồn tại dưới dạng hydrate, nhưng 2-


hydroxyethanal thì không thể tồn tại dưới dạng hemiacetal.

3) Dưới đây là một phương pháp điều chế các cyanohydrin. Đề xuất cơ chế chi
tiết cho quá trình này.

4) Triketone dưới đây gọi là ninhydrin, được dùng để phát hiện các amino acid.
Trong dung dịch nước, nó tồn tại ở dạng hydrate. Nhóm chức ketone nào bị
hydrate hoá? Tại sao?

5) Trichloroethanol có thể được tổng hợp bằng phản ứng khử trực tiếp chloral
hydrate trong nước với sodium borohydride. Đề xuất cơ chế cho phản ứng
này. Chú ý rằng sodium borohydride không thay thế hydroxide từ các nguyên
tử carbon.

6) Không thể tạo ra các sản phẩm cộng từ các aldehyde đơn giản và HCl. Nếu
có thể được tạo ra thì cấu trúc của những sản phẩm đó là như thế nào? Trình
bày cơ chế tạo thành và giải thích tại sao không thể điều chế những chất này?

421 | Câu hỏi lí thuyết Hóa học hữu cơ OlympiaVN


Hướng dẫn
1) Vẽ cấu trúc của BH4- và AlH4- anion, và nhóm carbonyl được đặt ở vị trí mà 1
trong các nguyên tử hydrogen có thể chuyển đến. Để tạo thành alcohol thì sau đó
phải có sự chuyển proton: ở trường hợp đầu là từ dung môi sẵn có; còn trường
hợp sau là khi xử lí với nước.

2) Tất cả các vòng 3 cạnh đều rất căng, bởi các góc liên kết là 60o, thay vì 109o
hay 120o. Cyclopropanone khá căng bởi carbon của nhóm carbonyl (lai hoá sp2)
phải có góc liên kết xấp xỉ 120o – do vậy sức căng vòng là khoảng 60o. Trong
dạng hydrate, nguyên tử carbon sp3, nên sức căng chỉ còn khoảng 49o. Tuy không
quá nhiều, nhưng dạng hydrate bền hơn dạng ketone.
Trường hợp thứ hai thì khác hoàn toàn. Dạng hydroxy-aldehyde thì không có sức
căng vòng, nhưng dạng hemiacetal thì có sức căng 49o. Thậm chí, nếu không có
sức căng thì các dạng hydrate và hemiacetal vẫn thường kém bền hơn các
aldehyde hoặc ketone tương ứng.
3) Silyl cyanide là tác nhân electrophile, trong khi cyanide ion trong xúc tác là
nucleophile. Cyanide cộng hợp vào nhóm carbonyl, và sản phẩm oxyanion tạo
thành phản ứng tiếp với silicon, giải phóng xúc tác (cyanide ion khác).

4) Cần nhớ rằng: qá trình tạo hydrate chịu sự khống chế nhiệt động.
Hai ketone cạnh vòng benzene được bền hoá bởi sự liên hợp, còn ketone trung
tâm không những không có liên hợp mà còn bị kém bền hoá bởi hai nhóm ketone
xung quanh (các nhóm hút electron lẫn nhau) - do vậy nhóm ketone trung tâm bị
hydrate hoá.

422 | Câu hỏi lí thuyết Hóa học hữu cơ OlympiaVN


5)

6) Trước tiên, xảy ra sự proton hoá nhóm carbonyl, sau đó là cộng nucleophile
của chloride ion.

Cơ chế này không có gì sai, chỉ có điều đây là một cân bằng sẽ chuyển dịch theo
chiều dịch. Các hemiacetal kém bền bởi chúng bị phân huỷ trở lại thành hợp chất
carbonyl. Hợp chất dạng RCH(OH)Cl thậm chí còn dễ phân huỷ hơn hemiacetal
vì tạo thành ion chloride rất bền.

423 | Câu hỏi lí thuyết Hóa học hữu cơ OlympiaVN


Bài 23
Đề xuất cơ chế cho chuyển hóa sau:

Hướng dẫn

424 | Câu hỏi lí thuyết Hóa học hữu cơ OlympiaVN


Bài 24
Đề xuất cơ chế cho chuyển hóa sau:

Hướng dẫn

425 | Câu hỏi lí thuyết Hóa học hữu cơ OlympiaVN


Bài 25
Phản ứng trực tiếp tạo thành ester từ carboxylic acid (R1CO2H) và alcohol (R2OH)
xảy ra trong dung dịch acid nhưng dung dịch base thì không. Vì sao không?
Ngược lại, phản ứng trực tiếp tạo thành ester từ alcohol (R2OH) và acid anhydride
[(R1CO)2O] hoặc acid chloride (R1COCl) thường tiến hành trong các dung dịch
có tính base (có mặt các base như pyridine). Giải thích.
Hướng dẫn
Phản ứng trực tiếp tạo thành ester được thực hiện trong môi trường acid do
carboxylic acid bị proton hoá (ở nhóm carbonyl), tạo thành một tác nhân
electrophile tốt, phản ứng với alcohol. Sau đó tiểu phân trung gian tứ diện này sẽ
bị proton hoá và tách 1 phân tử nước.

Trong dung dịch base, giai đoạn đầu tiên là tách proton khỏi carboxylic acid, tạo
thành anion giải toả bền. Tác nhân nucleophile này không thể tấn công anion và
phản ứng không xảy ra tiếp được nữa.

Acid anhydride và acid chloride không có hydrogen có tính acid, do vậy alcohol
tấn công ngay vào nhóm carbonyl và base dùng để loại bỏ proton có tính acid
khỏi tiểu phân trung gian. Có thể dùng các base yếu như pyridine (pKa của acid
liên hợp khoảng 5.5).

426 | Câu hỏi lí thuyết Hóa học hữu cơ OlympiaVN


Bài 26
Trình bày cơ chế của các phản ứng sau:
a) Thủy phân xúc tác acid của một amide, RCONH2.
b) Thủy phân xúc tác base của một ester, RCO2CH3.
c) Bromine hóa xúc tác acid của acetone.
d) Tự ngưng tụ xúc tác base của ethyl acetate.
Hướng dẫn
a) Đây là một ví dụ về phản ứng thế nucleophile ở nhóm carbonyl.

b) Đây là một ví dụ về phản ứng theo cơ chế BAC2.

c) Mặc dù acetone tồn tại chủ yếu ở dạng keto nhưng có một lượng nhỏ cân bằng
là của dạng enol:

Enol có liên kết đôi giàu electron và phản ứng với bromine, tái tạo phần ketone.

427 | Câu hỏi lí thuyết Hóa học hữu cơ OlympiaVN


Đây là ví dụ về phản ứng ngưng tụ Claisen. Trong giai đoạn đầu của phản ứng,
một proton bị tách ra khỏi một phân tử ethyl acetat, tạo thành enolate ion.

Enolate ion phản ứng với một phân tử ethyl acetate nữa:

Acetoacetic ester có lực acid mạnh hơn bất kì phân tử nào trong hệ, do đó nhanh
chóng deproton hóa tạo thành muối. Điều này làm cho giai đoạn cuối của phản
ứng gần như bất thuận nghịch.

428 | Câu hỏi lí thuyết Hóa học hữu cơ OlympiaVN


Bài 27
Dự đoán sản phẩm tạo thành khi các phản ứng thủy phân sau được tiến hành trong
nước đánh dấu đồng vị 18O.
a) Thủy phân CH3COOC(CH3)3
b) Thủy phân CH3COOCH3
Hướng dẫn
a) Đây là một ví dụ về phản ứng diễn ra theo cơ chế AAL1. Do nhóm t-butyl có
thể tạo thành carbocation tương đối bền, nên quá trình diễn ra với sự phân cắt
alkyl-oxygen.

b) Sự thủy phân methyl acetate là một ví dụ về cơ chế phổ biến BAC2. 18O được
tìm thấy ở carboxylic acid ở cuối phản ứng. Hai cấu trúc của carboxylate anion là
các ion không thể phân biệt về mặt hóa học.

429 | Câu hỏi lí thuyết Hóa học hữu cơ OlympiaVN


Bài 28
Việc tạo ester của các amino acid tồn tại trong tự nhiên (có cấu trúc tổng quát
dưới đây) là rất cần thiết để giữ chúng ở dạng muối hydrochloride. Điều gì sẽ xảy
ra khi trung hoà các hợp chất này?

Hướng dẫn
Các amino acid không thường tự phản ứng bởi chúng tồn tại chủ yếu ở dạng ion
lưỡng cực. Nhưng sau khi nhóm acid bị ester hoá thì nó có tính electrophile mạnh
hơn và nhóm amio là nucleophile.

Amine của phân tử này sẽ tấn công nhóm ester của phân tử khác để tạo thành một
dimer (peptide), có thể vòng hoá tạo thành một amide kép, gọi là
diketopiperazine. Phản ứng vòng hoá này thường phản ứng dimer hoá do nó là
phản ứng nội phân tử, tạo thành một vòng 6 cạnh bền.

430 | Câu hỏi lí thuyết Hóa học hữu cơ OlympiaVN


Bài 29
Có thể tạo ra diester hoặc monoester của butanedioic acid (succinic acid) từ
anhydride vòng dưới đây. Giải thích tại sao một phương pháp tạo ra diester, còn
phương pháp kia là monoester.

Hướng dẫn
Trong dung dịch base, tác nhân nucleophile là methoxide ion. Nucleophile mạnh
tấn công nhóm carbonyl, tạo thành tiểu phân trung gian tứ diện có hai nhóm có
thể rời đi. Ester anion ưu tiên hơn (pKa của RCO2H khoảng 5) so với alkoxide
ion (pKa của ROH khoảng 15). Carboxylate anion này không thể bị proton hoá
trong dung dịch base, do vậy không thể bị tấn công tiếp bởi methoxide ion.

Trong dung dịch acid, cơ chế xảy ra như sau:

431 | Câu hỏi lí thuyết Hóa học hữu cơ OlympiaVN


Bài 30
Khi xử lí ketone A với methanol trong môi trường acid thì không tạo thành sản
phẩm dự đoán acetal B mà thay vào đó là một sản phẩm C khác.

a) Trình bày cơ chế chuyển hóa ketone A thành sản phẩm C.


b) Giải thích tại sao sản phẩm dự đoán B không được tạo thành.
Hướng dẫn
a)

b) Ketone A có tính thơm (chứa 6 π-electron), nhưng acetal B thì không như vậy.
Do đó chuyển hóa A thành B sẽ làm mất tính thơm, không thuận lợi về mặt năng
lượng. Thay vào đó, có thể tạo thành sản phẩm C bởi nó cũng có tính thơm.

432 | Câu hỏi lí thuyết Hóa học hữu cơ OlympiaVN


Bài 31
Đề xuất cơ chế cho chuyển hóa sau:

Hướng dẫn

433 | Câu hỏi lí thuyết Hóa học hữu cơ OlympiaVN


Bài 32
Xử lí một carboxylic acid với tác nhân Vilssmeier, tạo thành một acid chloride.
Đề xuất cơ chế phản ứng cho chuyển hóa này:

Hướng dẫn

434 | Câu hỏi lí thuyết Hóa học hữu cơ OlympiaVN


Bài 33
Trình bày cơ chế phản ứng sau:

Hướng dẫn

435 | Câu hỏi lí thuyết Hóa học hữu cơ OlympiaVN


Bài 34
Như bạn đã biết, khi xử lí một nitrile với dung dịch acid sẽ dẫn đến sự thủy phân
tạo thành một carboxylic acid. Tuy nhiên, nếu nitrile được xử lí với acid khan và
một alcohol thì thay vào đó, một orthoester sẽ được tạo thành. Hãy đề xuất cơ chế
của chuyển hóa này.

Hướng dẫn

436 | Câu hỏi lí thuyết Hóa học hữu cơ OlympiaVN


Bài 35
Đề xuất cơ chế của phản ứng sau:

Hướng dẫn

437 | Câu hỏi lí thuyết Hóa học hữu cơ OlympiaVN


Bài 36
a) Phản ứng Ritter là một phương pháp thú vị để tổng hợp các amide. Hãy trình
bày cơ chế của phản ứng Ritter được cho dưới đây.

b) Phetermine là một chất ức chế sự thèm ăn, được dùng để giúp các bệnh nhân
béo phì giẩm cân. Hãy trình bày một quy trình tổng hợp chất tương tự
phetermine, sử dụng các nguyên liệu đầu cho trước dưới đây. Trong tổng hợp
của bạn, phản ứng Ritter sẽ đóng vai trò giai đoạn then chốt.
Nguyên liệu đầu Sản phẩm mong muốn

+ bất kì các tác nhân vô cơ, hữu cơ nào Chất tương tự phetermine
khác
Hướng dẫn
a)

b)

438 | Câu hỏi lí thuyết Hóa học hữu cơ OlympiaVN


439 | Câu hỏi lí thuyết Hóa học hữu cơ OlympiaVN
Bài 37
Đề xuất cơ chế cho phản ứng sau:

Hướng dẫn

440 | Câu hỏi lí thuyết Hóa học hữu cơ OlympiaVN


Bài 38
Đề xuất cơ chế cho các phản ứng sau, giải thích sự thành các sản phẩm.

Hướng dẫn
Acid chloride phản ứng nhanh với nước, sau đó carboxylic acid tạo thành phản
ứng nhanh với phân tử acid chloride thứ hai. Anhydride tạo thành phản ứng với
nước chậm hơn nhiều (pKa của HCl khoảng -7, còn pKa của RCOOH khoảng 5),
đặc biệt là với nồng độ nước trong acetone thấp, và phản ứng sẽ dừng lại.

Phản ứng thứ hai là ví dụ về sự thuỷ phân amide trong dung dịch kiềm. Mặc dù
nguyên tử nitrogen không phải nhóm rời đi tốt, nhưng nó có thể rời đi từ dianion
và sau đó bị proton hoá nhanh chóng.

441 | Câu hỏi lí thuyết Hóa học hữu cơ OlympiaVN


Bài 39
Một trong những phản ứng quan trọng nhất trong hóa hữu cơ là phản ứng của
nhóm carbonyl với một tác nhân nucleophile như minh hoạt dưới đây. Bạn có thể
suy luận chi tiết về chuyển hóa này dựa vào những hiểu biết về lí thuyết FMO:

a) Xác định các HOMO và LUMO tương ứng với quá trình này.
b) Vẽ giản đồ năng lượng bao gồm tất cả các orbital của mỗi tác nhân phản ứng.
Xác định rõ mỗi orbital (ví dụ σC-R) và điền các electron vào mỗi orbital.
c) Hãy phác thảo LUMO và chỉ ra nucleophile sẽ được định hướng tehes nào
khi tiếp cận với nhóm carbonyl trong hợp chất dưới đây.

Hướng dẫn
a)

b)

c) Nucleophile sẽ tấn công orbital π*C=O như trong minh hoạt dưới đây (bên
trái). Thực tế, orbital phản liên kết bị lệch (nghiêng) một chút khỏi góc 90o
và góc tiếp cận thực tế là khoảng 107o.

442 | Câu hỏi lí thuyết Hóa học hữu cơ OlympiaVN


443 | Câu hỏi lí thuyết Hóa học hữu cơ OlympiaVN
Bài 40
Cyanide là một nucleophile tốt, có thể cộng hợp trực tiếp [direct addition] vào
nguyên tử carbon có tính electrophile của nhóm carbonyl. Cyanide cũng có thể
thực hiện phản ứng cộng liên hợp [conjugate addition] - cộng hợp vào nguyên tử
β của một hợp chất α,β-carbonyl không no như A. Hai kiểu phản ứng được biểu
diễn dưới đây:

a) Đề xuất cơ chế cho mỗi phản ứng.


b) Ngay sau khi trộn A với cyanide ở pH 9, thì hợp chất B là cấu tử hữu cơ chính
trong hỗn hợp phản ứng. Sau một thời gian, nồng độ chất C tăng từ từ cho
đến khi nó trở thành hợp chất hữu cơ duy nhất còn hiện diện. Hãy giải thích
các quan sát này

c) Sử dụng các trục tọa độ được cho ở trên, hãy vẽ giản đồ năng lượng của hai
phản ứng với chất đầu được biểu diễn ở giữa. Chỉ rõ tương quan năng lượng
của chất đầu, các trạng thái chuyển tiếp, các trung gian và các sản phẩm.

444 | Câu hỏi lí thuyết Hóa học hữu cơ OlympiaVN


Hướng dẫn
a)

b) Hợp chất B thể hiện là sản phẩm khống chế động học [kinetic control], được
tạo thành nhanh hơn, còn hợp chất C là sản phẩm khống chế nhiệt động học
[thermodynamic control], bền hơn, năng lượng thấp hơn. Mặc dù sản phẩm động
học B được tạo thành nhanh hơn (năng lượng hoạt hóa thấp hơn), nhưng nó được
tạo thành thuận nghịch. Với năng lượng đủ để sản phẩm động học tái tạo trở lại
thành chất đầu A, một số sản phẩm nhiệt động học sẽ được tạo thành. Sau một
thời gian, sản phẩm nhiệt động học sẽ chiếm ưu thế; năng lượng hoạt hóa để sản
phẩm này chuyển trở lại thành chất đầu là quá lớn dưới những điều kiện phản ứng
thông thường.

445 | Câu hỏi lí thuyết Hóa học hữu cơ OlympiaVN


c) Giản đồ năng lượng:

446 | Câu hỏi lí thuyết Hóa học hữu cơ OlympiaVN


Giải thích cơ chế phản ứng 2
Bài 1
1) Giải thích tại sao (1S,3R)-3-tert-butylcyclohexyl tosylate trải qua phản ứng
tách E2 với potassium tert-butoxide rất chậm, trong khi (1R,3R) thì phản ứng
nhanh hơn rất nhiều.
2) Phản ứng của cis-2-pentene với iodine azide (IN3) trong dichloromethane tạo
thành (2S,3S)-3-azido-2-iodopentane và (2R,3R) -3-azido-2-iodopentane
nhưng không tạo thành bất kì đồng phân dia nào khác. Cho biết hóa lập thể
của phản ứng cộng và trình bày cơ chế để giải thích.
3) Phản ứng của trans-2-hexene với dung dịch peracetic acid tạo ra (2S,3R)- 2,3-
hexane diol và (2R,3S)-2,3-hexanediol nhưng không tạo thành bất kì đồng
phân dia nào khác. Cho biết hóa lập thể của phản ứng cộng.
4) Đun nóng (2S)-3-methyl-3-phenyl-2-butyl tosylate trong ethanol dẫn đến sự
chuyển vị khung carbon và tạo thành (3S)-2-ethoxy-2-methyl-3-
phenylbutane. Thông tin nào nói cho bạn biết về tiến trình hóa lập thể của
chuyển vị này.
5) Xử lí trans-2-phenylcyclohexyl tosylate với potassium tertbutoxide thì quá
trình xảy ra khá chậm, chủ yếu tạo thành 3-phenylcyclohexene, trong khi đó,
trong cùng điều kiện thì cis-2-phenylcyclohexyl tosylate tạo thành 1-
phenylcyclohexene với thời gian phản ứng nhanh. Giải thích ngắn gọn sự
khác biệt này.
Hướng dẫn
1)

447 | Câu hỏi lí thuyết Hóa học hữu cơ OlympiaVN


2)

3)

4)

448 | Câu hỏi lí thuyết Hóa học hữu cơ OlympiaVN


5)

449 | Câu hỏi lí thuyết Hóa học hữu cơ OlympiaVN


Bài 2
1) Phản ứng cộng osmium tetroxide vào norbornene 2, sau đó tách khử bởi
sodium sulfite tạo thành exo, exo diol 3. Tiến hành chuỗi phản ứng tương tự
với 7,7-dimethylnorbornene 4 lại tạo thành endo, endo diol 5. Từ các kết quả
này, hãy suy ra cơ chế phản ứng cộng và sự chọn lọc mặt phản ứng của hai
chất đầu.

2) Xử lí E-1-phenyl-2-butene 5 với I2 và silver benzoate (1:2) sau đó xà phòng


hóa tạo thành hỗn hợp đẳng mol của [2S,3R]-1-phenyl2,3-butanediol và
[2R,3S]-1-phenyl-2,3-butanediol. Xử lí 5 với I2 và silver acetate khi có mặt
nước, sau đó xà phòng hóa tạo thành hỗn hợp đẳng mol của [2R,3R]-1-
phenyl-2,3-butanediol và [2S,3S]-1-phenyl-2,3-butanediol. Xác định hóa lập
thể của các quá trình này. Giải thích sự khác nhau về cơ chế phản ứng.

3) Xử lí [4S]-4-t-butyl-1-methylcyclohexene với borane-THF, sau đó oxid hóa


với H2O2-NaOH tạo thành hỗn hợp [1S,2S,5S]-5-t-butyl-2-
methylcyclohexan-1-ol và [1R,2R,5S]-5-t-butyl-2-methylcyclohexan-1-ol và
không có các đồng phân dia khác. Có hai bước trong quá trình: cộng để tạo
thành cơ borane và oxid hóa để phân cắt liên kết carbon-boron thành alcohol.
a) Đây có phải quá trình đặc trưng lập thể toàn phần không?
b) Chỉ ra sự chọn lọc lập thể của mỗi giai đoạn phù hợp với sản phẩm quan
sát thấy.
c) Biết rằng phản ứng oxid hóa diễn ra với sự bảo toàn cấu hình. Vậy hóa
lập thể của giai đoạn hydroborate hóa là gì?

450 | Câu hỏi lí thuyết Hóa học hữu cơ OlympiaVN


4) Xử lí [2R,3R]-2,3-dibromo-3-methylpentane với Zn tạo ra (Z)-3-methyl-2-
pentene là sản phẩm duy nhất. Xác định hóa lập thể của phản ứng khử. Dựa
vào kết quả này dự đoán có thể sử dụng phản ứng của một trans olefin với
Br2, sau đó với Zn như một cách để chuyển trans thành cis olefin không?
Hướng dẫn
1) Thực tế rằng chỉ sản phẩm cis được tạo thành từ 2 hoặc 4, nghĩa là cả hai
oxygen trong sản phẩm đều đến từ tác nhân osmium. Đây có thể là kết quả
của một phản ứng cộng [3+2] tiếp hợp của OsO4 vào liên kết đôi hoặc một
chuỗi cộng lần lượt một oxygen sau đó đến một oxygen khác vào cùng phía
của liên kết đôi. So sánh 2 và 4 thì thấy rằng ưu tiên mặt phản ứng là hướng
tấn công exo. Chỉ khi mặt exo bị cản trở bởi nhóm methyl thì tác nhân mới
tấn công mặt endo.

2) Biết rằng cuối cùng thì hai nhóm hydroxyl được thêm vào mỗi phía của liên
kết đôi, có thể dễ dàng vẽ được các đồng phân có thể có:

451 | Câu hỏi lí thuyết Hóa học hữu cơ OlympiaVN


Xem xét kĩ lưỡng có thể thấy rằng các cặp đồng phân 2R,3S-2S,3R và 2R,3R-
2S,3S là các cặp đối quang. Khi chúng được vẽ ra, bạn có thể thấy cấu hình
E ban đầu của liên kết đôi. Do đó, cặp đầu tiên tạo ra từ phản ứng cộng hợp
đặc trưng lập thể trans qua liên kết đôi, trong khi cặp thứ hai đến từ từ phản
ứng cộng hợp đặc trưng lập thể cis. Rõ ràng, hóa lập thể của liên kết đôi vẫn
được bảo toàn qua phản ứng cộng. Do iodine là electrophile, nên khả năng
cao là có sự tạo thanh cầu iodonium ion. Phản ứng cộng ban đầu chỉ tạo thành
một cặp đối quang do nucleophile duy nhất là benzoate. Ngoài ra cũng chú
ý rằng iodine không có mặt trong sản phẩm, do đó phải có gì đó thay thế
iodine. Silver ion có thể giúp loại bỏ iodine. Do iodine có thể gây ra phản ứng
cộng trans và sản phẩm có các oxygen được cộng trans vào, do đó phải có
một sự thay thế iodine bởi một phối tử oxygen với sự bảo toàn cấu hình của
tâm gắn iodine. Vậy phải có sự tham gia của nhóm kề.

452 | Câu hỏi lí thuyết Hóa học hữu cơ OlympiaVN


Cặp đối quang còn lại là kết quả của sự nghịch đảo cấu hình. Sự hiện diện
của nước ngăn cản nhóm benzoyl không đóng vai trò như một nhóm kề.

453 | Câu hỏi lí thuyết Hóa học hữu cơ OlympiaVN


3)

a) Từ các sản phẩm, thấy rõ rằng một hydrogen và một nhóm hydroxyl đã
được cộng hợp syn vào một trong hai mặt của olefin.
b) Do một hydrogen và một boron được cộng hợp vào trong giai đoạn đầu
tiên và sau đó phối tử boron bị chuyển thành phối tử oxygen trong giai
đoạn thứ hai, các khả năng có thể xảy ra là (a) mô hình cộng hợp syn-anti
và (b) hóa lập thể của sự phân cắt bảo toàn-nghịch đảo. Chỉ chọn mặt
dưới cho giai đoạn cộng hợp ban đầu, bốn xác suất hóa lập thể là:

c) Do sản phẩm thực tế có nhóm hydroxyl và nhóm methyl ở vị trí cis với
nhau, nên chuỗi bảo toàn syn và nghịch đảo anti là khả năng duy nhất.
Nếu biết rằng sự phân cắt diễn ra với sự bảo toàn cấu hình, thì sau đó
phản ứng hydroborate phải diễn ra bởi cộng hợp syn qua liên kết đôi.

454 | Câu hỏi lí thuyết Hóa học hữu cơ OlympiaVN


4) Từ hóa lập thể được chỉ ra thì sự khử kẽm là quá trình tách trans của hai
bromine. Do phản ứng cộng hợp bromine vào olefin là quá trình trans nên
bạn không thể sử dụng chuỗi phản ứng này để đồng phân hóa olefin.

455 | Câu hỏi lí thuyết Hóa học hữu cơ OlympiaVN


Bài 3
Dự đoán sản phẩm của các phản ứng sau:
a) Dehydrate hóa (tách nước) 1,1,3-triphenyl-3-p-chlorophenylpropan-2-ol.
b) Xử lí (I) với (i) hydroxide ion và (ii) hydrogen bromide.

c) Tách bromine từ meso-1,2-dibromo-1,2-diphenylethane bằng cách xử lí với


iodide ion.
d) Xử lí CH3CHXCH2CH3 với base, (i) khi X = Cl, (ii) khi X = N+Me.
Hướng dẫn
a) Dehydrate hóa (tách nước) 1,1,3-triphenyl-3-p-chlorophenylpropan-2-ol.

Quá trình tách nước 1,1,3-triphenyl-3-p-chlorophenylpropan-2-ol trong môi


trường acid sẽ có sự tách proton ở H’ hoặc H’’. Do nhóm thế chlorine có hiệu
ứng hút electron với vòng phenyl nên H’ có thể có lực acid mạnh hơn một chút
so với H’’, dẫn đến sản phẩm chính là hỗn hợp đồng phân hình học. Sự mất proton
ở H’’ tạo thành sản phẩm phụ.

456 | Câu hỏi lí thuyết Hóa học hữu cơ OlympiaVN


b) Xử lí (I) với (i) hydroxide ion và (ii) hydrogen bromide.
Khi xử lí với hydroxide, một proton bị loại khỏi nhóm hydroxyl. Phản ứng thế
nội phân tử của bromide diễn ra với tốc độ nhanh hơn phản ứng thế trực tiếp bởi
hydroxide. Dạng hình học anti-periplanar6 của phản ứng tạo thành sản phẩm cụ
thể như bên dưới. Trong acid, phản ứng E2 diễn ra (tách nước). Dạng hình học
antiperiplanar của phản ứng quyết định dạng hình học của sản phẩm alkene.

c) Tách bromine từ meso-1,2-dibromo-1,2-diphenylethane bằng cách xử lí với


iodide ion.

Tương tự như trên, phản ứng lại diễn ra theo dạng hình học antiperiplanar, tạo
thành sản phẩm (E)-alkene.

457 | Câu hỏi lí thuyết Hóa học hữu cơ OlympiaVN


d) Xử lí CH3CHXCH2CH3 với base, (i) khi X = Cl, (ii) khi X = N+Me.
Khi X = Cl xảy ra phản ứng E2 (theo quy tắc Zaitsev), tạo thành alkene nhiều
nhóm nhế, với đồng phân (E) (bền hơn) được ưu tiên hơn.

Khi X = N+Me3, phản ứng tách E2 diễn ra tạo thành alkene ít nhóm thế hơn:

458 | Câu hỏi lí thuyết Hóa học hữu cơ OlympiaVN


Bài 4
Giải thích các phát biểu sau:
a) Khi một mẫu 2-iodobutane quang hoạt được xử lí với iodide ion phóng xạ
trong dung dịch, tốc độ đầu của quá trình racemic hóa gấp đôi tốc độ đầu của
sự hấp thụ phóng xạ.
b) (R)-α-bromopropionic acid tạo thành (S)-lactic acid với kiềm rất đặc nhưng
(R)-lactic acid với kiềm loãng.
c) (R)-2-butyl acetate bị thủy phân trong môi trường base tạo thành (R)-2-butyl
alcohol, với hydrochloric acid thì lại thành hỗn hợp (R)- và (S)-2-butyl
chloride.
d) Phản ứng tách hydrogen chloride dựa vào base diễn ra nhanh hơn với đồng
phân cis- hơn là trans-1-chloro-4-methylcyclohexane.

e) Trong khi các cấu dạng diequatorial (lưỡng biên) và diaxial (lưỡng trục) của
trans-1,2-dimethylcyclohexane tồn tại ở tỉ lệ khoảng 99:1 tại nhiệt độ phòng
thì các cấu dạng của trans-1,2-dibromocyclohexane lại tồn tại ở lượng bằng
nhau.
Hướng dẫn
a) Khi một mẫu 2-iodobutane quang hoạt được xử lí với iodide ion phóng xạ trong
dung dịch, tốc độ đầu của quá trình racemic hóa gấp đôi tốc độ đầu của sự hấp
thụ phóng xạ.
Sự nghịch đảo cấu hình của 2-iodobutane khi có mặt iodide ion diễn ra như sau:

Tốc độ racemic hóa được định nghĩa là tốc độ tại đó hỗn hợp racemci được tạo
thành. Sự racemic hóa của một mẫu đối quang tinh khiết đòi hỏi 50 % phân tử bị
nghịch đảo cấu hình, nghĩa là sự racemic diễn ra hoàn toàn khi 50 % phân tử đã
phản ứng. Do đó, trong trường hợp này, tốc độ đầu của quá trình racemic hóa gấp
đôi tốc độ hấp thụ iodide phóng xạ. Điều này giả định là nồng độ của iodide ion
phóng xạ phải lớn hơn nhiều so với nồng độ của ion không phóng xạ và bỏ qua
sự racemic hóa thứ cấp của sản phẩm; cả hai giả định phù hợp ở thời điểm đầu
của phản ứng.
b) (R)-α-bromopropionic acid tạo thành (S)-lactic acid với kiềm rất đặc nhưng
(R)-lactic acid với kiềm loãng.
Khi có kiềm, carboxylic acid bị deproton hóa. Có hai pản ứng cạnh tranh có thể
thay thế bromine: phản ứng thế liên phân tử bởi hydroxide hoặc phản ứng thế nội

459 | Câu hỏi lí thuyết Hóa học hữu cơ OlympiaVN


phân tử bởi carboxylate. Trong dung dịch kiềm rất đặc, phản ứng thế SN2 trực
tiếp nhanh hơn, dẫn đến sự nghịch đảo hóa lập thể.

Trong kiềm loãng, phản ứng nội phân tử nhanh hơn và α-lactone được tạo thành
với sự nghịch đảo hóa lập thể. Chất trung gian này phản ứng tiếp với hydroxide
tạo dẫn đến sự nghịch đảo hóa lập thể lần nữa.

c) (R)-2-butyl acetate bị thủy phân trong môi trường base tạo thành (R)-2-butyl
alcohol, với hydrochloric acid thì lại thành hỗn hợp (R)- và (S)-2-butyl chloride.
Phản ứng thủy phân ester diễn ra theo cơ chế BAC2 và vẫn bảo toàn hóa lập thể
của tâm chiral.

Khi xử lí với hydrochloric acid, các alcohol tinh khiết quang học (homochiral)
tạo thành hỗn hợp racemic, quá trình này diễn ra qua carbocation phẳng:

Chú ý rằng trong khi 2-iodobutane phản ứng qua cơ chế SN2 thì 2-butyl alcohol
lại theo cơ chế SN1. Điều này phản ánh thực tế là cơ chế của các hợp chất bậc hai
còn tùy thuộc vào bản chất nhóm thế và điều kiện phản ứng.
d) Phản ứng tách hydrogen chloride dựa vào base diễn ra nhanh hơn với đồng
phân cis- hơn là trans-1-chloro-4-methylcyclohexane.

460 | Câu hỏi lí thuyết Hóa học hữu cơ OlympiaVN


Các phản ứng tách diễn ra theo dạng hình học ưu tiên antiperiplanar. Trong các
hệ vòng 6 cạnh, phản ứng này chỉ xảy ra nếu các nhóm thế ở vị trí lưỡng trục
(diaxial). Nhìn chung, các hệ vòng 6 cạnh có nhóm thế bền hơn khi các nhóm thế
ở vị trí biên (equatorial). Trong cấu hình cis, mỗi cấu dạng có độ bền tương đương
do đều có 1 nhóm thế vị trí trục (axial) và 1 nhóm thế vị trí biên. Tuy nhiên, trong
cấu hình trans, cấu dạng lưỡng biên (diequatorial) thuận lợi hơn cấu dạng lưỡng
trục. Do đó, trong cấu hình trans, tỉ lệ phân tử ở cấu dạng cần thiết cho phản ứng
rất nhỏ và phản ứng chậm hơn. Chú ý rằng, do có hai proton có thể được tách ra
với xác suất tương đương nhua trong mỗi trường hợp, nên sản phẩm phản ứng là
racemic.
e) Trong khi các cấu dạng diequatorial (lưỡng biên) và diaxial (lưỡng trục) của
trans-1,2-dimethylcyclohexane tồn tại ở tỉ lệ khoảng 99:1 tại nhiệt độ phòng thì
các cấu dạng của trans-1,2-dibromocyclohexane lại tồn tại ở lượng bằng nhau.

461 | Câu hỏi lí thuyết Hóa học hữu cơ OlympiaVN


Tỉ lệ của các hợp chất thế dimethyl phản ánh trạng thái ưu tiên chung cho các
nhóm thế của vòng 6 cạnh để đáp ứng được cấu dạng biên. Trong hợp chất
dibromo, trạng thái chung này bị biến đổi bởi thực tế rằng mỗi liên kết carbon-
bromine là một moment lưỡng cực. Trong cấu dạng lưỡng biên, định hướng của
các liên kết này làm cho phân tử có moment lưỡng cực toàn phần lớn và các
nguyên tử bromine đẩy lẫn nhau. Tuy nhiên, trong cấu dạng lưỡng trục, hai
moment ngược nhau (do đó giảm thiểu tối đa lực đẩy tĩnh điện trong phân tử) và
moment lưỡng cực toàn phần của phân tử bị khử.

462 | Câu hỏi lí thuyết Hóa học hữu cơ OlympiaVN


Bài 5
Ester là những hợp chất hữu cơ gần gũi với chúng ta dưới dạng dung môi hữu cơ
trong thành phần các hương liệu trái cây. Cũng có nhiều phương pháp khác nhau
đã được biết để tổng hợp chúng, phụ thuộc vào mục đích và ứng dụng. Phương
pháp đơn giản nhất là thêm một lượng nhỏ xúc tác acid như sulfuric acid vào
nguyên liệu carboxylic acid và alcohol rồi đun nóng (được gọi là “phương pháp
tổng hợp ester Fisher”).

Trong phản ứng này, các phân tử nước được tách ra từ carboxylic và alcohol để
tạo thành ester, nhưng phản ứng không thể diễn ra hoàn toàn theo chiều thuận (về
bên phải). Và khi nồng độ của các chất trong hệ (gồm cả chất đầu và sản phẩm)
đạt tới một giá trị nhất định thì sẽ không thay đổi nữa, và rõ ràng phản ứng không
tiếp diễn. VÍ dụ, trong phản ứng trong đó ethyl acetate được tạo thành từ acetic
acid và ethanol như trên thì khi trộn mỗi 1.00 mol nguyên liệu đầu rồi giữ nhiệt
độ ở 76 oC thì khi 0.660 mol ethyl acetate được tạo thành, nồng độ của mỗi cấu
tử trong hệ sẽ không thay đổi.
Trạng thái mà tại đó phản ứng không tiếp diễn được gọi là “trạng thái cân bằng”.
Khái niệm cân bằng có ý nghĩa vô cùng quan trọng trong hóa học và phương trình
dưới đây được dùng để biểu diễn trạng thái cân bằng. Trong đó, kí hiệu [ ] chỉ
nồng độ của mỗi chất, và K là “hằng số cân bằng”. Do hằng số cân bằng là hằng
số của phản ứng ở nhiệt độ nhất định nên nếu biết hằng số cân bằng thì có thể biết
nồng độ của mỗi cấu tử ở nhiệt độ đó.

1) Tính hằng số cân bằng K ở 76 oC của phản ứng trên (tới chữ số thập phân thứ
2.)
Do đó, trong phản ứng ester hóa này, các ester không thể được tạo thành hoàn
toàn, và phải thực hiện các biến đổi để làm tăng lượng ester tạo thành. Ví dụ,
trong phản ứng trên, nếu một trong các nguyên liệu thô được lấy dư thì phản ứng
sẽ diễn ra theo chiều làm giảm tác động đó (chiều thuận), do đó hiệu suất ester sẽ
tăng lên. Theo đó, khi một yếu tố trong hệ ở trạng thái cân bằng bị thay đổi thì
phản ứng sẽ diễn ra theo hướng chống lại tác động của biến đổi đó. Đây là nội
dung chính của “nguyên lí Le Chatelier”.
2) Nếu cho 1.00 mol acetic acid phản ứng với 3.00 mol ethenol trong cùng điều
kiện như trên thì có bao nhiêu mol acetate được tạo thành? Chọn giá trị gần
nhất trong các số dưới đây.
① 0.60 mol ② 0.70 mol ③ 0.80 mol

463 | Câu hỏi lí thuyết Hóa học hữu cơ OlympiaVN


④ 0.90 mol ⑤ 1.00 mol
3) Dựa vào nguyên lí Le Chatelier, hãy giải thích cân bằng sẽ thay đổi thế nào
khi nước bị loại khỏi phản ứng bằng cách thêm tác nhân hút nước vào hỗn
hợp phản ứng ester hóa.
Trong trường hợp ester hóa một carboxylic acid có cấu trúc phức tạp, những vẫn
đề như các phản ứng khác ngoài phản ứng ester hóa có thể diễn ra trong điều kiện
phản ứng trên (đun nóng khi có mặt xúc tác acid). Do đó, một phương pháp đã
được phát triển để chuyển carboxylic acid thành một trạng thái hoạt động hơn
(khả năng phản ứng cao hơn) sau đó cho phản ứng với alcohol để thực hiện phản
ứng ester hóa ở nhiệt độ gần nhiệt độ phòng. Ví dụ, khi cho thionyl chloride
(SOCl2) phản ứng với acetic acid, phản ứng sau diễn ra tạo thành acetyl chloride
(CH3COCl), trong đó nhóm hydroxyl của acetic acid đã bị thay thế bởi chlorine.
Phản ứng của acetyl chloride với alcohol khi có mặt trimethylamine
((CH3CH2)3N) nhanh chóng tạo thành ester ở nhiệt độ phòng.

4) Giải thích vai trò của triethylamine trong phản ứng này.
Trong tất cả các phản ứng ester hóa đã được mô tả cho đến lúc này, nhóm
hydroxyl trên carboxylic acid và hydrogen của alcohol đã bị tách ra và phần còn
lại liên kết với nhau (phản ứng ngưng tụ tách nước). Mặt khác, một phản ứng
ester hóa với ý tưởng hoàn toàn khác đã được phát triển. Đó là dùng phản ứng với
một tác nhân oxide và hóa một tác nhân khử trên carboxylic acid và alcohol, mỗi
tác nhân nhận 2 nguyên tử hydrogen và 1 nguyên tử oxygen, và dẫn đến phản ứng
ngưng tụ tách nước.
Có thể thấy từ ví dụ được đề cập ở trên, trong một phản ứng tách nước thông
thường, những sáng kiến - như thay đổi điều kiện phản ứng khắc nghiệt hoặc
chuyển thành dẫn xuất hoạt tính cao - là cần thiết, thì trong phương pháp này, tự
thân phản ứng là quá trình kết hợp oxid hóa-khử, và có thể tạo thành ester dưới
những điều kiện hết sức êm dịu. Phản ứng này là một phương pháp ester hóa được
giáo sư Mitsunobu Ooyu ở ĐH Aoyama Gakuin phát triển, gọi là “phản ứng
Mitsunobu”, và nó được sử dụng bởi các nhà hóa học tổng hợp hữu cơ từ khắp
nơi trên thế giới.

464 | Câu hỏi lí thuyết Hóa học hữu cơ OlympiaVN


Chú thích:
トリフェニルホスフィン = Triphenylphosphine;
トリフェニルホスフィン オキシド = Triphenylphosphine oxide;
ジエチル アゾジカルボキシラート = Diethyl azodicarboxylate;
ジエチル ヒドラジンジカルボキシラート =
Diethylhydrazinedicarboxylate
5) Trong phương trình phản ứng, hãy chỉ ra những nguyên tử bị oxid hóa và
những nguyên tử bị khử.
Bây giờ, hãy nghiên cứu về hóa học thể của các hợp chất hữu cơ một chút. Khi 1
nguyên tử carbon tạo thành liên kết đơn với 4 nguyên tử khác, nó có thể tạo thành
cấu trúc ba chiều, trong đó nguyên tử carbon được đặt ở tâm của một tứ diện và
các (nhóm) nguyên tử khác được đặt ở 4 đỉnh xung quanh. Hãy xét hợp chất CABDE
(trong đó A, B, D, E là chỉ các (nhóm) nguyên tử khác nhau chứ không phải kí
hiệu của bất kì nguyên tố cụ thể nào). Trong cách biểu diễn cấu trúc này, khi 2
liên kết C-A và C-B được đặt lên cùng một mặt phẳng (trên giấy), liên kết C-D
[nét đậm] là liên kết hướng lên trên, còn liên kết C-E [nét đứt] là liên kết hướng
xuống dưới. Theo đó, nguyên tử carbon liên kết với 4 (nhóm) nguyên tử khác
nhau là “carbon bất đối” và tạo ra 2 đồng phân quang học. Chúng có mối liên hệ
“ảnh thật” và “ảnh gương” với nhau, không thể chồng khít lên nhau và gọi là các
“đồng phân ảnh gương” (đồng phân quang học).

465 | Câu hỏi lí thuyết Hóa học hữu cơ OlympiaVN


Chú thích: 鏡像異性体同士 = Đối quang
Như biểu diễn trong hình, A- đến gần mặt dưới của tứ diện CABDE (1) tạo thành
một liên kết mới với carbon trung tâm và đồng thời. liên kết C-A bị phân cắt. Hợp
chất (2) tạo thành là một đối quang của hợp chất (1). Điều này sẽ được thấy rõ
khi thử lật ngược cấu trúc ba chiều của (2). Khi liên kết C-A mới được tạo thành
thì 3 liên kết còn lại bị nghịch đảo (“sự nghịch đảo cấu hình”), như khi một chiếc
ô lật ngược trong gió mạnh. Thực tế, trong phản ứng Mitsunobu, khi một alcohol
bậc 2, trong đó nhóm hydroxyl liên kết với một carbon bất đối được sử dụng, thì
sẽ quan sát được hiện tượng sự sắp xếp của các nhóm thế quanh carbon bất đối bị
nghịch đảo khi ester được tạo thành. Mối quan hệ đặc biệt giữa alcohol ban đầu
và sản phẩm ester cũng là nguyên chính chính tại sao phản ứng Mitsunobu rất
hữu ích và được sử dụng rộng rãi.
6) Đồng phân lập thể ester nào được tạo thành bởi phản ứng giữa alcohol bậc
hai có nguyên tử carbon bất đối như hình dưới với acetic acid trong điều kiện
phản ứng Mitsunobu? Xác định cấu trúc ba chiều của sản phẩm.

Chú thích: トリフェニルホスフィン= Triphenyl phosphine; ジエチル アゾ


ジカルボキシラート = diethyl azodicarboxylate
Cuối cùng, hãy xét đến quá trình phân cắt và tạo thành liên kết của các phân tử
trong phản ứng Mitsunobu. Đặc trưng nghịch đảo hóa lập thể của phản ứng
Mitsunobu thực sự đã đưa ra một gợi ý lớn (thậm chí là bằng chứng) khi xem xét
liên kết nào bị phá vỡ, và liên kết nào được tạo thành. Hãy thử sức với câu hỏi
cuối.
7) Trong phản ứng Mitsunobu dưới đây, nghiên tử oxygen nào bị được coi là bị
loại bỏ? Hãy khoanh tròn nguyên tử đó. Giải thích ngắn gọn.

466 | Câu hỏi lí thuyết Hóa học hữu cơ OlympiaVN


467 | Câu hỏi lí thuyết Hóa học hữu cơ OlympiaVN
Hướng dẫn
1)

2) ④
3) Khi thêm tác nhân hút nước vào, nồng độ H2O trong hỗn hợp phản ứng giảm.
Để chống lại tác động này, phản ứng diễn ra theo chiều thuận và qua đó làm tăng
nồng độ ester (tăng hiệu suất).
4) Trong phản ứng này, hydrogen chloride được tạo thành cùng với ester, do đó
cần sử dụng base để giữ sản phẩm phụ này, ngăn cho dung dịch phản ứng có tính
acid.
5) P bị oxid hóa, N bị khử.
6)

7)

468 | Câu hỏi lí thuyết Hóa học hữu cơ OlympiaVN


Cơ chế phản ứng

469 | Câu hỏi lí thuyết Hóa học hữu cơ OlympiaVN


Bài 6
1) Những sản phẩm nào được tạo thành khi xử lí:
a) Meleic acid
b) Fumaric acid
Với bromine ở nhiệt độ thấp và không chiếu sáng. Giải thích qua cơ chế phản
ứng.
2) Vẽ các công thức chiếu Haworth của các sản phẩm tạo thành từ phản ứng
cộng hợp của bromine vào (R)-4-chlorocyclohex-1-ene.
3) Vẽ cấu trúc các sản phẩm tạo thành khi obtainedwhen (1S,2R)-1-bromo-2-
fluoro-1,2-diphenylethane trải qua sự tách β của HBr. Sản phẩm nào được tạo
thành khi chất phản ứng là đồng phân R,R hoặc S,S?
4) Xác định các sản phẩm tạo thành từ phản ứng của trans-2-
bromo-4-chlorocyclobutanone với LiAlH4 bởi sự tác kích
từ các mặt re (trên) và si (dưới)*. Suy luận cấu hình tuyệt
đối của các sản phẩm phản ứng.
(*) Các mặt re và si
của carbon lai hóa sp2.
5) Dự đoán sản phẩm tạo thành từ sự khử 2S,3R)-2,3-dichloro cyclobutanone
bởi LiAlH4 bởi sự tác kích từ mặt re.
6) Vẽ cấu trúc các sản phẩm từ phản ứng của bicyclo[2.2.2]octene với a) dung
dịch peracid và b) potassium permanganate. Cho biết sản phẩm có tính quang
hoạt hay không.
7) Sản phẩm nào được tạo thành từ phản ứng giữa benzaldehyde và butanone
khi có mặt base (NaH, nhiệt độ phòng). Vẽ cấu trúc tất cả các sản phẩm sản
phẩm ở dạng công thức chiếu Newman. Chọn ra cấu hình trong đó các nhóm
phenyl và carbonyl ở vị trí antiperplanar.
8) Dự đoán sản phẩm tạo thành khi 1-methylcyclopenta-1,3-diene phản ứng
như một diene với dienophile maleic anhydride.

470 | Câu hỏi lí thuyết Hóa học hữu cơ OlympiaVN


Hướng dẫn
1)
a) Trước tiên, maleci acid phản ứng với phân tử bromine, tách bromide ion và
tạo thành bromonium ion vòng. Ở giai đoạn thứ hai, nucleophile là bromide
ion có thể tác kích vào bromonium ion vòng. Sự tác kích này có thể diễn ra
theo cơ chế SN2 đặc trưng lập thể với sự nghịch đảo lập thể. Do xác suất tấn
công của bromide ở C2 và C3 của meso-bromonium ion bằng nhau nên sản
phẩm là hỗn hợp racemi của (2R,3R)- và (2S,3S)-dibromosuccinic acid. Chú
ý rằng phản ứng trên chỉ diễn ra trong điều kiện không chiếu sáng và nhiệt độ
thấp.

b) Tương tự:

471 | Câu hỏi lí thuyết Hóa học hữu cơ OlympiaVN


2)

3)

4) Do trans-2-bromo-4-chlorocyclobutanone có tính quang hoạt nên phải xét đến


phản ứng của cả hai đối quang. Với cả hai đối quang, hydride tác kích từ mặt re
phải dẫn đến sự tạo thành sản phẩm với cấu hình S ở C1, trong khi đó nếu sự tác
kích diễn ra ở mặt si thì nhận được sản phẩm với cấu hình S ở C1.

472 | Câu hỏi lí thuyết Hóa học hữu cơ OlympiaVN


5)

6) a) Phản ứng của bicyclo[2.2.2]octene với một peracid tạo thành một epoxide,
sản phẩm trung gian này bị thủy phân tiếp. Sự tác kích của nước diễn ra theo cơ
chế SN2. Do epoxide ban đầu có các tâm thủ tính nhưng không quang hoạt, sự tác
kích của nước có thể diễn ra với xác suất bằng nhau ở các tâm thủ tính, tạo thành
hai đối quang đồng phân trans-diol.

b) Xử lí bicyclo[2.2.2]octene với potassium permanganate thì ban đầu, cùng với


sự khử manganese, tạo thành một manganic(V) ester vòng, trung gian này sau đó
bị thủy phân tạo thành sản phẩm meso, nghĩ là cis-diol.

473 | Câu hỏi lí thuyết Hóa học hữu cơ OlympiaVN


7) Trong phản ứng aldol giữa butanone và benzaldehyde, cần chú ý rằng butanone
có thể tạo thành nhiều dạng enolate. Dưới những điều kiện phản ứng đã cho, sẽ
ưu tiên tạo thành enolate nhiều nhóm thế hơn và bền nhiệt động hơn, và nó có thể
có cấu hình E hoặc Z. Hơn nữa, sự tác kích electrophile ở nhóm aldehyde của
benzaldehyde phẳng có thể diễn ra ở cả phía re hoặc si. Do đó, có thể tạo thành
bốn sản phẩm.

8)

474 | Câu hỏi lí thuyết Hóa học hữu cơ OlympiaVN


Bài 7
Khi cho α-pinen phản ứng với bo hydrua có thể nhận được ba sản phẩm A, B và
C. Cả ba chất này khi cho phản ứng với H2O2/NaOH sau đó với tosyl clorua đều
thu được D. Lấy sản phẩm đầu tiên có phân tử khối nhỏ nhất cho phản ứng với
1-phenylxiclopenten thu được E. Cho E phản ứng với H2O2/NaOH thì thu được
F và G, cả hai chất này đều phản ứng với metyl sunfonyl clorua cho H và I. Viết
sơ đồ phản ứng và cấu tạo lập thể công thức các chất từ A đến I.
Hướng dẫn

475 | Câu hỏi lí thuyết Hóa học hữu cơ OlympiaVN


Bài 8
Cho biết sản phẩm các phản ứng sau và giải thích bằng cơ chế phản ứng (chỉ cần
chỉ rõ sự tạo sản phẩm chính, không cần quan tâm sản phẩm phụ)

Hướng dẫn

476 | Câu hỏi lí thuyết Hóa học hữu cơ OlympiaVN


477 | Câu hỏi lí thuyết Hóa học hữu cơ OlympiaVN
Bài 9
Các α-hydroxyacid có thể được điều chế từ nhiều amino acid, bằng cách xử lí
chúng với sodium nitrite trong dung dịch acid. Phản ứng diễn ra rất thuận lợi và
tạo thành sản phẩm mà trong đó cấu hình của nhóm hydroxyl được bảo toàn so
với chất đầu. Đề xuất cơ chế hoàn chỉnh để giải thích quan sát này.

Hướng dẫn
Sản phẩm thể hiện được sự bảo toàn hóa lập thể ở vị trí carbon có nhóm thế. Do
đó, cần phải diễn ra hai phản ứng SN2 ở amine nitrogen, mỗi lần đều nghịch đảo
cấu hình, để tạo thành sản phẩm với cấu hình như ban đầu.
Có ba chuyển hóa đã diễn ra:
- Chuyển amine thành diazonium, biến nó thành nhóm rời đi tốt.
- Thế SN2 của N2 bởi nhóm hydroxyl của carboxylic acid (nghịch đảo cấu
hình lần thứ nhất).

478 | Câu hỏi lí thuyết Hóa học hữu cơ OlympiaVN


Cơ chế được biểu diễn dưới đây:

479 | Câu hỏi lí thuyết Hóa học hữu cơ OlympiaVN


Bài 10
Alkyl hóa một enolate tạo thành một tâm quang hoạt mới. Với một tác nhân quang
hoạt, sản phẩm tạo thành là hỗn hợp racemic, như minh họa dưới đây.

Một enamine có thể được sử dụng như chất tổng hợp tương đương của một enolate
trong phản ứng alkyl hóa. Khi sử dụng amine quang hoạt, nó có thể khiến phản
ứng ưu tiên tạo thành một đối quang. Khi sử dụng (S,S)-2,5-dimethylpyrrolidine
để tạo enamine, cấu hình tuyệt đối của sản phẩm là R hay S?

Hướng dẫn
Giai đoạn đầu tiên tạo thành một enamine có tính nucleophile. Sự cản trở không
gian giữa mạch alkyl của ketone và methyl từ vòng pyrrolidine gây ra cân bằng
ưu tiên nhóm alkyl xuất hiện ở phía đối diện của alkene từ nitrogen.

Nhóm methyl cũng ngăn chặn sự tiếp cận của electrophile từ phía sau. Điều này
làm cho phản ứng cộng methyl từ phía trước được ưu tiên hơn.

480 | Câu hỏi lí thuyết Hóa học hữu cơ OlympiaVN


481 | Câu hỏi lí thuyết Hóa học hữu cơ OlympiaVN
Bài 11
Các enolate thường thể hiện phản ứng alkyl hóa chọn lọc đồng phân dia. Ví dụ,
hỗn hợp racemic của enolate thể hiện quá trình chọn lọc dưới đây cho sản phẩm
trans.

Sự chọn lọc này là kết quả của cấu hình nửa-ghế trở nên ổn định [bền] hơn khi
nhóm ethyl được gắn ở vị trí biên so với vòng. Cấu hình nửa-ghế sau được vẽ với
góc nhìn từ phía enolate.

Dựa vào hóa lập thể của cấu hình nửa-ghế của enolate, hãy đưa ra lời giải thích
hợp lí cho sự chọn lọc lập thể quan sát được trong sản phẩm.
Hướng dẫn
Cấu dạng bán-ghế thường được biểu diễn như một chiếc “nơ” cách điệu, tựa như
- nhưng không phải là đại diện cho - hình dạng thật của cyclohexene. Minh họa
dưới đây là một cách biểu diễn chính xác hơn của enolate nhìn từ cùng một góc,
cũng như từ phía trên mặt phẳng của phân tử.

482 | Câu hỏi lí thuyết Hóa học hữu cơ OlympiaVN


Theo các cách biểu diễn này, sẽ dễ thấy rằng carbon và hydrogen ở trên mặt phẳng
là ở phía đối diện với vòng từ enolate. Mặc dù chúng trông như thể sẽ ngăn trở
sự tiếp cận của electrophile nhưng thực tế thì hydrogen liền kề với alkene ở vị trí
“giả-axial” và gây ra nhiều hiệu ứng cản trở không gian hơn. Với đối quang này,
electrophile sẽ tấn công từ phía trên, không phải phía dưới.

483 | Câu hỏi lí thuyết Hóa học hữu cơ OlympiaVN


Bài 12
Đọc và trả lời các câu hỏi từ (110)-(133).
Trong lĩnh vực “khám phá thuốc” để chế tạo dược phẩm thì hóa học có vai trò rất
to lớn.
Trước tiên, chúng ta sẽ mô tả một loại thuốc có vòng benzene trong cấu trúc.
Aspirin (acetylsalicylic acid) là loại thuốc được biết đến rộng rãi như là tác nhân
hạ sốt, thuốc giảm nhức đầu và đau răng. Vào thời cổ đại, người Hi Lạp và Trung
Quốc đã biết chữa đau răng bằng cách sử dụng cành liễu và đến năm 1827, một
hợp chất có tên salicin (a) đã được chiết xuất từ nguồn gốc thực vật này. Salicin
bị thủy phân tạo thành salicyl alcohol và glucose. Salicyl alcohol được oxid hóa
để tổng hợp salicylic acid (b), và trong thập niên 1870, nó bắt đầu được sử dụng
làm thuốc giảm đau chống thấp khớp, tuy nhiên những tác dụng phụ gây ra rối
loạn tiêu hóa mạnh trở thành một vấn đề nghiêm trọng. F. Hoffmann, một nhà
hóa học ở thời kì này, nghĩ rằng tính acid mạnh của salicylic acid là nguyên nhân
gây ra tổn thương dạ dày nên đã tiến hành acetyl hóa nhóm hydroxyl để tổng hợp
acetylsalicylic acid, giúp giảm thiểu các tác dụng phụ. Năm 1899, thuốc aspirin
đã được giới thiệu.
Kể từ đó, aspirin đã được sản xuất bằng con đường tổng hợp hóa học. Trong điều
kiện nhiệt độ và áp suất cao, phenol (c) phản ứng với sodium hydroxide và (113)
để đưa nhóm carboxy vào vị trí ortho rồi trung hòa bằng sulfuric acid thì thu được
salicylic acid. Quá trình acetyl hóa được thực hiện bởi phản ứng giữa salicylic
acid với acetic anhydride, thu được aspirin. Ngoài aspirin còn thu được ibuprofen
và indomethacin, được biết đến như những tác nhân giảm đau có khả năng ức chế
viêm, và cấu trúc của chúng cũng có chứa các vòng benzene (hình 1).

Hình 1
Chú thích: アスピリン = Aspirin; イブプロフェン = Ibuprofen; インドメタ
シン = Indomethacin
Câu hỏi: Không có kết tủa copper(I) oxide xuất hiện khi cho “salicin” (phần gạch
chân a) vào dung dịch thuốc thử Fehling đun nóng. Xác định cấu trúc của salicin:
(110)

484 | Câu hỏi lí thuyết Hóa học hữu cơ OlympiaVN


Câu hỏi: Các hợp chất được tạo thành bởi phản ứng của “salicylic acid” (phần
gạch chân b) với sodium hydrogencarbonate được biết đến là có tác động chống
viêm. Hãy chọn phương trình phản ứng phù hợp: (111)

Câu hỏi: Phenol (phần gạch chân c) được sản xuất trong công nghiệp theo
phương pháp cumene. Chọn ra sự kết hợp các nguyên liệu thô phù hợp nhất cần
cho quá trình tổng hợp cumene và cho biết kiểu phản ứng. (112)
Nguyên liệu: a) benzene; b) propane; c) propene; d) acetone
Kiểu phản ứng: a) ngưng tụ; b) phản ứng cộng; c) dehydrogen hóa
Câu hỏi: Chọn tác nhân phù hợp với (113)
① oxygen ② hydrogen ③ carbon monoxide ④ carbon dioxide ⑤ acetic acid
⑥ acetic anhydride ⑦ ethanol ⑧ acetaldehyde
Acetaminophen là thuốc giảm đau hạ sốt nhẹ. Nó thường được sử dụng ở dạng
kết hợp, gọi là “công thức ACE” trong đó (114) và ethenzamide được thêm vào
acetaminophen. (114) đóng vai trò trợ lực cho hiệu quả của thuốc giảm đau.
Acetaminophen được tổng hợp từ phenol. Nitro hóa phenol, rồi khử 4-nitrophenol
(d) thu được 4-aminophenol, sau đó acetyl hóa với acetic anhydride (e) thu được
acetaminophen. Một phương pháp tổng hợp khác để sản xuất công nghiệp sử dụng
chuyển vị Beckmann (f) cũng đã được phát triển.

485 | Câu hỏi lí thuyết Hóa học hữu cơ OlympiaVN


Hình 2
Chú thích: アセトアミノフェ = Acetaminophen; エテンザミド =
Ethenzamide
Câu hỏi: (114) là một loại alkaloid có trong café, trà xanh, trà đen, … Xác định
tên gọi của hợp chất này.
① catechin ② catechol ③ caffeine ④ capsaicin ⑤ chitin ⑥ chitosan ⑦
glucosamine ⑧ collagen ⑨ cholesterol
Câu hỏi: Các nghiên cứu gần đây đã làm sáng tỏ rằng (114) được sinh tổng hợp
từ xanthosine trong thực vật. Xanthosine là nucleoside tạo thành từ ribose và
xanthine, cũng là purine base như adenine và guanine. Xác định cấu trúc
xanthosine. (115)

Hình 3
Câu hỏi: Biết phản ứng trong phần gạch chân d được tiến hành với hydrogen H2
sử dụng nickel làm xúc tác, hãy xác định bao nhiêu mol H2 sẽ phản ứng với 1.0
mol 4-nitrophenol. (116)
① 0.50 mol ② 1.0 mol ③ 1.5 mol ④ 2.0 mol ⑤ 2.5 mol ⑥ 3.0 mol
⑦ 3.5 mol ⑧ 4.0 mol
Kết quả phân tích nguyên tố sản phẩm phụ sinh ra bởi acetaminophen trong phản
ứng ở phần gạch chân e cho thấy có 62 % carbon và 5.7 % hydrogen. Ngay cả khi
thêm bột tẩy trắng vào cũng không có phản ứng màu xảy ra. Xác định cấu trúc
phù hợp của sản phẩm phụ này (117).

486 | Câu hỏi lí thuyết Hóa học hữu cơ OlympiaVN


Hình 4
Phản ứng chuyển vị Beckmann trong phần gạch chân f là một phản ứng để điều
chế amide từ ketone oxim và cũng được sử dụng trong tổng hợp ε-caprolactam là
nguyên liệu của nylon 6 như trong phản ứng (a) dưới đây (ベックマン転位 =
chuyển vị Beckmann)

Câu hỏi: Xác định nguyên liệu đầu để tổng hợp acetominophen bằng chuyển vị
Beckmann. (118)

Hình 5
Tiếp theo, hãy xem xét chiến lược tổng hợp các hợp chất trong khám phá thuốc.
Ở giai đoạn thiêst kế và tổng hợp các hợp chất tiềm năng cho những loại thuốc
mới, việc phân tích tổng hợp ngược - bằng cách đảo ngược quy trình phản ứng để
tổng hợp hợp chất mục tiêu - là rất quan trọng để xác định nguyên liệu đầu và các
phương pháp tổng hợp. Xét một phân tích tổng hợp ngược từ hai góc nhìn: (1)
chuyển hóa, đưa vào và loại bỏ nhóm chức; và (2) tạo liên kết carbon-carbon.
a) Chuyển hóa, đưa vào và loại bỏ nhóm chức
Khi tổng hợp nitrobenzene, có thể tiến hành phản ứng giữa benzene với hỗn hợp
sulfuric acid và nitric acid để nitro hóa. Tuy nhiên, khi tổng hợp aniline, rất khó
để trực tiếp đưa nhóm amino vào vòng benzene, do đó nó được tổng hợp qua
nhiều bước: nitro hóa, sau đó khử hóa.
Trong trường hợp đưa một nhóm thế thứ hai vào benzene một lần thế thì thế thì
nếu nhóm thế thứ nhất là nhóm đẩy (có tính chất nhường electron) thì nó thường
định hướng ortho-para, còn nếu nhóm thế thứ hai là nhóm hút (có tính chất nhận
electron) thì nó định hướng meta. Ví dụ, trong trường hợp tổng hợp 2,4,6-

487 | Câu hỏi lí thuyết Hóa học hữu cơ OlympiaVN


trinotrobenzoic acid, thay vì nitro hóa trực tiếp nitrobenzoic acid thì có thể tạo ra
sản phẩm cuối bằng cách oxid hóa nhóm methyl sau khi nitro hóa toluene.
Câu hỏi: Xác định sản phẩm chính A được tạo thành trong chuỗi chuyển hóa sau:
(119)

Hình 6
b) Tạo liên kết carbon-carbon
Các hợp chất thú vị trong khám phá thuốc thường là những phân tử lớn. Trong
trường hợp này, có thể tổng hợp các phân tử lớn bằng cách gắn kết các phân tử
nhỏ bởi các phản ứng tạo liên kết carbon-carbon. Lần này, chúng ta sẽ giới thiệu
2 phản ứng tập trung vào các nhóm carbonyl.
Trước tiên là phản ứng của hợp chất carbonyl với phosphonium ylide (sản phẩm
dehydrohalogen hóa của muối phosphonium tạo thành từ phản ứng của
triphenylphosphine (C6H5)3P và alkyl halide với base mạnh, có nguyên tử
phosphorus mang điện tích dương và nguyên tử carbon liên kết trực tiếp mang
điện tích âm). Đây là phản ứng Wittig, trong đó liên kết C=O bị chuyển thành liên
kết C=C.
Trong phản ứng Wittig, nguyên tử carbon mang điện tích âm của phosphonium
ylide (hình 7a) cộng hợp vào carbon của nhóm carbonyl, và nguyên tử oxygen
mang điện tích âm (của nhóm carbonyl) tạo thành từ sự cộng hợp này tấn công
vào nguyên tử phosphorus tạo thành tiểu phân trung gian vòng 4 cạnh (hình 7b)
kém bền. Sự phân hủy tiểu phân trung gian này tạo thành alkene.
Trong phản ứng này, khi có một nhóm hút electron như nhóm carbonyl liên kết
với nguyên tử carbon mang điện tích âm của phosphonium ylide thì điện tích âm
của nguyên tử carbon được giải tỏa (bền hóa), do đó nó được gọi là một ylide bền,
còn ngược lại thì gọi là ylide kém bền. Tùy thuộc vào ylide bền hay kém bền mà
sản phẩm chính là alkene dạng cis hay trans.

488 | Câu hỏi lí thuyết Hóa học hữu cơ OlympiaVN


Hình 7
Phản ứng Wittig chọn lọc đồng phân trans bởi ylide bền diễn ra kém bởi tính bền
của nó. Kết quả của nhiều nghiên cứu nhằm để cải thiện là phản ứng Horner-
Wadsworth-Emmons sử dụng phosphate ester thay vì muối phosphonium, và nó
được sử dụng rộng rãi để tổng hợp các trans alkene. Trong phản ứng này, một
ester không no α, β (ester trong đó liên kết đôi C=C liên hợp với liên kết đôi C=O)
đã được tổng hợp (hình 8).

Hình 8
Thứ hai là phản ứng Michael, là phản ứng cộng liên hợp carbanion vào hợp chất
carbonyl không no α, β. Ví dụ, xét phản ứng của ethyl acetoacetate với 3-buten-
2-one. Ethyl acetoacetate bị deproton hóa khi có mặt base tạo thành carbanion có
tính nucleophile được bền hóa bởi một nhóm carbonyl hút electron (hình 9).

Hình 9
Mặt khác, β carbon (nguyên tử carbon thứ hai so với nhóm carbonyl) của 3-
butene-2-one có điện tích dương riêng phần bởi nhóm carbonyl và liên kết đôi
C=C liên hợp tạo thành các cấu trúc cộng hưởng như sau (hình 10):

Hình 10
Nguyên tử carbon mang điện tích âm của carbanion tạo thành từ ethyl acetoacetat
được cộng hợp vào nguyên tử β carbon dương điện của 3-buten-2-one và anion

489 | Câu hỏi lí thuyết Hóa học hữu cơ OlympiaVN


mới tạo thành có điện tích âm trên nguyên tử oxygen. Anion này được bền hóa
cộng hưởng như trong hình 11 và sau khi tước proton từ dung môi và ethyl
acetoacetate, thì sản phẩm cuối được tạo thành. Phản ứng này được phát triển bởi
nhiều nhà nghiên cứu và đã được mở rộng cho các phản ứng trong đó tác nhân
nucleophile được cộng hợp vào liên kết không no thiếu electron có các nhóm thế
hút electron khác ngoài carbonyl.

Hình 11
Chú thích: 3-ブテン-2-オン = 3-butene-2-one; アセト酢酸エチル = Ethyl
acetoacetate
Câu hỏi: Để tổng hợp β-carotene cần sử dụng β-ionylideneacetaldehyde và một
hợp chất khác. Xác định hợp chất còn lại này. (120)

Hình 12a
Chú thích: β-カロテン = β-carotene; β-イオニリデンアセトアルデヒド =
β-ionylideneacetaldehyde

490 | Câu hỏi lí thuyết Hóa học hữu cơ OlympiaVN


Hình 12b
Câu hỏi: Bombycol, một pheromone của loài tằm, có thể được tổng hợp bởi 2
phản ứng Wittig, sau đó khử hóa ester bởi tác nhân khử mạnh để tạo thành alcohol
bậc một. Xác định cấu trúc của bombycol. (121)

Hình 13
Chú thích: 還 元剤 = tác nhân khử; ボンビコール = Bombycol
Câu hỏi: Xác định sản phẩm của chuỗi phản ứng sau. (122)

491 | Câu hỏi lí thuyết Hóa học hữu cơ OlympiaVN


Hình 14
Trong phần cuối của chiến lược tổng hợp, chúng ta sẽ xem xét cách thức các chiến
lược tổng hợp trong khám phá thuốc đã được mô tả được dùng trong tổng hợp
thuốc chữa cúm Tamiflu.
Tamiflu là thuốc chữa cúm có chứa thành phần hoạt tính là oseltamivir phosphate,
được sử dụng trong điều trị nhiễm virus cúm A và B vào năm 2001. Ban đầu, các
nguyên liệu của phản ứng tổng hợp chỉ được lấy từ các sản phẩm nguồn gốc thiên
nhiên, dẫn đến một vấn đề về sự ổn định của nguồn cung. Nhiều nhà nghiên cứu
đã nỗ lực thực hiện các tổng hợp toàn phần không cần đến sự trợ giúp của các quá
trình sinh học, và nhiều quy trình tổng hợp toàn phần đã được công bố từ năm
2006, và một quy trình được công bố vào năm 2009 là tổng hợp toàn phần của
Yujiro Hayashi và cộng sự (hình 15).
Trong quy trình này, các nguyên liệu đầu B và C tham gia vào phản ứng cộng
Michael, rồi phản ứng tiếp với hợp chất D theo phản ứng (1) cộng Michael; (2)
Horner-Wadsworth-Emmons tạo thành hợp chất E. Hợp chất mục tiêu oseltamivir
được tổng hợp qua 7 giai đoạn như là đưa vào nhóm bảo vệ, chuyển hóa nhóm
chức và loại nhóm bảo vệ.

492 | Câu hỏi lí thuyết Hóa học hữu cơ OlympiaVN


Hình 15
Câu hỏi: Xác định các chất đầu B, C. (123)

Hình 16
Độ tan trong nước của các hợp chất có phân tử khối thấp có thể ảnh hưởng đến
hiệu quả dược và độ an toàn của thuốc. Độ tan trong nước không chỉ bị ảnh hưởng
bởi ái lực với nước, là dung môi của hợp chất, mà còn bởi tính dễ vỡ. Do đó, cũng
có một số nghiên cứu để thay đổi tính dễ vỡ nhằm làm tăng độ tan trong nước.
Các tinh thể có xu hướng sụp đổ khi các phân tử cấu tạo thành trở thành không
phẳng hoặc bất đối xứng. Ví dụ, trong hợp chất F như trong hình 17, một nhóm
hydroxy gắn với vòng benzene, nhưng trong hợp chất G thì vị trí của nhóm
hydroxy đã thay đổi, do đó (124) diễn ra, tinh thể có khả năng sụp đổ. Ngoài ra,
khi một nhóm chức được đưa vào vị trí ortho của nhóm phenyl của hợp chất G
thì (125) xuất hiện trong tương quan về vị trí của hai vòng benzene do tương tác
với nhóm thế được đưa vào. Chính vì vậy tinh thể nhiều khả năng sẽ sụp đổ. Tinh
thể càng dễ sụp đổ thì càng dễ tan trong nước. Tính dễ vỡ cũng ảnh hưởng đến
điểm nóng chảy. Ví dụ, điểm nóng chảy của các hợp chất F và G trong hình 17
thì (126).

493 | Câu hỏi lí thuyết Hóa học hữu cơ OlympiaVN


Hình 17
Câu hỏi: Xác định các hiện tượng (124), (125).
① Giải tỏa ② Bất phẳng hóa ③ Bất đối xứng hóa ④ Không chia sẻ ⑤ Không
phân tử hóa
Câu hỏi: Xác định cụm từ thích hợp cho (126).
① F có điểm nóng chảy cao hơn. ② G có điểm nóng chảy cao hơn. ③ Điểm
nóng chảy của F và G bằng nhau. ④ Không thể đánh giá chất nào có điểm nóng
chảy cao hơn.
Tiếp theo, hãy xét các nguyên tử carbon của methane, ethylene và acetylene. Về
mặt cấu trúc, tất cả các liên kết từ các nguyên tử carbon trong phân tử ethylene
và acetylene đều thuộc cùng một mặt phẳng. Tuy nhiên, với các nguyên tử carbon
của methane, các liên kết (127). Nói cách khác, có thể thấy rằng sự tồn tại của các
nguyên tử carbon lai hóa sp3 như carbon trong methane là một chỉ số khi xét đến
tính phẳng của phân tử. Gọi tỉ lệ số nguyên tử carbon sp3 trên tổng số carbon trong
phân tử là Fsp3 thì Fsp3 của dimethylpiperidine trong hình 18 là 1.0, trong khi đó
Fsp3 của dimethylpyridine là (128). Do đó dimethylpyridine có tính phẳng cao
hơn.

Hình 18
Câu hỏi: Xác định cụm từ phù hợp cho (127)
① Giải tỏa ② Bất phẳng hóa ③ Bất đối xứng hóa ④ Không chia sẻ ⑤ Không
phân tử hóa
Câu hỏi: Xác định giá trị (128).
① 1.0 ② 0.34 ③ 0.29 ④ 0.18 ⑤ 0.060
Cuối cùng, chúng tôi giới thiệu các ví dụ về việc các tác dụng phụ đã được cải
thiện bằng cách thiết kế các phân tử thuốc có xem xét đến độ tan. Viêm mũi và

494 | Câu hỏi lí thuyết Hóa học hữu cơ OlympiaVN


ngứa dị ứng gây ra bởi sự giải phóng quá mức histamine vào máu do kích thích
của các chất dị ứng xâm nhập vào cơ thể, được kích hoạt bằng cách tạo liên kết
với một protein gọi là thụ thể histamine H1. Thụ thể histamine H1 được phân bố
rộng trong cơ thể người và khởi phát nhiều hiện tượng sinh lí liên quan đến kích
thích và tăng cường trí nhớ trong não.

Hình 19: Các kháng sinh histamine thế hệ đầu tiên.


Các chất trong hình 19 được gọi là các kháng histamine thế hệ đầu tiên và có
(129) tương tự như histamine. Thụ thể histamine H1 liên kết với chúng thay vì
histamine và vô hoạt nó để giảm bớt các chứng dị ứng. Mặt khác, chúng có tác
dụng phụ là gây buồn ngủ mạnh. Điều này là do hợp chất này là một chất có (130)
với 1 vòng benzene hoặc 1 mạch carbon, nó đi qua hàng rào máu não và cũng tác
động lên thụ thể histamine H1 trong não. Hàng rào máu não đóng vai trò như một
bộ lọc để chọn lọc các chất đi vào não và có xu hướng cho các chất có khối lượng
phân tử thấp như ethanol, caffeine hoặc có xu hướng đi qua các vật liệu (130) tốt.
Do đó, thế hệ kháng histamine thứ hai (hình 20) đã được phát triển. Đây là những
cấu trúc tương tự như kháng histamine thế hệ đầu tiên, nhóm chức có (132) như
(131) đã được đưa vào, và như dự kiến, sự dịch chuyển đến não đã ít đi và giảm
các tác dụng phụ.

Hình 20: Các kháng histamine thế hệ thứ hai.


Câu hỏi: Xác định các cụm từ thích hợp cho (129)-(132).
① nhóm amino ② nhóm carboxy ③ nhóm hydroxy ④ cầu ether ⑤ nhóm
phenyl ⑥ tính ưa nước ⑦ tính kị nước ⑧ hoạt động bề mặt ⑨ sự ăn mòn

495 | Câu hỏi lí thuyết Hóa học hữu cơ OlympiaVN


Câu hỏi: Histamine không chỉ được hấp thụ từ thực phẩm mà còn bởi amino acid
histidine trong cơ thể chuyển hóa thành bởi decarboxylase. Xác định cấu trúc của
histidine: (133)

Hình 21

496 | Câu hỏi lí thuyết Hóa học hữu cơ OlympiaVN


Hướng dẫn

497 | Câu hỏi lí thuyết Hóa học hữu cơ OlympiaVN


Bài 13
Chuyển vị Beckmann có thể diễn ra theo cơ chế từng bước hoặc cơ chế tiếp
hợp.

a) Trình bày cả hai cơ chế.


b) Giả sử bạn đã có oxime 1.
i. Nó có làm quay mặt phẳng ánh sáng phân cực không?
ii. Hãy xác định cấu hình các tâm chiral.
iii. Chỉ ra cách sử dụng 1 để phân biệt hai cơ chế bạn đã trình bày.

498 | Câu hỏi lí thuyết Hóa học hữu cơ OlympiaVN


Hướng dẫn

499 | Câu hỏi lí thuyết Hóa học hữu cơ OlympiaVN


Bài 14
Dự đoán sản phẩm của các phản ứng sau:

Hướng dẫn
Butanone có hai tâm hoạt động và có thể tạo thành carbanion bậc một hoặc bậc
hai:

Các anion này có thể trải qua các phản ứng aldol hóa. Phản ứng này dẫn đến sự
tạo thành sản phẩm chính phụ thuộc vào việc quá trình chịu sự khống chế động
học hay nhiệt động học. Enonate sau bền hơn, có thể là do sự bền hóa bởi các
nhóm methyl của cấu trúc cộng hưởng giống-alkene. Do đó, dưới những điều kiện
mà các enolate có thời gian để đạt tới cân bằng, như khi có mặt hydroxide, thì sản
phẩm của enolate sau sẽ chiếm ưu thế. Dưới những điều kiện khắc nghiệt, thường
khi đun nóng, thì các β-hydroxy ketone tạo thành sẽ bị tách nước:

Phản ứng aldol dưới các điều kiện cân bằng hiếm khi chọn lọc vùng (vị trí), và
sản phẩm của enolate kém bền hơn cũng có thể được quan sát thấy.

β-hydroxy ketone trung gian trong phản ứng sau có thể được điều chế bằng cách
thực hiện phản ứng trong các điều kiện khống chế động học: phản ứng của
butanone với LDA trong 1,2-dimethoxyethane ở nhiệt độ thấp (-78 oC) cho
enolate anion kém bền hơn nhưng dễ tạo thành hơn. Một phương pháp thay thế
khác để thực hiện phản ứng aldol hóa chọn lọc vùng là chuyển các enolate thành
các silyl ether, tách riêng các enol silyl ether đồng phân và cho chúng phản ứng

500 | Câu hỏi lí thuyết Hóa học hữu cơ OlympiaVN


riêng rẽ với aldehyde hoặc ketone, giải phóng enolate anion bởi phản ứng với
fluoride.

Hướng dẫn
Phản ứng ngưng tụ aldol hai lần giữa hai phân tử 2,3-butadienone (diacetyl) dẫn
đến sự tạo thành p-xyloquinone:

Hướng dẫn
Sơ đồ dưới đây minh họa một hướng trong đó 1 đương lượng acetone ngưng tụ
với 2 đương lượng diethyl oxalate tạo thành 4-pyrone. Đây là một trong nhiều kết
quả khả thi có thể được dự đoán hợp lí và biểu diễn sản phẩm bền nhất về mặt
nhiệt động học. Trong giai đoạn đầu tiên của phản ứng, một phân tử acetone bị
deproton hóa và phản ứng với một phân tử diethyl oxalate.

501 | Câu hỏi lí thuyết Hóa học hữu cơ OlympiaVN


Hướng dẫn
Các diester của các acid hai chức mạch ngắn không thể trải qua phản ứng
Dieckmann, nhưng có thể xảy ra phản ứng ngưng tụ liên phân tử (giữa hai phân
tử), sau đó vòng hóa tạo thành hệ cyclohexadienone.

Hướng dẫn
Acetophenone chứa các hydrogen có thể enol hóa và khi có mặt base, enolate
phản ứng với nhóm carbonyl hoạt động hơn của formaldehyde. Phản ứng nhạy
cảm với các điều kiện tiến hành và có thể tạo thành các sản phẩm cộng hoặc
ngưng tụ.

Tuy nhiên, trong thực tế, các sản phẩm thực tùy thuộc vào hệ số tỉ lượng tương
đối của các chất phản ứng. Với formaldehyde dư, có thể xảy ra phản ứng aldol
tiếp.

502 | Câu hỏi lí thuyết Hóa học hữu cơ OlympiaVN


Khi có ethoxide dư, có thể diễn ra phản ứng cộng Michael với enone. Phản ứng
ngưng tụ aldol và Michael liên tiếp tạo ra nhiều sản phẩm:

Khi có mặt acetophenone dư, có thể tạo thành 1,5-diketone:

Thậm chí, các sản phẩm phức tạp hơn như sau cũng có thể được tạo nên. Khi có
mặt lượng dư formaldehyde, có thể tạo thanh triol-ketone, và do nó không có các
hydrogen hoạt động nên nó có thể trải qua phản ứng Cannizzaro chéo tạo thành
hợp chất tetrahydroxyl:

Hướng dẫn
Anion của malonic ester được tạo ra bởi base sẽ trải qua phản ứng Michael (cộng
1,4) với acrylonitrile, diễn ra song song với phản ứng cộng 1,4- của các carbanion
vào enone:

503 | Câu hỏi lí thuyết Hóa học hữu cơ OlympiaVN


Hướng dẫn
Phản ứng nội phân tử của các dinitrile khi có mặt base gần tương tự với phản ứng
Dieckmann và đươc biết đến với tên gọi phản ứng Thorpe. Sản phẩm ban đầu, β-
imino-nitrile dễ bị thủy phân thành β-keto-nitrile.

Phản ứng thủy phân nitrile dưới những điều kiện phức tạp hơn tạo thành các amide
và carboxylic acid thế cyclopentyl.

504 | Câu hỏi lí thuyết Hóa học hữu cơ OlympiaVN


Bài 15
Trình bày cơ chế cho các chuyển hóa sau:

Hướng dẫn
a)

505 | Câu hỏi lí thuyết Hóa học hữu cơ OlympiaVN


b)

506 | Câu hỏi lí thuyết Hóa học hữu cơ OlympiaVN


Bài 16
Trình bày cơ chế cho các chuyển hóa sau:

Hướng dẫn
a)

507 | Câu hỏi lí thuyết Hóa học hữu cơ OlympiaVN


b)

508 | Câu hỏi lí thuyết Hóa học hữu cơ OlympiaVN


Bài 17
Chuyển hóa sau đây được công bố bởi nhóm nghiên cứu tại Merck Laboratories
vào năm 1939 khi tổng hợp Vitamin B6. Trình bày cơ chế của chuyển hóa này.

Hướng dẫn

509 | Câu hỏi lí thuyết Hóa học hữu cơ OlympiaVN


Bài 18
Trình bày cơ chế của chuyển hóa sau:

Hướng dẫn

510 | Câu hỏi lí thuyết Hóa học hữu cơ OlympiaVN


Bài 19
Trình bày cơ chế của chuyển hóa sau:

Hướng dẫn

511 | Câu hỏi lí thuyết Hóa học hữu cơ OlympiaVN


Bài 20
Trong quá trình tổng hợp hợp chất thiên nhiên longiborneol, phản ứng vòng hóa
sau của cyclopentenone có nhóm thế đã được công bố. Trình bày cơ chế phản ứng
của chuyển hóa này.

Hướng dẫn

512 | Câu hỏi lí thuyết Hóa học hữu cơ OlympiaVN


Bài 21
Trình bày cơ chế phản ứng của chuyển hóa sau:

Hướng dẫn

513 | Câu hỏi lí thuyết Hóa học hữu cơ OlympiaVN


Bài 22
Xác định sản phẩm của các phản ứng sau. Chỉ rõ liên kết carbon-carbon mới
nào đã được tạo thành và chỉ rõ nucleophile và electrophile.

514 | Câu hỏi lí thuyết Hóa học hữu cơ OlympiaVN


515 | Câu hỏi lí thuyết Hóa học hữu cơ OlympiaVN
516 | Câu hỏi lí thuyết Hóa học hữu cơ OlympiaVN
Hướng dẫn

517 | Câu hỏi lí thuyết Hóa học hữu cơ OlympiaVN


518 | Câu hỏi lí thuyết Hóa học hữu cơ OlympiaVN
519 | Câu hỏi lí thuyết Hóa học hữu cơ OlympiaVN
520 | Câu hỏi lí thuyết Hóa học hữu cơ OlympiaVN
Bài 23
Trình bày cách điều chế các hợp chất sau từ phenylmagnesium bromide:
a) Ph3C-OH
b) Ph2C=CH2
c) PhC(CH3)=CH2
d) PhCO2H
e) PhCH2Ph
f) PhCHO
g) PhCH2CH2OH
h) PhD (benzene được monodeuterium hóa)
Hướng dẫn
Trong phenylmagnesium bromide, nguyên tử carbon gắn trực tiếp vào nguyên tử
magnesium dương điện. Bởi vậy nên carbon bị phân cực hóa (mang điện âm) và
có thể được biểu diễn ở dạng lai hóa cộng hưởng theo hai cấu trúc sau:

Nguyên tử carbon bị phân cực hóa mang điện tích âm là tâm nucleophile mạnh
và phản ứng được với hầu hết các tâm electrophile. Do nhiều hợp chất hữu cơ có
nguyên tử carbon có tính electrophile nên đây là phương pháp phổ biến để tạo ra
các liên kết carbon-carbon mới. Trong mỗi phản ứng sau, phenylmagnesium
bromdie đóng vai trò nucleophile phản ứng với tâm electrophile.
Tác nhân Grignard thường được điều chế và sử dụng trong dung dịch ether, trong
đó chúng tồn tại ở dạng phức phối trí, monomer và dimer, ví dụ như

a) Ph3C-OH
Tâm electrophile được chọn là nguyên tử carbonyl carbon của ester. Trong giai
đoạn đầu tiên của phản ứng, nguyên tử magnesium của tác nhân Grignard tạo
phối trí với nguyên tử carbonyl oxygen của ester.

521 | Câu hỏi lí thuyết Hóa học hữu cơ OlympiaVN


Sự phối trí này làm tăng khả năng phản ứng của ester, tạo thuận lợi cho giai đoạn
thứ hai của phản ứng: sự tấn công của nhóm phenyl có tính nucleophile của phân
tử phenylmagnesium bromide, thay thế nhóm ethoxide. (Nếu ít hơn 1 đương
lượng mol tác nhân Grignard thì ester ban đầu sẽ được phục hồi.)

Dưới những điều kiện của phản ứng, ethoxide là nhóm đi ra tốt. Phản ứng nghịch
liên quan đến phản ứng cộng ethoxide, sau đó mất Ph- - là tiểu phân tương đối
kém bền. Kết quả là, phản ứng như trên thường là bất thuận nghịch.
Sản phẩm của giai đoạn thứ hai của phản ứng, Ph2C=O, ketone, cũng chứa một
tâm electrophile và do đó có thể phản ứng tiếp với một phân tử phenylmagnesium
bromide nữa. Do các ketone có khả năng phản ứng với các nucleophile mạnh hơn
ester, nên sản phẩm của giai đoạn đầu tiên của phản ứng sẽ phản ứng nhanh hơn
ester ban đầu – không cần đến sự phối trí xúc tác với nguyên tử magnesium:

522 | Câu hỏi lí thuyết Hóa học hữu cơ OlympiaVN


Do đó phản ứng cần 2 đương lượng mol phenylmagnesium bromide. Nếu sử dụng
ít hơn 2 đương lượng mol, các sản phẩm của phản ứng có thể là hỗn hợp của ester
chưa phản ứng, ketone và alcohol bậc 3. Sản phẩm của giai đoạn thứ ba này của
phản ứng là muối của alcohol mong muốn. Khi toàn bộ phenylmagnesium
bromide đã phản ứng thì acid (thường là dung dịch hydrochloric acid), có thể
được thêm vào hỗn hợp phản ứng để giải phóng sản phẩm mong muốn:

Phản ứng Grignard không thể tiến hành khi có mặt tác nhân nhường proton do
phenylmagnesium bromide có thể phản ứng với proton tạo thành benzene. Do đó,
acid và nước, bị loại bỏ khỏi bình phản ứng cho đến khi phản ứng mong muốn đã
diễn ra; sau đó chúng được thêm vào giai đoạn tiếp theo, thường gọi là “work-
up”. Alcohol có thể được tách khỏi muối magnesium bằng cách chiết với dung
môi hữu cơ, thường là diethyl ether, trong đó các sản phẩm phụ vô cơ không tan
được.
Chú ý rằng Ph2CO có thể được sử dụng làm nguyên liệu đầu thay cho PhCO2Et.
Trong các tổng hợp hữu cơ, thường có nhiều hướng khác nhau để tạo thành cùng
một hợp chất: hướng được chọn tùy thuộc vào tính sẵn có của các nguyên liệu
đầu thay thế, hiệu quả của các phản ứng, tính thuận tiện của việc tách sản phẩm
khỏi hỗn hợp và thậm chí là ưu tiên cá nhân của nhà thực nghiệm. Ngoài ra các
vấn đề về rác thải và ô nhiễm môi trường cũng cần được xem xét. Trong trường
hợp này, phản ứng của ester ban đầu với phenylmagnesium bromide được xác
định là có thể tiến hành với hiệu suất tốt.
b) Ph2C=CH2
Việc tổng hợp trực tiếp alkene mục tiêu từ phenylmagnesium bromide không hề
đơn giản. Tuy nhiên, cần nhớ rằng các alkene có thể dễ dàng được tạo thành từ
alcohol, và các alcohol bậc 3 đặc biệt dễ tách nước trong môi trường acid hoặc
qua phản ứng với thionyl chloride trong pyridine, nên hướng sau đây có thể được
đề xuất:

Bản chất hóa học của giai đoạn đầu tiên gần như tương đồng với giai đoạn thứ 3
của quá trình tổng hợp đã thảo luận ở ý trước. Để cô lập alcohol trước khi tách
nước, muối magnesium có thể được phân hủy bởi dung dịch ammonium chloride;
trong những điều kiện này, muối magnesium được kết tủa và alcohol còn lại được
hòa tan vào ether. Phản ứng dehydrate hóa alcohol bậc 3 được thực hiện với acid
hoặc với SOCl2 trong pyridine. Phương pháp saulaf phản ứng hữu dụng hơn do

523 | Câu hỏi lí thuyết Hóa học hữu cơ OlympiaVN


các sản phẩm phụ là sulfur dioxide khí và hdyrogen chloride tan trong nước; thế
nên dễ dàng tách sản phẩm khỏi hỗn hợp phản ứng.
c) PhC(CH3)=CH2
Khác biệt duy nhất giữa alkene mục tiêu này với alkene ở ý trước là nhóm thế thứ
hai của alkene là methyl thay vì phenyl. Do đó, quy trình tổng hợp được điều
chỉnh với việc lựa chọn ketone ban đầu phù hợp:

d) PhCO2H
Carbon dioxide là electrophile cần cho phản ứng này và nó phản ứng với
phenylmagnesium bromide tạo thành muối benzoate, khi xử lí với acid có thể giải
phóng benzoic acid.

Phản ứng này có thể được tiến hành bằng cách rót dung dịch ether của tác nhân
Grignard lên lượng dư carbon dioxide rắn, hoặc bằng cách sục carbon dioxide khí
vào dung dịch Grignard.
e) PhCH2Ph
Các alkyl halide có một nguyên tử carbon có tính electrophile và dễ tham gia phản
ứng thế nucleophile. Do đó, không có gì bất ngờ khi các tác nhân Grignard phản
ứng với các alkyl halide theo cơ chế SN2. Đáng tiếc là trong thực tế, các phản ứng
thường diễn ra chậm và có hiệu suất thấp. Tuy nhiên, các benzyl halide (cũng
giống như các dẫn xuất allyl tương đồng) lại dễ tham gia phản ứng SN2 với hiệu
suất cao đáng kể. Phản ứng sau đây là một tổng hợp khả thi.

f) PhCHO

524 | Câu hỏi lí thuyết Hóa học hữu cơ OlympiaVN


Nhìn sơ qua thì phản ứng sau của ethyl formate với phenylmagnesium bromide
có thể là một hướng tổng hợp đơn giản aldehyde mục tiêu:

Tuy nhiên do aldehdye được tạo thành khi có mặt phenylmagnesium bromide
chưa phản ứng (dễ phản ứng với sản phẩm hơn cả chất đầu) nên hỗn hợp sản
phẩm sẽ gồm: chất đầu, aldehyde và Ph2CH(OH) (chất cuối là kết quả của phản
ứng cộng phân tử phenylmagnesium bromide thứ hai và aldehyde mục tiêu.)
Những khó khăn này sẽ bị loại bỏ bằng cách sử dụng ethyl orthoformate. Chất
này phản ứng với phenylmagnesium bromide tạo thành acetal, có thể được cô lập
từ hỗn hợp phản ứng. Sau đó acetal bị thủy phân trong môi trường acid êm dịu
tạo thành aldehyde. Do aldehdye không được tạo thành khi có mặt
phenylmagnesium bromide nên không có thêm phản ứng nào diễn ra.

g) PhCH2CH2OH
Trong trường hợp này, electrophile thuận tiện nhất là sử dụng ethylene oxide, chất
này phản ứng với phenylmagnesium bromide tạo thành muối của alcohol mong
muốn:

Phản ứng có thể diễn ra bởi sự phối trí của epoxide oxygen với nguyên tử
magnesium. Các tác nhân Grignard thưởng không phản ứng với các ether, tuy
nhiên, sức căng nội tại của ethylene oxide giúp phản ứng diễn ra thuận lợi (cơ chế
chính xác chưa được biết đến).

525 | Câu hỏi lí thuyết Hóa học hữu cơ OlympiaVN


h) PhD
Sử dụng D2O làm nguồn electrophile D+.

Phản ứng tương đương với nước là phản ứng phụ với bất kì phản ứng nào có sự
tham gia của tác nhân Grignard, có thể diễn ra khi nước không được loại bỏ hoàn
toàn khỏi hệ. Nước được sử dụng để phân hủy phức kim loại dư.

526 | Câu hỏi lí thuyết Hóa học hữu cơ OlympiaVN


Bài 24
Dự đoán sản phẩm của phản ứng giữa methylmagnesium iodide với mỗi chất sau
đây:
a) CH3CO2Et
b) CH3CN
c) ClCO2Et
d) CH3CO-O-COCH3
e) ClCH2OCH3
f) CH2=CH-CH2Br
g) PhCOCH=CH2
h) (CH3)3C-CO-C(CH3)3
Hướng dẫn
Trong methylmagnesium iodide, nguyên tử carbon liên kết trực tiếp với nguyên
tử magnesium dương điện. Tương tự như bài trước, phân tử có thể được biểu diễn
ở dạng lai hóa cộng hưởng của hai cấu trúc sau:

Do methyl anion (CH3-) là tiểu phân cực kì kém bền nên đóng góp của cấu trúc
cộng hưởng thứ hai là không đáng kể. Methylmagnesium iodide là hợp chất rất
hoạt động, sẽ phản ứng với hầu hết các tâm electrophile. Nếu electrophile là
carbon thì một liên kết carbon-carbon mới sẽ được tạo thành.
a) CH3CO2Et
Do tâm electrophile là nguyên tử carbonyl carbon của ester. Bản chất hóa học của
phản ứng này tương tự như ý a của bài trước. Giai đoạn đầu của phản ứng ứng là
sự tạo phối trí của carbonyl oxygen với nguyên tử magnesium. Sau đó, nhóm
methyl có tính nucleophile thay thế nhóm ethoxy tạo thành một ketone:

Một phân tử methylmagnesium iodide nữa phản ứng với ketone tạo thành muối
của alcohol:

527 | Câu hỏi lí thuyết Hóa học hữu cơ OlympiaVN


Muối này được xử lí với acid, thường là hydrochloric acid, giải phóng alcohol.
Trong trường hợp này, alcohol bậc 3 được tạo thành có các nguyên tử hydrogen
ở vị trí carbon β, do đó, dễ bị tách nước trong môi trường acid của quá trình
“work-up”. Alcohol có thể được cô lập nếu quá trình work-up được tiến hành cẩn
thận. Quá trình này thường sử dụng dung dịch ammonium chloride để kết tủa
muối magnesium và để lại alcohol tan trong ether.

b) CH3CN
Electrophile của phản ứng này là nguyên tử carbon của nhóm nitrile, sẽ bị nhóm
methyl có tính nucleophile tấn công. Phản ứng này tương tự phản ứng cộng
nucleophile vào nhóm carbonyl:

Dẫn xuất magnesium của ketimine kém hoạt động, không tham gia phản ứng cộng
tiếp và khi thủy phân trong dung dịch acid tạo thành một ketone.

Do ketone không được tạo ra cho đến khi tác nhân Grignard đã bị phân hủy nên
không có phản ứng tiếp với ketone để tạo thành alcohol. Do đó, phản ứng của
nitrile với tác nhân Grignard là một phương pháp hữu ích để tổng hợp các ketone.
c) ClCO2Et
Nhóm methyl có tính nucleophile tấn công vào nhóm carbonyl có tính
electrophile tạo thành một trung gian tứ diện. Theo nguyên tắc thì trung gian này
bị mất hoặc chloride ion hoặc ethoxide ion. Do chloride ion là nhóm rời đi tốt hơn
nên đây là phản ứng được ưu tiên:

Sản phẩm của phản ứng này là một ester và tự nó có thể phản ứng tiếp với tác
nhân Grignard (nếu nó vẫn còn).

528 | Câu hỏi lí thuyết Hóa học hữu cơ OlympiaVN


d) CH3CO-O-COCH3

e) CICH2OCH3
Phản ứng SN2 của các haloalkene với tác nhân Grignard thường hiếm và hiệu suất
thấp. Tuy nhiên, ClCH2OCH3 có một hướng phản ứng thay thế. Chloride ion,
dưới ảnh hưởng của tác nhân Grignard tạo phối trí, bị thế nội phân tử bởi cặp
electron chưa liên kết của oxygen, để lại một tiểu phân có tính electrophile cao,
sẽ nhanh chóng phản ứng với nhóm methyl có tính nucleophile:

Sản phẩm tạo thành là một ether, không phản ứng tiếp.
f) CH2=CH-CH2Br
Như đã nói ở trên, phản ứng SN2 của các haloalkane với tác nhân Grignard thường
hiếm và hiệu suất thấp. Tuy nhiên, các hợp chất allyl lại rất dễ tham gia phản ứng
thế. Phản ứng sau là một trong số ít những ví dụ hữu ích trong tổng hợp hữu cơ
của tác nhân Grignard tại một tâm carbon bão hòa:

g) PhCOCH=CH2
Trong phản ứng, có hai tâm electrophile và mỗi tâm đều có thể phản ứng với
nhóm methyl có tính nucleophile. Nếu magnesium được sử dụng trong tổng hợp
tác nhân Grignard là kim loại tinh khiết thì phản ứng chiếm ưu thế là sự cộng hợp
vào nhóm carbonyl (phản ứng cộng 1,2):

529 | Câu hỏi lí thuyết Hóa học hữu cơ OlympiaVN


Sự xuất hiện của một lượng nhỏ đồng (vi lượng) trong magnesium để điều chế
tác nhân Grignard cũng dẫn đến phản ứng cộng chiếm ưu thế là vào liên kết đôi
carbon-carbon: Đây là phản ứng cộng 1,4- (hay cộng Michael). Nguyên nhân
chính xác của sự khác biệt về tỉ lệ sản phẩm vẫn chưa được thấu hiểu hoàn toàn;
một số nhà nghiên cứu cho rằng phản ứng cộng của đồng có thể thúc đẩy cho một
cơ chế luân chuyển trong đó diễn ra sự chuyển đơn electron. Như đã nói ở trên,
sản phẩm của phản ứng Grignard là muối của một enol, được giải phóng khi xử
lí với acid, và rồi nó nhanh chóng đồng phân hóa thành dạng keto:

h) (CH3)3C-CO-C(CH3)3
Mặc dù thoạt nhìn qua, ketone này có một tâm electrophile phù hợp cho phản ứng
với methyl magnesium iodide, nhưng trong thực tế thì sự cồng kềnh của các nhóm
thế alkyl như vậy khiến cho phản ứng cộng không thể diễn ra.

Trong một số trường hợp khi phản ứng nucleophile của tác nhân Grignard bị hạn
chế, các phản ứng phụ sẽ diễn ra. Tác nhân Grignard có thể đóng vai trò như một
base, nhưng trong trường hợp này phân tử không có proton có lực acid đủ mạnh.
Nếu tác nhân Grignard chứ 1 hydrogen ở vị trí β carbon, phản ứng khử có thể
diễn ra, tuy nhiên, trường hợp này cũng không xảy đến với methylmagnesium
iodide. Do đó, hệ này hoàn toàn không có phản ứng.

530 | Câu hỏi lí thuyết Hóa học hữu cơ OlympiaVN


Bài 25
Vì sao có thể tạo ra dẫn xuất lithium A bằng phản ứng trao đổi Br/Li nhưng không
thể điều chế dẫn xuất lithium B theo cách tương tự?

Hướng dẫn
Trong phản ứng đầu tiên, vinyl bromide tạo thành vinyl (sp2) carbanion bền hơn
carbanion no (sp3) bởi tính chất s trong liên kết C-Li lớn hơn. Trong ví dụ thứ hai,
nễu xảy ra phản ứng thì dẫn xuất bromide no tạo thành t-alkyl carbanion, kém
bền hơn carbanion bậc một trong BuLi. Do vậy phản ứng thứ hai không xảy ra
được.

531 | Câu hỏi lí thuyết Hóa học hữu cơ OlympiaVN


Bài 26
Đề xuất cơ chế cho 4 phản ứng sau:

532 | Câu hỏi lí thuyết Hóa học hữu cơ OlympiaVN


Hướng dẫn

533 | Câu hỏi lí thuyết Hóa học hữu cơ OlympiaVN


Bài 27
Dự đoán sản phẩm của các phản ứng sau

Hướng dẫn

534 | Câu hỏi lí thuyết Hóa học hữu cơ OlympiaVN


Bài 28
Xét chuyển hóa sau đây của phenyllithium, hexamethylacetone và carbon
monoxide:

a) Đề xuất cơ chế của chuyển hóa này. Chỉ rõ các tương tác orbital phân tử biên
trong mỗi giai đoạn của cơ chế.
b) Khi chuyển hóa tương tự được thực hiện, sử dụng acetone thay vì
hexamethylacetone thì ketone mong muốn được tạo thành với lượng rất nhỏ.
Hãy giải thích hiện tượng này, và đề xuất cơ chế đầu tiên của cơ chế minh
họa một sản phẩm phụ có thể xảy ra. (Gợi ý: Phân tích IR của sản phẩm phụ
chiếm ưu thế nhất thì thấy có một dải rộng ở 3391 cm-1 nhưng không có tín
hiệu nào trong khoảng 1750-1700 cm-1.)

Hướng dẫn
a) l.p = lone pair (cặp electron chưa liên kết).

b) Hexamethylacetone có sự án ngữ không gian nên phenyllithium ưu tiên tấn


công [tác kích] vào electrophile CO. Acetone thì ít bị án ngữ không gian hơn và
sẽ cạnh tranh với CO trong vài trò electrophile ở giai đoạn đầu tiên.
Cơ chế một phản ứng phụ có thể diễn ra:

535 | Câu hỏi lí thuyết Hóa học hữu cơ OlympiaVN


536 | Câu hỏi lí thuyết Hóa học hữu cơ OlympiaVN
Bài 28A
Có ba nhận xét được ghi trong một báo cáo thực nghiệm như sau:
i. Cho t-butyl clorua vào kim loại liti trong dietyl ete khan nước ở 35 oC. Một
phản ứng rất mạnh xảy ra và phóng thích khí hydrocacbon. Sau khi liti tan hết,
hỗn hợp phản ứng được rót vào đá cacbonic. Chỉ một axit được tạo thành với hiệu
suất không cao: axit 4,4-dimetylpentanoic .
ii. Cho t-butyl clorua vào kim loại liti trong dietyl ete khan nước ở -40 oC. Sau
khi liti tan hết, hỗn hợp được cacbonat hóa. Axit trimetylaxetic được tạo thành
với hiệu suất cao.
iii. Cho t-butyl clorua vào kim loại liti trong dietyl ete khan nước ở -40 oC. Sau
khi liti tan hết, sục khí etilen vào hỗn hợp cũng ở -40 oC cho đến khi phản ứng
chấm dứt. Hỗn hợp phản ứng được rót vào đá cacbonic. Một axit được tạo thành
với hiệu suất cao: axit 4,4-dimetylpentanoic.
a) Giải thích kết quả và cho biết cơ chế phản ứng của ba thí nghiệm kể trên.
b) Thay t-butyl clorua bằng metyl clorua sẽ thu được sản phẩm gì với ba thí
nghiệm kể trên.
c) Tại sao không thấy axit 6,6-dimetylheptanoic trong thí nghiệm 3?
Hướng dẫn
Ba thí nghiệm ghi trong một báo cáo thực nghiệm:
a) Giải thích kết quả và cho biết cơ chế phản ứng của ba thí nghiệm kể trên.
– Trong thí nghiệm 1: Kim loại liti tác dụng với t-butyl clorua cho ra hợp chất
hữu cơ kim loại liti, t-butylliti. Ở 35 oC, t-butylliti tác dụng với dietyl ete (dung
môi của phản ứng) cho ra 2-metylpropan và etilen. 2-metylpropan thoát ra khỏi
phản ứng dưới dạng khí hydrocacbon, còn etilen tác dụng với t-butylliti cho ra
3,3-dimetylbutylliti. Hợp chất hữu cơ kim loại liti này khi gặp CO2 sẽ cho ra axit
4,4,-dimetylpentanoic với hiệu suất không cao (vì một số t-butylliti tác dụng với
dietyl ete để tạo etilen).

537 | Câu hỏi lí thuyết Hóa học hữu cơ OlympiaVN


– Trong thí nghiệm 2: Vì phản ứng được thực hiện tại -40 oC, cho nên ở nhiệt độ
này hợp chất hữu cơ liti không cho phản ứng với ete. Sản phẩm t-butylliti sẽ chỉ
tác dụng với CO2 sẽ cho ra axit trimetylaxetic với hiệu suất cao.

– Trong thí nghiệm 3: Vì phản ứng được thực hiện tại -40 oC, cho nên hợp chất
hữu cơ liti không phản ứng với với ete mà phản ứng hoàn toàn với khí etilen sục
vào chuyển hóa hoàn toàn thành 3,3-dimetylbutylliti. Hợp chất này khi gặp CO2
sẽ cho ra axit 4,4-dimetylpentanoic với hiệu suất cao (vì etilen được cung cấp đầy
đủ).

538 | Câu hỏi lí thuyết Hóa học hữu cơ OlympiaVN


b) Thay t-butyl clorua bằng methyl clorua sẽ thu được sản phẩm gì với ba thí
nghiệm kể trên.
– Trong thí nghiệm 1: metylliti có hoạt tính kém hơn t-butylliti nên không tác
dụng với dietyl ete, toàn bộ metylliti tạo thành đều tác dụng với CO2 cho ra axit
axetic với hiệu suất cao.
– Trong thí nghiệm 2 và 3: đều cho ra axit axetic.
c) Không tìm thấy axit 6,6-dimetylheptanoic trong thí nghiệm 3, vì neohexylliti
là hợp chất hữu cơ liti bậc một nên có hoạt tính kém hơn t-butylliti nên không thể
tác dụng với etilen.

539 | Câu hỏi lí thuyết Hóa học hữu cơ OlympiaVN


Bài 29
Cho biết sản phẩm nào sẽ được tạo thành từ phản ứng của ammonia với mỗi chất
sau:
(a) Methyl iodide.
Hướng dẫn
Phản ứng của methyl iodide với ammonia ban đầu tạo thành acid liên hợp tương
ứng với amine, chất này sau đó phản ứng với ammonia trong một cân bằng acid-
base:

Methylamine được tạo thành sẽ phản ứng với methyl iodide và sản phẩm lại tiếp
tục phản ứng:

Hệ quả là thường thì một hỗn hợp sản phẩm sẽ được tạo thành và phương pháp
này không hiệu quả trong điều chế dimethylamine và trimethylamine.
Methylamine được điều chế bằng cách dùng lượng dư ammonia và muối
ammonium bậc bốn thì bằng cách sử dụng methyl iodide dư. Trimethylamine có
thể được tách riêng từ hỗn hợp bằng cách chưng cất. Nó cũng thể được điều chế
bởi phản ứng của lượng dư dimethylamine với methyl iodide.
(b) Acetyl chloride.
Hướng dẫn
Acetyl chloride phản ứng với ammonia bởi phản ứng tác kích nucleophile vào
nhóm carbonyl. Trung gian tứ diện mất chloride, tạo thành acetamide:

540 | Câu hỏi lí thuyết Hóa học hữu cơ OlympiaVN


Amide không phản ứng với acetyl chloride do cặp electron chưa liên kết trên
nitrogen bị vô hoạt tính nucleophile bởi sự liên hợp với nhóm carbonyl.
(c) Formaldehyde.
Hướng dẫn
Phản ứng của ammonia với nhóm carbonyl có thể theo sau bởi sự trao đổi proton,
tạo thành một sản phẩm cộng:

Trong một số ví dụ, sự tách nước từ sản phẩm cộng có thể diễn ra theo hướng liên
quan đến cơ chế E1cb:

Trong trường hợp formaldehyde, imine tạo thành quá hoạt động với nucleophile
nên sản phẩm cộng sơ cấp sẽ phản ứng tiếp, tạo thành urotropine
(hexamethylenetetramine) với cấu trúc tương tự như adamantane. Sự tạo thành
urotropine gồm nhiều bước:

(d) Acetaldehyde
Hướng dẫn
Ammonia phản ứng với các aldehyde nhưng thường thì các sản phẩm kém bền
trừ khi aldehyde carbon được gắn vào một nhóm hút electron mạnh. Với
acetaldehyde, một dung dịch trong ether hấp thụ ammonia và tạo thành kết tủa
kết tinh màu trắng, có thể là polymer của sản phẩm cộng.

541 | Câu hỏi lí thuyết Hóa học hữu cơ OlympiaVN


(e) Chloral.
Hướng dẫn
Khi một aldehyde carbon gắn vào một nhóm hút electron mạnh, như chloral, thì
sản phẩm cộng với ammonia sẽ bền.

Xu hướng tương tự cũng được quan sát thấy với tính bền của hydrate của các
aldehyde và ketone, cân bằng ưu tiên hydrate khi carbonyl được gắn vào nhóm
hút electron mạnh.
(f) Acetone.
Hướng dẫn
Ketone phản ứng với ammonia hơi khác so với aldehyde; sản phẩm cộng với
ketone không phân lập được mà bị tách nước tạo thành imine. Tuy nhiên, acetone,
thì thể hiện tính chất khác, trước tiên nó tự ngưng tụ dưới tác động của ammonia
- chất đóng vai trò như một base hơn là nucleophile. Có hai sản phẩm được phân
lập. Sản phẩm đầu tiên là từ phản ứng cộng kiểu-Michael:

Sản phẩm thứ hai được tạo thành theo chuỗi phản ứng dưới đây:

542 | Câu hỏi lí thuyết Hóa học hữu cơ OlympiaVN


(g) Benzaldehyde.
Hướng dẫn
Benzaldeyde tạo thành các sản phẩm ngưng tụ với ammonia; sự tạo thành chúng
liên quan đến sự tách nước sản phẩm cộng ban đầu, sau đó phản ứng tiếp. Một
trong các hướng khả dĩ là:

(h) Ethylene oxide.


Hướng dẫn
Ethylene oxide phản ứng với ammonia, lần lượt tạo thành các β-hydroxy amine
bậc một, bậc hai và bậc ba.

543 | Câu hỏi lí thuyết Hóa học hữu cơ OlympiaVN


Hoạt tính mạnh của ethylene oxide so với các ether khác là hệ quả của sức căng
vòng.
(i) 2,4-Pentanedione.
Hướng dẫn
2,4-Pentanedione phản ứng với ammonia tạo thành imine, rồi xảy ra hỗ biến thành
enamine liên hợp, bền:

(j) 2,5-Hexanedione.
Hướng dẫn

544 | Câu hỏi lí thuyết Hóa học hữu cơ OlympiaVN


(k) Acetonitrile.
Hướng dẫn
Đun nóng acetonitrile với ammonium chloride, một nguồn tạo ammonia, tạo
thành acetamidine:

(l) Phenyl isothiocyanate.


Hướng dẫn
Carbon của isothiocyanate có tính electrophile và phản ứng với ammonia tạo
thành N-phenylthiourea:

(m) Cyanamide.
Hướng dẫn

545 | Câu hỏi lí thuyết Hóa học hữu cơ OlympiaVN


Bài 30
Tổng hợp pyrrole Paal-Knorr là một phương pháp để tổng hợp các pyrrole từ tiền
chất 1,4-dicarbonyl và amine. Đề xuất cơ chế đầy đủ cho tổng hợp pyrrole này.

Hướng dẫn

546 | Câu hỏi lí thuyết Hóa học hữu cơ OlympiaVN


Bài 31
Thiourea, chất tương đồng chứa sulfur của urea, là một tác nhân hữu dụng để tổng
hợp các thiol từ alkyl halide, như dưới đây:

Đề xuất cơ chế đầy đủ cho chuyển hóa này.


Hướng dẫn

547 | Câu hỏi lí thuyết Hóa học hữu cơ OlympiaVN


Bài 32
Trình bày cơ chế của phản ứng sau:

Hướng dẫn

548 | Câu hỏi lí thuyết Hóa học hữu cơ OlympiaVN


Bài 33
Trình bày cơ chế phản ứng của các chuyển hóa sau:

Hướng dẫn
a)

549 | Câu hỏi lí thuyết Hóa học hữu cơ OlympiaVN


b)

550 | Câu hỏi lí thuyết Hóa học hữu cơ OlympiaVN


Bài 34
Bạn có thể đã đọc trong các giáo trình Hóa hữu cơ rằng phản ứng amine hóa-khử
có thể được thực hiện bằng cách xử lí một aldehyde hoặc ketone với một amine
khi có mặt sodium cyanoboronhydride (NaBH3CN) hoặc sodium
triacetoxyborohydride (NaBH(OAc)3). Chuyển hóa sau, gọi là phản ứng
Leuckart, là một cách thay thế để thực hiện phản ứng amine hóa-khử.

a) Trình bày cơ chế của chuyển hóa này.


b) Cho biết vai trò của formate anion trong phản ứng Leuckart.
Acid Base Chất khử
Chất oxid hóa Electrophile Nucleophile
Hướng dẫn
a)

b) Nucleophile và chất khử.

551 | Câu hỏi lí thuyết Hóa học hữu cơ OlympiaVN


Bài 35
Quy trình hai giai đoạn sau là một phương pháp thú vị để chuyển hóa các aldehyde
thành nitrile. Trình bày cơ chế của hai giai đoạn trong chuyển hóa sau, và xác
định rõ cấu trúc của trung gian A.

Hướng dẫn

552 | Câu hỏi lí thuyết Hóa học hữu cơ OlympiaVN


Bài 36
Trình bày cơ chế cho chuyển hóa thú vị sau đây giữa anthranilic acid và furan:

Hướng dẫn

553 | Câu hỏi lí thuyết Hóa học hữu cơ OlympiaVN


Bài 37
Viết phương trình phản ứng cho mỗi giai đoạn trong cơ chế của các chuyển hóa
sau:

554 | Câu hỏi lí thuyết Hóa học hữu cơ OlympiaVN


555 | Câu hỏi lí thuyết Hóa học hữu cơ OlympiaVN
Hướng dẫn

556 | Câu hỏi lí thuyết Hóa học hữu cơ OlympiaVN


557 | Câu hỏi lí thuyết Hóa học hữu cơ OlympiaVN
558 | Câu hỏi lí thuyết Hóa học hữu cơ OlympiaVN
559 | Câu hỏi lí thuyết Hóa học hữu cơ OlympiaVN
Bài 38
Dự đoán sản phẩm chính của các phản ứng sau. Nếu có nhiều sản phẩm tạo thành,
hãy chỉ ra sản phẩm chính:

Hướng dẫn

560 | Câu hỏi lí thuyết Hóa học hữu cơ OlympiaVN


Bài 39
Lynestrenol là một hormone tổng hợp được dùng để điều trị các chứng rối loạn
liên quan đến thiếu hụt progesterone. Dưới đây là một quy trình tổng hợp
lynestrenol từ nortestosterone.

a) Đề xuất một cơ chế chi tiết cho giai đoạn đầu của tổng hợp trên. (Gợi ý: BF3
là một Lewis acid, có thể xúc tác cho các phản ứng như một Brønsted acid;
Lewis acid chỉ đơn giản là thay cho H+ trong cơ chế.)
b) Xác định giai đoạn khử và oxid hóa trong tổng hợp.
c) Trình bày cơ chế cho chuyển hóa cuối và giải thích hóa lập thể của sản phẩm.

561 | Câu hỏi lí thuyết Hóa học hữu cơ OlympiaVN


Hướng dẫn
a) Giai đoạn đầu tiên:

b) Giai đoạn thứ ba là oxid hóa (mất 2H) từ alcohol thành carbonyl. Giai đoạn
thứ tư là khử hóa từ carbonyl thành alcolhol. Giai đoạn thứ hai cũng là khử hóa.

562 | Câu hỏi lí thuyết Hóa học hữu cơ OlympiaVN


c) Giai đoạn cuối:

563 | Câu hỏi lí thuyết Hóa học hữu cơ OlympiaVN


Bài 40
Viagra có thể được điều chế theo chuỗi phản ứng sau. Trình bày cơ chế của mỗi
phản ứng.

564 | Câu hỏi lí thuyết Hóa học hữu cơ OlympiaVN


Hướng dẫn
Tạo thành pyrazole

565 | Câu hỏi lí thuyết Hóa học hữu cơ OlympiaVN


Methyl hóa nitrogen, thủy phân ester:

Nitro hóa vòng thơm:

566 | Câu hỏi lí thuyết Hóa học hữu cơ OlympiaVN


Tạo thành amide qua acid chloride:

Khử hóa nhóm nitro thành amine:

567 | Câu hỏi lí thuyết Hóa học hữu cơ OlympiaVN


Tạo thành amide:

Vòng hóa bằng cách ghép cặp các nhóm amide:

568 | Câu hỏi lí thuyết Hóa học hữu cơ OlympiaVN


Thế nhân thơm với SO2Cl:

569 | Câu hỏi lí thuyết Hóa học hữu cơ OlympiaVN


Alkyl hóa amine bậc hai:

570 | Câu hỏi lí thuyết Hóa học hữu cơ OlympiaVN


Bài 41
Pentadienyl cation 1 tham gia phản ứng
đóng vòng electrocyclic (điện vòng), tạo
thành allylic cation 2. Dựa vào những suy
xét về tính đối xứng orbital, hãy suy luận
xem đây là quá trình quay cùng chiều
(conrotation) hay quay ngược chiều (disrotation) – bằng cách phân tích HOMO
của 1. Cuối cùng, hãy đưa ra hóa lập thể tương đối của 2.

Hướng dẫn
Orbital π2 (HOMO):

571 | Câu hỏi lí thuyết Hóa học hữu cơ OlympiaVN


Bài 42
Đề xuất cơ chế cho mỗi phản ứng Diels-Alder sau và dự đoán hóa lập thể cuả sản
phẩm. Chỉ rõ các tương tác orbital phân tử.

Thực tế thì sản phẩm của phản ứng c) không bền và phản ứng tiếp thành sản phẩm
bền hơn. Xác định sản phẩm cuối được tạo thành.

572 | Câu hỏi lí thuyết Hóa học hữu cơ OlympiaVN


Hướng dẫn

573 | Câu hỏi lí thuyết Hóa học hữu cơ OlympiaVN


Bài 43
Đọc và trả lời các câu hỏi từ (26)-(46).
Tổng hợp toàn phần là phương pháp tổng hợp nhân tạo một hợp chất với cấu trúc
phức tạp đi từ các chất đơn giản, sẵn có. Nếu hợp chất mục tiêu có tiềm năng trở
thành dược phẩm, như các mô tả dưới đây, nhưng chỉ có thể nhận được lượng nhỏ
từ tự nhiên thì nhu cầu tổng hợp toàn phần là vô cùng thiết yếu để điều chế sản
lượng lớn. Nhiều phản ứng hữu cơ đã được sử dụng trong các tổng hợp toàn phần
và một trong những loại quan trọng nhất là tạo liên kết carbon-carbon. Mặc dù có
nhiều phản ứng tạo liên kết carbon-carbon nhưng phổ biến nhất là các phản ứng
Grignard, ngưng tụ aldol, ghép cặp chéo và Diels-Alder. Phản ứng Diels-Alder
được khám phá bởi các nhà hóa học Đức Otto Diels và Kurt Alder - cả hai đều
được vinh danh với giải Nobel Hóa học năm 1950. Trong phản ứng Diels-Alder,
diene liên hợp tham gia phản ứng vòng hóa với hợp chất có liên kết bội (tác nhân
dienophile), và một dẫn xuất cyclohexene được tạo thành. (Hệ liên hợp là hợp
chất với 2 liên kết bội được xen kẽ bởi 1 liên kết đơn ở giữa.) Phản ứng Diels-
Alder đơn giản nhất là phản ứng sử dụng 1,3-butadiene và ethylene, tao thành
cyclohexene (hình 1). Tuy nhiên, phản ứng Diels-Alder đơn giản nhất này lại rất
khó để tiến hành và xảy ra rất chậm ở nhiệt độ phòng. aĐể tăng hiệu quả phản
ứng, có thể dựa vào áp suất.

Hình 1: Phản ứng Diels-Alder của 1,3-butadiene và ethylene.


Trong bài này, các nguyên tử carbon và hydrogen được lược bỏ và cấu trúc của
các hợp chất được biểu diễn bởi cấu trúc khung như trong hình 1. Trong liên kết
đôi carbon-carbon thì có 1 liên kết khó phá vỡ, tham gia tạo thành bộ khung
carbon của phân tử, gọi là liên kết σ (sigma). Liên kết còn lại gọi là liên kết π (pi).
Cặp electron được dùng cho liên kết π gọi là electron π và khi diễn ra phản ứng
có thể di chuyển trong bộ khung carbon. Trong phản ứng Diels-Alder, tổng cộng
có 3 liên kết π bị phá vỡ và 2 liên kết σ cùng 1 liên kết π mới được tạo thành. Sự
dịch chuyển của cặp electron được xác định bởi các mũi tên cong. Điểm đầu của
mũi tên cong chỉ cặp electron dịch chuyển, còn điểm cuối (mũi tên) là ở giữa các
nguyên tử sẽ tạo thành liên kết mới hoặc trên nguyên tử mà các electron sẽ chuyển
tới.
Câu hỏi ア: Bộ khung carbon dưới đây là của phân tử nào? (26)

574 | Câu hỏi lí thuyết Hóa học hữu cơ OlympiaVN


Câu hỏi イ: Trong số các alcohol có công thức C5H12O dưới đây, hãy chọn ra
chất không có carbon bất đối. (27)

Câu hỏi ウ: Chất nào sau đây không có liên kết đôi liên hợp. (28)

Câu hỏi エ: Điều nào sau đây phù hợp nhất với phần gạch chân (a) ở trên. (29)
① Cao ② Thấp ③ Không đổi
Phản ứng Diels-Alder phụ thuộc nhiều vào các nhóm thế của diene liên hợp và
tác nhân dienophile. Nhìn chung, khi một nhóm nhường electron được gắn vào
diene liên hợp, còn một nhóm hút electron được gắn vào dienophile thì phản ứng
có xu hướng diễn ra. Nói cách khác, phản ứng sẽ thuận lợi khi có sự tham gia của
một diene giàu điện tử và dienophile thiếu hụt điện tử. Một nguyên tử có cặp
electron chưa liên kết gắn trực tiếp vào liên kết đôi hoặc một nhóm thế có khả
năng nhường cặp electron chưa liên kết vào hệ electron π được xem là nhóm
nhường electron. Còn nhóm thế có liên kết không no hoặc nguyên tử với độ âm
điện cao thì được xem là nhóm hút electron. Liên kết đôi bị thiếu hụt điện tử nếu
các electron π bị kéo về nguyên tử có độ âm điện cao gắn với liên kết.
Câu hỏi オ: Chọn ra 2 nhóm hút electron trong các nhóm dưới đây: (30) (31)
① Nhóm nitro (-NO2) ② Nhóm hydroxy (-OH)
③ Nhóm amino (-NH2) ④ Nhóm cyano (-CN)
⑤ Nhóm metyl (-CH3)
Câu hỏi カ: Tôi muốn tổng hợp hợp chất 1 sau đây bằng phản ứng Diels-Alder.
Hãy chọn ra sự kết hợp phù hợp nhất giữa diene liên hợp và dienophile trong số
các phương án dưới đây. (32)

575 | Câu hỏi lí thuyết Hóa học hữu cơ OlympiaVN


Trong các diene liên hợp, đồng phân cấu dạng được tạo thành bởi sự quay của
liên kết đơn. Với diene liên hợp, nếu các liên kết đôi carbon-carbon nằm cùng
phía của liên kết đơn ở giữa thì đó là đồng phân s-cis (hình 2). Còn nếu các liên
kết đôi ở hai phía ngược nhau của liên kết đơn thì đó là đồng phân s-trans. Chữ s
là kí hiệu của single bond (liên kết đơn). Để thực hiện phản ứng Diels-Alder thì
diene liên hợp phải đạt được cấu dạng s-cis và diene càng dễ đạt được cấu dạng
này thì phản ứng càng dễ xảy ra.

Hình 2: Các cấu dạng s-cis và s-trans của 1,3-butadiene.


Câu hỏi キ: Diene lien hợp nào sau đây dễ và khó tham gia phản ứng Diels-Alder
nhất?
Dễ tham gia nhất: (33)
Khó tham gia nhất: (34)

Tiếp theo, hãy tìm hiểu về hóa lập thể của phản ứng Diels-Alder. Trong phản ứng
giữa 1,3-butadiene và ethylene, các phân tử phản ứng không tiếp cận với nhau
theo hướng phẳng mà theo cách như dưới đây. Khi phản ứng xảy ra, các phân tử
lại gần nhau theo hướng thẳng đứng của của mặt phẳng tạo nên từ các liên kết
đôi. Trong hình 3, 4 nguyên tử hydrogen của dienophile ethyelen được phân biệt
bởi các kí hiệu Ha đến Hd. Phản ứng Diels-Alder là quá trình đặc trưng lập thể, và
mối quan hệ về vị trí của các nhóm thế của diene liên hợp và dienophile vẫn được
bảo toàn trong sản phẩm.

576 | Câu hỏi lí thuyết Hóa học hữu cơ OlympiaVN


Hình 3: Hóa lập thể của phản ứng Diels-Alder.
Câu hỏi ク: Trong phân tử cyclohexene ở hình 3, hãy chọn ra nguyên tử phù hợp
với nhóm được kí hiệu Z.
①Ha ② Hb ③ Hc ④ Hd
Hãy xét trường hợp dienophile có nhóm thế. Hợp chất A ở dạng … (a), được tạo
thành từ 1,3-butadiene và dimethyl fumarate và hợp chất B ở dạng … (b), được
tạo thành từ 1,3-butadiene và dimethyl maleate (hình 4). Khi đo phổ 13C NMR
của hợp chất B thì quan sát được … (c) tín hiệu của 13C NMR là kĩ thuật phân
tích được dùng để xác định cấu trúc hợp chất hữu cơ và các nguyên tử carbon với
[kiểu] môi trường hóa học khác nhau sẽ có thể được quan sát ở dạng các tín hiệu
riêng biệt.

Hình 4: Phản ứng Diels-Alder của 1,3-butadiene với dimethyl fumarate (フマ
ル酸ジメチル) và dimethyl maleate (マレイン酸ジメチル).
Câu hỏi ケ: Xác định các hợp chất A và B trong số các phương án sau đây. Cho
biết trong cấu trúc thì các đường nét đậm và nét đứt để xác định cấu hình. Đường
nét đậm chỉ các liên kết hướng lên trên mặt phẳng giấy, đường nét đứt chỉ các
liên kết hướng xuống dưới.
Hợp chất A: (37)
Hợp chất B: (38)

577 | Câu hỏi lí thuyết Hóa học hữu cơ OlympiaVN


Câu hỏi コ: Hãy chọn từ/cụm từ phù hợp với các khoảng trống (a), (b) ở đoạn
trên.
① chất quang hoạt ② hỗn hợp racemic
③ hỗn hợp đồng phân dia ④ dạng meso
 Racemate (hay hỗn hợp racemic) là hỗn hợp đẳng mol của các đối quang,
không thể hiện tính quang hoạt.
 Đồng phân dia: các đồng phân lập thể không phải đối quang.
 Dạng meso: các hợp chất có carbon bất đối nhưng có mặt phẳng đối xứng
trong phân tử và không thể hiện tính quang hoạt.
Câu hỏi サ: Chọn số tương ứng với khoảng trống (c).
①5 ②6 ③7
④8 ⑤9 ⑥ 10
Như đã đề cập ở phần trước, phản ứng Diels-Alder là phản ứng tạo liên kết
carbon-carbon cực kì quan trọng và thường được sử dụng trong tổng hợp toàn
phần các hợp chất hữu cơ có cấu trúc phức tạp. Hãy tìm hiểu một ví dụ về ứng
dụng của phản ứng này qua tổng hợp các prostaglandins (hình 5) được tiến hành
bởi Elias James Corey - nhà hóa học đã giành giải Nobel Hóa học 1990. Hơn 20
prostaglandin đã được biết tới và thể hiện những tác động dược lí khác nhau như
hạ huyết áp, giãn mạch, ức chế kết tập [lắng đọng] tiểu cầu, … Một trong số chúng
là prostaglandin F2α, được dùng làm thuốc kích thích chuyển dịch. Hợp chất này
được sinh tổng hợp (các chuyển hóa tạo thành hợp chất hữu cơ bởi tác động của
enzyme) trong cơ thể người, nhưng chiếm hàm lượng cực kì thấp nên việc tổng
hợp toàn phần loại thuốc này là nhu cầu thiết yếu. Trong tổng hợp prostaglandin
của Corey và đồng nghiệp, phản ứng Diels-Alder được sử dụng trong giai đoạn
đầu.

578 | Câu hỏi lí thuyết Hóa học hữu cơ OlympiaVN


Hình 5: (A) Tổng hợp toàn phần prostaglandin F2α bởi Corey và đồng nghiệp;
(B) Phản ứng oxid hóa Baeyer-Villiger; (C) Tạo thành trung gian iodonium ion
vòng.
Chú thích: ディールス・アルダー反応 = Phản ứng Diels-Alder; バイヤー・
ビリガー酸化 = Phản ứng oxid hóa Baeyer-Villiger; けん化 = xà phòng hóa; 弱
塩基 = base yếu; 3段階 = 3 giai đoạn; コーリーラクトン = Corey lactone; プ
ロスタグランジン F2α = prostaglandin F2α; 環状ヨードニウム中間体 = trung
gian iodonium ion vòng
Hợp chất C thu được bởi phản ứng của dẫn xuất cyclopentadiene 2 và dienophile
3 và ketone được tạo thành khi xử lí với base (KOH). Tiến hành phản ứng oxid
hóa Baeyer-Villiger với ketone này thì thu được hợp chất D có vòng lactone (ester
vòng). Chú ý rằng phản ứng Baeyer-Villiger là phản ứng tạo thành ester từ ketone
như trong hình 5 (trong khung nét đứt, mục B). Từ ketone bất đối sẽ tạo thành
ester trong đó nguyên tử oxygen liên kết nhóm thế R’ có bậc cao hơn.
Xà phòng hóa hợp chất D với base, sau đó xử lí với acid tạo thành hợp chất 4.
Khi chất này phản ứng với I2, có mặt base yếu, thì nhóm carboxyl phản ứng với
trung gian iodonium vòng - được tạo thành bởi phản ứng giữa liên kết đôi với
iodine (hình 5, trong khung nét đứt, mục C) - thu được hợp chất E. Hợp chất
nhận được từ chất E trong phản ứng 3 giai đoạn là trung gian quan trọng trong
tổng hợp prostaglandin và được gọi là Corey lactone.
Câu hỏi シ: Hãy chọn ra trong số các phương án đây 2 cấu trúc phù hợp với hợp
chất C. (41) (42)

579 | Câu hỏi lí thuyết Hóa học hữu cơ OlympiaVN


Câu hỏi ス: Hãy chọn ra trong số các phương án đây 1 cấu trúc phù hợp với hợp
chất D. (43)

Câu hỏi セ: Hãy chọn ra trong số các phương án đây 1 cấu trúc phù hợp với hợp
chất E. (44)

Phản ứng Diels-Alder không chỉ được sử dụng trong những tổng hợp toàn phần
như trên mà còn có thể xuất hiện trong sinh tổng hợp. Ví dụ, có một giả thuyết
cho rằng Pinnatoxin A (hình 6) - được xem là tác nhân gây ra ngộ độc bởi thực
phẩm nguồn gốc từ các loại động vật có vỏ ở Nhật Bản - được sinh tổng hợp bởi
phản ứng Diels-Alder. Pinnatoxin A được cho là đã được sinh tổng hợp từ một
hợp chất có mạch carbon dài.
Câu hỏi ソ: Hãy chọn ra hai vòng được xem là tạo thành bởi phản ứng Diels-
Alder trong quá trình sinh tổng hợp pinnatoxin A. (45), (46).

580 | Câu hỏi lí thuyết Hóa học hữu cơ OlympiaVN


Hình 6: Cấu trúc của pinnatoxin A.

581 | Câu hỏi lí thuyết Hóa học hữu cơ OlympiaVN


Hướng dẫn

Gợi ý cho một số câu hỏi


Câu hỏi オ: Ảnh hưởng của các nhóm hút electron đến liên kết đôi.

Câu hỏi カ: Nhóm ester (-COOCH3) đóng vai trò nhóm hút electron và nhóm
methoxy (CH3O-) đóng vai trò là nhóm đẩy electron (như minh họa dưới đây).
Khi nhóm đẩy electron gắn vào diene liên hợp và nhóm hút electron gắn vào
dienophile thì phản ứng Diels-Alder sẽ dễ diễn ra hơn, do đó ② là phương án
phù hợp nhất.

582 | Câu hỏi lí thuyết Hóa học hữu cơ OlympiaVN


Câu hỏi ケ: Phản ứng với dimethyl fumarate diễn ra như sau, tạo thành sản phẩm
trans.

Khi tiến hành phản ứng với dimethyl maleate thì trong sản phẩm tạo thành các
nhóm thế ở vị trí cis với nhau.
Câu hỏi シ: Dẫn xuất cyclopentadiene sẽ dễ phản ứng hơn nếu hướng tiếp cận
với dienophile có nguyên tử hydrogen nhỏ hơn. Vị trí của CN và Cl thì dù thay
đổi cũng không có khác biệt gì lớn.

Khi xử lí sản phẩm Diels-Alder với base và nước, thì có phản ứng tạo thành
ketone. Cả cyanide ion (CN-) lẫn chloride ion (Cl-) đều là nhóm rời đi tốt. Ketone
tạo thành có thể nhận được bởi phản ứng Diels-Alder giữa hợp chất 2 với ketene
(CH2=C=O) nhưng ketene này kém bền, và sẽ tự phản ứng ngay với chính nó,
tạo thành dimer, chính vì vậy nên hợp chất 3 mới được sử dụng thay thế.

583 | Câu hỏi lí thuyết Hóa học hữu cơ OlympiaVN


Câu hỏi ソ: Một số nhà nghiên cứu cho rằng pinnatoxin A được tạo thành bởi
phản ứng Diels-Alder nội phân tử từ các trung gian dưới đây. Vòng ② được cho
là tạo thành từ phản ứng ngưng tụ tách nước nội phân tử của amine và ketone.

Tổng hợp toàn phần pinnatoxin A đầu tiên được thực hiện bởi Kishi Yoshito của
ĐH Harvard, với phản ứng Diels-Alder nội phân tử tương tự như trong quá trình
sinh tổng hợp đã đề xuất là giai đoạn then chốt.

584 | Câu hỏi lí thuyết Hóa học hữu cơ OlympiaVN


Bài 44
Một cách để khống chế sự chọn lọc lập thể của phản ứng Diels-Alder, nhằm ngăn
sự tạo thành hỗn hợp racemic, là thêm amino acid proline vào phản ứng của các
aldehyde dienophile. Sự bổ sung proline làm biến đổi phản ứng như thế nào để
ngăn sự tạo thành của một trong các đối quang có thể có?

Hướng dẫn
Proline có thể tạo thành imine với carbon của carbonyl:

Khi dienophile tiếp cận từ hướng endo, carboxylic acid ngăn sự tiếp cận từ phía
trên, chỉ cho phép tiếp cận từ phía dưới diene.

585 | Câu hỏi lí thuyết Hóa học hữu cơ OlympiaVN


Bài 45
Bổ sung các trung gian và sản phẩm còn thiếu trong chuyển hóa sau:

586 | Câu hỏi lí thuyết Hóa học hữu cơ OlympiaVN


Hướng dẫn

587 | Câu hỏi lí thuyết Hóa học hữu cơ OlympiaVN


Bài 46
Trình bày cơ chế phản ứng của các chuyển hóa sau và chỉ rõ các loại phản ứng
peri hóa (pericyclic):

Hướng dẫn
a)

588 | Câu hỏi lí thuyết Hóa học hữu cơ OlympiaVN


b)

589 | Câu hỏi lí thuyết Hóa học hữu cơ OlympiaVN


Bài 47
Trình bày cơ chế phản ứng của chuyển hóa sau và chỉ rõ các loại phản ứng peri
hóa (pericyclic):

Hướng dẫn

590 | Câu hỏi lí thuyết Hóa học hữu cơ OlympiaVN


Bài 48
Với mỗi phản ứng dưới đây, hãy bổ sung các mũi tên mô tả cơ chế phản ứng và
trả lời các câu hỏi:
i) Có bao nhiêu electron π tham gia vào chuyển hóa;
ii) Thuộc loại phản ứng peri nào: điện vòng [electrocylic], cộng vòng
[cycloaddition], hoặc sigmatropic? Nếu thích hợp, hãy gán các kí
hiệu [m,n] hoặc [m+n] cho phản ứng.
iii) Phản ứng này (giả sử điều kiện nhiệt) là dạng quay cùng chiều
[conrotatory] hay ngược chiều [disrotatory]?
iv) Phản ứng này (giả sử điều kiện nhiệt) là kiểu suprafacial (supra) hay
antarafacial (anta)?
v) Phản ứng này cho phép [thermally allowed] hay không thể diễn ra
[thermally forbidden] trong điều kiện nhiệt?

591 | Câu hỏi lí thuyết Hóa học hữu cơ OlympiaVN


592 | Câu hỏi lí thuyết Hóa học hữu cơ OlympiaVN
Hướng dẫn

593 | Câu hỏi lí thuyết Hóa học hữu cơ OlympiaVN


594 | Câu hỏi lí thuyết Hóa học hữu cơ OlympiaVN
595 | Câu hỏi lí thuyết Hóa học hữu cơ OlympiaVN
Bài 49
Mỗi chuyển hóa sau đều thuộc loại chuyển vị sigmatropic [3,3] Cope hoặc
Claisen. Trong mỗi trường hợp, hãy dự đoán sản phẩm với hóa lập thể chính xác
bằng cách vẽ trạng thái chuyển tiếp dạng ghế tối ưu nhất.

Hướng dẫn
a)

b)

596 | Câu hỏi lí thuyết Hóa học hữu cơ OlympiaVN


c)

597 | Câu hỏi lí thuyết Hóa học hữu cơ OlympiaVN


Bài 50
Phản ứng mở vòng cyclobutene trong điều kiện nhiệt chỉ tạo thành một trong bốn
đồng phân sau. Xác định đồng phân được tạo thành và giải thích.

Hướng dẫn
Đây là phản ứng mở vòng điện vòng 4p. Trong điều kiện nhiệt, phản ứng điện
vòng 4p là dạng conrotatory (cùng chiều). Do đó, hai sản phẩm đối xứng được
phép là các đồng phân (E,Z) và (Z,E). Đồng phân (Z,E) thuận lợi bởi sự án ngữ
không gian: nó có nhóm t-Bu hướng ra ngoài thay vì vào trong.

598 | Câu hỏi lí thuyết Hóa học hữu cơ OlympiaVN


Bài 51
a) Sử dụng mũi tên cong, hãy đề xuất cơ chế của chuyển hóa dưới đây. Phân
loại tất cả các phản ứng pericyclic. (Gợi ý: Giai đoạn đầu tiên là sự tạo thành
trung gian cation).

b) Dựa vào sự đối xứng orbital, hãy giải thích sự tạo thành đồng phân trans 2.
Hướng dẫn
a) Cơ chế phản ứng:

b) Phản ứng peri hóa của pentadienyl cation.

599 | Câu hỏi lí thuyết Hóa học hữu cơ OlympiaVN


Bài 52
Đề xuất cơ chế cho chuyển hóa sau và phân loại tất cả các phản ứng peri hóa. Dự
đoán hóa lập thể của sản phẩm.

Hướng dẫn

600 | Câu hỏi lí thuyết Hóa học hữu cơ OlympiaVN


Bài 53
Đề xuất cơ chế cho chuyển hóa sau và phân loại tất cả các phản ứng peri hóa.

Hướng dẫn

601 | Câu hỏi lí thuyết Hóa học hữu cơ OlympiaVN


Bài 54
Các phản ứng nucleophile của các cyclopropane liên quan đến phản ứng thế kiểu
SN1 dẫn đến sự tạo thành các sản phẩm mở mạch. Nguyên nhân là carbocation
cyclopropyl - được tạo thành khi nhóm rời đi chloride - trải qua phản ứng mở
vòng điện vòng.

a) Trình bày cơ chế để giải thích cho hướng phản ứng.


b) Sự mở vòng trong phản ứng này cùng chiều [conrotatory] hay ngược chiều
[disrotatory]?

Hướng dẫn
a)

b) Phản ứng mở vòng sẽ diễn ra qua carbocation vòng ba cạnh, như trong ý a.
HOMO của carbocation sản phẩm sẽ là Ψ1. Để các orbital cùng phase trong Ψ1,
phản ứng mở vòng cần phải ngược chiều - như dưới đây:

Phản ứng mở vòng carbocation sẽ mang đến hóa lập thể ban đầu cho carbocation
sản phẩm.

602 | Câu hỏi lí thuyết Hóa học hữu cơ OlympiaVN


603 | Câu hỏi lí thuyết Hóa học hữu cơ OlympiaVN
Bài 55
Các endriandric acids là họ hợp chất thiên nhiên phức tạp được phân lập từ một
loài thực vật ở Australia. Các hợp chất này rất không bình thường bởi chúng tồn
tại ở dạng racemic, gợi ý rằng các phản ứng hóa học mà thực vật sử dụng để tạo
thành chúng không liên quan đến các enzyme. Ba trong số các endriandric acids
(D, E, và A) tạo nên từ một tiền chất chung. Các endriandric acid D và E tạo
thành bởi một phản ứng vòng hóa điện vòng nhiệt 8π, sau đó đóng vòng nhiệt 6π.
Đồng phân A được tạo thành khi endriandric acid E trải qua phản ứng Diels-Alder
nội phân tử. Sử dụng các cơ chế phản ứng để dẫn ra cấu trúc của ba đồng phân
(D, E, và A) này.

Hướng dẫn

Vẽ lại cấu trúc:

604 | Câu hỏi lí thuyết Hóa học hữu cơ OlympiaVN


605 | Câu hỏi lí thuyết Hóa học hữu cơ OlympiaVN
Bài 56
Khi đun nóng, (R,2E,4Z)-6-Methylocta-2,4-diene trải qua phản ứng chuyển vị
hydrogen [1,5] thay vì đóng vòng điện vòng. Hãy giải thích quan sát này.

Hướng dẫn
Sản phẩm phản ứng đóng vòng có sức căng vòng lớn và do đó sẽ có năng lượng
rất cao. Tính đối xứng của phản ứng đòng vòng (cùng chiều) sẽ mang hai nhóm
thế alkyl lớn về cùng phía của vòng cyclobutene và gây ra các tương tác cản trở
không gian.

Chuyển vị [1,5] H không có gây ra cản trở không gian nào lớn nên là cơ chế được
ưu tiên.

606 | Câu hỏi lí thuyết Hóa học hữu cơ OlympiaVN


Bài 57
Giải thích sự tạo thành A và B trong phản ứng sau. Xác định hướng quay và đưa
ra tên gọi mỗi phản ứng. Xác định hóa lập thể của vòng dung hợp trong các hợp
chất A và C.

Hướng dẫn

607 | Câu hỏi lí thuyết Hóa học hữu cơ OlympiaVN


Bài 58
Xác định sản phẩm các phản ứng sau:

Hướng dẫn

608 | Câu hỏi lí thuyết Hóa học hữu cơ OlympiaVN


Bài 59
Chuyển hóa sau đây liên quan đến sự chuyển dời “rõ ràng” nhóm methyl. Hãy
trình bày cơ chế phản ứng và phân loại các phản ứng peri hóa (pericyclic) liên
quan.

Hướng dẫn

609 | Câu hỏi lí thuyết Hóa học hữu cơ OlympiaVN


Bài 60
Đề xuất cơ chế cho chuyển hóa sau và phân loại tất cả các phản ứng peri hóa.

Hướng dẫn

610 | Câu hỏi lí thuyết Hóa học hữu cơ OlympiaVN


Bài 61
Trình bày cơ chế phản ứng của các chuyển hóa sau và chỉ rõ các loại phản ứng
peri hóa (pericyclic):

Hướng dẫn
a)

611 | Câu hỏi lí thuyết Hóa học hữu cơ OlympiaVN


b)

612 | Câu hỏi lí thuyết Hóa học hữu cơ OlympiaVN


Bài 62
Trình bày cơ chế phản ứng của chuyển hóa sau và chỉ rõ các loại phản ứng peri
hóa (pericyclic):

Hướng dẫn

613 | Câu hỏi lí thuyết Hóa học hữu cơ OlympiaVN


Bài 63
Trình bày cơ chế phản ứng của chuyển hóa sau và chỉ rõ các loại phản ứng peri
hóa (pericyclic):

614 | Câu hỏi lí thuyết Hóa học hữu cơ OlympiaVN


Hướng dẫn

615 | Câu hỏi lí thuyết Hóa học hữu cơ OlympiaVN


Bài 64
Chuyển hóa dưới đây là giai đoạn cuối trong tổng hợp toàn phần 1-O-
methylforbesione. Trình bày cơ chế phản ứng của chuyển hóa và chỉ rõ các loại
phản ứng peri hóa (pericyclic):

Hướng dẫn

616 | Câu hỏi lí thuyết Hóa học hữu cơ OlympiaVN


Bài 65
Trình bày cơ chế phản ứng của các chuyển hóa sau và chỉ rõ các loại phản ứng
peri hóa (pericyclic):

Hướng dẫn
a)

617 | Câu hỏi lí thuyết Hóa học hữu cơ OlympiaVN


b)

618 | Câu hỏi lí thuyết Hóa học hữu cơ OlympiaVN


Bài 66
Đề xuất cơ chế cho mỗi chuyển hóa sau. Hãy phân loại tất cả các phản ứng peri
hóa và chỉ rõ hóa lập thể của sản phẩm cuối:

Hướng dẫn
a)

b)

619 | Câu hỏi lí thuyết Hóa học hữu cơ OlympiaVN


c)

620 | Câu hỏi lí thuyết Hóa học hữu cơ OlympiaVN


Bài 67
Đề xuất cơ chế cho mỗi chuyển hóa sau và phân loại tất cả các phản ứng peri hóa
liên quan.

Hướng dẫn
a)

621 | Câu hỏi lí thuyết Hóa học hữu cơ OlympiaVN


b)

622 | Câu hỏi lí thuyết Hóa học hữu cơ OlympiaVN


Bài 68
Bây giờ, bạn hẳn đã quen thuộc với tính hữu dụng về mặt tổng hợp của phản ứng
Diels-Alder. Không chỉ là phản ứng tạo thành hai liên kết carbon-carbon, nó còn
có khả năng thiết lập tới bốn trung tâm lập thể liên tiếp. Khi xét đến tầm quan
trọng của phản ứng Diels-Alder, bạn không nên ngạc nhiên khi biết rằng rất nhiều
nỗ lực đã được dành cho việc phát triển các phiên bản bất đối xứng của phản ứng
nhằm tạo thành sản phẩm được làm giàu đối quang [enantioenriched product].
Các chất trợ Evans, được biết đến từ các phản ứng alkyl hóa và aldol hóa bất đối,
cũng có thể được ứng dụng cho các phản ứng Diels-Alder bất đối. Acryloylimide
được hoạt hóa, như dưới đây, bằng cách cộng hợp hai đương lượng Lewis acid
diethylaluminum chloride.

a) Xác định các sản phẩm phụ của phản ứng và giải thích kết quả của việc sử
dụng hai đương lượng Lewis acid trong trường hợp này. Giải thích tác động
của việc hoạt hóa Lewis acid với cấu trúc điện tử và cấu dạng của dienophile
trong phản ứng Diels-Alder.
Bây giờ, hãy xét đến phản ứng với diene như dưới đây:

b) Vẽ trạng thái chuyển tiếp dẫn đến sự tạo thành sản phẩm cộng Diels-Alder X
và đưa ra một lời giải thích hợp lí ngắn gọn.
c) Sử dụng trạng thái chuyển tiếp trên, dự đoán cấu trúc và hóa lập thể của sản
phẩm Y.

Hướng dẫn
a)

623 | Câu hỏi lí thuyết Hóa học hữu cơ OlympiaVN


Sau khi phức hóa trước một trong các nhóm imide carbonyl bởi Et2AlCl, đương
lượng Lewis acid thứ hai sẽ tách chloride anion từ tâm aluminum (nhôm) liên kết
với chất nền, tạo thành phức cation aluminum như trên, phức này làm tăng cường
thêm tính electrophile của dienophile, so với việc chỉ sử dụng một đương lượng
Et2AlCl.
Lewis acid đóng hai vai trò. Thứ nhất, làm hạ thấp năng lượng của LUMO của
alkene; thực tế, nó làm giảm năng lượng LUMO đủ để phản ứng Diels-Alder có
thể diễn ra ở nhiệt độ thấp. Thứ hai, nó tạo phức vòng càng với cả hai nhóm
carbonyl và khóa vị trí của chúng. Amide bị khóa vào cấu hình s-cis.
b) Diene cần phải tấn công từ phía trên, do phía
dưới bị khóa bởi nhóm isopropyl cồng kềnh.
Ngoài ra, diene được định hướng để cực đại hóa
xen phủ orbital thứ cấp và tạo thành sản phẩm
endo.

624 | Câu hỏi lí thuyết Hóa học hữu cơ OlympiaVN


c)

625 | Câu hỏi lí thuyết Hóa học hữu cơ OlympiaVN


Bài 69
Chuyển hóa sau đây là giai đoạn then chốt trong tổng hợp cephalotaxine, một hợp
chất thiên nhiên với các tính chất chống ung thư. Trình bày cơ chế phản ứng của
các chuyển hóa.

Hướng dẫn

626 | Câu hỏi lí thuyết Hóa học hữu cơ OlympiaVN


Bài 70
Trình bày cơ chế của chuyển hóa sau đây:

Hướng dẫn

627 | Câu hỏi lí thuyết Hóa học hữu cơ OlympiaVN


Bài 71
Chất đầu đa vòng A bị phân mảnh khi xử lí với dung dịch acid, tạo thành
carboxylic acid B.

a) Trình bày cơ chế phản ứng của chuyển hóa này.


b) Chỉ rõ các orbital tham gia vào phản ứng phân mảnh then chốt.
Hướng dẫn
a)

b)

628 | Câu hỏi lí thuyết Hóa học hữu cơ OlympiaVN


Bài 72
Đề xuất cơ chế cho các chuyển hóa sau:

Hướng dẫn
a)

b)

629 | Câu hỏi lí thuyết Hóa học hữu cơ OlympiaVN


Bài 73
Một diol ba vòng trải qua phản ứng phân mảnh Grob trong các điều kiện như sau.
Hãy đưa ra cơ chế phản ứng, bắt đầu từ cấu dạng năng lượng cực tiểu của chất
đầu và dự đoán sản phẩm tạo thành, cùng tất cả thông tin hóa lập thể.

630 | Câu hỏi lí thuyết Hóa học hữu cơ OlympiaVN


Hướng dẫn

631 | Câu hỏi lí thuyết Hóa học hữu cơ OlympiaVN


Bài 74
Trình bày cơ chế phản ứng của các chuyển hóa sau:

632 | Câu hỏi lí thuyết Hóa học hữu cơ OlympiaVN


Hướng dẫn
a)

633 | Câu hỏi lí thuyết Hóa học hữu cơ OlympiaVN


b)

c)

634 | Câu hỏi lí thuyết Hóa học hữu cơ OlympiaVN


635 | Câu hỏi lí thuyết Hóa học hữu cơ OlympiaVN
Bài 75
a) Một nhà hóa học đã cố gắng thực hiện chuyển vị Beckmann với oxime A. Xử
lí oxime A với SOCl2 tạo thành trung gian B. Nhà hóa học dự đoán rằng khi
đun nóng trung gian B trong nước sẽ tạo thành sản phẩm C. Hãy đề xuất cấu
trúc của C đã dự đoán (expected product).

b) Thực tế, sản phẩm dự đoán C không được tạo thành. Thay vào đó, trung gian
B phản ứng tạo thành sản phẩm D với công thức phân tử C9H8ClNS. Xác định
cấu trúc sản phẩm D và đưa ra cơ chế cho chuyển hóa tạo thành nó.

Hướng dẫn
a)

636 | Câu hỏi lí thuyết Hóa học hữu cơ OlympiaVN


b)

637 | Câu hỏi lí thuyết Hóa học hữu cơ OlympiaVN


Bài 76
Tính tổng số electron hóa trị của mỗi phức kim loại sau, và xác định số oxid hóa
của kim loại.

638 | Câu hỏi lí thuyết Hóa học hữu cơ OlympiaVN


Hướng dẫn

639 | Câu hỏi lí thuyết Hóa học hữu cơ OlympiaVN


Bài 77
Chuyển hóa cơ kim sau thường được gọi là một “tầng khóa kéo” (zipper cascade),
bởi chất đầu về cơ bản là bị “nén lại” để tạo thành sản phẩm. Trình bày cơ chế
tất cả các bước trong chuyển hóa này. Chú thích chính xác từng biến đổi và
luôn bao gồm số oxid hóa của kim loại. Bạn không cần biểu diễn các mũi tên
cong hoặc vẽ một chu trình xúc tác. Bạn có thể sử dụng [Pd0] để bắt đầu vẽ cơ
chế của mình.

640 | Câu hỏi lí thuyết Hóa học hữu cơ OlympiaVN


Hướng dẫn

641 | Câu hỏi lí thuyết Hóa học hữu cơ OlympiaVN


Bài 78
Manzamine A là một alkaloid được phân lập từ tảo biển, thể hiện hoạt tính kháng
nhiều loại bệnh kí sinh trùng. Hãy đề xuất cơ chế cho chuyển hóa then chốt sau
để tạo thành lõi ba vòng của Manzamine A. Chú thích chính xác từng biến đổi
và luôn bao gồm số oxid hóa của kim loại. Bạn không cần biểu diễn các mũi
tên cong hoặc vẽ một chu trình xúc tác. Bạn có thể sử dụng [Pd0] để bắt đầu vẽ
cơ chế của mình.

642 | Câu hỏi lí thuyết Hóa học hữu cơ OlympiaVN


Hướng dẫn

643 | Câu hỏi lí thuyết Hóa học hữu cơ OlympiaVN


Bài 79
Palladium có thể xúc tác cho phản ứng cyanide hóa các aryl halide. Hãy đưa ra
chu trình xúc tác hoàn chỉnh cho chuyển hóa dưới đây. Chú thích chính xác từng
biến đổi và luôn bao gồm số oxid hóa của kim loại. Mặc dù chuyển hóa này sử
dụng xúc tác palladium(II) nhưng bạn có thể giả sử rằng xúc tác trước tiên bị khử
thành palladium(0) tại chỗ (in situ). Ngoài ra, bạn không cần quan tâm về việc có
bao nhiêu phối tử liên kết đồng thời với xúc tác. Do đó, bạn có thể đơn giản là sử
dụng [Pd]0 để bắt đầu cơ chế của mình.

Hướng dẫn

644 | Câu hỏi lí thuyết Hóa học hữu cơ OlympiaVN


Bài 80
Chuyển hóa cơ kim sau được sử dụng để tạo thành nhân estrone, là một trong ba
loại estrogen. Hãy đưa ra cơ chế hoàn chỉnh với tất các giai đoạn trong chuyển
hóa sau. Chú thích chính xác từng biến đổi và luôn bao gồm số oxid hóa của
kim loại. Bạn không cần biểu diễn các mũi tên cong hoặc vẽ một chu trình xúc
tác. Mặc dù chuyển hóa này sử dụng xúc tác palladium(II) nhưng bạn có thể giả
sử rằng xúc tác trước tiên bị khử thành palladium(0) tại chỗ (in situ). Ngoài ra,
bạn không cần quan tâm về việc có bao nhiêu phối tử liên kết đồng thời với xúc
tác. Do đó, bạn có thể đơn giản là sử dụng [Pd]0 để bắt đầu cơ chế của mình (Pd0
cộng oxid hóa vào vinyl bromide nhanh hơn aryl bromide.) Cuối cùng, chú ý rằng
KOAc được sử dụng như một base để trung hòa sản phẩm phụ HBr.

645 | Câu hỏi lí thuyết Hóa học hữu cơ OlympiaVN


Hướng dẫn

646 | Câu hỏi lí thuyết Hóa học hữu cơ OlympiaVN


Bài 81
Khi thực hiện một ghép cặp Stille dưới khí quyển CO, thì sản phẩm tạo thành là
một ketone. Phản ứng như vậy được gọi là phản ứng ghép cặp Stille carbonyl hóa.
Hãy đưa ra một chu trình xúc tác hoàn chỉnh cho chuyển hóa này. Chú thích
chính xác từng biến đổi và luôn bao gồm số oxid hóa của kim loại. Bạn có thể
sử dụng [Pd0] để bắt đầu vẽ cơ chế của mình.

Hướng dẫn

647 | Câu hỏi lí thuyết Hóa học hữu cơ OlympiaVN


Bài 82

Chuyển hóa trên là một ví dụ của hoán vị enyne, là một biến thể của phản ứng
hoán vị olefin. Hãy đưa ra một cơ chế hoàn chỉnh cho chuyển hóa sử dụng xúc
tác ruthenium này. Bạn không cần vẽ chu trình xúc tác. Để bắt đầu cơ chế, bạn
có thể kí hiệu xúc tác là:

648 | Câu hỏi lí thuyết Hóa học hữu cơ OlympiaVN


Hướng dẫn

649 | Câu hỏi lí thuyết Hóa học hữu cơ OlympiaVN


Bài 83
Trong bài này, chúng ta sẽ nghiên cứu hai phản
ứng hữu ích, được xúc tác bởi hợp chất
rhodium(I), được gọi là xúc tác Wilkinson (biểu
diễn ở hình bên). Dẫu bạn chưa bao giờ thấy xúc
tác Wilkinson trước đây, bạn cũng có thể hình
dung được các phản ứng diễn ra như thế nào bằng
cách kết hợp các phản ứng cơ bản của các kim
loại chuyển tiếp bạn đã biết thành các chu trình
xúc tác. Khi vẽ quy trình xúc tác liên quan đến xúc tác Wilkinson, bạn có thể biểu
diễn xúc tác là [RhI]Cl.
a) Khi có mặt khí H2, xúc tác Wilkinson có thể được sử dụng để hydrogen hóa
một alkene, như trong chuyển hóa dưới đây. Hãy đưa ra một quy trình xúc
tác hoàn chỉnh cho chuyển hóa này. Chú thích chính xác từng biến đổi và
luôn bao gồm số oxid hóa của kim loại. Hãy đảm bảo chu trình xúc tác của
bạn có xem xét đến thực tế rằng cả hai nguyên tử hydrogn đều được đưa đến
cùng một phía của liên kết đôi (cộng syn). (Gợi ý: xúc tác Wilkinson có thể
trải qua phản ứng cộng oxid hóa với khí H2).

b) Khi có mặt một aldehyde và một alkene, xúc tác Wilkinson có thể được sử
dụng để thực hiện phản ứng hydroacyl hóa, như biểu diễn dưới đây. Hãy đưa
ra một chu trình xúc tác hoàn chỉnh cho chuyển hóa này. Chú thích chính xác
từng biến đổi và luôn bao gồm số oxid hóa của kim loại.

650 | Câu hỏi lí thuyết Hóa học hữu cơ OlympiaVN


Hướng dẫn
a)

b)

651 | Câu hỏi lí thuyết Hóa học hữu cơ OlympiaVN


Bài 84
Hãy đề xuất cơ chế hoàn chỉnh cho chuyển hóa cơ kim dưới đây, dẫn đến sự tạo
thành một vòng thơm. Chú thích chính xác từng biến đổi và luôn bao gồm số oxid
hóa của kim loại. Mặc dù chuyển hóa này sử dụng xúc tác palladium(II) nhưng
bạn có thể giả sử rằng xúc tác trước tiên bị khử thành palladium(0) tại chỗ (in
situ). Ngoài ra, bạn không cần quan tâm về việc có bao nhiêu phối tử liên kết
đồng thời với xúc tác. Do đó, bạn có thể đơn giản là sử dụng [Pd]0 để bắt đầu cơ
chế của mình.

Hướng dẫn

652 | Câu hỏi lí thuyết Hóa học hữu cơ OlympiaVN


653 | Câu hỏi lí thuyết Hóa học hữu cơ OlympiaVN
Bài 85
Xét chuyển hóa xúc tác ruthenium sau:

Hãy đưa ra cơ chế hoàn chỉnh cho chuyển hóa trên. Bạn không cần biểu diễn
các mũi tên cong hoặc vẽ một chu trình xúc tác. Để bắt đầu cơ chê,s bạn có thể
kí hiệu xúc tác là:

Hướng dẫn

654 | Câu hỏi lí thuyết Hóa học hữu cơ OlympiaVN


Bài 86
Các phản ứng Heck, ghép cặp Stille và ghép cặp Suzuki đều được xúc tác bởi Pd0.
Như bạn đã thấy, xúc tác Pd0 có thể xúc tác cho các phản ứng này là Pd(PPh3)4.
Tuy nhiên, thay vào đó, các phản ứng này được bắt đầu với một “tiền-xúc tác”
PdII, chất này được xử lí với một tác nhân khử và bị khử thành Pd0 tại chỗ (in
situ). Một tiền-xúc tác PdII phổ biến là PdCl2(PPh3)2, bền trong không khí và dễ
được điều chế từ PdCl2. Tiền xúc tác này có thể được xử lí với nhiều tác nhân
khử, bao gồm ethylene và NEt3, để tạo thành xúc tác hoạt tính Pd0(PPh3)2. Với
mỗi (trong số hai) tác nhân khử trên, hãy đưa ra cơ chế chuyển tiền xúc tác
PdIICl2(PPh3)2 thành xúc tác hoạt tính Pd0(PPh3)2. Chú thích chính xác từng biến
đổi và luôn bao gồm số oxid hóa của kim loại. Bạn không cần biểu diễn các mũi
tên cong hoặc vẽ chu trình xúc tác.

Hướng dẫn
a)

b)

655 | Câu hỏi lí thuyết Hóa học hữu cơ OlympiaVN


Bài 87
a) Khi xử lí hợp chất A với xúc tác Pd0 và methyl acrylate, sẽ tạo thành hợp chất
B. Hãy trình bày cơ chế hoàn chỉnh của chuyển hóa này. Chú thích chính xác
từng biến đổi và luôn bao gồm số oxid hóa của kim loại. Bạn không cần biểu
diễn các mũi tên cong hoặc vẽ chu trình xúc tác.

b) Khi hợp chất C được đưa vào các điều kiện như trên, không tạo thành chất
tương tự D. Thay vào đó, quan sát được một sản phẩm E khác. Hãy đề xuất
cơ chế hoàn chỉnh cho sự tạo thành E từ C, và giải thích sự khác biệt trong
các hướng phản ứng so với ý a.

Hướng dẫn
a)

b) Trung gian Y có một nguyên tử β-hydrogen và do đó có thể trải qua phản ứng
tách syn β-H. Ngược lại, trung gian X (như ở ý a) không có β-hydrogen và do đó
trải qua chuyển vị (migratory insertion) syn với methyl acrylate.

656 | Câu hỏi lí thuyết Hóa học hữu cơ OlympiaVN


657 | Câu hỏi lí thuyết Hóa học hữu cơ OlympiaVN
Bài 88
Hãy đưa ra cơ chế hoàn chỉnh với tất cả các giai đoạn trong chuyển hóa sau đây.
Chú thích mỗi giai đoạn với mô tả bản chất (ví dụ, cộng oxid hóa) và luôn kèm
theo số oxid hóa của kim loại. Bạn không cần biểu diễn các mũi tên cong hoặc
vẽ một chu trình xúc tác. Mặc dù chuyển hóa này sử dụng xúc tác palladium(II)
nhưng bạn có thể giả sử rằng xúc tác trước tiên bị khử thành palladium(0) tại chỗ
(in situ). Sử dụng kí hiêu [Pd]0 cho phức kim loại tham gia vào chuỗi phản ứng.
Cuối cùng, chú ý rằng silver carbonate được sử dụng như một base để trung hòa
sản phẩm phụ HBr.

Hướng dẫn

658 | Câu hỏi lí thuyết Hóa học hữu cơ OlympiaVN


Bài 89
Guanacastepene A là hợp chất được chiết xuất từ một loại nấm Costa Rica, thể
hiện các họa tính kháng sinh hứa hẹn. Chuyển hóa xúc tác ruthenium sau đây
được sử dụng để tạo nên lõi ba vòng của Guanacastepene A. Hãy đề xuất cơ chế
của chuyển hóa này. Bạn không cần vẽ một chu trình xúc tác. Bạn có thể kí hiệu
xúc tác là [Ru]=CHPh để bắt đầu cơ chế của mình.

Hướng dẫn

659 | Câu hỏi lí thuyết Hóa học hữu cơ OlympiaVN


Bài 90
Asteriscanolide là một hợp chất tự nhiên được phân lập từ loài hoa dại Asteriscus
aquaticus. Chuyển hóa xúc tác ruthenium sau đây, được thực hiện khi đun nóng
trong khí quyển ethylene, được sử dụng để tạo nên cấu trúc lõi ba vòng của hợp
chất này. Hãy đề xuất cơ chế của chuyển hóa này. Bạn không cần vẽ một chu
trình xúc tác. Bạn có thể kí hiệu xúc tác là [Ru]=CHPh để bắt đầu cơ chế của
mình.

Hướng dẫn

660 | Câu hỏi lí thuyết Hóa học hữu cơ OlympiaVN


Xác định cấu trúc hợp chất &
Hoàn thành sơ đồ phản ứng
Bài 1
Sự hình thành liên kết đôi C=C có vai trò rất quan trọng trong tổng hợp Hữu cơ
vì từ nối đôi có thể tổng hợp ra được rất nhiều nhóm chức khác nhau.
Các đồng phân hình học của olefin được phân biệt bởi các kí hiệu lập thể E/Z.
Nếu hai nhóm có độ ưu tiên cao hơn (theo sắp xếp của quy tắc Cahn - Ingold -
Prelog, hay CIP) nằm ở hai phía ngược nhau của liên kết đôi, thì đó là cấu hình
E. Đồng phân còn lại có cấu hình là Z.
1. Gọi tên IUPAC (kèm theo kí hiệu lập thể E/Z) của hợp chất sau:

Một trong các phương pháp được dùng nhiều nhất để tạo liên kết đôi carbon-
carbon là sử dụng phản ứng tách β như sau:

Cấu trúc lập thể để phản ứng tách lưỡng phân tử (E2) xảy ra như trên đòi hỏi sự
phân bố hình học dạng trans (trans geometry) của nguyên tử β-hydrogen và nhóm
rời đi (X). Trong đa số trường hợp, sản phẩm chính là olefin nhiều nhóm thế nhất
(ở C=C). Tuy nhiên, trong phản ứng tách của các muối ammonium và sulphonate
thì olefin ít thế nhất mới là sản phẩm chính.
2. Xác định sản phẩm chính (A-D) trong các phản ứng sau. Xác định hóa lập thể
của sản phẩm nếu có thể.

i.

661 | Câu hỏi lí thuyết Hóa học hữu cơ OlympiaVN


ii.

iii.

iv.

Phản ứng tách nhiệt là một hướng tổng hợp khác để tạo ra alkene. Phản ứng được
thực hiện đồng bộ qua một trạng thái chuyển tiếp dạng vòng. Phản ứng nhiệt phân
thường được tiến hành với: carboxylate ester, sulfoxide, xanthate, … Các phản
ứng này diễn ra theo cơ chế syn, với nguyên tử hydrogen và nhóm X tách ra cùng
một phía.
3. Xác định sản phẩm chính (F-H) trong các phản ứng sau. Xác định hóa lập thể
của sản phẩm nếu có thể.

i.

ii.

Các phản ứng phân mảnh, tương tự phản ứng tách β, cũng được sử dung để tổng
hợp các olefin. Dưới đây là một ví dụ:

662 | Câu hỏi lí thuyết Hóa học hữu cơ OlympiaVN


Dưới đây là chuỗi phản ứng để điều chế một chất trung gian cho quá trình tổng
hợp juvenile hormone – có chức năng điều hóa sinh lý côn trùng.
4. Xác định các sản phẩm (I-K), chỉ rõ hóa lập thể, trong chuỗi phản ứng sau.

Một trong các hướng tổng hợp E-olefin phổ biến là sử dụng phản ứng olefin hóa
Julia, đi từ các hợp chất sulfone có một nhóm rời đi tốt ở vị trí β.

Các sphingolipid là nhóm hợp chất thiên nhiên có vai trò bảo vệ bề mặt tế bào
khỏi các yếu tố nguy hiểm từ môi trường. Dưới đây là sơ đồ tổng hợp tiền chất
của sphingosin - chất tạo ra bộ khung của sphingolipid.
5. Xác định các sản phẩm L-N.

Có thể cộng hợp dễ dàng các diborane vào alkene để điều chế các hợp chất borane
hữu cơ dạng monoalkyl, dialkyl hoặc trialkyl. Borane hữu cơ có thể chuyển hóa
thành các liên kết C-O, C-N, C-C qua phản ứng của nucleophile, theo sau đó là
sự chuyển vị như trong sơ đồ sau đây:

6. Hoàn thành chuỗi phản ứng sau bằng cách xác định các hợp chất O-Q.

663 | Câu hỏi lí thuyết Hóa học hữu cơ OlympiaVN


7. Viết công thức cấu tạo các chất R-T trong các phản ứng sau và xác định hóa
lập thể của chúng nếu có thể.
i.

3 đương lượng
ii.

Các olefin thường có tính nucleophilic (ái nhân), nhưng khi có nhóm hút electron
gắn vào liên kết bội thì nó sẽ có tính electrophilic (ái điện tử). Các olefin này dễ
xảy ra phản ứng thế thông qua hệ liên hợp khi có các nhóm rời đi tốt ở vị trí β.
8. Xác định sản phẩm của phản ứng sau:

9. Hoàn thành sơ đồ tổng hợp thuốc chống loét Tagamet sau bằng cách xác định
công thức cấu tạo các chất U-W.

664 | Câu hỏi lí thuyết Hóa học hữu cơ OlympiaVN


Hướng dẫn
1. 3-E, 5-Z-3-bromo-5-chloro-4-cyclohexylocta-3, 5-diene-7-ynal
2.

3.

665 | Câu hỏi lí thuyết Hóa học hữu cơ OlympiaVN


4.

666 | Câu hỏi lí thuyết Hóa học hữu cơ OlympiaVN


5.

6.

7.

8.

9.

667 | Câu hỏi lí thuyết Hóa học hữu cơ OlympiaVN


Bài 2
Đọc và trả lời các câu hỏi từ (109)-(123).
Thường thì, khi biểu diễn công thức cấu tạo, các nguyên tử carbon và hydrogen
liên kết với carbon thường bị lược bỏ - như cách biểu diễn trong hình dưới đây -
trừ khi chúng có đặc trưng cụ thể. Ngoài ra, các kí hiệu R và E trong công thức
cấu tạo là chỉ các nguyên tử hoặc nhóm nguyên tử. Các chỉ dẫn cũng được thêm
vào khi cần thiết.

Các phân tử hữu cơ có tên là benzene, trong đó 6 nguyên tử nối với nhau thành
vòng và tạo thành một lục giác đều, là phần cấu trúc quan trọng có trong bộ khung
của nhiều hợp chất hữu cơ. Trong bài tập này, chúng ta tập trung vào việc làm thế
nào để tạo nên lục giác đều này (gọi là vòng benzene) bởi phương pháp hóa học,
và chúng ta sẽ tìm hiểu cách xác định các hợp chất hữu cơ bằng phương pháp
khối phổ.
Giáo trình phổ thông cho biết các phân tử hữu cơ có 2 nguyên tử carbon nối với
nhau bởi liên kết ba, gọi là acetylene, sẽ tam hợp hóa [trimer hóa] tạo thành vòng
benzene (hình 1). Từ khi phản ứng được nhà hóa học người Pháp M. P. E.
Berthelot phát hiện vào năm 1866, nhiều xúc tác (chất thúc đẩy tiến trình [tốc độ]
phản ứng) đã được phát triển để thực hiện phản ứng hiệu quả và chọn lọc. Đặc
biệt, nhiều xúc tác phát triển trên cơ sở phức kim loại đã được W. J. Reppe báo
cáo vào năm 1948.

Hình 1
Ngoài acetylene đơn giản, các phân tử hữu cơ với liên kết ba (như propyne, …)
cũng có cách thức phản ứng tương tự, nhưng nhiều hợp chất sẽ được tạo thành
tùy thuộc vào định hướng của các phân tử phản ứng (định hướng R trong hình
2). Ví dụ, khi một phân tử có liên kết ba như trong hình 2 phản ứng, sử dụng xúc
tác chứa molybdenum (Mo), thì có 2 loại phân tử được tạo thành.

668 | Câu hỏi lí thuyết Hóa học hữu cơ OlympiaVN


Hình 2
Câu hỏi ア: Trong phản ứng ở hình 2, khi định hướng của phân tử khi phản ứng
không bị ảnh hưởng bởi R thì tỉ lệ tối giản của sản phẩm phản ứng là: I : II =
(109) : (110)
Trong hình 3, khi các kiểu phân tử khác nhau phản ứng đồng thời thì cũng có
nhiều phân tử được tạo thành.

Hình 3
Câu hỏi イ: Khi 2 loại phân tử phản ứng đồng thời, như trong hình 3, thì số loại
phân tử có thể được tạo thành là (111), bao gồm cả 2 kiểu đã liệt kê trong hình.
Câu hỏi ウ: Biết rằng trong phản ứng ở hình 3, nếu sử dụng một xúc tác nhất
định chứa iridiium (Ir), thì có thể tổng hợp chọn lọc với chỉ duy nhất một hợp
chất được tạo thành. Khối lượng phân tử của hợp chất này là 370. Hãy chọn trong
số các chất (từ ①〜⑧) dưới đây cấu trúc của hợp chất này. (112)

Phản ứng này cũng có thể được áp dụng với các chất có đồng thời 3 liên kết ba
(triyne) trong phân tử.
Câu hỏi エ: Trong số các phương án ①〜⑥, hãy chọn ra sản phẩm có thể được
tạo thành khi các liên kết 3 của hợp chất (triyne) sau đây phản ứng với nhau. (113)

669 | Câu hỏi lí thuyết Hóa học hữu cơ OlympiaVN


Phản ứng này thường được sử dụng để tổng hợp các hợp
chất thiên nhiên. Ví dụ, bộ khung của hợp chất thiên
nhiên alcyopterosin E có thể được tạo thành bởi phương
pháp này.
Câu hỏi オ: Tôi muốn tổng hợp triyne phù hợp để điều
chế alcyopterosin E bởi phản ứng ester hóa. Hãy chọn
trong số các phương án dưới đây, carboxylic acid và alcohol có thể sử dụng. Chú
ý rằng trong phản ứng ester hóa thì 1 phân tử nước được tách từ -COOH của
carboxylic và -OH của alcohol, tạo thành liên kết ester -COO-.
Carboxylic acid (114)

Alcohol (115)

Đến lúc này thì tôi đã thấy một phương pháp để tạo nên benzene từ acetylene.
Liệu bạn có thể tạo nên benzene từ alkane không?
Một phương pháp điều chế benzene từ nguyên liệu đầu hexane (C6H14) sử dụng
hệ xúc tác chứa iridium (Ir) đã được báo cáo. Trong phản ứng này, nhiều phân tử
hydrogen đã bị tách loại khỏi hexane để tạo thành benzene, nhưng một alkene
(3,3-dimethyl-1-butene) đã được thêm vào để đóng vai trò chất nhận hydrogen
(hình 4).

670 | Câu hỏi lí thuyết Hóa học hữu cơ OlympiaVN


Hình 4
Câu hỏi カ: Khi 1 phân tử hexane có thể tạo thành 1 phân tử thì có bao nhiêu
phân tử 3,3-dimethyl-1-butene đã phản ứng? (116)
Phản ứng này được cho là diễn ra qua các hợp chất trung gian A đến D (mỗi trung
gian này là hydrocarbon).

Hình 5 biểu diễn lượng hexane và sản phẩm quan sát thấy trong phản ứng trên
trục hoành (tỉ lệ lượng chất khi lấy lượng hexane ở thời điểm ban đầu là 100) theo
thời gian phản ứng - lấy từ trục tung. Các đồ thị (a)-(c) là benzene hoặc trung gian
A hoặc trung gian B, còn các đồ thị được biểu diễn bởi các dấu ♦ và x lần lượt là
hexane và các trung gian khác.

Hình 5
Chú thích: 各物質の物質量の相対値 = giá trị lượng chất tương đối (%) của
mỗi chất; 反応時間 (h) = thời gian phản ứng (giờ)
Câu hỏi キ: Hãy cho biết các đồ thị (a)-(c) tương ứng với những chất nào trong
số benzene, trung gian A và trung gian B. Chọn ra phương án phù hợp dưới đây.
(117)

671 | Câu hỏi lí thuyết Hóa học hữu cơ OlympiaVN


Chú thích: ベンゼン = benzene; 中間体 A = trung gian A; 中間体 B = trung
gian B
Câu hỏi ク: Phản ứng nào sau đây tạo thành cấu trúc vòng? (118)
① từ hexane thành trung gian A
② từ trung gian A thành trung B
③ từ trung gian B thành trung C
④ từ trung gian C thành trung D
⑤ từ trung gian D thành trung benzene
Khi thay đổi chất đầu từ hexane thành octane (C8H18) thì có 2 loại sản phẩm được
tạo thành như trong phương trình (1) và tỉ lệ sản phẩm là o-xylene:ethylbenzene
= 87:13.

Mặt khác, khi chất đầu là decane (C10H22) thì có 3 loại sản phẩm được tạo thành
như trong phương trình (2) và tỉ lệ sản phẩm là 1-methyl-2-propylbenzene: 1,2-
diethylbenzene: butylbenzene = 86:10:4.

Dựa vào phản ứng của octane và decane trong phương trình (1) và (2) để trả lời
câu hỏi dưới đây.
Câu hỏi ケ:
(a) Nếu chất đầu là dodecane (C12H26) thì có bao nhiêu loại sản phẩm được tạo
thành? (119)

672 | Câu hỏi lí thuyết Hóa học hữu cơ OlympiaVN


(b) Xác định các nhóm thế (120) và (121) của cấu trúc được cho là sản phẩm
chính.

(c) Xác định các nhóm thế (120) và (121) của cấu trúc được cho là tạo thành ít
nhất?

Hướng dẫn

(または = hoặc)
Gợi ý cho một số câu hỏi
Câu hỏi イ: Các cách để tạo thành các sản phẩm I và II.

673 | Câu hỏi lí thuyết Hóa học hữu cơ OlympiaVN


Câu hỏi イ:

Câu hỏi オ: ssCó thể phân tích tổng hợp ngược để tìm các nguyên liệu đầu.

Câu hỏi ケ:
Từ phản ứng của octane và decane, có thể thấy đặc trưng của phản ứng này là
không có sự thay đổi về số lượng carbon trong vòng (6) và sản phẩm tạo thành là
các dẫn xuất benzene một nhóm thế hoặc thế 1,2- (với các nhóm thế đều là alkyl
khong phân nhánh). Do đó, có thể dự đoán rằng có 4 sản phẩm hexylbenzene, 1-
methyl-2-pentylbenzene, 1-butyl-2-ethylbenzene và 1,2-dipropylbenzene. Tuy
nhiên, có một sản phẩm - dù thực tế không tạo thành, nhưng chỉ từ các dữ kiện đã
cho thì cũng không thể bác bỏ khả năng diễn ra phản ứng vòng hóa tiếp nhóm
hexyl để tạo thành biphenyl. Vậy nên câu trả lời là 5 cũng được chấp nhận.

674 | Câu hỏi lí thuyết Hóa học hữu cơ OlympiaVN


1-methyl-2-pentylbenzene có lượng lớn nhất [sản phẩm chính] và hexylbenzene
có lượng ít nhất.

675 | Câu hỏi lí thuyết Hóa học hữu cơ OlympiaVN


Bài 3
Cấu trúc của benzene đã làm các nhà hóa học bối rối suốt một thời gian dài, kể từ
khi nó được khám phá vào năm 1825. Một số cấu trúc được đề nghị cho benzene
đều là các hệ thống vòng rất căng. Trong những thập kỉ gần đây, các nhà hóa học
Hữu cơ đã tổng hợp được nhiều hợp chất đa vòng căng bất thường như vậy. Một
chiến lược quen thuộc cho các quá trình tổng hợp này là sử dụng các phản ứng
pericyclic. Pericyclic là các phản ứng mà dòng electron chuyển dịch một cách
đồng bộ thông qua một trạng thái chuyển tiếp vòng, tức trong tiến trình đó sự tạo
thành và phá vỡ liên kết diễn ra đồng thời. Trong các phản ứng này, hóa lập thể
của các nhóm thế trong chất đầu đóng vai trò quyết định tới hóa lập thể của sản
phẩm.
Dưới đây là sự chuyển hóa qua lại giữa cis-3,4-dimethylcyclobutene (A) và
2E,4Z-hexadiene (B) khi đun nóng.

Dewar đã đề nghị cấu trúc C cho benzene (gọi là Dewar benzene). Cấu trúc này
gồm hai vòng xiclobutan ngưng tụ với nhau:

Để làm giảm sức căng vòng, C dễ bị chuyển vị dưới tác dụng nhiệt, tạo thành hợp
chất đơn vòng D.
1. Xác định công thức cấu tạo và hóa lập thể của D. Gọi tên D theo danh pháp
IUPAC, sử dụng kí hiệu lập thể E/Z.
Điều thú vị là dù có sức căng rất lớn nhưng hợp chất C không chuyển hóa ngay
thành hợp chất E (gọi là cấu trúc Kekule) vốn rất bền.
2. Cho C phản ứng với một acid vô cơ thu được E. Đề xuất một cơ chế phản ứng
(sử dụng mũi tên cong) cho chuyển hóa này.
Chì tetraacetate (LTA) là một tác nhân hay dùng để decaboxyl hóa oxy hóa các
dicarboxylic acid có hai nhóm COOH kế cận nhau để tạo thành alkene. Dự đoán
công thức cấu tạo của hợp chất F (C8H8O4) có thể phản ứng với LTA tạo thành
C.

676 | Câu hỏi lí thuyết Hóa học hữu cơ OlympiaVN


Ladenburg cũng đã đề nghị một cấu trúc khác cho benzene (gọi là Ladenburg
benzene), sau này được biết đến với tên gọi Prismane (H). Cấu trúc này cố gắng
giải thích cho sự hình thành một sản phẩm thế mono với benzene và ba đồng phân
thế 2 lần của benzene.
Quang phân G tạo thành một lượng nhỏ H.

4. Xác định công thức cấu tạo của H.


5. Xác định công thức cấu tạo dạng Ladenburg của tất cả các đồng phân
dibromobenzene.
Một cấu trúc khác được đề nghị cho benzene là benzvalene (I) (còn được gọi là
Hückel benzene).

1
H-NMR là công cụ phổ quan trọng để xác định các nguyên tử hydrogen trong
các cấu trúc hóa học khác nhau. Mỗi loại nguyên tử hydrogen tạo ra một tín hiệu
đặc trưng trên phổ.
6. Trong phổ 1H-NMR của I có bao nhiêu peak?
7. Xác định các chất J, K trong chuỗi phản ứng tổng hợp benzvalene (I) sau:

Trimethylenecyclopropane (N) là một đồng phân khác của benzene, có thể được
tổng hợp từ chất L theo sơ đồ sau.

677 | Câu hỏi lí thuyết Hóa học hữu cơ OlympiaVN


8. Xác định các chất M, N.
9. Vẽ cấu trúc các đồng phân lập thể của L.
10. Dự đoán công thức cấu tạo của sản phẩm O tạo thành từ phản ứng Diels-Alder
giữa N và acetylene.
Hướng dẫn
1.

2.

3.

678 | Câu hỏi lí thuyết Hóa học hữu cơ OlympiaVN


4.

5.

6. ba.
7.

8.

9.

679 | Câu hỏi lí thuyết Hóa học hữu cơ OlympiaVN


10.

680 | Câu hỏi lí thuyết Hóa học hữu cơ OlympiaVN


Bài 4
Khái niệm độ bất bão hòa (trước đây gọi là số đương lượng liên kết đôi - Double
Bond Equivalent, DBE), là tổng số liên kết π hoặc vòng trong phân tử, được sử
dụng rất nhiều trong các tính toán để xác định công thức hợp chất hữu cơ.
Ví dụ, quý ngài Mr. Bond dưới đây có 007 DBE (độ bất bão hòa bằng 7)*

*Đây là một cách chơi chữ, liên tưởng tới nhân vật điệp viên 007, James Bond,
vốn là một người Anh.
a) Công thức tổng quát của alkane là CnH2n+2. Hãy xác định công thức tổng quát
của
i) alkene hoặc cycloalkane
ii) alkyne
iii) cycloalkene
iv) dialkyne
Sự xuất hiện của mỗi liên kết π hoặc vòng sẽ làm phân tử hydrocarbon giảm đi 2
nguyên tử hydrogen so với alkane tương ứng.
b) Bảng dưới đây cung cấp các giá trị về số vòng, liên kết đôi và ba có thể có
trong các phân tử có DBE bằng 1, 2. Hãy mở rộng bảng này cho các trường hợp
DBE bằng 3, 4.

681 | Câu hỏi lí thuyết Hóa học hữu cơ OlympiaVN


Độ bất bão hòa của hydrocarbon có thể tính từ số nguyên tử hydrogen thực tế (gọi
là X) trong phân tử với số hydrogen (gọi là A) trong alkane có cùng số carbon:
1
DBE  A  X
2
Ví dụ: Để tính độ bất bão hòa của cyclohexene, C6H10 thì cần xác định số nguyên
tử hydrogen trong alkane có cùng số carbon. Đó là C6H14, do đó:
1
DBE  14  10   2
2
c) Tính độ bất bão hòa của

Lưu ý: Trong các phân tử đa vòng, có thể tính số vòng bằng cách đếm số nhát cắt
tối thiểu qua các liên kết để tạo ra cấu trúc không vòng.
Có thể sử dụng các kĩ thuật quang phổ, như NMR (cộng hưởng từ), để phân biệt
các loại DBE (của liên kết đôi, ba hay vòng?)
Trong phổ 13C NMR, số tín hiệu phổ phụ thuộc vào số lượng các nguyên tử carbon
có cấu tạo khác nhau trong phân tử. Ví dụ, trong benzene, các nguyên tử carbon
tương đương nhau và phổ 13C NMR chỉ có duy nhất 1 peak. Trong khi đó, 1,3-
dimethylbenzene có 5 loại carbon khác nhau.

Trong benzene, tất cả các Trong 1,3-dimethylbenzene, vẽ một mặt phẳng


nguyên tử carbon tương đương đối xứng (hoặc trục quay) chạy qua phân tử
nhau, do vậy trong phổ đồ chỉ (biểu diễn bằng đường nét đứt) thì thấy có 5
xuất hiện duy nhất 1 peak ở loại carbon khác nhau trong phân tử. Do vậy,
128 ppm. trong phổ đồ xuất hiện 5 peak.

Dưới đây là vùng phổ đặc trưng của một số loại nguyên tử carbon:
• carbon trong liên kết ba alkyne: 70-100 ppm.

682 | Câu hỏi lí thuyết Hóa học hữu cơ OlympiaVN


• carbon trong liên kết đôi: 100-160 ppm.
• carbon no (tạo ra 4 liên kết đơn với hydrogen hoặc nguyên tử carbon
khác): 0-50 ppm.
Các allene (dạng R2C=C=CR2) là trường hợp khá bất thường. Nguyên tử carbon
trung tam tạo ra một peak trên 200 ppm, còn các nguyên tử carbon liên kết trực
tiếp với carbon trung tâm (còn gọi là nguyên tử carbon “cạnh sườn”) thì nằm cùng
khoảng với các nguyên tử carbon của liên kết ba.
• carbon trung tâm của allen: >200 ppm.
• carbon ở cạnh sườn: 70-100 ppm.
Khi sử dụng các kĩ thuật NMR tiên tiến, ngoài việc xác định được có bao nhiêu
nguyên tử carbon thuộc cùng một loại thì còn có thể biêt có bao nhiêu nguyên tử
hydrogen gắn với mỗi carbon bất kì. Chúng ta kí hiệu (CH3), (CH2), (CH) hoặc
(C) cho các nguyên tử carbon gắn với 3, 2, 1 hoặc 0 nguyên tử hydrogen.
Ví dụ, phổ đồ của 1,3-dimethylbenzene có thể tóm tắt như sau:
2 × 138 (C), 130 (CH), 128 (CH), 2 × 126 (CH), 2 × 21 (CH3)
Các dữ liệu sau được lấy từ phổ 13C NMR của các đồng phân có cùng công thức
C8H8 (5 DBE).

Hãy hoàn thành bảng dữ liệu về số loại nguyên tử carbon, từ đó hãy cho biết có
bao nhiêu liên kết đôi, ba và vòng trong mỗi phân tử.
Dự đoán thông tin cấu
Số nguyên tử carbon thuộc loại
trúc
Số Số
Liên kết Liên kết Liên Trung Cạnh
liên liên Số
ba đôi kết tâm sườn
kết kết vòng
(alkyne) (alkene) đơn allene allene
ba đôi

683 | Câu hỏi lí thuyết Hóa học hữu cơ OlympiaVN


A
B
C
D
E
F
G
H
I
Dự đoán công thức cấu tạo của các chất A-I.

684 | Câu hỏi lí thuyết Hóa học hữu cơ OlympiaVN


Hướng dẫn
a)

b)

c)

d)

685 | Câu hỏi lí thuyết Hóa học hữu cơ OlympiaVN


686 | Câu hỏi lí thuyết Hóa học hữu cơ OlympiaVN
687 | Câu hỏi lí thuyết Hóa học hữu cơ OlympiaVN
Bài 5
Các nhà hóa học Hữu cơ đã tổng hợp được nhiều hợp chất có tính chất bất thường,
ví dụ như những hợp chất có sức căng trong phân tử rất lớn. Nhiều hợp chất trong
số đó đã được đặt tên dựa vào sự tương đồng của cấu tạo phân tử với các vật thể
quanh ta như basketane (giống cái giỏ), pagodane (giống ngôi chùa), penguinone
(giống chim cánh cụt), garudane (giống chim Ca Câu La, hay Kim Sí Điểu – chim
thần trong Ấn Độ giáo), …. Samanene (4) (bắt nguồn từ tiếng Phạn của Ấn Độ:
suman nghĩa là bông hoa) cũng là một hợp chất được đặt tên theo cách như vậy
vì phân tử của nó có dạng hình bông hoa. Quá trình tổng hợp chất (4) bắt đầu từ
một hợp chất hai vòng ngưng tụ đơn giản là norbornadiene (1). Trimer hóa (1) sẽ
tạo ra hợp chất trung gian quan trọng (2) (C21H18).

1. Xác định công thức cấu tạo hai đồng phân lập thể có thể có của (2).
Khi có xúc tác Ru, (2) chuyển vị thành (3) - là tiền chất của Sumanene. Oxid hóa
(3) với tác nhân phù hợp, thu được (4).
2. Viết công thức cấu tạo của (3).
Oxid hóa (4) bằng KMnO4 trong môi trường kiềm, thu được (5).
3. Viết công thức cấu tạo của 5 (C21H6O3).
Cubane (10) là một phân tử bị kéo căng rất thú vị khác. Cubane được tổng hợp
theo chuỗi phản ứng sau:

688 | Câu hỏi lí thuyết Hóa học hữu cơ OlympiaVN


4. Phản ứng của ethylene glycol và (6) xảy ra chọn lọc vào nhóm carbonyl (a) khi
i) nhóm carbonyl (b) liên hợp với liên kết đôi.
ii) nhóm carbonyl (a) ít bị cản trở không gian.
iii) nhóm carbonyl (b) ít bị cản trở không gian.
iv) liên kết đôi cạnh nhóm carbonyl (a) hỗ trợ phản ứng.
Dimer hóa hợp chất X bằng phản ứng Diels-Alder thu được chất (6). Chất X được
tạo ra từ cyclopentanone (11) theo chuỗi phản ứng sau:

689 | Câu hỏi lí thuyết Hóa học hữu cơ OlympiaVN


**NBS là tác nhân bromo hóa chọn lọc carbon ở vị trí allyl.
5. Xác định công thức cấu tạo các sản phẩm trung gian (12)-(15) và X.
Chuyển vị Favorskii là phản ứng chuyển α-haloketone thành carboxylic acid khi
có xúc tác base.

Giai đoạn chuyển hóa (8)  (9) trong tổng hợp cubane là chuyển vị Favorskii.
6. Xác định công thức cấu tạo của tiểu phân trung hòa điện tạo thành trong giai
đoạn chuyển hóa (8)  (9).
Chloro hóa cubane tạo ra dichlorocubane.
7. Viết công thức cấu tạo tất cả các đồng phân của dichlorocubane.
8. Cubane là

690 | Câu hỏi lí thuyết Hóa học hữu cơ OlympiaVN


(i) hợp chất 4 vòng. (ii) hợp chất 5 vòng. (iii) hợp chất 6 vòng.(iv) hợp
chất 8 vòng.
Cũng giống như cubane, propellane (C5H6) là một phân tử rất căng đầy thú vị.
Chất này có thể được tổng hợp từ trung gian carbene.
Khi đun nóng haloform, là phân tử có chứa hydrogen acid, với dung dịch kiềm sẽ
tạo ra một tiểu phân trung gian thuộc nhóm carbene. Carbene là một tiểu phân có
hóa trị hai và một cặp e tự do.
9. Viết phương trình phản ứng giữa bromoform với NaOH, tạo thành carbene
(16).
Dưới đây là chuỗi phản ứng điều chế một chất trung gian (19) được dùng để tổng
hợp propellane.

10. Xác định công thức các hợp chất từ (17)-(19).


Carbene phản ứng với nối đôi C=C tạo thành cyclopropane. Carbene (16) phản
ứng với (19) tạo thành hợp chất (20).
11. Xác định công thức cấu tạo của chất (20).
Xử lí chất (20) với base mạnh (CH3Li) thì xảy ra sự tách halogen, tạo ra
propellane.
12. Xác định công thức cấu tạo của propellane.

691 | Câu hỏi lí thuyết Hóa học hữu cơ OlympiaVN


Hướng dẫn
1.

2.

3.

4. i) nhóm carbonyl (b) liên hợp với liên kết đôi.

692 | Câu hỏi lí thuyết Hóa học hữu cơ OlympiaVN


5.

6.

7.

8. ii) Hợp chất 5 vòng.


9.

693 | Câu hỏi lí thuyết Hóa học hữu cơ OlympiaVN


10.

11.

12.

694 | Câu hỏi lí thuyết Hóa học hữu cơ OlympiaVN


Bài 6
Carbocation là tiểu phân trung gian có nguyên tử carbon mang điện tích. Chúng
được tạo thành trong nhiều phản ứng hóa học, bao gồm các quá trình tổng hợp
hợp chất thiên nhiên.
1. Cho biết dạng lai hóa của nguyên tử carbon dương điện trong carbocation dưới
đây.

2. Sắp xếp các carbocation sau theo thứ tự độ bền giảm dần:

3. Hãy dự đoán xem liệu những chuyển hóa sau có đi qua tiểu phân carbocation
trung gian không?

Carbocation sinh ra tong phản ứng hóa học thường bị chuyển vị để tạo thành các
tiểu phân bền hơn, như trong ví dụ sau:

695 | Câu hỏi lí thuyết Hóa học hữu cơ OlympiaVN


4. Xác định công thức cấu tạo của carbocation C tạo ra ban đầu.
5. C chuyển vị thành carbocation D bền hơn. Xác định công thức cấu tạo của D.
6. Để tạo ra hợp chất B thì carbocation D đã tách loại tiểu phân nào?
7. B là hợp chất
a) đơn vòng.
b) hai vòng.
c) ba vòng.
d) bốn vòng.
8. Xác định công thức cấu tạo của sản phẩm tạo thành khi cho B phản ứng với
HBr có mặt peroxide.
Có nhiều cách tạo ra carbocation. Một chuỗi phản ứng liên quan đến cacbocation
và sự chuyển vị được trình bày dưới đây:

9. Xác định công thức cấu tạo các chất F, G, H.


Trong phản ứng của một số phân tử, carbocation tạo ra ban đầu sẽ trải qua một
chuỗi chuyển vị nối tiếp nhau để tạo thành sản phẩm. I là một phân tử như vậy.

696 | Câu hỏi lí thuyết Hóa học hữu cơ OlympiaVN


10. Xác định công thức cấu tạo của sản phẩm J tạo thành khi cho chất I thực hiện
một chuỗi chuyển vị trong môi trường acid.
Terpene là các hợp chất thiên nhiên có nguồn gốc thực vật. Terpen K, một thành
phần của dầu sả, là tiền chất để tổng hợp một số hợp chất được dùng trong nước
hoa. Chuỗi tổng hợp dưới đây có sự tạo thành của một tiểu phân carbocation trung
gian.

11. Xác định công thức cấu tạo các chất L, M, N. Biết L phản ứng được với hai
đương lượng bromine.
Chuỗi tổng hợp này cũng tạo ra một sản phẩm P, là đồng phân vị trí của O.
12. Xác định công thức cấu tạo của P.
13. Tính số đồng phân quang học có thể có của O.
14. Tính số đồng phân quang học có thể có của P.
15. Xác định các sản phẩm từ phản ứng ozone phân liên kết đôi anken của O.

697 | Câu hỏi lí thuyết Hóa học hữu cơ OlympiaVN


Hướng dẫn
1. sp2
2. IV > III > I > II
3. i. Có; ii. Không; iii. Có
4.

5.

6. proton
7. hai vòng (bicyclic).
8.

9.

698 | Câu hỏi lí thuyết Hóa học hữu cơ OlympiaVN


10.

11.

12.

13. 2.
14. 4.
15.

699 | Câu hỏi lí thuyết Hóa học hữu cơ OlympiaVN


Bài 7
Nhiều phản ứng hữu cơ có sự tạo thành các tiểu phân trung gian như carbocation,
carbanion, gốc carbon tự do, carbene, … Cấu trúc và độ bền của các tiểu phân
trung gian là những yếu tố quyết định cơ chế phản ứng. Ví dụ, trong một phản
ứng có sự hình thành của carbocation thì độ bền của carbocation sẽ quyết định
tốc độ phản ứng.
Định đề Hammond là công cụ quan trọng để nghiên cứu về trạng thái chuyển tiếp.
Định đề này chỉ ra rằng trạng thái chuyển tiếp có cấu trúc gần giống với hệ có
năng lượng tự do gần nó hơn.
1. Sắp xếp các olefin sau theo thứ tự đúng về tốc độ phản ứng cộng HI.

Chuyển vị carbocation là phản ứng khá phổ biến. Nhóm bị chuyển vị thường là
nhóm giàu electron. Do phản ứng chuyển vị nên thi thoảng vẫn có một sản phẩm
khác so với dự kiến được tạo thành.
Cho hợp chất A phản ứng với một Bronsted acid thu được sản phẩm C. Trong
phản ứng này, B là tiểu phân trung gian tạo ra qua quá trình chuyển vị.
2. Xác định B, C.

Cho hợp chất D phản ứng với một lượng nhỏ clo dưới tác dụng của tia tử ngoại
(UV) ở diều kiện thường, tạo ra một hỗn hợp sản phẩm.
3. Xác định sản phẩm chính của phản ứng.

700 | Câu hỏi lí thuyết Hóa học hữu cơ OlympiaVN


Cho các hợp chất sau:

4. Hãy cho biết những chất nào


i. có tính thơm?
ii. phản thơm?
iii. không thơm?
iv. không thơm nhưng có cấu trúc cộng hưởng có tính thơm?
v. không thơm nhưng có base liên hợp có tính thơm?
vi. không thơm, có pKa xấp xỉ -3,8.
Các hợp chất thơm rất khó bị khử. Tuy nhiên, nhà hóa học người Úc, A.J.Birch,
đã phát triển thành công một phương pháp để khử các hợp chất thơm thành diene
không liên hợp bằng phản ứng với Li/K/Na trong ammonia lỏng, có mặt alcohol.
Ví dụ, có thể khử benzene thành 1,4-cyclohexadiene. Phương pháp này được gọi
là phản ứng khử Birch. Trong phản ứng, nguyên tử kim loại nhường 1 electron
cho vòng thơm để tạo thành anion gốc (tiểu phân trung gian dạng anion có 1

701 | Câu hỏi lí thuyết Hóa học hữu cơ OlympiaVN


electron chưa ghép cặp). Trong tiểu phân trung gian này, vị trí tương đối của anion
và gốc là 1,4.
Năm 1944, Birch đã thực hiện phản ứng khử 3-methylanisole (3-
methylmethoxybenzene), thu được sản phẩm F. Đun nóng F với acid vô cơ loãng,
thu được chất G (C7H10O).
5. Xác định các công thức cấu tạo có thể có của F.
6. Xác đinh các công thức cấu tạo có thể có của G.
Chất G thu được trong thực tế không phải là hợp chất quang hoạt. Khi phản ứng
với Br2, G tạo ra H. Đun nóng H với KOH trong alcohol, thu được I (C7H9BrO).
7. Xác định G, H, I.
E.Vogel nổi tiếng vì những nghiên cứu về các annulene có cầu nối. Annulene là
các hợp chất vòng lớn, chứa các hệ liên hợp nối tiếp. Annulene O được tổng hợp
từ naphthalene (J) theo chuỗi phản ứng dưới đây. Biết rằng:
- Hợp chất K cộng hợp được 3 đương lượng bromine;
- Carbene là tiểu phân trung gian rất hoạt động, phản ứng được với liên
kết đôi carbon-carbon tạo thành vòng cyclopropane.
8. Thiếu dữ kiện.
9. Xác định công thức cấu tạo các chất còn thiếu trong chuỗi phản ứng.

702 | Câu hỏi lí thuyết Hóa học hữu cơ OlympiaVN


Hợp chất carbonyl với nguyên tử hydrogen có tính acid có thể xảy ra hiện tượng
hỗ biến hay còn gọi là tautome hóa và có thể tồn tại ở dạng enol. Base liên hợp
của enol được gọi là enolate.
10. i) Viết công thức cấu tạo của enol bền nhất và enolate tương ứng của hợp chất
sau:

ii) Xác định sản phẩm chính trong các phản ứng sau:

703 | Câu hỏi lí thuyết Hóa học hữu cơ OlympiaVN


Aza-enolate là các hợp chất chứa nitrogen có cấu tạo tương tượng enolate.
11. Xác định công thức cấu tạo của aza-enolate của hợp chất sau:

12. Có thể điều chế hợp chất trên bằng phản ứng ngưng tụ của
⃝ một aldehyde và một amine bậc II.
⃝ một ketone và một amine bậc II.
⃝ một aldehyde và một amine bậc I.
⃝ một ketonee và một amine bậc I.
Phản ứng Mannich là phản ứng giữa formaldehyde, amine và ketone. Ví dụ:

13. Xác định công thức cấu tạo của các hợp chất P-S trong sơ đồ tổng hợp thuốc
gây mê cục bộ Tutocaine hydrochloride (S). Biết: giai đoạn đầu của sơ đồ là phản
ứng Mannich.

704 | Câu hỏi lí thuyết Hóa học hữu cơ OlympiaVN


705 | Câu hỏi lí thuyết Hóa học hữu cơ OlympiaVN
Hướng dẫn
1. III > I > II
2.

3.

4.

706 | Câu hỏi lí thuyết Hóa học hữu cơ OlympiaVN


5.

6.

7.

9.

10.

707 | Câu hỏi lí thuyết Hóa học hữu cơ OlympiaVN


11.

12. iii) Một aldehyde và một amine bậc một.

708 | Câu hỏi lí thuyết Hóa học hữu cơ OlympiaVN


13.

709 | Câu hỏi lí thuyết Hóa học hữu cơ OlympiaVN


Bài 8
Gốc carbon tự do là những tiểu phân có carbon hóa trị III với một electron chưa
ghép cặp. Chúng tham gia vào nhiều quá trình hữu cơ (tổng hợp chất, hóa sinh,
phản ứng trong môi trường tự nhiên, …). Chúng được tạo ra qua quá trình nhiệt
phân, quang phân hay phản ứng oxy hóa khử thông qua phân cắt đồng ly. Các
nhóm đẩy hay hút electron đều làm bền hóa gốc và các gốc chứa những nhóm này
lần lượt được gọi là gốc nucleophin (gốc chứa nhóm đẩy) và gốc electrophin (gốc
chứa nhóm rút).
Cho các gốc sau:
A) (CH3)3C• B) CH3CH2•
C) HC≡C• D) PhCH2•
1. Dãy sắp xếp các gốc theo trật tự chính xác về độ bền là
a) A > B > C > D. c) D > A > B > C.
b) B > D > A > C. d) C > A > B > D.
Một trong các phản ứng thường gặp nhất của gốc tự do là sự tách hydrogen để tạo
thành gốc tự do mới. Cho hợp chất sau:

2. Trật tự chính xác về khả năng tách hydrogen của các nguyên tử carbon (được
đánh số 1-6) là
a) 1 > 6 > 4 = 5 > 1 > 3. c) 1 > 2 > 6 > 3 > 4 = 5.
b) 2 > 1 > 6 > 3 > 4 = 5. d) 6 > 1 > 2 > 4 = 5 > 3.
Phản ứng halogen hóa alkane bằng đơn chất halogen, có ánh sáng, diễn ra theo
cơ chế gốc. Khi chloro hóa n-butane bằng Cl2, có ánh sáng thì hiệu suất tạo thành
1-chlorobutane và 2-chlorobutane lần lượt là 28 % và 72 %.
3. Tính tỉ lệ khả năng phản ứng tương đối của nguyên tử halogen ở các vị trí C1
và C2.
4. Cho (S)-2-chlorobutane phản ứng với chlorine, có ánh sáng, thì có thể thu được
bao nhiêu sản phẩm dichloro, tính cả các đồng phân quang học?
Oxygen tồn tại dưới dạng gốc kép và nhiều hợp chất hữu cơ bị oxid hóa bởi không
khí khi có ánh sáng tạo ra hydroperoxide.
5. Xác định công thức cấu tạo của hydroperoxide (A) tạo thành khi oxid hóa
tetrahydrofuran (THF) bằng không khí.

710 | Câu hỏi lí thuyết Hóa học hữu cơ OlympiaVN


Dầu mỡ dễ bị ôi thiu là do các acid béo không no, nhiều nối đôi bị oxid hóa bởi
không khí với xúc tác enzym.
6. Xác định công thức cấu tạo các hợp chất gây ra sự ôi thiu linoleic acid. Biết
trong sản phẩm có chứa hệ liên hợp.

Những chất chống oxid hóa như vitamin E được dùng làm chất bảo quản cho các
hợp chất dễ bị oxid hóa. Các chất chống oxid hóa phản ứng với các gốc tự do (là
tác nhân gây ra sự oxid hóa).
7. Xác định công thức gốc C tạo ra khi vitamin E phản ứng với gốc RO•.

8. Tính bền của gốc C là do


a) hiệu ứng siêu liên hợp. b) sự cản trở không gian.
c) sự giải tỏa electron. d) hiệu ứng liên hợp
Sự nhị hợp (dimer hóa) của các gốc trung gian gây ra bởi sự chuyển 1e electron
độc thân (SET mechanism: single electron transfer) từ các ion kim loại là phản
ứng hữu ích để tạo liên kết carbon-carbon. Dưới đây là một số ví dụ điển hình:
a) Ghép đôi pinacol:

711 | Câu hỏi lí thuyết Hóa học hữu cơ OlympiaVN


b) Ngưng tụ acyloin:

c) Ghép đôi alkyne:

9. Đề xuất các chất đầu phù hợp để điều chế các hợp chất i và ii sau đây bằng một
phản ứng trực tiếp có đi qua trung gian gốc tự do.
i) E là hợp chất trung gian trong quá trình tổng hợp thuốc chống ung thư taxol.

ii) F là hợp chất trung gian trong quá trình tổng hợp Exatolide, được dùng trong
nhiều loại nước hoa.

Sự tách halogen (dehalogen hóa) khỏi dẫn xuất halide hữu ocw có thể thực hiện
qua một phản ứng gốc, sử dụng tributyltin hydride (Bu3SnH) và
azobiisobutyronitrile (AIBN). AIBN đóng vai trò như một chất khơi màu để tạo
ra gốc Bu3Sn•. Gốc này phản ứng với RX tạo ra gốc alkyl R• có khả năng phản
ứng cao. Bu3SnH cũng đóng vai trò một chất nhường electron.
(Chú ý rằng sự hình thành các hệ vòng 5 hoặc 6 cạnh thì thuận lợi hơn các hệ
vòng lớn hoặc nhỏ hơn.)
10. Xác định công thức cấu tạo của sản phẩm G trong phản ứng sau. Biết 1 mol
G phản ứng hết với 1 mol hydrogen.

712 | Câu hỏi lí thuyết Hóa học hữu cơ OlympiaVN


Chất K, một hợp chất đóng vai trò quan trọng trong việc tiết dịch của nhiều loài
côn trùng được tổng hợp qua sơ đồ dưới đây
11. Xác định công thức cấu tạo của các chất H-K.
Biết: hợp chất J phản ứng với 2,4-DNPH tạo ra kết tủa màu đỏ cam, còn K thì
không.

Mn(OAc)3 được dùng làm chất oxid hóa bằng electron độc thân trong các phản
ứng tạo liên kết carbon-carbon. Các ion kim loại Mn+ có số oxid hóa cao có thể
tách hydrogen khỏi hợp chất hữu cơ để tạo ra gốc tự do:

Các ion kim loại Mn+ cũng có thể tách 1 electron từ gốc để tạo ra carbocation.
12. Xác định công thức cấu tạo các chất M-P trong sơ đồ phản ứng sau.

713 | Câu hỏi lí thuyết Hóa học hữu cơ OlympiaVN


714 | Câu hỏi lí thuyết Hóa học hữu cơ OlympiaVN
Hướng dẫn
1. C
2. B
3. C1: C2 = 1 : 3.85
4. D
5.

6.

7.

8. B
9.

715 | Câu hỏi lí thuyết Hóa học hữu cơ OlympiaVN


10.

11.

716 | Câu hỏi lí thuyết Hóa học hữu cơ OlympiaVN


12.

P không làm mất màu dung dịch Br2.

717 | Câu hỏi lí thuyết Hóa học hữu cơ OlympiaVN


Bài 9
Năm 2004, đã có thông báo LỊCH SỬ TỰ NHIÊN CỦA OSMIUM
rằng một vụ tấn công khủng bố
với kế hoạch sử dụng osmium Đây là một kim loại mới, được phát hiện
tetroxide (OsO4). Dù rằng bởi Mr.Tennant. Tên gọi của Osmium
osmium tetroxide là chất cực được đặt cho nó vì oxide của dễ bay hơi
độc nhưng nó không phù hợp và khuếch tán. Khi bay hơi, nó có mùi
để sử dụng cho mục đích này hăng đặc biệt. Kim loại này tồn tại ở dạng
do chi phí đắt đỏ của nguyên tố bột màu đen, trộn lẫn với quặng platina.
hiếm này (mỗi gam OsO4 có Trích từ “Hệ thống Hóa học lí thuyết và thực
giá khoảng 100 bảng Anh). nghiệm” của Fredrick Accum, 1808.
Tuy nhiên, các hợp chất
osmium lại phù hợp để dùng
làm xúc tác cho một số phản ứng hữu cơ.
Trong dung dịch, osmium tồn tại dưới dạng một số tiểu phân cụ thể.
a) Xác định số oxid hóa của osmium trong các tiểu phân sau:
i. OsO2.
ii. OsO4.
iii. OsO4-.
Một tiểu phân chứa osmium có thể xúc tác cho chuyển hóa của alkene thành diol
có hai nhóm hydroxyl thuộc hai nguyên tử carbon cạnh nhau.
b) Phản ứng trên thuộc loại nào
A. Thủy phân
B. Tách nước
C. Oxid hóa
D. Khử hóa
E. Đồng phân hóa
Chỉ duy nhất một tiểu phân osmium trong phần (a) có thể xúc tác cho phản ứng
trên. Dựa vào sự phân loại phản ứng ở phần (b), hãy cho biết tiểu phân nào của
osmium được dùng làm xúc tác cho phản ứng?
Phản ứng này rất hữu ích vì dựa vào cách các liên kết C-O hình thành thì có thể
xác định chính xác hóa lập thể của sản phẩm. Các liên kết C-O mới đều được hình
thành đồng thời ở cùng một phía của liên kết đôi. Giai đoạn đầu tiên là sự hình
thành của hợp chất vòng tại vị trí mà tiểu phân osmium có thể tấn công vào một
trong hai phía của liên kết đôi.

718 | Câu hỏi lí thuyết Hóa học hữu cơ OlympiaVN


d) Tính số oxid hóa của nguyên tử osmium trong vòng.
Sau khi thủy phân, các liên kết O-H mới được tạo ra đồng thời ở cùng một phía
(trên hoặc dưới). Do vậy, chỉ có các đồng phân lập thể nhất định mới được tạo
thành.

e) Diol dưới đây có thể tồn tại dưới dạng bốn đồng phân lập thể. Hãy vẽ các
alkene tương ứng mà khi phản ứng với tiểu phân osmium tạo thành các mỗi đồng
phân đó.

f) i. Diol sau đây có bao nhiêu đồng phân lập thể?

ii. Hãy vẽ cấu trúc lập thể của từng đồng phân và cấu trúc các alkene tương ứng
tạo thành chúng.
Trong một số phân tử nhất định chứa hai liên kết đôi, có thể chỉ có 1 nguyên tử
O tham gia tạo vòng với 2 nguyên tử C. Phản ứng này cũng có lập thể xác định.
Trong số 4 liên kết C-O mới được hình thành, liên kết 1 và 2 được hình thành
đồng thời trên cùng một phía của liên kết đôi A, liên kết 3 và 4 được hình thành
đồng thời cùng phía của liên kết đôi B.

719 | Câu hỏi lí thuyết Hóa học hữu cơ OlympiaVN


ii. Sản phẩm vòng từ phản ứng của hợp chất sau với tiểu phân osmium phù hợp
có thể tồn tại dưới dạng 16 đồng phân quang học. Dựa vào thông tin ở trên, hãy
tính số đồng phân quang học có thể tạo thành từ phản ứng này.

Hướng dẫn
a) i. +4; ii. +8; iii. +6.
b) Sự oxid hóa.
c) OsO4.
d) +6.
e)

f) i. 3
ii.

iii.

720 | Câu hỏi lí thuyết Hóa học hữu cơ OlympiaVN


721 | Câu hỏi lí thuyết Hóa học hữu cơ OlympiaVN
Bài 10
Carboxylic acid tồn tại nhiều trong tự nhiên. Các dẫn xuất carboxylic acid phổ
biến như: acid chloride, ester, amide được dùng làm phần tử trung gian trong
nhiều quá trình tổng hợp hữu cơ. Tuy nhiên, hoạt tính của các dẫn xuất này có
nhiều khác biệt.
1. Vẽ các cấu trúc cộng hưởng (Lewis) của amide và ester.

2. Trật tự đúng về độ bền cộng hưởng của các dẫn xuất là


a) Acid Chloride > Amide > Ester
b) Ester > Amide > Acid Chloride
c) Amide > Ester > Acid Chloride
d) Acid Chloride > Ester > Amide
e) Amide > Acid Chloride > Ester
f) Ester > Acid Chloride > Amide
3. Nguyên tử oxygen trong nhóm carbonyl của dẫn xuất nào có tính base mạnh
nhất:
a) Amide.
b) Ester.
c) Acid chloride.
Peak của nhóm carbonyl trong phổ IR (hồng ngoại) nằm trong khoảng 1600-1800
cm-1. Liên kết càng bền, vùng hấp thụ trên phổ IR càng lớn. Phổ IR của các mẫu
A (acetamide), B (acetyl chloride) và C (ethyl acetate) đã được ghi lại.
4. Cho biết peak hấp thụ sau đây ứng với chất nào trong ba chất A, B, C:
1650 cm-1

722 | Câu hỏi lí thuyết Hóa học hữu cơ OlympiaVN


1750 cm-1
1800 cm-1

Các dẫn xuất acid (RCOX) tham gia phản ứng nucleophile ở vị trí nhóm carbonyl.
Tốc độ phản ứng phụ thuộc vào bản chất nhóm X. Phản ứng diễn ra qua các giai
đoạn sau:

6. Chất nào sau đây có tốc độ phản ứng thế nucleophile nhanh nhất?
a) CH3CH2CONH2
b) CH3CH2COOCH3
c) CH3CH2COCl
Dựa vào cơ chế phản ứng trên, có thể vẽ ra bốn giản đồ năng lượng sau:

6. Bốn giản đồ trên ứng với nhóm đi ra nào trong số ba nhóm NH2, OEt, Cl.
Phản ứng điều chế ester từ carboxylic acid với xúc tác acid được gọi là phản ứng
ester hóa Fischer. Thực hiện phản ứng ester hóa 3-nitrophthalic acid (4) với
methanol khi có xúc tác sulfuric acid, thu được ester (5). Cho (5) phản ứng với
dung dịch nước vôi trong, tạo ra một hợp chất mà khi clo hóa sẽ tạo ra sản phẩm
chính là một dẫn xuất monochloro.

723 | Câu hỏi lí thuyết Hóa học hữu cơ OlympiaVN


7. Xác định công thức chất (5) và dẫn xuất monochloro.
Cho ester (6) phản ứng với n-BuLi và diisopropylamine (DIPA), thu được enolate
(7). Cho (7) phản ứng với hợp chất (8), thu được hợp chất (9).
8. Xác định công thức cấu tạo các chất (7), (8).

Các enolate phản ứng tiếp, tương tác nội hoặc ngoại phân tử với các nguyên tử
carbon electronphin, tạo ra liên kết C-C mới. Cho hợp chất (9) phản ứng với
NaOEt, thu được hỗn hợp sản phẩm (10), (11), (12).

9. Xác định công thức cấu tạo (10), (11), (12).


Steroid là một hợp chất thiên nhiên, tồn tại trong nhiều hệ sinh học. Cho steroid
(13) phản ứng với một base, tạo ra hợp chất bền (14).

10. Xác định công thức cấu tạo của sản phẩm chính (14).

724 | Câu hỏi lí thuyết Hóa học hữu cơ OlympiaVN


11. i) Trong (13) có bao nhiêu tâm lập thể?
ii) Xác định cấu hình tuyệt đối của các tâm bất đối trong (a), (b).
Cho (14) phản ứng với chromium trioxide, thu được hợp chất (15).

12. Xác định công thức cấu tạo của (15).


Oxime của ketone và ester của oxime bị chuyển vị khi đun nóng hoặc khi có xúc
tác acid, tạo thành amide (phản ứng chuyển vị Beckmann).

Cho chất (15) phản ứng với hydroxylamide hydrochloride và base, sau đó xử lí
tiếp với 4-MePhSO2Cl, thu được sản phẩm (16).

13. Xác định công thức cấu tạo của (16).


Đun nóng hợp chất (16) rồi thủy phân, thu được hỗn hợp hai chất (17), (18).

725 | Câu hỏi lí thuyết Hóa học hữu cơ OlympiaVN


14. Xác định công thức cấu tạo của (17), (18). Gợi ý: Các sản phẩm đều có vòng
5 cạnh và có peak cường độ mạnh trong khoảng 1620-1650 cm-1.
Tham khảo: Các vùng đặc trưng trong phổ hấp thụ IR.
Nhóm chức Vùng (cm-1)
Hydroxyl 3000-4000
Carbonyl 1620-1800
Ether 1050-1150
Olefin 1580-1620

726 | Câu hỏi lí thuyết Hóa học hữu cơ OlympiaVN


Hướng dẫn
1.

2. c) Amide > Ester > Acid Chloride


3. Amide.
4.

5. CH3CH2COCl
6. Tốt nhất: C; kém nhất: B
7.

8.

727 | Câu hỏi lí thuyết Hóa học hữu cơ OlympiaVN


9.

10.

11. a) 9.
b) cả hai là S.

728 | Câu hỏi lí thuyết Hóa học hữu cơ OlympiaVN


Bài 11
Acyl chloride, RCOCl, là các hợp chất có hoạt tính cao, phản ứng nhanh với nước
tạo thành carboxylic acid và với alcohol tạo thành ester. Những oxochloride khác,
chứa các liên kết với chlorine và các liên kết đôi với oxygen, như phosphoryl
chloride (phosphorus trichloride oxide, POCl3) và thionyl chloride (sulfur
dichloride oxide, SOCl2) cũng có phản ứng tương tự.
a) Viết phương trình phản ứng giữa ethanoyl chloride và nước.
b) Viết phương trình phản ứng giữa propanoyl chloride và ethanol.
Phosgene (carbonyl chloride, COCl2), được xem như là một diacyl chloride, cũng
có phản ứng tương tự với alcohol. Tuy rất độc nhưng nó thường được dùng trong
tổng hợp hữu cơ, ví dụ như trong quá trình tổng hợp thuốc trừ sâu có hoạt tính
trên diện rộng Carbaryl.

c) Xác định công thức cấu tạo của hợp chất A.


d) Vẽ công thức cấu tạo của phosphoryl chloride và thionyl chloride. Xác định rõ
dạng hình học của chúng và số oxid hóa của phosphorus, sulfur.
e) Viết phương trình phản ứng giữa thionyl chloride và nước.
f) Xác định công thức cấu tạo của phosphonate ester tạo thành khi cho 1 mol
POCl3 phản ứng với
i. 1 mol methanol.
ii. 2 mol methanol.
1 mol POCl3 phản ứng với 3 mol nước tạo ra phosphoric acid, H3PO4. Đun nóng
phosphoric acid với POCl3 thì tạo thành một acid chứa phosphorus khác, là B.
Thêm dung dịch bạc nitrate vào dung dịch acid B thì tạo ra kết tủa trắng chứa
71,3 % bạc và 10,2 % phosphorus về khối lượng, phần còn lại là oxygen.
g) Xác định công thức cấu tạo của acid B và viết phương trình tạo thành nó từ
phosphoric acid và POCl3. Cho biết rằng sản phẩm còn lại là HCl.
Hướng dẫn
a)

729 | Câu hỏi lí thuyết Hóa học hữu cơ OlympiaVN


b)

c)

d)

e)

f)

730 | Câu hỏi lí thuyết Hóa học hữu cơ OlympiaVN


g)

h)

731 | Câu hỏi lí thuyết Hóa học hữu cơ OlympiaVN


Bài 12
Các hợp chất thiên nhiên đóng vai trò then chốt trong sự phát triển của hóa học
Hữu cơ. Việc tìm ra hướng cụ thể để tổng hợp một hợp chất thiên nhiên chính là
phương pháp để xác định cấu trúc của nó.
Một hợp chất thiên nhiên quan trọng (đã được thương mại hóa), có thể tạo kết tủa
vàng với thuốc thử 2,4-DNP nhưng không phản ứng với dung dịch
[Ag(NH3)2]NO3. Khử hóa A bằng NaBH4 thu được alcohol bậc hai, còn nếu oxid
hóa bằng nitric acid sẽ tạo ra một dicarboxylic acid (đều có cùng số nguyên tử
carbon). Khi bromo hóa A, thu được một sản phẩm monobromo.
1. Dựa vào những dữ kiện trên, có thể kết luận trong A có chứa
⃝ C=O aldehyde. ⃝ C=O ketone.
⃝ nhóm chức ester. ⃝ không có liên kết bội.
⃝ C=C.
Oxid hóa A bằng HNO3 tạo ra B (C10H16O4), oxid hóa B tiếp thì tạo ra C
(C9H14O6). Để trung hòa hoàn toàn 78 mg B cần 15,6 mL KOH 0,05 M. Để trung
hòa hoàn toàn 85 mg C cần 23,4 mL KOH 0,05 M.
2. Tính số nhóm chức có tính acid trong B và C.
Perkin. Jr và Thorpe đã tông hợp chất C đi từ methyl acetoacetate theo chuỗi phản
ứng sau:

3. Xác định công thức cấu tạo của tác nhân P, các sản phẩm trung gian D, E và
sản phẩm C. Biết: Phản ứng giữa α-haloester và Zn tạo ra một tác nhân cơ kim
tương tự như tác nhân Grignard.
4. Thiếu dữ kiện.

732 | Câu hỏi lí thuyết Hóa học hữu cơ OlympiaVN


5. Vẽ công thức chiếu Fischer các đồng phân quang học của C và chỉ rõ cấu hình
các tâm bất đối.
Hợp chất H là chất trung gian cần thiết cho quá trình tông hợp A. H được tổng
hợp từ diethyl malonate theo chuỗi phản ứng sau.

6. Xác định công thức tác nhân Y, chất trung gian F, G và sản phẩm H.
Diethyl 3,3-dimethyl pentanedioate được dùng để tổng hợp B theo chuỗi phản
ứng sau.

733 | Câu hỏi lí thuyết Hóa học hữu cơ OlympiaVN


7. Xác định công thức cấu tạo của I, J, K, B.
8. Xác định tên IUPAC của B.

734 | Câu hỏi lí thuyết Hóa học hữu cơ OlympiaVN


Hướng dẫn
1. Có nhóm C=O ketone và không có liên kết bội.
2. Trong B có 2, trong C có 3.
3.

5.

6.

735 | Câu hỏi lí thuyết Hóa học hữu cơ OlympiaVN


7.

8. 1, 2, 2-Trimethylcyclopentane-1,3-dicarboxylic acid

736 | Câu hỏi lí thuyết Hóa học hữu cơ OlympiaVN


Bài 13
Một số các hợp chất quan trọng trong tự nhiên như alkaloid, amino acid, protein
và peptide đều có chứa nitrogen. Các dị vòng chứa nitrogen ở dạng pyrimidine
và purine base đóng vai trò quan trọng đối với chức năng của các nucleic acid, là
những chất chứa đựng thông tin di truyền của cơ thể sống và có thể truyền lại cho
thế hệ kế cận.
1. Hợp chất nào có thể phân giải được thành đối quang?
i. ii. iii. iv.

2. Tiểu phân có tính acid mạnh nhất là


i. ii. iii. iv.

3. Vẽ cấu trúc những sản phẩm có thể phân lập được của các phản ứng sau:
i.

ii.

4. Xác định B và C trong dãy sau.

737 | Câu hỏi lí thuyết Hóa học hữu cơ OlympiaVN


5. Với cặp hợp chất sau đây thì ý nào dưới đây đúng:

i) D có tính base mạnh hơn E.


ii) E có tính base mạnh hơn D.
iii) D và E có tính base như nhau.
Alkaloid là các hợp chất hữu cơ chứa nitơ trong tự nhiên có hoạt tính sinh học
mạnh được phân lập từ cây cỏ. Việc xác định cấu trúc của alkaloid bao gồm nhiều
công đoạn và một vài bước trong số chúng được mô tả dưới đây.
(a) Phương pháp Zeisel: Được dùng để xác định số nhóm methoxyl. Ở đây
alkaloid được đun nóng với HI ở nhiệt độ sôi (126C) để tạo thành methyl iodide,
chất này được hấp thụ bằng dung dịch AgNO3 trong ethanol. Kết tủa AgI tạo
thành được tách ra và cân.
(b) Phương pháp metyl hóa hoàn toàn theo Hoffmann: Được dùng để xác định
khung sườn của alkaloid. Nó bao gồm việc methyl hóa hoàn toàn amine bằng
CH3I sau đó tiến hành nhiệt phân muối amonium bậc bốn này với Ag2O. Ở đó
xảy ra quá trình tách được biểu diễn như sau.

Chú ý: Tiến trình này tạo thành alkene ít bền hơn.


Xác định sản phẩm ‘G’và ‘H’ nhận được từ phương pháp methyl hóa hoàn toàn
Hoffmann rồi sau đó thoái phân N-methylpiperidine (F) theo thứ tự sau. Biết rằng:
H hấp thụ UV ở vùng trên 200 nm.

738 | Câu hỏi lí thuyết Hóa học hữu cơ OlympiaVN


Papaverine (C20H21NO4) là một alkaloid không quang hoạt. Cấu trúc của nó đã
Goldschmidt và các cộng sự xác lập được vào năm 1888 và công việc họ đã làm
là một ví dụ điển hình của phương pháp oxid hóa thoái phân định cấu trúc.
7. Papaverine phản ứng với 4 mol HI ở 126C để tạo thành nhiều sản phẩm, một
trong số đó là papaveroline. Công thức của papaveroline là
i) C16H13NO4
ii) C16H9NO4
iii) C16H14NO4
iv) C16H15NO4
Papaverine, khi bị oxid hóa bằng KMnO4 đặc nóng sẽ bị cắt thành nhiều phân
mảnh nhỏ bao gồm veratric acid, C9H10O4 (I), metahemipinic acid (J), pyridine
tricarboxylic acid (K) và 6,7-dimethoxyisoquinoline-1-carboxylic acid. Veratric
acid (I) được tổng hợp từ p-hydroxybenzoic acid theo sơ đồ sau.

8. Xác định công thức các chất còn thiếu.


Đun nóng metahemipinic acid J (C10H10O6) với CaO, thu được veratrole. Chất
này cũng được tạo ra bằng cách đun nóng veratric acid (I) với CaO. J chuyển
thành anhydride khi đun nóng và chỉ tạo ra một monoester.
9. Công thức metahemipinic acid là
i. ii. iii. iv.

Oxid hóa lepidine L (4-methylquinoline) thu được pyridine tricarboxylic acid K.


10. Xác định công thức của K.

739 | Câu hỏi lí thuyết Hóa học hữu cơ OlympiaVN


11. Dựa vào tất cả các thông tin trên thì cấu trúc của papaverine là
i. ii.

iii. iv.

Oxid hóa pepaverine bằng KMnO4 loãng, đun nóng sẽ tạo ra papaverinic acid
(C16H13NO7), chất này có khả năng tạo thành một oxime.
12. Xác định công thức cấu tạo papaverinic acid.
Hướng dẫn
1. Chất ii)
2. Chất ii)
3.
a) CH3COCHN2

740 | Câu hỏi lí thuyết Hóa học hữu cơ OlympiaVN


b) MeO COOH

MeO COOH

4. Br
Br
or
N NH2 N NH2

5. i) D có tínhBbase mạnh hơn E.


6. C
OH
_
N CH2 C
N CH2
H

7. i) C16H13NO4
8. COOH COOH COOH

SO3H OH OCH3
9. OH OH OCH3
i) COOH
HOOC
10.
HOOC N

N
H
G

K
741 | Câu hỏi lí thuyết Hóa học hữu cơ OlympiaVN
11. MeO
N
iii) MeO
CH2

OMe
12. OMe
HOOC

N
HOOC
C=O

OMe
OMe

742 | Câu hỏi lí thuyết Hóa học hữu cơ OlympiaVN


Bài 14
Alkaloid là các hợp chất thiên nhiên chứa nitrogen. Tuy độc nhưng nếu được sử
dụng với lượng rất nhỏ thì chúng là những dược chất giá trị.
Thủy phân alkaloid A (C17H23NO3) với Ba(OH)2, tạo ra hai sản phẩm: B
(C9H10O3) và C (C8H15NO). Hợp chất B không có phản ứng cộng hợp bromine
nhưng tan được trong dung dịch NaHCO3. Đun nóng B tạo ra hợp chất D
(C9H8O2).
1. Hãy xác định tính đúng/sai của các phát biểu sau về chất B.
a) B có một nhóm hydroxyl.
b) B là olefin.
c) B có một nhóm carboxyl.
d) B là hợp chất thơm.
Oxid hóa hợp chất D thì thu được hỗn hợp sản phẩm có chứa benzoic acid.
2. Những công thức cấu tạo nào sau đây có thể là của hợp chất D:

Hợp chất B được tổng hợp theo chuỗi phản ứng sau.
3. Xác định công thức cấu tạo của B và các chất trung gian E-H.

Chú ý: Sản phẩm H cũng được tạo thành khi cho G phản ứng với HBr, có mặt
H2O2.
Có nhiều phương pháp vật lí và hóa học để xác định rõ cấu trúc của alkaloid. Một
trong số đó là phản ứng tách Hoffmann để xác định cấu tạo của các dị vòng
nitrogen. Dị vòng bị phá vỡ do bị mất nitrogen (dưới dạng amine) và tạo thành
một alkene ít nhóm thế (ở C=C). Dưới dây là phản ứng tổng quát:

743 | Câu hỏi lí thuyết Hóa học hữu cơ OlympiaVN


Chú ý: Thực hiện phản ứng tách Hoffmann hai lần với amine vòng sẽ tạo ra một
amine và một diene.
Hợp chất C không làm mất màu bromine và tan được trong dung dịch HCl. Cho
C phản ứng với H2SO4 đặc, tạo ra J. Thực hiện phản ứng tách Hoffmann J, thu
được 1,3,5-cycloheptatriene.
4. Những công thức cấu tạo nào sau đây có thể là của C?

Khi oxid hóa có kiểm soát C sẽ tạo thành chất K. Chất K tạo kết tủa với 2,4-DNP
và cũng phản ứng được với 2 đương lượng benzaldehyde (khi có base), tạo thành
một dẫn xuất dibenzylidine, có công thức cấu tạo tổng quát như sau:

5. Xác định các công thức cấu tạo có thể có của C.

744 | Câu hỏi lí thuyết Hóa học hữu cơ OlympiaVN


6. i) Các chất a và b (ở mục 5) có bao nhiêu nguyên tử carbon bất đối?
ii) Những hợp chất nào trong số các chất a-e (ở mục 5) có tính quang hoạt?
Oxid hóa C với CrO3 tạo ra acid 2 chức L. Đun nóng L với HI ở 150-300 oC, tạo
ra 1 đương lượng methyl iodide.
Hòa tan 0,580 gam L vào 100 cm3 nước cất. Để trung hòa 10 cm3 dung dịch thu
được, cần 6,2 cm3 KOH 0,1 N.
7. Tính khối lượng phân tử của L. Trình bày chi tiết các bước tính toán.
8. Xác định công thức phân tử của L.
Oxid hóa L thu được sản phẩm M dưới đây:

9. Xác định công thức cấu tạo của L.


10. Vẽ các cấu trúc hỗ biến có thể có của M.
11. Dựa vào các dữ kiện từ mục 4-9, hãy xác định công thức cấu tạo của C.
12. Xác định công thức cấu tạo của alkaloid A.
Trong quá trình trao đổi chất của thực vật, alkaloid được tạo thành từ các amino
acid. Ví dụ, alkaloid hygrine (N) được tạo thành từ amino acid Ornithine (O).

745 | Câu hỏi lí thuyết Hóa học hữu cơ OlympiaVN


Giai đoạn đầu tiên trong quá trình tổng hợp này là phản ứng decarboxyl hóa
Ornithine với xúc tác pyridoxal.
13. Xác định công thức cấu tạo chất Q.

14. Xác định công thức cấu tạo các chất S, T, U, V trong chuỗi phản ứng sau.

746 | Câu hỏi lí thuyết Hóa học hữu cơ OlympiaVN


Chú ý: Phản ứng methyl hóa bởi SAM xảy ra ở nguyên tử nitrogen có tính
nucleophilic lớn hơn.
Con đường sinh tổng hợp được xác định bằng phương pháp đánh dấu đồng vị ở
các tiền chất phản ứng. Xác định công thức cấu tạo của hygrin, chỉ rõ các nguyên
tử được đánh dấu nếu Ornithine được đánh dấu bằng các đồng vị 14C ở C2 và 15N
ở nguyên tử nitrogen của C2.

747 | Câu hỏi lí thuyết Hóa học hữu cơ OlympiaVN


Hướng dẫn
1. a, c, d.
2. a, b.
3.

4. a, b, d.
5. b, d.
6. a) i. 3; ii. 0.
b) a, b, c, d, e
7. 187
8. C8H13NO4
9.

10.

748 | Câu hỏi lí thuyết Hóa học hữu cơ OlympiaVN


11. b.
12.

13.

749 | Câu hỏi lí thuyết Hóa học hữu cơ OlympiaVN


14.

15.

750 | Câu hỏi lí thuyết Hóa học hữu cơ OlympiaVN


Bài 15
Có rất nhiều hợp chất chứa nitrogen được tìm thấy trong thế giới động-thực vật,
điển hình là: alkaloid, nucleic acid, vitamin, … và các hợp chất này đều có ảnh
hưởng sinh lý.
Trung tâm lập thể có thể là nguyên tử hoặc nhóm nguyên tử, trong đó sự thay đổi
vị trí của 2 nhóm bất kì tạo thành một đồng phân lập thể khác. Các hợp chất chứa
nitrogen cũng có thể có cấu trúc bất đối và có tính quang hoạt.
Cho ba hợp chất: (I): không quang hoạt; (II): quang hoạt nhưng không tách riêng
các đối quang được; (III): quang hoạt và có thể tách riêng các đối quang.
1. Hãy cho biết I, II, III ứng với những hợp chất nào dưới đây:

Alkaloid là nhóm hợp chất chứa nitrogen có tính base, có nguồn gốc thực vật.
(+)-Muscarine là một alkaloid có độc tính, được tìm thấy trong một số loại nấm.

2. Vẽ và xác định cấu hình tuyệt đối của đối quang (+)-Muscarine.
Trong thuốc phiện có nhiều alkaloid, một trong số đó là morphine - có thể dùng
làm thuốc giảm đau.

751 | Câu hỏi lí thuyết Hóa học hữu cơ OlympiaVN


3. Trong morphine có bao nhiêu tâm bất đối?
Phản ứng tách Hoffmann là một phương pháp để xác định cấu trúc alkaloid. Phản
ứng này gồm giai đoạn tạo ra muối ammonium bậc IV rồi tách nitrogen để tạo
thành alkene. Phản ứng tổng quát được mô tả như sau:

Một dẫn xuất diacetyl của morphine là heroin, cũng được dùng làm thuốc gây mê.
Thực hiện phản ứng tách Hoffmann với heroin, thu được sản phẩm A.
4. Tính số mol methyl iodide cần để tách hoàn toàn nitrogen khỏi heroin.
5. Xác định công thức cấu tạo của A.
Cho A phản ứng với bromine dư.
6. Mỗi phân tử A phản ứng được với bao nhiêu phân tử bromine?
Một nhóm alkaloid chứa nitrogen thú vị khác là Indole alkaloid - trong phân tử
có chứa vòng indole.

Indole và dẫn xuất của nó được tổng hợp bằng phương pháp Fischer Indole, trong
đó có một giai đoạn thú vị là sự chuyển vị (với xúc tác acid) của arylhydrazone
như sau:

752 | Câu hỏi lí thuyết Hóa học hữu cơ OlympiaVN


Dẫn xuất Indole sau cũng được tổng hợp bằng phương pháp Fischer, đi từ hợp
chất carbonyl B và dẫn xuất thế phenyl hydrazine C.

7. Xác định công thức cấu tạo của B, C.


Muối arene diazonium là các hợp chất trung gian quan trọng, có thể bị khử trực
tiếp thành aryl hydrazine hoặc được dùng để tổng hợp các aryl hydrazone. Các
muối này cũng được dùng để tổng hợp thuốc nhuộm azo, trong trường hợp này
chúng đóng vai trò tác nhân electrophile.
8. Vẽ các cấu trúc cộng hưởng quan trọng nhất của benzene diazonium ion.
Muối arene diazonium ghép cặp được với các hợp chất béo có chứa nguyên tử
carbon có tính acid (hợp chất chứa nhóm methylene hoạt động) tạo thành dẫn xuất
azo. Hợp chất azo bị hỗ biến tạo thành dẫn xuất hydrazo. Nếu một hợp chất không
thể hỗ biến thì nó sẽ loại bỏ (chuyển vị) một nhóm chức phù hợp ở vị trí tạo nối
đôi để cho phép xảy ra sự hỗ biến).
9. Hoàn thành phương trình phản ứng sau:

Alkaloid J là redox (oxid hóa-khử) cofactor của enzyme alcohol dehydrogenase


(tách hydrogen khỏi alcohol) và đóng vai trò quan trọng trong việc truyền tín hiệu
và giao lưu thông tin tế bào.

753 | Câu hỏi lí thuyết Hóa học hữu cơ OlympiaVN


I là hợp chất trung gian quan trọng trong quá trình tổng hợp J. Quá trình tổng hợp
này có sự hình thành một hydrazone.
10. Xác định các chất còn thiếu trong sơ đồ phản ứng sau.

754 | Câu hỏi lí thuyết Hóa học hữu cơ OlympiaVN


Từ hợp chất I, có thể tổng hợp K là tiền chất của J.
11. Xác định nguyên tử có tính base mạnh nhất của K.

Nhiều bằng chứng cho thấy các hợp chất thiên nhiên chứa vòng imidazole có hoạt
tính sinh lí. Do vậy, đã có nhiều hợp chất như vậy được tổng hợp nhân tạo. Một
trong số đó là dược chất Pentostatin - dùng làm hoạt chất chống virus và kháng

755 | Câu hỏi lí thuyết Hóa học hữu cơ OlympiaVN


u. Hợp chất O (C12H10N4O5) là hợp chất trung gian quan trọng trong quá trình
tổng hợp Pentostatin.

12. Xác định công thức cấu tạo các chất còn thiếu trong sơ đồ phản ứng sau.

756 | Câu hỏi lí thuyết Hóa học hữu cơ OlympiaVN


Hướng dẫn
1. a. III; b. II. c. III.
2.

3. d) 6.
4. b) 2.
5.

6. b) 3.
7.

8.

757 | Câu hỏi lí thuyết Hóa học hữu cơ OlympiaVN


9.

10.

11. C.
12.

758 | Câu hỏi lí thuyết Hóa học hữu cơ OlympiaVN


759 | Câu hỏi lí thuyết Hóa học hữu cơ OlympiaVN
Bài 16
Cholesterol (A) là steroid có nhiều nhất trong động vật. Nó là tiền chất của tất cả
các steroid khác và là một thành phần quan trọng của màng tế bào.

(Lưu ý: Nét đậm để biểu diễn các liên kết phía trên mặt phẳng còn nét đứt là các
liên kết phía dưới mặt phẳng)
1. Tổng số đồng phân lập thể có thể có của A là
a) 32
b) 256
c) 128
d) 64
2. Vẽ cấu trúc este của cholesterol với một acid béo 16 nguyên tử carbon với liên
kết đôi ở C4 có cấu hình E.
3. a) Hoàn chỉnh cấu trúc sản phẩm với đầy đủ lập thể khi cho B2H6 phản ứng với
cholesterol trong tetrahydrofuran, sau đó xử lý với H2O2 trong kiềm.

b) Cấu trúc 3D của khung steroid ở câu a được cho dưới đây. Hãy hoàn chỉnh cấu
trúc của sản phẩm ở câu a, bằng cách đặt các nhóm thế vào đúng vị trí lập thể của
nó.

c) Hoàn chỉnh cấu trúc sản phẩm (với đầy đủ lập thể) khi cho A phản ứng với
i. H3O+ (ở nhiệt độ phòng)

760 | Câu hỏi lí thuyết Hóa học hữu cơ OlympiaVN


ii. peroxyacetic acid, CH3COOOH

iii. Br2/H2O

Trong số các loại hợp chất tự nhiên thì terpene là một họ chất rất thú vị do nó
tham gia được vào nhiều kiểu phản ứng khác nhau. Khi xử lý ancol borneol (B)
C10H18O, với một BrØnsted acid sẽ tạo thành F, (C10H16).

4. Xác định cấu trúc các chất từ C-F.


Diels-Alder là một phản ứng quan trọng và rất có ích trong việc tổng hợp các dẫn
xuất của cyclohexene. Phản ứng này là phản ứng của một “diene” giàu electron
với một olefin nghèo electron mà thường gọi là “dienophile”.

Có hai hướng tổng hợp steroid cortisone, chúng đi qua hai trung gian G và H
tương ứng.

761 | Câu hỏi lí thuyết Hóa học hữu cơ OlympiaVN


5. Vẽ cấu trúc những nguyên liệu ban đầu cần thiết để tổng hợp G và H bằng phản
ứng Diels-Alder.

I là một hợp chất rất thú vị. Nó phản ứng được với Na để tạo thành hợp chất J,
chất này bền vững cho đến 225oC thì tạo thành K (C4H6; UV - max 215nm). Một
mol HBr phản ứng với K ở 45oC để cho L (C4H7Br) và M (C4H7Br) với tỉ lệ mol
80:20.
6. Xác định các chất từ J-K.
2-aminobenzoic acid phản ứng với NaNO2/HCl tạo thành N. Trong phổ khối
lượng thì khi đun nóng hợp chất ở nhiệt độ cao trong chân không và chiếu xạ nó
bằng chùm electron năng lượng cao thì sẽ bị ion hóa để tạo thành cation. Cation
này sẽ chịu sự phân mảnh. Khối lượng của các phân mảnh này được ghi lại trên
phổ.
Dung dịch kiềm của N được tách ra riêng và ngay lập tức được quét phổ khối
lượng với tốc độ rất nhanh để xác định cấu trúc sản phẩm. Kết quả phân tích phổ
khối lượng của N cho các peak có khối lượng 28, 44 và 76.
7. Xác định cấu trúc các mảnh có khối lượng 28, 44, 76 và của N.

762 | Câu hỏi lí thuyết Hóa học hữu cơ OlympiaVN


Hướng dẫn
1. b) 256
2.
CH3
H3C-(CH2)10

H
C C
H

(CH2)2
O

C O
}
3.
a)
CH3
H3C-(CH2)10

H
C C
H

(CH2)2
O

C O
}
b)

CH3
CH3

HO

HO

c)

763 | Câu hỏi lí thuyết Hóa học hữu cơ OlympiaVN


764 | Câu hỏi lí thuyết Hóa học hữu cơ OlympiaVN
4.

5.

765 | Câu hỏi lí thuyết Hóa học hữu cơ OlympiaVN


6.

7.

766 | Câu hỏi lí thuyết Hóa học hữu cơ OlympiaVN


Bài 17
Hợp chất cơ sulphur (cơ lưu huỳnh) hữu dụng trong hóa hữu cơ một phần là do
khả năng sử dụng các orbital 3d để tạo liên kết hóa học; bên cạnh đó là khả năng
tạo ra số liên kết cộng hóa trị lớn hơn 2.
1. Vẽ công thức Lewis của thiosulphate ion (S2O32-)
2. Vẽ ba cấu trúc cộng hưởng không tương đương của S2O32-.
Sulphur ylide là các hợp chất trong phân tử có nguyên tử lưu huỳnh mang điện
tích dương liên kết với nguyên tử carbon mang điện tích âm (carbanion). Sulphur
ylide là một trung gian hữu ích trong tổng hợp hữu cơ.
3. Hoàn thành chuỗi phản ứng sau. Biết sản phẩm H có một peak ở 1740 cm-1
trong phổ IR. (Tham khảo dữ liệu cuối bài)

767 | Câu hỏi lí thuyết Hóa học hữu cơ OlympiaVN


Sulphur ylide cũng tham gia các phản ứng chuyển vị, khi một trong các nhóm thế
của nguyên tử S có dạng allyl. Phản ứng này được gọi là chuyển vị [2,3]
sigmatropic (do có sự dịch chuyển của liên kết sigma), với 1 liên kết sigma mới
được tạo ra giữa các vị trí 3’ và 2 như sau:

4. Viết công thức cấu tạo của sản phẩm tạo thành từ phản ứng chuyển vị [2,3]
sigmatropic ylide I sau đây:

5. Thực hiện chuyển vị [1,2] sigmatropic ylide I thì sẽ thu được hợp chất nào?
Sulphur ylide được dùng làm hợp chất trung gian trong quá trình tổng hợp chất P
(C9H9N) sau đây.

768 | Câu hỏi lí thuyết Hóa học hữu cơ OlympiaVN


6. Hãy xác định công thức các chất còn thiếu trong sơ đồ trên.
Sự có mặt của một số nhóm thế nhất định sẽ làm đảo chiều phân cực nhóm
carbonyl. Hiện tượng đảo chiều phân cực này được gọi là “UMPOLUNG” (tiếng
Đức, nghĩa là ‘chuyển hóa lưỡng cực’) và được sử dụng để tổng hợp các hợp chất
khó tổng hợp theo các phương pháp thông thường.
7. Xác định các chất còn thiếu trong sơ đồ phản ứng sau.

769 | Câu hỏi lí thuyết Hóa học hữu cơ OlympiaVN


770 | Câu hỏi lí thuyết Hóa học hữu cơ OlympiaVN
Tham khảo:

Hướng dẫn
1.

2.

771 | Câu hỏi lí thuyết Hóa học hữu cơ OlympiaVN


3.

4.

772 | Câu hỏi lí thuyết Hóa học hữu cơ OlympiaVN


5.

6.

773 | Câu hỏi lí thuyết Hóa học hữu cơ OlympiaVN


7.

774 | Câu hỏi lí thuyết Hóa học hữu cơ OlympiaVN


Bài 18
Diene không liên hợp là chất trung gian quan trọng để tổng hợp các hợp chất thiên
nhiên. Dưới đây là chuỗi phản ứng đơn giản để tổng hợp một diene như vậy.
1. Xác định công thức cấu tạo các sản phẩm A và B trong chuỗi sau đây:

Chuyển vị di-pi methane là dạng phản ứng quang hóa quan trọng, trong đó diene
không liên hợp chuyển thành cyclopropane có nhóm thế vinyl như sau:

Zizaene là một terpene thiên nhiên có trong dầu cỏ hương lau (cỏ Vetiver) và
được dùng để tổng hợp α-vetivone (sử dụng trong nhiều loại nước hóa). Zizaene
được tổng hợp từ hợp chất C sau:

2. Xác định công thức cấu tạo tiền chất của C biết rằng C được tạo thành từ phản
ứng chuyển vị di-pi methane.
Chuyển vị vinyl cyclopropane thành cyclopentane (như hình dưới) là một phương
pháp quan trọng dể tổng hợp các hệ vòng carbon 5 cạnh. Các phản ứng chuyển vị
này có thể xảy ra dưới tác dụng nhiệt, quang hóa hoặc khi có xúc tác. Dưới tác
dụng nhiệt và quang hóa, sự chuyển vị diễn ra qua sự hình gốc kép bền nhất.

Phản ứng chuyển vị vinyl cyclopropane là giai đoạn chính trong chuỗi tông hợp
Zizaene dưới đây.
3. Xác định công thức cấu tạo các chất D-F.

775 | Câu hỏi lí thuyết Hóa học hữu cơ OlympiaVN


E phản ứng với LiN(i-Pr)2 (LDA – một base có cấu tạo cồng kềnh) và PhS-SPh
tạo ra hợp chất G. Oxid hóa G bằng NaIO4 tạo ra hợp chất trung gian H, và cuối
cùng là sản phẩm I.

4. Xác định công thức cấu tạo các chất G, H, I.


5. Xác định công thức cấu tạo của J, K trong chuỗi phản ứng sau:

Hợp chất N là chất trung gian trong chuỗi phản ứng tổng hợp Zizaene. N được
tổng hợp theo cách sau:

6. Xác định các chất L, M, N.


Biết thành phần nguyên tố trong L là: %C = 60,55; %H = 6,41; %O = 20,95; còn
trong N là: %C = 75,63; %H = 9,97.
Ylide là tiểu phân có điện tích dương trên nguyên tử dị tố và điện tích âm trên
nguyên tử carbon liền kề. Phản ứng Wittig là phản ứng có sự tham gia của
phosphorus ylide.

776 | Câu hỏi lí thuyết Hóa học hữu cơ OlympiaVN


7. Xác định công thức của O, P và Zizaene.

8. Zizaene có bao nhiêu đồng phân quang học?

777 | Câu hỏi lí thuyết Hóa học hữu cơ OlympiaVN


Hướng dẫn
1.

2.

3.

778 | Câu hỏi lí thuyết Hóa học hữu cơ OlympiaVN


4.

5.

6.

779 | Câu hỏi lí thuyết Hóa học hữu cơ OlympiaVN


7.

8. 4.

780 | Câu hỏi lí thuyết Hóa học hữu cơ OlympiaVN


Bài 19
A. Các hợp chất dị vòng có vai trò quan trọng trong ngành Hóa học Hữu cơ.
Nhiều hợp chất dị vòng có đặc tính sinh học quan trọng. Chúng có thể là hợp
chất thơm hoặc không thơm. Do có mặt các dị tố nên chúng có tính chất khác
biệt so với các hợp chất thơm chứa hệ vòng carbon.
1. Chất nào trong số các dị vòng sau có tính thơm:

2. Sắp xếp các dị vòng sau theo thứ tự giảm dần khả năng phản ứng với CH3I:

Aziridine, một hợp chất dị vòng chứa nitrogen, có thể đóng vai trò như một tác
nhân nucleophile. Nó tác dụng với acetyl chloride theo các hướng khác nhau, tùy
thuộc vào điều kiện phản ứng.
3. Xác định công thức cấu tạo các chất H, I.

B. Hợp chất dị vòng 5 cạnh chứa nitrogen pyrrole là thành phần của nhân “heme”
của hemoglobin.
4. a) Vẽ tất cả các cấu trúc cộng hưởng của pyrrole (K).

781 | Câu hỏi lí thuyết Hóa học hữu cơ OlympiaVN


b) Xác định sản phẩm chính của phản ứng sau:

Pyrrole phản ứng với 4-(N,N-dimethylamino)benzaldehyde trong môi trường


acid, tạo ra sản phẩm L (C13H15N2) có màu đậm.

5. Xác định công thức cấu tạo của L.


Tổng hợp Paal-Knorr là một trong các phương pháp phổ biến được sử dụng để
tổng hợp pyrrole và dẫn xuất. Ví dụ:

6. Xác định các chất đầu và tỉ lệ mol của chúng để tổng hợp chất J sau:

C. Phản ứng đóng vòng là quá trình kết hợp hai chất chứa liên kết bội với nhau,
tạo thành sản phẩm vòng. Trong những phản ứng này, các đầu mạch của hai tác
nhân nối lại với nhau. Diels-Alder là một ví dụ về phản ứng đóng vòng [4+2]
trong đó 4 và 2 là số π-electron trong hai chất phản ứng.

782 | Câu hỏi lí thuyết Hóa học hữu cơ OlympiaVN


Indolizine (M) là hợp chất thuộc nhóm di vòng có dị tố nitrogen nằm ở cầu nối
hai vòng.

Indolizine tham gia phản ứng đóng vòng [8+2] với diethyl acetylenedicarboxylic
tạo thành P. Dehydrogen hóa P với xúc tác Pd tạo ra hợp chất Q.

7. Xác định công thức cấu tạo của Q.


8. Q có tính thơm không?
9. Dị vòng V có nhiều ứng dụng hữu ích được tổng hợp theo sơ đồ sau:

783 | Câu hỏi lí thuyết Hóa học hữu cơ OlympiaVN


9. Xác định các chất trung gian S, T, U và sản phẩm V.

784 | Câu hỏi lí thuyết Hóa học hữu cơ OlympiaVN


Hướng dẫn
1. B, C.
2. F > G > E.
3.

4.

785 | Câu hỏi lí thuyết Hóa học hữu cơ OlympiaVN


5.

6.

7.

8. Có.
9.

786 | Câu hỏi lí thuyết Hóa học hữu cơ OlympiaVN


Bài 20
Chất độc màu da cam là hóa chất làm rụng lá cây rừng có hoạt tính mạnh, từng
được sử dụng trong chiến tranh Việt Nam. Nó là hỗn hợp của hai loại thuốc diệt
cỏ 2,4-D và 2,4,5-T với tỉ lệ 1:1. Dưới đây là chuỗi phản ứng tổng hợp 2,4,5-T.

Giai đoạn 1: 1. Đun nóng với NaOH trong dung môi methanol-H2O dưới áp suất
cao; 2. Acid hóa. Giai đoạn 2: 1. Đun nóng với NaOH và ethanoic acid ở 140 oC;
2. Acid hóa.
Phổ 1H NMR của chất đầu (1,2,4,5-tetrachlorobenzene), hợp chất trung gian X và
2,4,5-T được cho ở cuối bài.
Sau giai đoạn đầu tiên của chuỗi phản ứng, nếu hỗn hợp phản ứng bị acid hóa bởi
D3O+ (chứ không phải acid thông thường, H3O+) thì tín hiệu ở 5,8 ppm trong phổ
1
H NMR của X sẽ biến mất. (Cho biết: Deuterium, D, là đồng vị 2H của
hydrogen).
a) Xác định công thức cấu tạo của 1,2,4,5-tetrachlorobenzene.
b) Xác định công thức cấu tạo của hợp chất trung gian X.
c) Gọi tên X theo danh pháp hệ thống.
d) Xác định công thức cấu tạo của X nếu hỗn hợp phản ứng được acid hóa bởi
D3O+ thay vì H3O+.
e) Vẽ cấu trúc của chloroethanoic acid.
g) Xác định các nguyên tử hydrogen tương ứng với các tín hiệu (A-D) trong phổ
1
H NMR của 2,4,5-T.
Tuy 2,4,5-T là một loại thuốc diệt cỏ hiệu quả cao nhưng nó không còn được tiếp
tục sản xuất đại trà do có sự hiện diện của một tạp chất vô cùng độc hại, gọi là
Dioxin, có thể được tạo thành từ hợp chất trung gian X trong quá trình tổng hợp
2,4,5-T.

Phổ 1H NMR của Dioxin chỉ có một peak đơn ở 7.2 ppm.
h) Xác định công thức cấu tạo của Dioxin.

787 | Câu hỏi lí thuyết Hóa học hữu cơ OlympiaVN


788 | Câu hỏi lí thuyết Hóa học hữu cơ OlympiaVN
Hướng dẫn

c) 2,4,5–trichlorophenol
d)

789 | Câu hỏi lí thuyết Hóa học hữu cơ OlympiaVN


790 | Câu hỏi lí thuyết Hóa học hữu cơ OlympiaVN
Bài 21
Từ lâu chúng ta đã biết rằng: sau khi ăn măng tây thì nhiều người có thể ngửi thấy
mùi lạ trong nước tiểu của họ. Hiện tượng này gọi là “asparagus-pee” - nước tiểu
bốc mùi sau khi ăn măng tây. [Việc có người ngửi thấy, có người không dường
như là do di truyền].
Vào thế kỉ thứ 19, các nhà khoa học cho rằng hợp chất gây ra mùi của “asparagus-
pee” là methanethiol, CH3SH. Tuy nhiên, nhiều nghiên cứu gần đây cho thấy rằng
không phát hiện methanethiol trong “asparagus-pee”, thay vào đó là hai hợp chất
chứa sulfur: A và B, được xác định qua phương pháp khối phổ. Ion M+ của hợp
chất A có peak ở m/e = 102, còn với hợp chất B là m/e = 150.
Để đối chứng, các mẫu A, B đã được tổng hợp trong phòng thí nghiệm theo sơ đồ
sau:

*by-product: sản phẩm thứ cấp.


Hợp chất A phản ứng với HBr tạo ra Y, còn với Br2 thì tạo ra hai đồng phân Z1
và Z2. Z1 và Z2 là đối quang và ion M+ của chúng có các peak ở m/e = 260, 262
và 264 (tỉ lệ 1:2:1). Y không có tính quang hoạt và có hai peak với ion M+ ở m/e
= 182 và 184 (tỉ lệ 1:1).
a) Vai trò của ‘DCC’ là
i) xúc tác
ii) tác nhân hydrate hóa
iii) tác nhân dehydrate hóa
iv) tác nhân hydrogen hóa
v) tác nhân dehydrogen hóa
b) Xác định công thức cấu tạo của các hợp chất A, B, W, X, Y, Z1, Z2.

791 | Câu hỏi lí thuyết Hóa học hữu cơ OlympiaVN


c) Cả A và B đều có thể bị thủy phân trong nước, giải phóng methanethiol (vậy
nên trước đây mới các nhà khoa học mới nhầm lẫn chất này là nguyên nhân gây
mùi của asparagus-pee). Viết phương trình phản ứng thủy phân A, B.
(Biết: Trong tự nhiên, bromine tồn tại ở dạng hỗn hợp đồng vị 79Br và 81Br với tỉ
lệ 1:1)
Hướng dẫn
a) iii. tác nhân dehydrate hóa.
b)

792 | Câu hỏi lí thuyết Hóa học hữu cơ OlympiaVN


c)

793 | Câu hỏi lí thuyết Hóa học hữu cơ OlympiaVN


Bài 22
Vào năm 2004, công ty dược phẩm Sanofi-Synthelabo đã công bố một hợp chất
mới có thể chống béo phì lẫn nghiện thuốc lá. Dưới đây là công thức cấu tạo
thuốc cùng với sơ đồ tổng hợp Rimonabant (được thương mại hóa dưới tên gọi
Acomplia).
Xác định công thức cấu tạo của các chất đầu (chlorobenzene, propanoyl chloride)
và các hợp chất A-F.

Hướng dẫn

794 | Câu hỏi lí thuyết Hóa học hữu cơ OlympiaVN


795 | Câu hỏi lí thuyết Hóa học hữu cơ OlympiaVN
Bài 23
Sơ đồ tổng hợp ngược dựa trên sự tổng hợp sildenafil đầu tiên, thành phần hoạt
tính trong Viagra - loại thuốc được dùng để chữa trị chứng rối loạn chức năng
cương dương ở nam giới. Chú ý rằng các sản phẩm phụ không được liệt kê trong
sơ đồ này.
a) Xác định công thức cấu tạo của các hợp chất B, D, E, F, G, I, J.
b) i. Đề nghị (các) tác nhân phù hợp cho chuyển hóa B  C.
ii. Đề nghị tác nhân cho chuyển hóa E  F.
c) Trong chuyển hóa H  I, giai đoạn đầu tiên trong cơ chế phản ứng là sự
deproton hóa bởi dung dịch sodium hydroxide. Hãy cho biết nguyên tử hydrogen
nào của H bị deproton hóa.
d) Vẽ cấu trúc N-methylpiperazine, tác nhân cần dể chuyển hóa J  sildenafil.
Hợp chất A được tổng hợp bởi phản ứng giữa hydrazine, N2H4 và tác nhân K theo
phương trình sau: K + N2H4  A + 2H2O.
e) i. Vẽ công thức cấu tạo của hydrazine.
ii. Xác định công thức cấu tạo của K.

796 | Câu hỏi lí thuyết Hóa học hữu cơ OlympiaVN


797 | Câu hỏi lí thuyết Hóa học hữu cơ OlympiaVN
Hướng dẫn
a)

798 | Câu hỏi lí thuyết Hóa học hữu cơ OlympiaVN


b) i. HCl; ii. NH3
c)

d)

e)

i.
ii.

799 | Câu hỏi lí thuyết Hóa học hữu cơ OlympiaVN


Bài 23
Tamiflu là thuốc chống virus cúm, hiện đang được dùng để chữa “cúm gia cầm”.
Dưới đây là công thức cấu tạo của tamiflu. Đường nét đậm biểu diễn liên kết
hướng lên trên mặt phẳng giấy. Đường nét đứt biểu diễn liên kết hướng xuống
dưới.

Tamiflu được bán ra dưới dạng muối phosphate để làm tăng tính tan.
a) Xác định nguyên tử sẽ bị proton hóa để tạo thành muối.
Một vấn đề đặt ra trong quá trình tổng hợp thuốc là phải đảm bảo chính xác về
hóa lập thể của mỗi tâm bất đối trong cấu trúc.
b) i. Đánh dấu hoa thị (*) vào các tâm bất đối.

b) Có thể có bao nhiêu đồng phân quang học tương ứng với cấu tạo trên.
Hai quá trình tổng hợp tamiflu đã được thử nghiệm, bắt đầu từ các hợp chất tự
nhiên là Quinic acid và Shikimic acid. Ưu điểm của shikimic acid là đã có sẵn
liên kết đôi, còn quinic cid thì lại có giá thành rẻ hơn.

800 | Câu hỏi lí thuyết Hóa học hữu cơ OlympiaVN


Dù đi từ acid nào thì một vấn đề nảy sinh là phải kiểm soát được phản ứng của
các nhóm OH. Do đó, giai đoạn đầu tiên của quá trình tổng hợp đi từ quinic acid
là phải bảo vệ hai nhóm OH cùng phía (so với mặt phẳng vòng), ở hai carbon
cạnh nhau, bằng phản ứng với propanone, có xúc tác acid để tạo thành nhóm
acetal.

Dưới đây là các giai đoạn tiếp theo của quá trình tổng hợp. Các sản phẩm vô cơ
không được liệt kê trong sơ đồ.
D phản ứng với một lượng dư pentan-3-one tạo ra hợp chất G.

*major product: sản phẩm chính; unwanted isomer: đồng phân không mong
muốn; left over intermediate: hợp chất trung gian còn sót lại.
c) Xác định công thức cấu tạo của các chất B, C, D, E, F. Chỉ rõ hóa lập thể của
từng chất bằng cách sử dụng các đường in đậm và nét đứt.
Có thể dễ dàng tổng hợp G từ shikimic acid đắt tiền hơn theo sơ đồ sau. Các sản
phẩm vô cơ không được liệt kê trong sơ đồ.

d) Xác định công thức cấu tạo của I, J, K (với hóa lập thể phù hợp).
Có thể khử hóa để mở vòng acetal G, tạo ra sản phẩm chính L. Khi phản ứng với
base, L bị vòng hóa, tạo ra hợp chất M chứa một vòng ba cạnh. Cho M phản ứng
với NaN3 để mở vòng mới, tạo ra hỗn hợp đồng phân N và O. Các đồng phân này
phản ứng với trimethylphosphine đều tạo ra sản phẩm P. Cho P phản ứng tiếp với
NaN3 để mở vòng mới thì thu được sản phẩm chính là Q. Chất này bị acyl hóa rồi
hydrogen hóa, tạo ra sản phẩm trung hòa Tamiflu.

801 | Câu hỏi lí thuyết Hóa học hữu cơ OlympiaVN


e) Khoanh tròn proton có tính acid mạnh nhất trong L (dễ bị bị loại bỏ bởi base).
f) Xác định công thức cấu tạo của M, N, O, P, Q, R (chỉ rõ hóa lập thể của từng
chất).

802 | Câu hỏi lí thuyết Hóa học hữu cơ OlympiaVN


Hướng dẫn
a)

b)

Số đồng phân: 8
c)

803 | Câu hỏi lí thuyết Hóa học hữu cơ OlympiaVN


d)

804 | Câu hỏi lí thuyết Hóa học hữu cơ OlympiaVN


e)

f)

M=

O hoặc N =

N hoặc O =

805 | Câu hỏi lí thuyết Hóa học hữu cơ OlympiaVN


P=

Q=

R=

806 | Câu hỏi lí thuyết Hóa học hữu cơ OlympiaVN


Bài 24
Các loại thuốc kháng histamine được dùng để làm giảm ảnh hưởng của các phản
ứng dị ứng trong cơ thể. Một trong số đó là thuốc fexofenadine có tác dụng chữa
các chứng hắt hơi, sổ mũi và ngứa mắt - gây ra bởi bệnh viêm mũi dị ứng - mà
không gây buồn ngủ.
Fexofenadine được bán dưới dạng muối hydrochloride.
a) i. Xác định nguyên tử sẽ bị proton hóa để tạo thành muối.

ii. Cho biết trong 120,14 mg muối thì thực tế có 112,00 mg fexofenadine. Tính
khối lượng mol của fexofenadine.
iii. Biết R là một gốc hữu cơ phổ biến. Hãy xác định công thức của R.
Dưới đây là sơ đồ tổng hợp Fexofenadine.

b) Hãy chỉ ra những nguyên tử hydrogen nào trong X sẽ bị tách ra bởi base trong
phản ứng đầu tiên.

807 | Câu hỏi lí thuyết Hóa học hữu cơ OlympiaVN


c) Xác định công thức cấu tạo của các hợp chất và tiểu phân trung gian từ A-G.
Hướng dẫn
a) i.

iii. R = C6H5 (phenyl)


b)

c)

808 | Câu hỏi lí thuyết Hóa học hữu cơ OlympiaVN


809 | Câu hỏi lí thuyết Hóa học hữu cơ OlympiaVN
Bài 24
Dưới đây là công thức cấu tạo của loperamide - thành phần hoạt tính trong các
loại thuốc chống tiêu chảy như imodium. Cũng giống như nhiều loại thuốc khác,
loperamide thường được bán dưới dạng muối hydrochloride để tăng tính tan trong
nước.

a) Khoanh tròn nguyên tử bị proton hóa trong loperamide khi tạo muối.
Dưới đây là sơ đồ tổng hợp loperamide.

b) i. Ester A có thể được tạo thành bằng cách trộn một hỗn hợp alcohol và
carboxylic acid với một lượng sulfuric acid vừa đủ, đóng vai trò xúc tác. Xác định
công thức cấu tạo alcohol và carboxylic acid.

810 | Câu hỏi lí thuyết Hóa học hữu cơ OlympiaVN


ii. Deproton hóa ester A bằng base, thu được anion B. Xác định công thức cấu tạo
của A và chỉ rõ proton nào bị tách ra bởi base để tạo thành anion B.
B mở vòng ba cạnh epoxyethane tạo ra tiểu phân trung gian là anion C. Tiểu phân
này bị vòng hóa và tách ethoxide, C2H5O-, tạo ra ester D.
c) Xác định công thức cấu tạo của anion C và ester vòng D.
Ester D phản ứng với HBr thì bị mở vòng, tạo ra carboxylic E. E phản ứng với
SOCl2 tạo ra F.
d) Xác định công thức cấu tạo của các hợp chất E và F.
F phản ứng với dimethylamine tạo ra G. G có thể có hai đồng phân cấu tạo: hoặc
là amide mạch hở, hoặc là muối bromide vòng. Trong phổ 13C NMR của muối
bromide vòng có 10 tín hiệu.
e) Xác định công thức cấu tạo hai đồng phân G: i. dạng mạch hở và ii. dạng muối
bromide vòng.

811 | Câu hỏi lí thuyết Hóa học hữu cơ OlympiaVN


Hướng dẫn
a)

b) i.

ii.

c)

d)

e) i.

812 | Câu hỏi lí thuyết Hóa học hữu cơ OlympiaVN


ii.

813 | Câu hỏi lí thuyết Hóa học hữu cơ OlympiaVN


Bài 25
Long diên hương, một sản phẩm trao đổi chất của cá nhà táng, là một trong các
thành phần giá trị nhất để sản xuất nước hoa, mỹ phẩm. Gần đây, nó đã bị thay
thế bởi các sản phẩm tổng hợp khác, như Ambrox® - chất sở hữu mùi hương hổ
phách mạnh mẽ. Nhiều nhóm nghiên cứu khác nhau đã đề xuất các phương pháp
tổng hợp Ambrox từ những hợp chất thiên nhiên có nguồn gốc thực vật. Dưới đây
là một sơ đồ tổng hợp đi từ (-)-drimenol, được chiết xuất từ vỏ cây Drimys winteri
ở Chile.

a) Biết mỗi năm có 10 tấn Ambrox được sản xuất. Lượng chất này tương đương
với bao nhiêu mol?
b) Xác định công thức cấu tạo các hợp chất từ A-D.
Hiệu suất của mỗi giai đoạn trong sơ đồ tổng hợp được cho bên dưới các mũi tên
phản ứng.
c) i. Tính hiệu suất toàn quá trình tổng hợp Ambrox.
ii. Tính khối lượng (-)-drimenol cần dùng mỗi năm, nếu toàn bộ Ambrox thương
mại đều được sản xuất theo phương pháp này.
iii. Trong vỏ cây Drimys winteri chứa khoảng 0,5 % (-)-drimenol về khối lượng.
Tính khối lượng vỏ cây cần dùng mỗi năm, nếu toàn bộ Ambrox thương mại đều
được sản xuất theo phương pháp này.

814 | Câu hỏi lí thuyết Hóa học hữu cơ OlympiaVN


Hướng dẫn
b)

815 | Câu hỏi lí thuyết Hóa học hữu cơ OlympiaVN


Bài 26
Barbiturate là các dẫn xuất của barbituric acid – là chất có khả năng làm ức chế
thần kinh trung ương. Các barbiturate được sử dụng trong điều trị rối loạn co giật.
Một ví dụ về barbiturate được dùng làm thuốc chống co giật trong nhiều năm qua
là phenobarbital. Phenobarbital có thể đươc tổng hợp từ hai chất A và G.

a) Hợp chất A là một sản phẩm trao đổi chất có mặt trong nước tiểu con người.
Sử dụng các thông tin trên và dữ kiện dưới đây để xác định công thức phân tử của
hợp chất A. Trình bày chi tiết các phân tích của bạn.
 Đốt cháy 0,250 gam hợp chất A tạo ra 0,178 gam CO2 và 0,146 gam H2O.
 Đun sôi 0,250 gam hợp chất A với dung dịch kiềm dư thì toàn bộ nitrogen
trong A bị giải phóng dưới dạng NH3. Lượng NH3 sinh ra từ phản ứng
này vừa đủ để trung hòa 40,8 cm3 dung dịch hydrochloric acid 0,200
mol.dm-3.
 Kết quả phân tích khối phổ cho thấy khối lượng mol của A xấp xỉ 60
gam.mol-1.
Hợp chất G có thể được tổng hợp theo sơ đồ sau. (Chú ý: NBS là N-
bromsuccinimide - nguồn tạo gốc bromine; Et là ethyl)

816 | Câu hỏi lí thuyết Hóa học hữu cơ OlympiaVN


b) Xác định công thức cấu tạo của các hợp chất A-G.

817 | Câu hỏi lí thuyết Hóa học hữu cơ OlympiaVN


Hướng dẫn
a) CH4N2O
b)

818 | Câu hỏi lí thuyết Hóa học hữu cơ OlympiaVN


Bài 27

Với doanh số hàng năm đạt hàng tỉ bảng, Lipitor - thuốc làm giảm cholesterol của
hãng Pfizer - là một trong những loại thuốc bán chạy nhất thế giới nhiều năm qua.
Dưới đây là những giai đoạn đầu tiên trong quá trình tổng hợp của Lipitor.
Các tần số dao động đặc trưng trong phổ IR của các hợp chất trung gian được ghi
kèm trong các ô tương ứng. Trong số đó không có các dao động của bất kì liên
kết C-C hoặc C-H nào. Tần số dao động của các liên kết đơn (không phải C-H)
cũng không được liệt kê. Bạn không thể chỉ dựa vào các giá trị tần số dao động
để dự đoán, nhưng có thể kết hợp với suy luận cẩn thận, để xác định cấu trúc các
chất chưa biết.
Chú ý: Tất cả các sản phẩm phụ đều không được liệt kê trong sơ đồ.

a) Xác định công thức cấu tạo của các hợp chất B – G. Trong quá trình làm toàn
bộ các câu hỏi, hãy hoàn thành bảng giá trị hấp thụ IR trong các chất A – M dưới
đây:

819 | Câu hỏi lí thuyết Hóa học hữu cơ OlympiaVN


Khi phản ứng với base mạnh, ester H bi deproton hóa, tạo ra anion I - là một
carbon nucleophile có hoạt tính mạnh. Gốc R trong cấu trúc là một mạch alkyl
không bị biến đổi trong toàn quá trình tổng hợp.

b) Xác định công thức cấu tạo của anion I.


Dưới đây là phần tiếp theo của quá trình tổng hợp.

c) Xác định công thức cấu tạo các hợp chất J, K, L.


Trong một nhánh khác của quá trình tổng hợp, hợp chất N phản ứng với
phenylamine tạo thành hợp chất O. Chất này bị deproton hóa trong base, tạo thành
anion P, là một carbon nucleophile khác. Anion P phản ứng với benzaldehyde tạo
thành Q. Dehydrate hóa Q thu được hợp chất S. Cho S phản ứng với
fluorobenzaldehyde khi có xúc tác, tạo thành hợp chất T.

820 | Câu hỏi lí thuyết Hóa học hữu cơ OlympiaVN


d) Xác định công thức cấu tạo của phenylamine và 4-fluorobenzaldehyde.
e) Xác định công thức cấu tạo của chất O, anion P và các chất Q, S.
Trong giai đoạn cuối của quá trình tổng hợp, T và M phản ứng với nhau tạo ra
hợp chất U. Thủy phân U trong môi trường acid, thu được dược chất Lipitor.

f) Xác định công thức cấu tạo của hợp chất U.

821 | Câu hỏi lí thuyết Hóa học hữu cơ OlympiaVN


Hướng dẫn
a)

b)

822 | Câu hỏi lí thuyết Hóa học hữu cơ OlympiaVN


c)

823 | Câu hỏi lí thuyết Hóa học hữu cơ OlympiaVN


d)

824 | Câu hỏi lí thuyết Hóa học hữu cơ OlympiaVN


e)

825 | Câu hỏi lí thuyết Hóa học hữu cơ OlympiaVN


f)

826 | Câu hỏi lí thuyết Hóa học hữu cơ OlympiaVN


Bài 28
Các dược phẩm diazepam (Valium®) và alprazolam (Xanax®) đều thuộc nhóm
hợp chất benzodiazepine, từng được ca ngợi như thần dược trị bá bệnh. Năm
1988, Ủy ban An toàn Thuốc đã khuyến cáo các bác sĩ nên hạn chế các loại thuốc
này khi kê đơn do sự lạm dụng quá mức. Tuy nhiên, trong năm 2010, có hơn 6,6
triệu đơn thuốc chống trầm cảm chứa benzodiazepine đã được bán ra ở Anh.
Heath Ledger, Michael Jackson và Whitney Houston là vài người trong số những
bệnh nhân được cho là đã chết vì sốc các loại thuốc liên quan đến benzodiazepine.
Dưới đây là sơ đồ tổng hợp diazepam từ 4-chloroaniline. Con số ghi dưới các mũi
tên là hiệu suất của từng phản ứng.

827 | Câu hỏi lí thuyết Hóa học hữu cơ OlympiaVN


a) i. Tính hiệu suất toàn quá trình chuyển hóa 4-chloroaniline thành diazepam.
ii. Một bệnh nhân được khuyên dùng diazepam trong 3 năm với liều lượng 4 lần
mỗi ngày và mỗi lần dùng 5 mg. Tính khối lượng diazepam và 4-chloroaniline
cần để tạo ra đủ lượng thuốc cho bệnh nhân trên.
b) Xác định công thức cấu tạo của các chất trung gian A-H trong sơ đồ tổng hợp
diazepam.
Trong gan, diazepam bị demethyl hóa (tách methyl). Sản phẩm của phản ứng này
là tiền chất cho quá trình tổng hợp dược chất alprazolam.

c) Dưới đây là sơ đồ tổng hợp alprazolam. Xác định công thức cấu tạo các chất
trung gian I, J, K.

828 | Câu hỏi lí thuyết Hóa học hữu cơ OlympiaVN


d) Trong giai đoạn cuối của sơ đồ, hợp chất L phản ứng với một chất gọi là
‘DEAD’, tạo ra alprazolam. DEAD là viết tắt của diethyl azodicarboxylate.

Phản ứng chuyển L  alprazolam thuộc loại


i) Đồng phân hóa
ii) Thủy phân
iii) Ngưng tụ
iv) Oxid hóa
v) Khử hóa

829 | Câu hỏi lí thuyết Hóa học hữu cơ OlympiaVN


Hướng dẫn
b)

830 | Câu hỏi lí thuyết Hóa học hữu cơ OlympiaVN


831 | Câu hỏi lí thuyết Hóa học hữu cơ OlympiaVN
c)

d) Sự oxid hóa.

832 | Câu hỏi lí thuyết Hóa học hữu cơ OlympiaVN


Bài 29
Dược chất tazrotene (được bán dưới tên thương mại là Zorac® hoặc Tazorac®)
được khuyên dùng làm kem trị mụn và một số loại bệnh khác trên da. Thông
thường, các loại kem được bán ra chứa khoảng 0,05 % tazarotene về khối lượng.

a) Khối lượng mol của tazarotene là 351,46 gam.mol-1. Giả sử rằng khối lượng
riêng của tazarotene là 0,90 gam.cm-3. Tính nồng độ tazarotene trong kem theo
mol.dm-3.
Dưới đây là sơ đồ tổng hợp tazarotene. Tất cả các sản phẩm phụ không được nhắc
đến. Sơ đồ này bắt đầu với giai đoạn chuyển 2-chloro-5-methylpyridine thành
ester B.

b) Xác định công thức cấu tạo hợp chất A và ester B.


Phần thứ hai của sơ đồ tổng hợp bắt đầu từ thiophenol Chất này được chuyển hóa
thành hợp chất I qua nhiều giai đoạn.

833 | Câu hỏi lí thuyết Hóa học hữu cơ OlympiaVN


c) Xác định công thức cấu tạo các chất D, E, F, I và các anion C-, G-.
d) Phản ứng chuyển H  I thuộc loại
i) oxid hóa
ii) khử hóa
iii) cộng hợp
iv) tách
v) thế
Cuối cùng, cho hợp chất I phản ứng với một base rất mạnh, thu được anion J. Cho
J phản ứng với hợp chất B, thu được tazarotene.

e) Xác định công thức cấu tạo anion J.


f) Có thể quan sát thấy bao nhiêu tín hiệu trong phổ 13C NMR cuả tazarotene.
Tazarotene thực chất là một tiền thuốc (pro-drug), nghĩa là khi đi vào cơ thể, nó
được chuyển hóa trong quá trình trao đổi chất để chuyển thành dạng có hoạt tính.
Dạng hoạt động của tazarotene có khối lượng mol bằng 323,41 gam.mol-1 và trong
phổ 13C NMR có nhiều hơn tazarotene 2 tín hiệu.
g) Xác định công thức cấu tạo cho dạng hoạt động của thuốc.

834 | Câu hỏi lí thuyết Hóa học hữu cơ OlympiaVN


835 | Câu hỏi lí thuyết Hóa học hữu cơ OlympiaVN
Hướng dẫn

836 | Câu hỏi lí thuyết Hóa học hữu cơ OlympiaVN


d) Phản ứng tách.
e)

f) 20 tín hiệu.

837 | Câu hỏi lí thuyết Hóa học hữu cơ OlympiaVN


g)

838 | Câu hỏi lí thuyết Hóa học hữu cơ OlympiaVN


Bài 30
Tên gọi của bọ xịt hơi cay bắt nguồn từ cơ chế phòng vệ của chúng. Khi bị tấn
công, loài bọ này sẽ phun một loại hóa chất cay nồng vào kẻ thù. Trong bụng của
chúng có hai khoang riêng biệt, một khoang chứa dung dịch hydrogen peroxide
và khoang còn lại chứa dung dịch nước của một chất hữu cơ, kí hiệu là A. Khi
loài bọ này bị tấn công, chất lỏng từ hai khoang được xịt vào một khoang trộn có
chứa các enzyme. Một trong các enzym đó, catalase, xúc tác cho phản ứng phân
hủy hydrogen peroxide thành oxygen và nước.
a) i. Viết phương trình (có cân bằng) cho phản ứng này.
ii. Phản ứng này thuộc loại
1) oxid hóa
2) khử hóa
3) dị phân (tự oxid-hóa khử)
4) thủy phân
5) tách nước
Một số tiểu phân trung gian tạo thành trong phản ứng phân hủy hydrogen peroxide
phản ứng với hợp chất hữu cơ A tạo ra sản phẩm B. Phản ứng tổng quát có thể
được viết dưới dạng phản ứng của A với toàn bộ lượng oxygen tạo thành trong
phản ứng ở mục a.
1
A  O2  B  H 2 O
2
b) Viết phương trình tổng của phản ứng giữa hydrogen peroxide và hợp chất A,
tạo thành B.
Nhiệt độ của hỗn hợp phun ra từ bọ xịt hơi cay đạt tới nhiệt độ sôi của nước, nghĩa
là nóng hơn rất nhiều so với nhiệt độ cơ thể loài bọ này (xấp xỉ 20 oC).
c) i. Tính năng lượng cần để đun 1 dm3 hỗn hợp này từ 20 oC lên 100 oC. Giả sử
rằng nhiệt dung riêng và khối lượng riêng của hỗn hợp này giống với nước tinh
khiết, lần lượt bằng 4,18 J.gam-1.K-1 và 100 gam.cm-3.
ii. Giả sử hai dung dịch được trộn lẫn với thể tích bằng nhau. Tính nồng độ tối
thiểu của dung dịch hydrogen peroxide trong khoang chứa riêng. Biết biến thiên
enthalpy chuẩn của phản ứng tổng (ở câu b) trên mỗi mol A là -203 kJ.mol-1.
Hợp chất A là dẫn xuất thế ở vị trí 1, 2, 4 của benzene. Các nhóm thế được kí hiệu
là X.

839 | Câu hỏi lí thuyết Hóa học hữu cơ OlympiaVN


d) Dẫn xuất thế trên có thể có bao nhêu đồng phân nếu:
i. Các nhóm thế khác nhau.
ii. Có hai nhóm thế giống.
Kết quả phân tích định tính bằng phương pháp đốt cháy cho thấy trong hợp chất
A chỉ có carbon, hydrogen và oxygen. Trong hợp chất A, dao động của một số
liên kết chứa nhóm thế X góp phần vào sự xuất hiện các peak đặc trưng trong phổ
IR.

Các peak đặc trưng được kí hiệu là I, II.


e) Xác định các dao động liên kết ứng với peak I, II.
Trong hợp chất A, có hai nhóm thế giống hệt nhau (gọi các nhóm thế trong A là
Xa, Xa, Xb).
Phổ 13C NMR của A có 7 tín hiệu. Còn phổ 1H NMR được cho dưới đây. Peak
III, IV trong phổ 1H NMR biến mất khi thêm D2O vào mẫu phân tích.

840 | Câu hỏi lí thuyết Hóa học hữu cơ OlympiaVN


f) i. Xác định hai nhóm thế giống nhau (Xa).
ii. Xác định nhóm thế còn lại (Xb).
Có thể xác định các proton của vòng benzene trong hợp chất A (H1, H2, H3) bằng
cách phân tích các hằng số ghép cặp của chúng trong phổ 1H NMR.

Sự chẻ tín hiệu gây ra bởi sự ghép cặp của 2 proton không tương đương trong
vòng benzene ở vị trí 1,2 lớn hơn giá trị của hai proton ở vị trí 1,3. Còn giá trị
tương ứng với hai proton ở vị trí 1,4 thì quá nhỏ, gần như không quan sát được
trên phổ.
g) Gán các proton H1, H2, H3 với tín hiệu phổ tương ứng trong số các tín hiệu từ
III-VIII.
Bọ xịt hơi cay cũng sử dụng một hợp chất hữu cơ C đơn giản hơn cho mục đích
tương tự. Hợp chất C cũng tương tự hợp chất A, trong đó một trong các nhóm thế
X bị thay thế bằng một nguyên tử hydrogen (C là một dẫn xuất hai nhóm thế của
benzene).
Phổ 1H NMR của C chỉ có 2 tín hiệu.
h) Xác định công thức cấu tạo của C, từ đó xác định cấu trúc của A.
Oxid hóa hợp chất C theo cách oxid hóa A  B thì thu được hợp chất D. Phổ 1H
NMR của D chỉ có 1 tín hiệu.
i) Xác định công thức cấu tạo của B và D.
Hướng dẫn

841 | Câu hỏi lí thuyết Hóa học hữu cơ OlympiaVN


e)

f)

g)

h)

842 | Câu hỏi lí thuyết Hóa học hữu cơ OlympiaVN


i)

843 | Câu hỏi lí thuyết Hóa học hữu cơ OlympiaVN


Bài 31
Từ lâu, thuốc Ritalin® dã được sử dụng để điều trị chứng rối loạn tăng động giảm
chú ý (thường gọi là ADHD). Gần dây, đã có thông tin về việc về việc các sinh
viên sử dụng dược chất này làm thuốc kích thích khi ôn thi. Dưới đây là công
thức cấu tạo của Ritalin, trong đó R là một gốc hydrocarbon.

Ritalin được tổng hợp theo sơ đồ sau. Trong sơ đồ, một vài tần số đặc trưng cho
dao động liên kết trong phổ IR được ghi kèm với các hợp chất trung gian.

Sơ đồ tổng hợp đi từ phản ứng của benzyl chloride với sodium cyanide, tạo thành
hợp chất A.
a) Xác định công thức cấu tạo của A.
Deproton hóa (tách H+) chất A, thu được anion B-.
b) Xác định công thức cấu tạo của B-.
c) Xác định công thức cấu tạo các chất C, D, E, F. Không cần xác định gốc
hydrocarbon.
Trong thuốc viên, Ritalin tồn tại ở dạng muối hydrochloride (muối HCl).

844 | Câu hỏi lí thuyết Hóa học hữu cơ OlympiaVN


d) Khoanh tròn nguyên tử bị proton hóa để tạo thành muối hydrochloride.

e) i. Trong mỗi viên thuốc chứa 10,00 mg muối hydrochloride tương đương với
8,647 mg Ritalic. Sử dụng thông tin này để xác định khối lượng mol của Ritalic.
Trình bày cách tính.
ii. Xác định công thức gốc hydrocarbon R.
f) Xác định các nhóm chức tương ứng với các giá trị tần số dao động trong phổ
IR của các chất trong sơ đồ. Dùng kí hiệu mũi tên để chỉ rõ liên kết dao động.

Đồng phân quang học của một phân tử là các đồng phân có trật tự sắp xếp các
nguyên tử giống nhau nhưng khác nhau về cấu trúc không gian ba chiều. Dược
tính của một loại thuốc phụ thuộc vào cấu trúc không gian của phân tử.
Sơ đồ tổng hợp trên có thể tạo ra hỗn hợp 4 đồng phân của Ritalin như dưới đây.
Một số đồng phân có hoạt tính cao hơn các đồng phân khác.

g) Hãy cho biết những đồng phân nào là đối quang của nhau.
Một số đồng phân lập thể có hoạt tính yếu có thể được chuyển thành dạng có hoạt
tính mạnh hơn bằng cách deproton hóa Ritalin bằng alkoxide base OR-, tạo ra
anion G-. Sau đó, proton hóa lại để tạo ra đồng phân lập thể khác.

845 | Câu hỏi lí thuyết Hóa học hữu cơ OlympiaVN


h) Xác định công thức cấu tạo anion G-.
i) Những đồng phân nào có thể chuyển hóa lẫn nhau qua trạng thái trung gian
anion G-.
Cặp đồng phân Có thể chuyển Không thể
hóa qua anion G- chuyển hóa qua
anion G-
1 và 2

1 và 3

1 và 4

2 và 3

2 và 4

3 và 4

846 | Câu hỏi lí thuyết Hóa học hữu cơ OlympiaVN


Hướng dẫn
a)

b)

c)

d)

847 | Câu hỏi lí thuyết Hóa học hữu cơ OlympiaVN


f)

848 | Câu hỏi lí thuyết Hóa học hữu cơ OlympiaVN


g)

849 | Câu hỏi lí thuyết Hóa học hữu cơ OlympiaVN


h)

i)

850 | Câu hỏi lí thuyết Hóa học hữu cơ OlympiaVN


Bài 32
Vào năm 2015, dược chất Addyi (có tên Hóa học là Flibanserin) được công bố là
loại thuốc đầu tiên được chấp thuận trong điều trị rối loạn (giảm ham muốn) tình
dục của nữ giới. Dưới đây là công thức cấu tạo của Adyyi:

Hai tiền chất của Flibanserin, A và C, được điều chế theo các sơ đồ sau:

a) Xác định công thức thực nghiệm (công thức phân tử) của A.
b) Xác định công thức cấu tạo của A, B, C.
Phần còn lại của quá trình tổng hợp được bắt đầu với chất Z như sau:

c) Xác định công thức cấu tạo các hợp chất D, E, F, G và sản phẩm phụ H.

851 | Câu hỏi lí thuyết Hóa học hữu cơ OlympiaVN


Khi mới được tổng hợp, Flibanserin tồn tại ở dạng muối monoprotic mang 1 điện
dương.
d) Hãy khoanh tròn nguyên tử bị proton hóa trong cấu trúc Flibanserin,

Hợp chất Z được tổng hợp từ 1,2-diaminobenzene và ethyl acetoacetate theo sơ


đồ sau:

e) Xác định công thức cấu tạo X, Y.

852 | Câu hỏi lí thuyết Hóa học hữu cơ OlympiaVN


Hướng dẫn

853 | Câu hỏi lí thuyết Hóa học hữu cơ OlympiaVN


854 | Câu hỏi lí thuyết Hóa học hữu cơ OlympiaVN
e)

855 | Câu hỏi lí thuyết Hóa học hữu cơ OlympiaVN


Bài 33
a) Dưới đây là quy trình tổng hợp của giáo sư E. J. Corey với 12S-hydroxy-
5,8,14-(Z)-10-(E)-eicosatetraenoic acid (12-HETE), một chất chuyển hóa của
of arachidonic acid được tạo thành trong tiểu cầu con người. Hãy bổ sung các
trung gian và tác nhân còn thiếu trong sơ đồ. Chú ý rằng nhóm
tetrahydropyranyl (THP) là nhóm bảo vệ alcohol, bị tách ra trong môi trường
protic dưới tác động của acid.

856 | Câu hỏi lí thuyết Hóa học hữu cơ OlympiaVN


* Ar = mesityl, hoặc 2,4,6-trimethylphenyl.
Tại sao bạn cho rằng một tác nhân sulfonyl chloride bị cản trở không gian như
vậy lại thuận lợi trong trường hợp này?
b) Quy trình tổng hợp ở ý a trình bày hai phương pháp phổ biến nhất để điều
chế các Z-alkene hai nhóm thế. Gọi tên của chúng.
c) Nghiên cứu phản ứng của muối phosphonium C và hợp chất B để tạo thành
allylic alcohol D, đã được suy luận trong ý a. Bạn nên chú ý việc sử dụng hai
đương lượng base mạnh methyllithium cũng như cấu hình E bất thường của
sản phẩm alkene. Hãy trình bày cơ chế của phản ứng giả định của muối
phosphonium C với chỉ một đương lượng methyllithium ở -78 oC, sau đó gia
nhiệt tới 23 oC. Dự đoán sản phẩm tạo thành.
d) Bây giờ, xét phản ứng thực của C với hai đương lượng methyllithium ở -78
o
C. Chỉ rõ trung gian được tạo thành ở -25 oC trong quá trình từng bước này.
Sau đó, xét phản ứng tiếp của trung gian này với aldehyde B. Bạn cần nhanh
chóng nhận ra rằng, về mặt lí thuyết, có thể tạo thành nhiều trung gian đồng
phân lập thể oxaphosphetane (hoặc betaine). Có bao nhiêu trung gian có thể
được tạo thành? Chỉ có hai trung gian đồng phân lập thể oxaphosphetane có
thể tạo thành sản phẩm (E)-alkene quan sát được, hãy xác định chúng. Bây
giờ, rõ ràng chuyển hóa quan sát được có độ chọn lọc rất đáng chú ý!
Hướng dẫn
a)

857 | Câu hỏi lí thuyết Hóa học hữu cơ OlympiaVN


858 | Câu hỏi lí thuyết Hóa học hữu cơ OlympiaVN
Chuyển hóa của acetonide thành phosphonium alcohol C diễn ra qua sự sulfonyl
hóa chọn lọc nhóm chức alcohol bậc một của diol, tạo thành từ phản ứng loại
nhóm bảo vệ acetonide trong môi trường acid. Sự sulfonyl hóa nhóm chức alcohol
bậc một chọn lọc hơn alcohol bậc hai khi sử dụng một tác nhân sulfonyl chloride
gây án ngữ không gian.
b) Phản ứng Wittig và phản ứng hydrogen hóa alkyne không hoàn toàn khi có
mặt xúc tác Lindlar.
c)

859 | Câu hỏi lí thuyết Hóa học hữu cơ OlympiaVN


d)

Xác định hai trung gian:

Trước đó, giáo sư Corey đã chỉ rằng nói chung thì đồng phân dia trong ô vuông
(hoặc oxophosphenate tương ứng của nó) là sản phẩm chính của phản ứng giữa
một "beta-oxido ylide" với một aldehyde. Mặc dù nguồn gốc của sự chọn lọc
này chưa được biết đến chắc chắn, nhưng nó có thể liên quan đến sự cực tiểu
hóa lực đẩy tĩnh điện. Sản phẩm của phản ứng này được dự đoán trên cơ sở các
quan sát trước đó của giáo sư Corey với các tác nhân tương tự.

860 | Câu hỏi lí thuyết Hóa học hữu cơ OlympiaVN


Bài 34
A (C13H10S3O) là một hợp chất có khả năng phát quang xanh. Oxy hóa A thu được
B có thể làm mất màu nước brom và cho kết tủa trắng bạc với AgNO3/NH3 Oxy
hóa mạnh A bằng KMnO4/H2SO4 thu được axit thiophen-2,5-dicacboxylic và một
chất C (C9H6S2O2) có khả năng hóa hồng quỳ tím. C đồng nhất với sản phẩm khi
cho 2,2’-bithienyl phản ứng với POCl3/DMF rồi xử lý trung gian thu được với
Ag2O. Xác định cấu trúc các chất chưa biết.
Hướng dẫn

861 | Câu hỏi lí thuyết Hóa học hữu cơ OlympiaVN


Bài 35
Thiamin hydroclorua có công thức phân tử C12H18ON4SCl2 là dẫn xuất của
vitamin B1 có trong gạo. Xử lý thiamin với Na2SO3 cho hai mảnh A (C6H9N3O3S)
và B (C6H9NOS). A tan được trong kiềm và giải phóng H2SO4 khi đun với nước
sôi. A phản ứng với HNO2 thấy có thoát Nitơ và xử lý A với Na/NH3 cho 4-
amino-2,5-dimetylpyrimidin. B không cho phản ứng với HNO2. Oxy hóa B bằng
PCC thu được sản phẩm cho phản ứng tráng gương. B không quang hoạt và không
cho phản ứng iodoform. Oxy hóa B bằng HNO3 thu được axit 4-metyl-thiazole-
5-carboxylic. Khử hóa thiamin bằng Na/NH3 cho 4-amino-5-aminometyl-2-
metylpyrimidin. Từ các thông tin trên hãy xác định cấu trúc A, B và Vitamin B1.
Hướng dẫn
A (C6H9N3OS) tan được trong kiềm và giải phóng H2SO4 khi đun nóng với nước
sôi. Trong A phải có nhóm SO3H. A cho phản ứng thoát N2 với HNO2, tức A
phải có nhóm amin, và dựa vào cấu trúc sản phẩm sau khi cho A phản ứng với
NH3 thì nhóm amin trong A phải ở vị trí số 4.
Oxy hóa B bằng HNO3 thu được axit 4-metylthiazole-5-carboxylic cho thấy trong
Vitamin B1 tồn tại khung này, và từ dữ kiện oxy hóa bằng PCC cho sản phẩm
phản ứng tráng gương. B không quang hoạt và không có phản ứng iodoform cho
thấy trong B buộc phải có nhánh CH2CH2OH.
Vì B có sẵn một S, tức trong mảnh pyrimidin phải không có lưu huỳnh, nhóm
SO3H chỉ được đưa vào sau khi phân mảnh bằng Na2SO3. Tức nhóm SO3H không
thể nằm ở vị trí 6, vậy nhóm SO3H phải thế một hydro của nhóm metyl.
Như vậy cấu trúc các mảnh A và B như sau:

Xử lý thiamin bằng Na/NH3 cho 4-amino-5-aminometyl-2-metylpyrimidin cho


thấy A nối với B qua nhóm metylen C5, như vậy mảnh A1 sẽ là cấu trúc phù hợp.
Như vậy phản ứng giữa thiamin và Na2SO3 là một phản ứng SN2 với sự đi ra của
mảnh B, tức nguyên tử N sẽ nối trực tiếp với nhóm CH2 ở mảnh A1 (Nối trên S
sẽ có sự tạo thành R3S+ kém bền nên khả năng này bị loại trừ).
Vitamin B1 có công thức như sau:

862 | Câu hỏi lí thuyết Hóa học hữu cơ OlympiaVN


863 | Câu hỏi lí thuyết Hóa học hữu cơ OlympiaVN
Bài 36
Chất A (C11H21N) chịu sự tách Hoffmann (CH3I dư, rồi đun nóng với Ag2O) thu
được B (C12H23N). Ozon phân B thu được HCHO và chất C (C11H21NO) cho kết
tủa vàng với I2/NaOH. Xử lý C với mCPBA rồi đun nóng trong KOH thu được D
(C7H12O). D phản ứng với PCC thu được E (C7H10O). E phản ứng với Na/NH3
thu được 2,3-dimetylxiclopentanon. Xác định cấu trúc các chất chưa biết.
Hướng dẫn

864 | Câu hỏi lí thuyết Hóa học hữu cơ OlympiaVN


Bài 37
Hợp chất A (C10H14O5) cho phản ứng iodoform, phản ứng với NaOH loãng cho
axit axetic và hợp chất B (C7H10O). Đun nóng A với dung dịch K2CO3 thu được
C (C7H12O2) cho phản ứng iodoform và phản ứng tráng gương. C phản ứng với
amoni cacbonat thu được D (C7H11N) không có tính bazơ, phản ứng được với
MeLi thoát ra khí CH4. C phản ứng với AgNO3/NH3 cho E (C7H12O3), chất này
phản ứng với I2/NaOH thu được axit 2,3-dimetylbutandioic. Mặt khác D có thể
chuyển hóa thành C trong môi trường kiềm. Hãy xác định cấu trúc các chất chưa
biết.
Hướng dẫn

865 | Câu hỏi lí thuyết Hóa học hữu cơ OlympiaVN


Bài 38
Hợp chất X C19H22O6 không tác dụng với 2,4-dinitrophenylhidrazin; không có
phản ứng iođofom. Khi tác dụng với HI dư, X cho 4 mol CH3I và chất Y
C15H14O5I2. Oxi hóa Y bằng dung dịch KMnO4 đun nóng thu được axit 2,4-
đihidroxibenzoic, axit 2,4,6-trihidroxibenzoic, khí CO2 có số mol mỗi chất bằng
số mol Y đã dùng. Khi cho Y tác dụng với dung dịch NaOH loãng sau đó cho sản
phẩm tác dụng với HIO4 thu được hợp chất Y1 và axit 2,4-dihidroxibenzoic. Xác
định công thức cấu tạo của X, Y và Y1, biết X không chứa vòng nhỏ hơn 6 cạnh.
Hướng dẫn
Khi oxy hóa Y C15H16O5I2 bằng KMnO4 thu được axit 2,4-đihidroxibenzoic, axit
2,4,6-trihidroxibenzoic, khí CO2 có số mol mỗi chất bằng số mol Y đã dùng tức
trong Y có hai vòng thơm nối với nhau bằng một mạch nhánh có 3 cacbon. Như
vậy khung cacbon của Y có dạng:

Mặt khác khi cho Y tác dụng với NaOH loãng sau đó cắt mạch bằng HIO4 thu
được axit 2,4-đihidroxibenzoic nên Y và Y1 phải có cấu trúc:

Khi thủy phân X thu được 4 mol CH3I và một chất Y có 5 nguyên tử oxy và 2
nguyên tử iot suy ra trong X có 4 nhóm OCH3, một nhóm OH tự do và một nguyên
tử O nối với mạch nhánh có 3 cacbon. Cùng với việc X không phản ứng với
KMnO4 hay iodoform, và không chứa vòng nhỏ hơn 6 cạnh nên X sẽ có cấu trúc
như sau:

866 | Câu hỏi lí thuyết Hóa học hữu cơ OlympiaVN


Bài 39
Dendrolasin (D) có công thức phân tử C15H22O là cấu phần của một loại tinh dầu.
Nó không có tính quang hoạt, không tan trong axit và kiềm, bảo quản lâu không
sử dụng sẽ chuyển thành một chất lỏng nhớt. Khi có mặt axit mạnh D chuyển
thành sản phẩm giống như nhựa. D không phản ứng với phenylhydrazin và không
cho thoát khí khi phản ứng với Na nhưng phản ứng nhanh với brom. D tạo sản
phẩm màu đỏ khi đun nóng với p-N,N-dimetylaminbenzandehit khi có mặt axit.
Có thể hydro hóa D thu được sản phẩm tetrahydro X và octahydro Y, nhưng thực
tế cho rằng khả năng khống chế phản ứng để tạo thành X là không thể. Ozon phân
D thu được một hỗn hợp sản phẩm trong đó có andehit levulinic (4-oxopentanal)
và axeton.
Dẫn xuất octahydro Y được tổng hợp theo sơ đồ dưới đây:

a) Xác định cấu trúc các hợp chất chưa biết


b) Hãy đưa ra cấu trúc hợp lý của dendrolasin. Nếu biết rằng dendrolasin là một
terpenoid và các nối đôi của nó đều có cấu hình trans thì có thể kết luận được
chính xác cấu trúc của hợp chất này hay không.
Hướng dẫn
Dendrolasin không có nhóm OH (do không phản ứng với PhNCO), không có
nhóm carbonyl (do không phản ứng với phenylhydrazin). Dựa vào kết quả phản
ứng ozon phân và các đặc tính theo đề bài có thể thấy rằng trong dendrolasin có
chứa dị vòng furan và nhánh Me2C=CHCH2CH2C(CH3)= . Sơ đồ tổng hợp dẫn
xuất octahydro của dendrolasin như sau:

Dựa vào đó có thể kết luận dendrolasin có thể có bốn cấu trúc thỏa mãn

867 | Câu hỏi lí thuyết Hóa học hữu cơ OlympiaVN


Do D là terpenoid nên sự sắp xếp phân tử phải tuân theo quy tắc isopren. Bên
cạnh đó nối đôi chỉ có cấu hình trans duy nhất nên D1 phải là cấu trúc chính xác
của dendrolasin.

868 | Câu hỏi lí thuyết Hóa học hữu cơ OlympiaVN


Bài 40
Phản ứng giữa một dieste A (C4H6O4) với hợp chất B (C6H10O5) trong sự có mặt
của NaH thu được dị vòng C (C8H8O7). C phản ứng với CH2N2 thu được D. Đun
nóng D với KOH sau đó axit hóa thu được E (C6H8O3). E phản ứng với anhydrit
maleic trong môi trường kiềm rồi sau đó đun nóng trong axit sẽ thu được axit 4,5-
dimetoxyphtalic. Hãy xác định cấu trúc các chất chưa biết.
Hướng dẫn

869 | Câu hỏi lí thuyết Hóa học hữu cơ OlympiaVN


Bài 41
Hai hợp chất hữu cơ A và B đều chỉ chứa 3 nguyên tố. Ở nhiệt độ thường, A là
chất khí có khối lượng phân tử là 31 g/mol, còn B có khối lượng phân tử nhỏ hơn
120 g/mol. Khi đốt cháy B chỉ thu được CO2 và H2O, phần trăm khối lượng oxy
trong B là 27,58%. Chất B không phản ứng vói H2/Ni ở 200oC. Đun nóng B và
axit axetic dư trong môi trường axit thu được hợp chất C có khối lượng phân tử
tăng 42 g/mol so với B. Cho B phản ứng với axetyl clorua dư có mặt trietylamin,
thu được hợp chất D có khối lượng phân tử tăng 84 g/mol so với B. Oxi hóa B
bằng cách đun nóng với dung dịch KMnO4 trong môi trường axit thì tách được
hợp chất E từ hỗn hợp sản phẩm. Đun nóng E với A thu được hợp chất F. Cho F
phản ứng với diazometan thu được hợp chất G (C7H11NO) và đơn chất H.
Lập luận để tìm ra cấu tạo các chất từ A đến H.
Hướng dẫn
A: B1, B2: C1, C2:
MeNH2

D1, D2: E: F: G: H:
N2

870 | Câu hỏi lí thuyết Hóa học hữu cơ OlympiaVN


Bài 42
Hợp chất A (C15H26O) tham gia phản ứng ozon phân – khử hóa cho 4-(3,3-dimetyl
oxiran-2-yl) butan-2-on, 4-oxopentanal và axetandehit. Đun nóng A trong dung
dịch HBr đặc, dư thu được chủ yếu hợp chất B (C15H28Br4). Đun nhẹ A với xúc
tác axit thu được hợp chất C (C15H26O). Ozon phân oxi hóa hoặc ozon phân khử
hóa C đều cho hợp chất D (C15H26O3). Hydrogen hóa C với xúc tác Niken Raney
thì thu được hợp chất E. Cho E phản ứng với tosyl clorua trong dung môi pyridin,
rồi khử hóa sản phẩm thu được bằng LiAlH4 thu được hợp chất F.
Xác định cấu tạo các chất từ E đến F.
Hướng dẫn
A: B: C:

D: E: F:

871 | Câu hỏi lí thuyết Hóa học hữu cơ OlympiaVN


Bài 43
Cantharidin (C10H12O4) là một hợp chất có độc tính cao, có vai trò góp phần tạo
nên lớp vỏ cứng của côn trùng. Các khảo sát cấu trúc cho thấy dung dịch
cantharidin (trong CH2Cl2) có khả năng làm hồng quỳ tím ẩm nhưng không làm
đổi màu quỳ khô. Khi cho Cantharidin tác dụng với lượng dư NaOH rồi axit hóa
thu được B (C10H14O5) cũng làm hồng quỳ ẩm. Cho B phản ứng với Pb(OAc)4
thu được C. Tiến hành ozon phân oxy hóa hay ozon phân khử C đều thu được
một chất D (C8H12O3). Xử lý D với I2/NaOH thu được kết tủa vàng và hợp chất
E (C6H8O5). E tác dụng với HgO/Br2 rồi xử lý tiếp trong HCl đặc cho nhiều sản
phẩm, một trong số đó là HOCCH2CH2CHO. Biết rằng Cantharidin không cho
sản phẩm khi xử lý với Pb(OAc)4. Hãy xác định cấu trúc các chất chưa biết.
Hướng dẫn
Cấu trúc các chất

872 | Câu hỏi lí thuyết Hóa học hữu cơ OlympiaVN


Bài 44
Cà chua, đặc biệt là cà chua xanh, có khả năng sinh ra axit clorogenic giúp tiêu
diệt sâu bệnh. Khi bị sâu tấn công thì axit clorogenic có trong quả cà chua sẽ bị
oxy hóa thành dạng quinon và gây độc với kẻ thù bằng cách ức chế khả năng hoạt
động của protein (khóa nhóm NH-CO trong protein).

Hãy vẽ công thức cấu tạo các hợp chất từ A đến E (không cần xét đến yếu tố lập
thể) và giải thích khả năng diệt sâu bệnh của axit clorogenic bằng phản ứng hóa
học.
Hướng dẫn
Công thức cấu tạo các chất:
A: B: C:

D: E: F:

Cơ chế bảo vệ của cà chua:

873 | Câu hỏi lí thuyết Hóa học hữu cơ OlympiaVN


874 | Câu hỏi lí thuyết Hóa học hữu cơ OlympiaVN
Bài 45
Hoàn thành các sơ đồ chuyển hóa sau:
a)

b)

Chất đầu tiên có thể được tổng hợp bằng cách cho furfural phản ứng với oxy
singlet trong etanol. Hãy đề nghị cơ chế cho phản ứng này.
c)

875 | Câu hỏi lí thuyết Hóa học hữu cơ OlympiaVN


d)

Hướng dẫn
a)

b)

876 | Câu hỏi lí thuyết Hóa học hữu cơ OlympiaVN


c)

d)

877 | Câu hỏi lí thuyết Hóa học hữu cơ OlympiaVN


Bài 46
Năm 2015, các nhà khoa học thuộc Đại học Johns Hopkins (Baltimore, Hoa Kì)
đã tổng hợp được hợp chất X, có đặc trưng cấu trúc rất thú vị. Dưới đây là sơ đồ
tổng hợp hợp chất X:

1. Xác định cấu trúc các hợp chất A-G và X.


2. Trong X có thể có liên kết hydro nội phân tử hay không? Nếu có, hãy biểu diễn
liên kết hydro này.
Hướng dẫn
1) Cấu trúc các chất chưa biết:

2) Trong X có liên kết hydro nội phân tử giữa nhóm OH và liên kết C=C.

878 | Câu hỏi lí thuyết Hóa học hữu cơ OlympiaVN


Bài 47
Tadalafil là hoạt chất để bào chế thuốc điều trị rối loạn cương dương thông qua
tín hiệu thụ thể đặc hiệu trên tế bào biểu mô thành mạch máu. Tadalafil được tổng
hợp theo sơ đồ sau:

Xác định cấu tạo các chất chưa biết.


Hướng dẫn
A: B: C: D:

E: F: G: H:

J: K: L:

879 | Câu hỏi lí thuyết Hóa học hữu cơ OlympiaVN


880 | Câu hỏi lí thuyết Hóa học hữu cơ OlympiaVN
Luyện tập viết cơ chế phản ứng
Bài 1
Đề xuất cơ chế cho phản ứng sau:

Hướng dẫn
Các bước phản ứng:
- A: Cộng hợp hydroxide ion vào nhóm carbonyl tạo thành trung gian tứ diện.
- B: Tách methoxide ion nhờ cặp electron chưa liên kết của oxygen.
- C: Deproton hóa (pKa, AcOH = 4.8, H2O = 15.7).
- D: Proton hóa khi xử lí sản phẩm [work-up].

881 | Câu hỏi lí thuyết Hóa học hữu cơ OlympiaVN


Bài 2
Đề xuất cơ chế cho phản ứng sau:

Hướng dẫn
Các bước phản ứng:
- A: Proton hóa để hoạt hóa nhóm carbonyl.
- B: Cộng hợp EtOH vào nhóm carbonyl đã hoạt hóa.
- C: Deproton hóa oxonium ion.
- D: Proton hóa để chuyển nhóm hydroxyl thành nhóm rời đi tốt.
- E: Tách nước nhờ cặp electron chưa liên kết của oxygen.
- F: Deproton hóa.

882 | Câu hỏi lí thuyết Hóa học hữu cơ OlympiaVN


Bài 3
Đề xuất cơ chế cho phản ứng sau:

Hướng dẫn
Các bước phản ứng:
- A: Carboxylic acid tấn công [tác kích] SOCl2, tạo thành anhydride.
- B: Cộng hợp chloride ion vào nhóm carbonyl tạo thành trung gian tứ diện.
- C: Tạo thành acylium ion.
- D: Cộng hợp chloride vào acylium ion.

Hoặc

883 | Câu hỏi lí thuyết Hóa học hữu cơ OlympiaVN


Bài 4
Đề xuất cơ chế cho phản ứng sau:

Hướng dẫn
Các bước phản ứng:
- A: Cộng hợp PhMgBr vào nhóm carbonyl của ester, tạo thành trung gian tứ
diện.
- B: Tách ethoxide ion tạo thành ketone.
- C: Cộng hợp PhMgBr vào ketone hoạt động hơn, tạo thành alkoxide bậc ba.

884 | Câu hỏi lí thuyết Hóa học hữu cơ OlympiaVN


Bài 5
Đề xuất cơ chế cho phản ứng sau:

Hướng dẫn
Các bước phản ứng:
- A: Cộng hợp PhMgBr vào nitrile tạo thành imine anion.
- B: Cộng hợp nước vào iminium tạo thành hemiaminal.
- C: Sự proton hóa diễn ra trên nhóm amino có tính base cao hơn (pKa, H3O+
= -1.7, EtNH3+ = 10.6).
- D: Tách ammonia nhờ cặp electron chưa liên kết của oxygen.
- E: Deproton hóa.

885 | Câu hỏi lí thuyết Hóa học hữu cơ OlympiaVN


Bài 6
Đề xuất cơ chế cho phản ứng sau:

Hướng dẫn
Các bước phản ứng:
- A: Cộng hợp PhMgBr vào nhóm carbonyl. Trung gian tứ diện tạo thành khá
bền bởi alkoxide anion không thể tạo thành amine anion (pKa,i-PrOH = 17,
Et2NH = 36).
- B: Sự proton hóa diễn ra khi xử lí [work-up].
- C: Sự proton hóa diễn ra trên nhóm amino có tính base mạnh hơn (pKa, H3O+
= 1.7, EtNH3+ = 10.6.
- D: Tách loại amine nhờ cặp electron chưa liên kết của oxygen.
- E: Deproton hóa.

886 | Câu hỏi lí thuyết Hóa học hữu cơ OlympiaVN


Bài 7
Đề xuất cơ chế cho phản ứng sau:

Hướng dẫn
Các bước phản ứng:
- A: Proton hóa để hoạt hóa DCC.
- B: Cộng hợp carboxylate vào DCC được proton hóa.
- C: Cộng hợp DMAP vào nhóm carbonyl.
- D: Tách urea anion (sẽ lấy proton từ alcohol).
- E: Cộng hợp alkoxide anion vào nhóm carbonyl tạo thành trung gian tứ diện.
- F: Tách DMAP tạo thành sản phẩm.

887 | Câu hỏi lí thuyết Hóa học hữu cơ OlympiaVN


888 | Câu hỏi lí thuyết Hóa học hữu cơ OlympiaVN
Bài 8
Đề xuất cơ chế cho phản ứng sau:

Hướng dẫn
Các bước phản ứng:
- A: Proton hóa để hoạt hóa nhóm carbonyl.
- B: Cộng hợp ethylene glycol vào nhóm carbonyl được hoạt hóa.
- C: Chuyển proton.
- D: Tách nước nhờ cặp electron chưa liên két của oxygen.
- E: Cộng hợp nội phân tử nhóm hydroxy thứ hai.

889 | Câu hỏi lí thuyết Hóa học hữu cơ OlympiaVN


Bài 9
Đề xuất cơ chế cho phản ứng sau:

Hướng dẫn
Các bước phản ứng:
- A: Proton hóa để hoạt hóa nhóm carbonyl.
- B: Cộng hợp MeOH vào nhóm carbonyl được hoạt hóa.
- C: Chuyển proton.
- D: Tách nước nhờ cặp electron chưa liên két của oxygen.
- E: Cộng hợp MeOH và proton hóa để tạo thành dimethyl acetal.
- F: Trimethyl orthoformate đóng vai trò như chất thu nước để cân bằng chuyển
dịch theo chiều tạo thành sản phẩm. Proton hóa, sau đó tách loại MeOH.
- G: Cộng hợp nước.
- H: Chuyển proton.
- I: Tách loại MeOH, sau đó deproton hóa tạo thành HCO2Me.

890 | Câu hỏi lí thuyết Hóa học hữu cơ OlympiaVN


891 | Câu hỏi lí thuyết Hóa học hữu cơ OlympiaVN
Bài 10
Đề xuất cơ chế cho phản ứng sau:

Hướng dẫn
Các bước phản ứng:
- A: Proton hóa.
- B: Phân cắt vòng dioxolane nhờ cặp electron chưa liên kết của oxygen.
- C: Deproton hóa tạo thành enol ether.
- D: Bromine hóa enol ether giàu electron.
- E: Cộng hợp nội phân tử nhóm hydroxy. Sự mở vòng dioxolane của sản phẩm
khó hơn bởi có nguyên tử bromine hút electron.

892 | Câu hỏi lí thuyết Hóa học hữu cơ OlympiaVN


Bài 11
Đề xuất cơ chế cho phản ứng sau:

Hướng dẫn
Các bước phản ứng (đây là tổng hợp amino acid theo phương pháp Strecker):
- A: Proton hóa nhóm carbonyl.
- B: Cộng hợp NH3 vào nhóm carbenyl, sau đó deproton hóa tạo thành
hemiaminal.
- C: Proton hóa, sau đó tách nước nhờ cặp electron chưa liên kết của nitrogen,
tạo thành iminium ion.
- D: Cộng hợp cyanide ion tạo thành aminonitrile.
- E: Thủy phân nitrile bởi acid. Nhóm amino bị proton hóa sau phản ứng này.
- F: Proton hóa nitrile tạo thành nitrilium ion hoạt động.
- G: Cộng hợp nước vào nitrilium ion.
- H: Deproton hóa và tautomer hóa [hỗ biến].
- I: Proton hóa amide tạo thành, sau đó cộng hợp nước.
- J: Chuyển hóa proton.
- K: Tách NH3, sau đó deproton hóa tạo thành sản phẩm.

893 | Câu hỏi lí thuyết Hóa học hữu cơ OlympiaVN


894 | Câu hỏi lí thuyết Hóa học hữu cơ OlympiaVN
Bài 12
Đề xuất cơ chế cho phản ứng sau:

Hướng dẫn
Các bước phản ứng (phản ứng Vilsmeier):
- A: Oxygen giàu eletron của DMF tấn công vào POCl3 (oxygen của các amide
thường dễ phản ứng với các electrophile hơn trong các điều kiện trung tính).
- B: Cộng hợp chloride ion, sau đó tách dichlorophosphate ion tạo thành tác
nhân Vilsmeier.
- C: Cộng hợp vòng thơm giàu electron vào tác nhân Vilsmeier, sau đó thơm
hóa lại.
- D: Tách loại chloride ion nhờ cặp electron chưa liên kết của nitrogen, tạo
thành iminium ion.
- E: Cộng hợp nước vào iminium ion.
- F: Chuyển proton hóa, sau đó tách loại Me2NH.

895 | Câu hỏi lí thuyết Hóa học hữu cơ OlympiaVN


896 | Câu hỏi lí thuyết Hóa học hữu cơ OlympiaVN
Bài 13
Đề xuất cơ chế cho phản ứng sau:

Hướng dẫn
Các bước phản ứng (phản ứng Mannich):
- A: Proton hóa formaldehyde, sau đó cộng hợp Me2NH vào nhóm carbonyl.
- B: Chuyển proton, sau đó tách nước tạo thành iminium ion.
- C: Sự tautomer hóa [hỗ biến] của nhóm carbonyl tạo thành enol.
- D: Enol giàu electron tấn công vào iminium ion.

897 | Câu hỏi lí thuyết Hóa học hữu cơ OlympiaVN


Bài 14
Đề xuất cơ chế cho chuyển hóa sau:

Hướng dẫn

898 | Câu hỏi lí thuyết Hóa học hữu cơ OlympiaVN


Bài 15
Sự khử ketone cấu hình R dưới đây tạo thành hai alcohol là đồng phân dia (xuyên
lập thể phân) với tỉ lệ 86:14. Dự đoán đồng phân dia nào là sản phẩm chính, và
giải thích dự đoán của bạn.

Hướng dẫn

Giải thích: Sự tấn công nucleophile nhóm


carbonyl diễn ra theo quỹ đạo Bürgi–Dunitz.
Trong cấu dạng ghế năng lượng cực tiểu sự
cộng hợp hydride tạo thành sản phẩm phụ bị
cản trở không gian bởi một nhóm methyl
trục (axial). Sự tấn công vào mặt còn lại ít
cản trở không gian hơn, tạo thành sản phẩm
chính như trên.

899 | Câu hỏi lí thuyết Hóa học hữu cơ OlympiaVN


Bài 16
Dự đoán sản phẩm và trình bày cơ chế hoàn chỉnh của các chuyển hóa dưới đây:

Hướng dẫn
a) Giải thích sự chọn lọc hóa lập thể dia: Trong cấu dạng năng lượng cực tiểu, sự
tấn công diễn ra ưu tiên trên phía ngược với nhóm ethyl của liên kết π.

900 | Câu hỏi lí thuyết Hóa học hữu cơ OlympiaVN


b)

901 | Câu hỏi lí thuyết Hóa học hữu cơ OlympiaVN


Bài 17
Đề xuất cơ chế cho các chuyển hóa sau:

Hướng dẫn
a)

b)

902 | Câu hỏi lí thuyết Hóa học hữu cơ OlympiaVN


903 | Câu hỏi lí thuyết Hóa học hữu cơ OlympiaVN
Bài 18
a) Alizarine Yellow R là một thuốc nhuộm màu vàng, có thể được dùng làm chỉ
thị pH: nó có màu vàng ở pH dưới 10 và màu đỏ ở pH trên 12. Hãy đề xuất
cơ chế hoàn chỉnh cho phản ứng ghép cặp dưới đây, dùng để tổng hợp thuốc
nhuộm này.

b) Trong một chuyển hóa có tính hữu dụng cao, phản ứng của amino acid L-
alanine với sodium nitrite, hydrobromic acid, và potassium bromide, được
mô tả dưới đây, dẫn đến sự thay thế nhóm α-amino bởi α-bromo với sự bảo
toàn cấu hình hoàn toàn. Hãy đề xuất cơ chế phản ứng để giải thích quan sát
này.

c) Khi các tế bào tiếp xúc với nitrous acid (HNO2) thì cytosine – một nucleobase
thành phần của DNA – có thể bị chuyển hóa thành uracil, chất nay có thể dẫn
đến đột biến mã hóa liên quan đến phát sinh ung thư. Hãy đề xuất cơ chế cho
chuyển hóa đột biến này.

904 | Câu hỏi lí thuyết Hóa học hữu cơ OlympiaVN


Hướng dẫn
a)

b)

c)

Xe m ý a

905 | Câu hỏi lí thuyết Hóa học hữu cơ OlympiaVN


Bài 19
Trình bày cơ chế hoàn chỉnh cũng như cấu trúc của sản phẩm A trong chuyển hóa
sau. Phổ cộng hưởng từ proton NMR của A được cho bên dưới, hãy sử dụng nó
để suy luận và xác thực cấu trúc của A.

Hướng dẫn
Các suy luận từ phổ proton NMR của sản phẩm A:
- A có hợp phần alkene ba nhóm thế với hai nhóm methyl và một nhóm alcohol
gắn vào nhóm methylene.

906 | Câu hỏi lí thuyết Hóa học hữu cơ OlympiaVN


Cơ chế phản ứng:

907 | Câu hỏi lí thuyết Hóa học hữu cơ OlympiaVN


Bài 20
Phản ứng dưới đây giữa benzonitrile A và tert-butyl acetate B trong dung dịch
sulfuric acid tạo thành hai sản phẩm C và D. Hãy đề xuất cơ chế phản ứng cho sự
tạo thành C và D. Sử dụng phổ NMR cho dưới đây để suy luận và xác thực cấu
trúc các sản phẩm.

Phổ 1H và 13C NMR của hợp chất C:

908 | Câu hỏi lí thuyết Hóa học hữu cơ OlympiaVN


Phổ 1H (dưới) và 13C (trên) của hợp chất D:

Hướng dẫn
Phân tích các phổ đồ:

909 | Câu hỏi lí thuyết Hóa học hữu cơ OlympiaVN


910 | Câu hỏi lí thuyết Hóa học hữu cơ OlympiaVN
911 | Câu hỏi lí thuyết Hóa học hữu cơ OlympiaVN
Bài 21
Trình bày cơ chế phản ứng hoàn chỉnh cũng như xác định cấu trúc sản phẩm B
của chuyển hóa sau. Từ phổ khối lượng phân giải cao, xác định được khối lượng
phân tử chính xác của sản phẩm B tương ứng với công thức C6H10S.

Hướng dẫn

912 | Câu hỏi lí thuyết Hóa học hữu cơ OlympiaVN


Bài 22
a) Các alkaloid aspidosperma là nhóm hợp chất thiên nhiên được cô lập từ các
loại cây Aspidosperma và thể hiện hoạt tính sinh học đa dạng với cơ thể
người. Ketone đa vòng A đã bị khử theo phản ứng Wolff-Kishner để tạo thành
một tiền chất quan trọng của hợp chất thiên nhiên aspidospermidine. Trình
bày cơ chế hoàn chỉnh của phản ứng khử Wolff-Kishner A và vẽ cấu trúc sản
phẩm B. Bạn có thể sử dụng một cấu trúc được viết tắt cho chất đầu trong cơ
chế của mình.

b) Thực hiện một phản ứng tương tự trong dung môi triethylamine khi có mặt
iodine, từ hydrazone A sẽ tạo thành inyliodide C, qua trung gian diazo B.
Nhóm chức diazo được tạo thành bởi phản ứng oxid hóa trợ lực bởi iodine
của hydrazone. Bạn không cần phải đưa ra cơ chế phản ứng, dù rằng bạn có
thể dự đoán nó. Thay vào đó, hãy đề xuất ít nhất một cơ chế khả thi cho
chuyển hóa tiếp đó của B thành C. Bạn có thể sử dụng một cấu trúc được viết
tắt của B cho cơ chế của mình.

913 | Câu hỏi lí thuyết Hóa học hữu cơ OlympiaVN


Hướng dẫn
a)

b)

914 | Câu hỏi lí thuyết Hóa học hữu cơ OlympiaVN


Bài 23
Các muối 2-halopyridinium muối A có thể được sử dụng để hoạt hóa các
carboxylic acid để tạo thành ester trong các điều kiện êm dịu. Sử dụng của A để
tạo thành lactone 12 vòng B như dưới đây. Đề xuất cơ chế tạo thành lactone B
cũng như sản phẩm phụ C. Sử dụng dữ kiện phổ NMR và IR được cho ở dưới để
dự đoán và xác thực cấu trúc của sản phẩm phụ C.

Phổ 1H (dưới) và 13C (trên) của hợp chất C:

Phổ hồng ngoại của C:

915 | Câu hỏi lí thuyết Hóa học hữu cơ OlympiaVN


Hướng dẫn
Phân tích các phổ đồ:

916 | Câu hỏi lí thuyết Hóa học hữu cơ OlympiaVN


917 | Câu hỏi lí thuyết Hóa học hữu cơ OlympiaVN
Bài 24
Trình bày cơ chế cho phản ứng sau:

Hướng dẫn

918 | Câu hỏi lí thuyết Hóa học hữu cơ OlympiaVN


Bài 25
Trình bày cơ chế cho các phản ứng sau:

Hướng dẫn
a)

b)

919 | Câu hỏi lí thuyết Hóa học hữu cơ OlympiaVN


Bài 26
Phản ứng khử Birch là phương pháp truyền thống được sử dụng để khử các vòng
thơm. Hãy đề xuất cơ chế của chuyển hóa này. Chú ý: Trong câu trả lời của bạn,
chỉ vẽ các tiểu phân trung gian chứ không vẽ các trạng thái chuyển tiếp.

Điều kiện phản ứng: Kim loại Li, Na hoặc K được hòa tan trong NH3 lỏng, rồi
thêm ROH vào.
Hướng dẫn
Giai đoạn tốc định là sự proton hóa anion gốc.

Orbital phân tử của anion gốc:

920 | Câu hỏi lí thuyết Hóa học hữu cơ OlympiaVN


Bài 27
Đề xuất cơ chế cho các chuyển vị sau:

Hướng dẫn
a)

921 | Câu hỏi lí thuyết Hóa học hữu cơ OlympiaVN


b)

c)

922 | Câu hỏi lí thuyết Hóa học hữu cơ OlympiaVN


Bài 28
Dưới đây là minh họa về phản ứng epoxide hóa olefin với sự chọn lọc đáng ngạc
nhiên được báo cáo trên J. Org. Chem. 1991, 56, 5553. Trong phản ứng này, olefin
B trong hợp chất 1 thể hiện hoạt tính kém hơn nhiều so với olefin A.

Dựa vào hiểu biết về các hiệu ứng chọn lọc lập thể, hãy đưa ra lời giải thích cho
việc suy giảm hoạt tính của olefin B trong diene 1.
Hướng dẫn
Orbital phản liên kết C-Cl có sự định hướng tốt trong
tương tác với orbital π C=C. Tương tác làm bền hóa này
sẽ làm giảm tính nucleophile của liên kết, dẫn đến hoạt
tính trong phản ứng epoxide hóa suy giảm.

923 | Câu hỏi lí thuyết Hóa học hữu cơ OlympiaVN


Bài 29
Xử lí 5-hydroxypentanal với xúc tác H2SO4 trong methanol tạo thành các acetal
vòng A và B.

a) Tỉ lệ sản phẩm A:B chịu sự khống chế động học hay nhiệt động học? Giải
thích.
b) Hiệu ứng lập thể điện tử nào chịu trách nhiệm cho sự ưu đãi cấu trạng của
đồng phân trục A?
Hướng dẫn
a) Phản ứng tạo thành acetal là quá trình thuận nghịch, do đó nó chịu sự khống
chế nhiệt động học.
b) Hiệu ứng anomer. Các electron trong orbital không liên kết trên oxygen
(trong vòng) (HOMO) được nhường vào orbital σ* của liên kết C-Ome
(LUMO)

924 | Câu hỏi lí thuyết Hóa học hữu cơ OlympiaVN


Bài 30
Tốc độ thủy phân các acetal A và B (hình dưới) không phụ thuộc vào nòng độ
acid.

a) Đề xuất cơ chế cho chuyển hóa này


b) Giải thích sự khác nhau về tốc độ thủy phân của hai acetal này.
Hướng dẫn
a)

b) Trong đồng phân A, nhóm rời đi (acetate) ở vị trí trục. Vị trí này cho phép
tương tác nhường electron trên orbital chưa liên kết oxygen (trong vòng) đến
σ*C-O của acetate đạt hiệu quả cao nhất. Kết quả là, tốc độ phân cắt liên kết
tăng lên. Đây là một ví dụ về hiệu ứng anomer. Trong đồng phân B, tương
tác điện tử thứ cấp không xảy ra do sự cản trở về mặt hình học.

925 | Câu hỏi lí thuyết Hóa học hữu cơ OlympiaVN


Bài 31
Đề xuất cơ chế cho chuyển hóa sau:

Hướng dẫn

926 | Câu hỏi lí thuyết Hóa học hữu cơ OlympiaVN


Bài 32
Trình bày cơ chế hoàn chỉnh của chuyển hóa sau. Hãy chú ý vẽ các cấu dạng năng
lượng cực tiểu của trung gian để giải thích hợp lí sản phẩm tạo thành của phản
ứng.

Hướng dẫn

927 | Câu hỏi lí thuyết Hóa học hữu cơ OlympiaVN


Bài 33
Trình bày cơ chế của chuyển hóa sau:

Hướng dẫn

928 | Câu hỏi lí thuyết Hóa học hữu cơ OlympiaVN


Bài 34
Đề xuất cơ chế cho chuyển hóa sau:

Hướng dẫn

929 | Câu hỏi lí thuyết Hóa học hữu cơ OlympiaVN


Bài 35
Đề xuất cơ chế cho chuyển hóa sau:

Hướng dẫn

930 | Câu hỏi lí thuyết Hóa học hữu cơ OlympiaVN


Bài 36
Đề xuất cơ chế cho chuyển hóa sau:

Hướng dẫn

931 | Câu hỏi lí thuyết Hóa học hữu cơ OlympiaVN


Bài 37
Sinh tổng hợp endiandric acid (B) được đề xuất là diễn ra qua một loạt các phản
ứng peri hóa. Điều thú vị, và bất thường, là endiandric acid tự nhiên là racemic,
củng cố cho đề xuất rằng nó có thể được tạo ra bởi phản ứng vòng hóa không-
enzyme, tự diễn biến, theo hướng mà bạn sẽ phải xác định. Đề xuất này đã được
thực hiện trong phòng thí nghiệm, và dưới đây là giai đoạn thứ hai của chuỗi tổng
hợp hai giai đoạn. Hãy đề xuất cơ chế phản ứng cho chuyển hóa và phân loại tất
cả các phản ứng peri hóa liên quan. Cuối cùng, hãy dự đoán hóa lập thể của nhóm
phenyl trong sản phẩm cuối B.

932 | Câu hỏi lí thuyết Hóa học hữu cơ OlympiaVN


Hướng dẫn

933 | Câu hỏi lí thuyết Hóa học hữu cơ OlympiaVN


Bài 38
Trình bày cơ chế cho chuyển hóa của isopulegone A thành sản phẩm hai vong B,
là kết quả của một loạt chuyển vị sigmatropic qua 4 giai đoạn. Loạt phản ứng
được kết thúc bởi phản ứng carbonyl-ene. Với những thông tin dưới đây, bạn có
thể suy luận và biểu diễn cơ chế phản ứng. Sẽ cần phân tích các cấu dạng năng
lượng cực tiểu của tất cả các trung gian để giải thích cho hóa lập thể quan sát
được của sản phẩm. Hãy phân loại các phản ứng peri hóa liên quan.

934 | Câu hỏi lí thuyết Hóa học hữu cơ OlympiaVN


Hướng dẫn

935 | Câu hỏi lí thuyết Hóa học hữu cơ OlympiaVN


Bài 39
Đề xuất cơ chế phản ứng cho chuyển hóa dưới đây và dự đoán hóa lập thể của
sản phẩm. Vẽ cấu dạng năng lượng cực tiểu của tất cả các trung gian cần thiết cho
giải thích của bạn. Hãy phân loại các phản ứng peri hóa liên quan. Gợi ý: Sự tạo
thành ester enolate dưới các điều kiện đã cho sẽ tạo thành enolate với nhóm arene
ở vị trí cis so với enoxy anion.

Hướng dẫn

936 | Câu hỏi lí thuyết Hóa học hữu cơ OlympiaVN


937 | Câu hỏi lí thuyết Hóa học hữu cơ OlympiaVN
Bài 40
Trình bày cơ chế cho các chuyển hóa sau (phần hướng dẫn được đóng khung phía
dưới):

938 | Câu hỏi lí thuyết Hóa học hữu cơ OlympiaVN


939 | Câu hỏi lí thuyết Hóa học hữu cơ OlympiaVN
940 | Câu hỏi lí thuyết Hóa học hữu cơ OlympiaVN
941 | Câu hỏi lí thuyết Hóa học hữu cơ OlympiaVN
942 | Câu hỏi lí thuyết Hóa học hữu cơ OlympiaVN
943 | Câu hỏi lí thuyết Hóa học hữu cơ OlympiaVN
944 | Câu hỏi lí thuyết Hóa học hữu cơ OlympiaVN
945 | Câu hỏi lí thuyết Hóa học hữu cơ OlympiaVN
946 | Câu hỏi lí thuyết Hóa học hữu cơ OlympiaVN
947 | Câu hỏi lí thuyết Hóa học hữu cơ OlympiaVN
948 | Câu hỏi lí thuyết Hóa học hữu cơ OlympiaVN
949 | Câu hỏi lí thuyết Hóa học hữu cơ OlympiaVN
950 | Câu hỏi lí thuyết Hóa học hữu cơ OlympiaVN
951 | Câu hỏi lí thuyết Hóa học hữu cơ OlympiaVN
952 | Câu hỏi lí thuyết Hóa học hữu cơ OlympiaVN
953 | Câu hỏi lí thuyết Hóa học hữu cơ OlympiaVN
954 | Câu hỏi lí thuyết Hóa học hữu cơ OlympiaVN
955 | Câu hỏi lí thuyết Hóa học hữu cơ OlympiaVN
Bài 41
Đề xuất cơ chế cho chuyển hóa sau của enol A thành enone B, là một phần trong
tổng hợp steroid hormone Cortisone do R. B. Woodward công bố lần đầu tiên.

Hướng dẫn

956 | Câu hỏi lí thuyết Hóa học hữu cơ OlympiaVN


Bài 42
Đề xuất cơ chế cho phản ứng của aldehyde A với aminoester B khi có mặt tác
nhân khử trung bình sodium triacetoxyborohydride trong acetic acid, tạo ra
ketopiperazine C.

Hướng dẫn

957 | Câu hỏi lí thuyết Hóa học hữu cơ OlympiaVN


Bài 43
Trong nỗ lực khám phá ra một chất ức chế HIV protease mới, Ghosh và đồng
nghiệp đã quan sát thấy rằng khi xử lí (Z)-C với sulfuric acid trong methanol khan
thì tạo thành hỗn hợp đẳng mol của hai sản phẩm 5-carbon là D và E. Đề xuất cấu
trúc D và E, cũng như cơ chế phản ứng dẫn đến sự tạo thành chúng.

958 | Câu hỏi lí thuyết Hóa học hữu cơ OlympiaVN


Hướng dẫn

959 | Câu hỏi lí thuyết Hóa học hữu cơ OlympiaVN


Bài 44
Diphenylphosphoryl azide (DPPA) là tác nhân hữu ích với rất nhiều ứng dụng
tổng hợp. Khi xử lí một carboxylic acid với tert-butanol và DPPA thì tạo thành
amine được bảo vệ bởi Boc. Hãy trình bày cơ chế cho chuyển hóa này.

Hướng dẫn

960 | Câu hỏi lí thuyết Hóa học hữu cơ OlympiaVN


Bài 45
Đề xuất cơ chế cho chuyển hóa sau. Chỉ rõ hóa lập thể tương đối của tất cả các
trung gian.

Hướng dẫn

961 | Câu hỏi lí thuyết Hóa học hữu cơ OlympiaVN


Bài 46
Đề xuất cơ chế cho chuyển hóa hai-giai đoạn của aminoester A thành racemic
amino acid B. Vẽ các cấu dạng năng lượng cực tiểu của các trung gian hoạt động
để giải thích hóa lập thể của sản phẩm. Phân loại tất cả các phản ứng peri hóa đã
diễn ra.

Hướng dẫn

962 | Câu hỏi lí thuyết Hóa học hữu cơ OlympiaVN


Bài 47
Đề xuất cơ chế cho chuyển hóa xúc tác thiamine sau. Bạn có thể sử dụng kí hiệu
viết tắt [abbreviation] cho xúc tác.

Hướng dẫn

963 | Câu hỏi lí thuyết Hóa học hữu cơ OlympiaVN


Bài 48
Đề xuất cơ chế phản ứng cho các chuyển hóa sau. Chỉ rõ mỗi phản ứng liên quan
đến sự cộng hoặc phân mảnh các nhóm carbonyl (ví dụ phản ứng aldol, retro-
aldol, …)

Hướng dẫn

964 | Câu hỏi lí thuyết Hóa học hữu cơ OlympiaVN


Bài 49
Nghiên cứu phản ứng dưới đây ở hai nhiệt độ khác nhau, kết quả được ghi nhận
lại như sau.

-78 oC: 0% 88%


Nhiệt độ phòng: 48% 46%
Đề nghị cơ chế cho phản ứng này và giải thích kết quả thu được.
Hướng dẫn
Cơ chế tạo thành hai sản phẩm:

965 | Câu hỏi lí thuyết Hóa học hữu cơ OlympiaVN


Cacbocation trung gian tạo thành B được bền hóa bởi nhóm OMe nên được ưu
tiên tạo thành. Ở nhiệt độ thấp (-78 oC), phản ứng tạo thành B chiếm ưu thế hoàn
toàn. Ở nhiệt độ cao hơn (nhiệt độ phòng) phản ứng trở thành thuận nghịch, cân
bằng sẽ dịch chuyển về phía sản phẩm bền hơn là hợp chất thơm A.

966 | Câu hỏi lí thuyết Hóa học hữu cơ OlympiaVN


Bài 50
1) Đề nghị cơ chế cho một số chuyển hóa sau đây. Riêng câu 1b chỉ vẽ cơ chế sự
tạo thành X.

2) Tùy dung môi sử dụng mà chất đầu có thể chuyển hóa theo nhiều hướng khác
nhau như sau. Hãy giải thích sự tạo thành các sản phẩm tương ứng bằng cơ chế
phản ứng.

Hướng dẫn
1)

967 | Câu hỏi lí thuyết Hóa học hữu cơ OlympiaVN


2)

968 | Câu hỏi lí thuyết Hóa học hữu cơ OlympiaVN


Bài 51
Đề nghị cơ chế cho một số phản ứng sau:
a)

b)

c)

d)

Hướng dẫn
a)

b)

969 | Câu hỏi lí thuyết Hóa học hữu cơ OlympiaVN


c)

d)

970 | Câu hỏi lí thuyết Hóa học hữu cơ OlympiaVN


Monosaccharide
Bài 1
1) Xác định cấu hình R/S của các tâm thủ tính (chiral) sau:

2) Vẽ lại cấu trúc các đường sau ở dạng công thức chiếu Newman và cho biết
tên thông thường của chúng.

Hướng dẫn
1)

2)

971 | Câu hỏi lí thuyết Hóa học hữu cơ OlympiaVN


972 | Câu hỏi lí thuyết Hóa học hữu cơ OlympiaVN
Bài 2
Do tất cả các carbohydrate đều có các nguyên tử carbon
quang hoạt nên cần có phương pháp tiêu chuẩn để biểu
diễn chúng. Một trong những phương pháp thường được
dùng là công thức chiếu Fischer, lấy glyceraldehyde tồn
tại trong tự nhiên làm tiêu chuẩn để kí hiệu tâm lập thể.
Các đường ngang biểu diễn các liên kết hướng ra ngoài
mặt phẳng giấy, các đường dọc biểu diễn các liên kết
hướng vào trong mặt phẳng giấy. Các đường có cùng cấu
hình như D-glyceraldehyde ở nguyên tử carbon ở xa nhóm carbonyl nhất (nhóm
hydroxyl ở nguyên tử quang hoạt thấp nhất hướng sang phải) thì được gọi là
đường D. Ngược lại với đường D là các đường L có nhóm hydroxyl ở nguyên tử
carbon quang hoạt thấp nhất hướng sang trái trong hình chiếu Fischer.
1) Sử dụng quy tắc Cahn, Ingold, Prelog để xác định liệu D-glyceraldehyde có
cấu hình R hay S?
2) Vẽ công thức chiếu Fischer của các đồng phân lập thể của 2,3,4-
trihydroxypentane. Những chất nào là đối quang? Những chất nào là đồng
phân dia? Những chất nào quang hoạt?
Hình dưới biểu diễn hình chiếu Fischer của D-glucose.

3) Gọi tên hệ thống của D-glucose, sử dụng quy tắc Cahn, Ingold, Prelog.

973 | Câu hỏi lí thuyết Hóa học hữu cơ OlympiaVN


Hướng dẫn
1) (2R)-2,3-dihydroxypropanal.
2) Các cấu trúc giống nhau: a = e, b = f, g = h, c = d

Các đối quang:

Quang hoạt Quang hoạt

974 | Câu hỏi lí thuyết Hóa học hữu cơ OlympiaVN


Đồng phân dia:

Dạng meso Dạng meso


3) (2R, 3S, 4R, 5R)-2,3,4,5,6-Pentahydroxyhexanal.

975 | Câu hỏi lí thuyết Hóa học hữu cơ OlympiaVN


Bài 3
1) Phân loại mỗi carbohydrate sau là aldose hay ketose, và chèn vào thuật ngữ
phù hợp để chỉ số nguyên tử carbon của chúng, ví dụ aldopentose.

2) Chỉ rõ mỗi carbohydrate sau là đường D hay đường L và gắn cấu hình R/S
cho mỗi tâm thủ tính. Bạn có nhận ra bất kì xu hướng nào để gán cấu hình
nhanh hơn cho mỗi tâm thủ tính của carbohydrate không?

3) Xác định cấu hình mỗi tâm thủ tính trong các hợp chất sau:

4) Phân loại các monosaccharide sau thuộc loại D hay L, là đường aldo hay keto,
và là tetrose, pentose hay hexose?

5) Xác định tên gọi các aldohexose sau:

976 | Câu hỏi lí thuyết Hóa học hữu cơ OlympiaVN


6) Với mỗi hợp chất sau, hãy xác định liệu chúng là đối quang, epimer, đồng
phân dia mà không phải epimer, hay là các hợp chất giống nhau:

Hướng dẫn
1) (a) aldohexose; (b) aldopentose; (c) ketopentose; (d) aldotetrose; (e)
ketohexose.
2) Tất cả đều là đường D, ngoại trừ (b) là đường L. Cấu hình của mỗi tâm thủ
tính là:
(a) 2S, 3S, 4R, 5R
(b) 2R, 3S, 4S
(c) 3R, 4R
(d) 2S, 3R
(e) 3S, 4S, 5R
Bạn có thể nhận thấy cấu hình của mỗi tâm thủ tính là R khi nhóm OH ở bên phải
của công thức chiếu Fischer, và cấu hình là S khi nhóm OH ở bên trái.
3)

977 | Câu hỏi lí thuyết Hóa học hữu cơ OlympiaVN


4) Phân loại:
(a) D-aldotetrose
(b) L-aldopentose
(c) D-aldopentose
(d) D-aldohexose
(e) D-ketopentose
5) Tên gọi:
(a) D-glucose
(b) D-mannose
(c) D-galactose
(d) L-glucose
6)
(a) Epimer
(b) Đồng phân dia
(c) Đối quang
(d) Cùng một chất (-D-glucopyranose).

978 | Câu hỏi lí thuyết Hóa học hữu cơ OlympiaVN


Bài 4
1) Vẽ công thức chiếu Haworth cho các hợp chất sau:
(a) β-D-Galactopyranose
(b) α-D-Mannopyranose
(c) α-D-Allopyranose
(d) β-D-Mannopyranose
(e) β-D-Glucopyranose
(f) α-D-Glucopyranose
2) Xác định tên gọi hoàn chỉnh cho hợp chất sau:

3) Sự nghịch đảo gây ra sự chuyển hóa β-D-mannopyranose thành α-D-


mannopyranose. Sử dụng công thức Haworth, vẽ cân bằng giữa hai dạng
pyranose và dạng mạch hở của D-mannose.
4) Khi hoa tan D-talose trong nước, một cân bằng sẽ được thiết lập, trong đó có
sự hiện diện của hai dạng pyranose. Vẽ các dạng pyranose và gọi tên chúng.
Hướng dẫn
1)
a) b) c)

d) e) f)

2) Cấu trúc này biểu diễn đồng phân β anomer của dạng vòng của D-galactose và
do đó có tên hoàn chỉnh là β-D-galactopyranose.

979 | Câu hỏi lí thuyết Hóa học hữu cơ OlympiaVN


3)

4)

980 | Câu hỏi lí thuyết Hóa học hữu cơ OlympiaVN


Bài 4A
Đưa ra các cấu trúc phù hợp để làm ví dụ minh họa cho mỗi trường hợp sau:
(a) Một aldopentose
(b) Một ketohexose
(c) Một L-monosaccharide
(d) Một glycoside
(e) Một aldonic acid
(f) Một aldaric acid
(g) Một aldonolactone
(h) Một pyranose
(i) Một furanose
(j) Một đường khử
(k) Một pyranoside
(l) Một furanoside
(m) Các epimer
(n) Các anomer
(o) Một phenylosazone
(p) Một disaccharide
(q) Một polysaccharide

981 | Câu hỏi lí thuyết Hóa học hữu cơ OlympiaVN


Hướng dẫn

982 | Câu hỏi lí thuyết Hóa học hữu cơ OlympiaVN


983 | Câu hỏi lí thuyết Hóa học hữu cơ OlympiaVN
984 | Câu hỏi lí thuyết Hóa học hữu cơ OlympiaVN
Bài 5
1) Cấu dạng bền nhất của đa số aldopyranose là cấu dạng mà trong đó nhóm lớn
nhất, -CH2OH, ở vị trí biên (equatorial). Tuy nhiên, D-idopyranose chủ yếu
tồn tại ở cấu dạng với nhóm -CH2OH trục (axial). Biểu diễn hai cấu dạng ghế
của α-D-idopyranose (một với nhóm -CH2OH ở vị trí trục, một với nhóm -
CH2OH ở vị trí biên) và giải thích cho sự chiếm ưu thế của cấu dạng trục.
2) Một trong hai anomer của methyl 2,3-anhydro-D-ribofuranoside, I, là dạng β
có nhiệt độ nóng chảy thấp hơn đến mức đáng kinh ngạc (so với dạng còn
lại). Giải thích dựa vào cấu trúc của chúng.

Hướng dẫn
1) Cấu dạng của D-idopyranose với bốn nhóm OH biên và một nhóm CH2OH
trục bền hơn cấu dạng với bốn nhóm OH trục và một nhóm CH2OH biên.

985 | Câu hỏi lí thuyết Hóa học hữu cơ OlympiaVN


2) Dạng β-anomer có thể tạo thành liên kết hydrogen nội phân tử:

Ngược lại, dạng α-anomer chỉ tạo thành liên kết hydrogen liên phân tử, dẫn đến
nhiệt độ sôi cao hơn nhiều.

986 | Câu hỏi lí thuyết Hóa học hữu cơ OlympiaVN


Bài 6
2) Vẽ cấu trúc hemiacetal vòng được tạo thành khi xử lí mỗi hợp chất lưỡng
chức sau với dung dịch acid:

3) Vẽ cấu trúc hemiacetal vòng được tạo thành khi xử lí các hợp chất lưỡng chức
sau với dung dịch acid:

4) Xác định hydroxyaldehyde bị vòng hóa trong môi trường acid tạo thành
hemiacetal sau:

5) Hợp chất dưới đây có một nhóm aldehyde và hai nhóm OH. Trong môi trường
acid, một trong hai nhóm OH có thể đóng vai trò như một nucleophile và tác
kích nhóm carbonyl, nên có thể tạo thành hai kích thước vòng.
a) Bỏ qua yếu tố hóa lập thể, hãy vẽ hai vòng có thể tạo thành.
b) Sự biến dạng vòng có mối liên hệ với các kích thước vòng khác nhau. Hãy
dự đoán trong trường hợp này thì hemiacetal vòng nào sẽ chiếm ưu thế.

987 | Câu hỏi lí thuyết Hóa học hữu cơ OlympiaVN


6) Chất nào sau đây có thể nghịch quay?

7) Chất nào sau đây có thể nghịch quay?

8) Mặc dù các ketone thường bền hơn đồng phân aldehyde xấp xỉ 12 kJ mol-1,
tuy nhiên fructose-6-phosphate lại kém bền hơn glucose-6-phosphate khoảng
1.7 kJ mol-1. Giải thích tại sao.

988 | Câu hỏi lí thuyết Hóa học hữu cơ OlympiaVN


Hướng dẫn
1) Chúng ta sử dụng hệ thống đánh số để xác định kích thước của vòng được tạo
thành. Ví dụ với chất (a) thì bốn nguyên tử carbon và một nguyên tử oxygen kết
hợp thành vòng năm cạnh, có hai nhóm methyl ở C4 sẽ phải được biểu diễn trong
sản phẩm. Các chất còn lại cũng tương tự.
a) b)

c) d)

2)
(a)

(b)

(c)

3)

4) Nhóm carbonyl có thể bị tác kích bởi nhóm OH-C4 tạo thành vòng năm cạnh:

989 | Câu hỏi lí thuyết Hóa học hữu cơ OlympiaVN


Hoặc bị tác kích bởi nhóm OH-C5, tạo thành vòng sáu cạnh:

Vòng sáu cạnh chiếm ưu thế do nó có biến dạng vòng ít hơn vòng năm cạnh.
5) Hợp chất a và hemiacetal và b là hemiketal, Do đó chúng có thể nghịch quay,
còn c là ketal và không thể nghịch quay.
6) Chất a có tâm hemiacetal ở vòng bên phải, có thể nghịch quay. Hợp chất b có
các tâm acetal trên cả hai vòng và không thể nghịch quay.
7) Glucose-6-phosphate và fructose-6-phosphate không tồn tại trong dung dịch ở
dạng mạch hở, mà ở dạng các dẫn xuất hemiacetal và hemiketal vòng. Do đó, độ
bền tương đối phản ánh những khác biệt khác giữa các dẫn xuất vòng, như cấu
trúc vòng và các tương tác không gian.

990 | Câu hỏi lí thuyết Hóa học hữu cơ OlympiaVN


Bài 7
Đường kính là sucrose tinh khiết, được tìm thấy trong củ cải đường và mía. Trong
nước, sucrose bị thủy phân thành dung dịch của D-glucose và D-fructose. Hỗn
hợp này được gọi là “đường nghịch chuyển”. Phản ứng thủy phân có thể được
khảo sát bởi phân cực kế. Dung dịch sucrose có tính hữu triền (quay phải) (   D
20

= 66.53o cm3 g-1), trong khi đó dung dịch đường nghịch chuyển lại có tính tả triền
(quay trái) (   D của glucose và fructose lần lượt là +52.7 o và -92.4o cm3 g-1).
20

Tốc độ thủy phân phụ thuộc nhiều vào nồng độ hydrogen ion.
1) Viết phương trình phản ứng (sơ đồ phản ứng) thủy phân sucrose, sử dụng
công thức chiếu Haworth để biểu diễn tác nhân và sản phẩm phản ứng.
Sự nghịch chuyển sucrose được khảo sát ở 25 oC. Các giá trị sau đây của góc
quay, αt, ở dạng hàm của thời gian, t, quan sát được là:

2) α∞ tương ứng với chuyển hóa hoàn toàn. Do đó αt - α∞ là thước đo nồng độ


của sucrose. Sử dụng các giá trị cho ở trên để chứng minh rằng phản ứng có
bậc 1 theo nồng độ sucrose, và tính hằng số tốc độ.
3) Trong HCl 0.1 M, tốc độ tăng gấp đôi khi tăng nhiệt độ từ 25 lên 30 oC. Tính
năng lượng hoạt hóa của phản ứng.

991 | Câu hỏi lí thuyết Hóa học hữu cơ OlympiaVN


Hướng dẫn
1)

2) ln(αt - α∞) ở dạng hàm của thời gian là một đường thẳng, có nghĩa đây là phản
ứng bậc 1. Độ dốc là -0.0053 min-1.

3)

992 | Câu hỏi lí thuyết Hóa học hữu cơ OlympiaVN


Bài 8
Sự chuyển hóa [bởi tác động của] vi khuẩn của saccharose tạo thành (S)-(+)-2-
hydroxypropanoic acid (L-(+)- lactic acid), chất này tạo thành ester vòng từ 2
phân tử. Dilactide này có thể bị trùng hợp tạo thành một polylactide, được sử
dụng trong phẫu thuật.
1) Xác định cấu trúc không gian và công thức chiếu của L-(+)-lactic acid và
dilactide của nó.
2) Vẽ cấu trúc của polylactide được nói đến ở trên (với ít nhất 3 mắt xích). Xác
định tính điều hòa của nó (isotactic, syndiotactic hay atatic?).
3) Vẽ công thức các dilactide đồng phân được tạo thành từ racemic của lactic
acid. Chỉ rõ cấu hình của các tâm chiral.
L-(+)-lactic acid được sử dụng để điều chế thuốc diệt cỏ Barnon. Trong trường
hợp này, (+)-lactic acid được ester hóa với 2-propanol, sau đó nhóm hydroxyl
được xử lí với methanesulfonyl chloride. Sau đó, sản phẩm tạo thành tham gia
vào phản ứng thế SN2 với 3-fluoro-4-chloro-phenylamine, trong đó nhóm
methanesulfonate rời đi ở dạng CH3SO3-. Cuối cùng, nhóm benzoyl được đưa vào
qua phản ứng với benzoyl chloride.
4) Hoàn thành chuỗi phản ứng. Biểu diễn các sản phẩm ở dạng công thức chiếu
Fischer.

993 | Câu hỏi lí thuyết Hóa học hữu cơ OlympiaVN


Hướng dẫn
1)

2)

3)

994 | Câu hỏi lí thuyết Hóa học hữu cơ OlympiaVN


4)

995 | Câu hỏi lí thuyết Hóa học hữu cơ OlympiaVN


Bài 9
1,3-Dihydroxyacetone có thể được chuyển hóa thành glyceraldehyde. Sau một
khoảng thời gian, glyceraldehyde này có thể chuyển thành dimer vòng 6 cạnh
C6H12O6. Phổ hồng ngoại của dimer này cho thấy không có peak hấp thụ giữa
1600-1800 cm-1, và moment lưỡng cực của dimer được xác định là bằng 0.
1) Vẽ công thức chiếu Fischer của glyceraldehyde tạo thành và chỉ rõ cấu hình
của nó là dạng D(+) hay L(-).
2) Xác định công thức cấu tạo của trung gian phản ứng trong chuyển hóa của
1,3- dihydroxyacetone thành glyceraldehyde.
3) Viết công thức cấu tạo của dimer tạo thành.
4) Sử dụng công thức chiếu Haworth, hãy biểu diễn các đồng phân lập thể có
thể có phù hợp với dữ kiện về moment lưỡng cực.
5) Xác định cấu hình tuyệt đối (R/S) của mỗi nguyên tử chiral carbon trong các
công thức ở trên.
Hướng dẫn
1)

2)

3)

4)

996 | Câu hỏi lí thuyết Hóa học hữu cơ OlympiaVN


5)

997 | Câu hỏi lí thuyết Hóa học hữu cơ OlympiaVN


Bài 10
Có thể chuyển hóa R(+) – glyxerandehit thành axit(-)-malic (axit 2-
hydroxibutandioic) qua con đường như sau: Đầu tiên xử lý R(+) – glyxerandehit
với hydro xianua để thu được hai đồng phân A và B có thể tách ra được bằng
phương pháp kết tinh phân đoạn. Chỉ B được chuyển hóa thành C (C4H8O5) mà
khi oxy hóa chất này bởi axit nitric cho D (C4H6O6). Xử lý tiếp D bằng photpho
tribromua cho E (C4H5BrO5) để cuối cùng chuyển thành axit(-)-malic bằng cách
khử với kẽm trong axit.
Viết các phản ứng xảy ra ở dạng công thức chiếu Fischer. Giải thích tại sao chỉ
duy nhất E được tạo thành từ D. Cho biết nếu sử dụng A thay cho B thì sẽ tạo
thành bao nhiêu đồng phân chất E ?
Hướng dẫn

Phản ứng chuyển từ D sang E là SN2 nên quay cấu hình. Chính vì vậy nếu sử dụng
đồng phân A sẽ tạo hai đồng phân E do lúc này dạng meso D sẽ chuyển thành
dạng đối quang có hai C* mang cấu hình khác nhau, tức sản phẩm thu được nhiều
khả năng là hỗn hợp raxemic.

998 | Câu hỏi lí thuyết Hóa học hữu cơ OlympiaVN


Bài 11
1) Vẽ cấu dạng ghế bền hơn của mỗi hợp chất sau:
(a) β-D-Galactopyranose
(b) α-D-Glucopyranose
(c) β-D-Glucopyranose
2) Vẽ dạng mạch hở của monosaccharide vòng sau:

3) β-D-glucopyranose có hai cấu dạng ghế. Vẽ và xác định cấu dạng kém bền
hơn.
Hướng dẫn
1)
a) b) c)

2)

3) Cấu dạng kém bền hơn là cấu dạng có tất cả các nhóm thế chiếm vị trí trục
(axial).

999 | Câu hỏi lí thuyết Hóa học hữu cơ OlympiaVN


1000 | Câu hỏi lí thuyết Hóa học hữu cơ OlympiaVN
Bài 12
Hai liên kết carbon-fluorine trong phân tử A dưới đây có độ dài khác nhau, chênh
lệch khoảng 0.015 Å. Dựa vào quan điểm thuyết FMO, hãy giải thích quan sát
này và chỉ rõ liên kết C-F nào dài hơn.

Hướng dẫn

1001 | Câu hỏi lí thuyết Hóa học hữu cơ OlympiaVN


Bài 13
1) Xét cấu trúc của hai D-aldotetrose sau:

Mỗi hợp chất này tồn tại ở dạng vòng furanose, được tạo thành khi OH ở C4 tác
kích vào nhóm aldehyde. Vẽ cấu trúc các vòng furanose sau:
(a) α-D-Erythrofuranose
(b) β-D-Erythrofuranose
(c) α-D-Threofuranose
(d) β-D-Threofuranose
2) Vẽ cơ chế cho phản ứng vòng hóa xúc tác acid của L-threose thành β-L-
threofuranose.
3) Vẽ cơ chế cho phản ứng vòng hóa xúc tác acid của D-fructose thành β-D-
fructofuranose.
Hướng dẫn
1)
a) b) c) d)

2) Trước tiên nhóm carbonyl bị proton hóa, khiến cho nó có tính electrophile
mạnh hơn và phù hợp hơn cho sự tác kích nucleophile bởi một trong các nhóm
OH. Nhóm OH ở C4 sẽ tác kích nhóm carbonyl được proton hóa tạo thành một
vòng furanose (năm cạnh). Sự deproton hóa (với nước đóng vai trò như một base)
tạo thành sản phẩm.

1002 | Câu hỏi lí thuyết Hóa học hữu cơ OlympiaVN


3)

1003 | Câu hỏi lí thuyết Hóa học hữu cơ OlympiaVN


Bài 14
1) Vẽ sản phẩm nhận được khi xử lí mỗi hợp chất sau với acetic anhydride khi
có mặt pyridine:
(a) α-D-Galactopyranose
(b) α-D-Glucopyranose
(c) β-D-Galactopyranose
2) Vẽ sản phẩm nhận được khi xử lí mỗi hợp chất ở ý 1 với methyl iodide khi
có mặt silver oxide (Ag2O).
3) Vẽ các sản phẩm dự đoán tạo thành khi xử lí α-D-galactopyranose với lượng
dư methyl iodide khi có mặt silver oxide, sau đó xử lí với dung dịch acid.
4) Khi xử lí D-glucose với dung dịch nước bromine (trong đệm pH 6) thì tạo
thành một aldonic acid gọi là D-gluconic acid. Xử lí D-gluconic acid với xúc
tác acid tạo thành một lactone (ester vòng) có vòng sáu cạnh.
a) Vẽ cấu trúc D-gluconic acid.
b) Vẽ cấu trúc lactone tạo thành từ D-gluconic acid, biểu diễn cấu hình mỗi
tâm thủ tính.
c) Dự đoán lactone này có tính quang hoạt không?
d) Giải thích làm thế nào mà bạn có thể phân biệt được D-gluconic acid và
lactone bởi phổ IR.
5) Glycoside nào được tạo thành khi xử lí mỗi monosaccharide này với
CH3CH2OH, HCl: (a) β-D-mannose; (b) α-D-gulose; (c) β-D-fructose?
6) Trình bày cơ chế phản ứng sau:

1004 | Câu hỏi lí thuyết Hóa học hữu cơ OlympiaVN


Hướng dẫn
1) Khi xử lí với acetic anhydride và pyridine, tất cả các nhóm OH bị acetyl hóa:
(a)

(b)

(c)

1005 | Câu hỏi lí thuyết Hóa học hữu cơ OlympiaVN


2) Khi xử lí với lượng dư methyl iodide khi có mặt silver oxide, tất cả các nhóm
OH bị chuyển thành nhóm methoxy:
(a)

(b)

(c)

3)

4)

1006 | Câu hỏi lí thuyết Hóa học hữu cơ OlympiaVN


a)

b)

c) Hợp chất này có các tâm thủ tính và không phải hợp chất meso. Do đó nó
quang hoạt.
d) Gluconic acid là một carboxylic acid và trong phổ IR được dự đoán sẽ có một
tín hiệu rộng trong khoảng 2200 và 3600 cm-1. Phổ IR của lactone thì không
có tín hiệu rộng này.

1007 | Câu hỏi lí thuyết Hóa học hữu cơ OlympiaVN


5)

6)

1008 | Câu hỏi lí thuyết Hóa học hữu cơ OlympiaVN


Bài 15
Glucozơ phản ứng với metanol trong HCl để sinh ra metyl glucopyranozơ.
a) Đề nghị cơ chế phản ứng này.
b) Trên lý thuyết, sản phẩm thu được sẽ là hỗn hợp giữa hai đồng phân α và
β-metyl glycozit. Tuy nhiên thực tế chỉ thu được α-glycozit. Hãy giải
thích điều này.
c) Đề nghị một phương thức tổng hợp chọn lọc β-D-metyl glucopyranozơ
từ glucozơ.
Hướng dẫn
a) Cơ chế phản ứng như sau:

b) Có thể giải thích theo một trong ba cách sau:


Cách 1: Obitan p của carbocation phải nằm thẳng góc với cặp electron n của oxy
để tạo được sự liên hợp bền.
Cách 2: Sản phẩm α-glycozit không xảy ra sự đẩy lưỡng cực giữa cặp electron n
của oxy và liên kết C-O như sản phẩm β-glycozit.

Cách 3: Tồn tại tương tác n - *C-O giữa cặp e của O1 với liên kết C2 – O(CH3).

c) Sơ đồ tổng hợp.

1009 | Câu hỏi lí thuyết Hóa học hữu cơ OlympiaVN


1010 | Câu hỏi lí thuyết Hóa học hữu cơ OlympiaVN
Bài 16
a) Đề xuất cơ chế cho phản ứng dưới đây:

b) Sử dụng những hiểu biết về lí thuyết FMO và các hiệu ứng điện tử, hãy giải
thích sự tạo thành ưu tiên của anomer ở trên.
Hướng dẫn
a)

b) Phản ứng xúc tác acid


tạo thành acetal như mô
tả ở trên là một phản ứng
thuận nghịch, tạo thành
sản phẩm bền nhiệt động.
α-anomer bền nhiệt động
hơn β-anomer do chỉ có
α-anomer được hưởng lợi
từ sự nhường [donation]
electron của cặp electron chưa liên kết trên oxygen vào liên kết C-O liền kề.

1011 | Câu hỏi lí thuyết Hóa học hữu cơ OlympiaVN


Bài 17
Xác định các sản phẩm được tạo thành khi xử lí β-D-galactose với mỗi tác nhân
sau:
a) Ag2O + CH3I
b) NaH + C6H5CH2Cl
c) Sản phẩm ở (b), sau đó với H3O+
d) Ac2O + pyridine
e) C6H5COCl + pyridine
f) Sản phẩm ở (c), sau đó với C6H5COCl + pyridine
Hướng dẫn

1012 | Câu hỏi lí thuyết Hóa học hữu cơ OlympiaVN


Bài 17A
-D-(+)-mannopyranose là một epimer của -D-(+)-glucopyranose.
- Vẽ cấu dạng ghế bền nhất của nó.
- Xác định sản phẩm của -D-(+)-mannopyranose với các tác nhân sau:
a) Cu2+ (trong hệ đệm pH >7)
b) Br2, H2O (pH = 6)
c) HNO3
d) CH3OH, HCl khan
e) sản phẩm phản ứng d + (CH3)2SO2, NaOH
f) 1. NaBH4; 2. H2O
g) 5 đương lượng HIO4
h) acetic anhydride dư trong pyridine
i) 3 đương lượng phenylhydrazine, H+
j) 1. Br2/H2O; 2. Fe(III) sulfate, H2O2
k) 1. HCN; 2. Ba(OH)2; 3. H3O+; 4. Na-Hg, H2O, pH 3-5
Hướng dẫn

a) b) c)

d) e) f)

1013 | Câu hỏi lí thuyết Hóa học hữu cơ OlympiaVN


g) h) i)

j) k)

1014 | Câu hỏi lí thuyết Hóa học hữu cơ OlympiaVN


Bài 18
1) Nguyên liệu đầu để tổng hợp thương mại vitamin C là L-sorbose. Nó có thể
được tổng hợp từ D-glucose qua chuỗi phản ứng dưới đây. Giai đoạn thứ hai
của chuỗi phản ứng này minh họa việc sử dụng quá trình oxid hóa xúc tác bởi
vi khuẩn; vi sinh vật A. suboxydans hoàn thành giai đoạn này với hiệu suất
90%. Kết quả tổng thể của quy trình tổng hợp là chuyển hóa của một D-
aldohexose (D-glucose) thành một L-ketohexose (L-sorbose). Điều này có ý
nghĩa gì về tính đặc hiệu của quá trình oxid hóa vi khuẩn?

2) Hai aldose nào cũng tạo thành phenylosazone như L-sorbose?


3) Ngoài fructose và sorbose, có hai 2-ketohexose khác là psicose and tagatose.
D-psicose tạo thành cùng phenylosazone như D-allose (hoặc D-altrose); D-
tagatose tạo thành cùng osazone như D-galactose (hoặc D-talose). Xác định
cấu trúc của D-psicose và D-tagatose.
4) A, B, C là các aldohexose. Hợp chất A và B tạo thành cùng alditol quang hoạt
khi chúng bị khử bởi hydrogen xúc tác; A và B tạo thành các phenylosazone
khác nhau khi xử lí với phenylhydrazine; B và C tạo thành phenylosazone
giống nhau nhưng alditol khác nhau. Giả sử chúng đều là đường D, xác định
tên và cấu trúc của A-C.
Hướng dẫn
1) Vi sinh vật oxid hóa chọn lọc nhóm -CHOH của D-glucitol tương ứng với
C5 của D-glucose.

2) L-Gulose và L-idose

1015 | Câu hỏi lí thuyết Hóa học hữu cơ OlympiaVN


3)

1016 | Câu hỏi lí thuyết Hóa học hữu cơ OlympiaVN


4) A = D-altrose, B = D-talose, C = D-galactose

1017 | Câu hỏi lí thuyết Hóa học hữu cơ OlympiaVN


Bài 19
D-idose có cấu hình ngược với D-glucose ở C-2, C-3, C-4.
D-idose tồn tại trong một cân bằng của hai dạng pyranose (75 %) và furanose (25
%).
a) Vẽ cấu dạng vòng cyclohexane của các đồng phân anomer  và -D-
idopyranose. Theo bạn thì dạng nào bền nhất với mỗi anomer? Tại sao?
b) Qua chuyển hoá Lobry de Bruyn — Alberda van Ekenstein, D-idose có thể
đồng phân hoá thành một 2-ketose (D-sorbose). Vẽ cấu trúc dạng furanose
của D-sorobose.
c) Khi đun nóng D-idose bị mất nước và tồn tại chủ yếu ở dạng 1,6-anhydro-D-
idopyranose. Phản ứng này ưu tiên tạo thành anomer nào hơn? Vẽ cấu trúc
của anomer đó. Giải thích tại sao phản ứng tương tự không xảy ra với
glucose?
Hướng dẫn
a) Dưới đây là cấu dạng ghế của hai anomer D-idose:

Bền hơn

Bền hơn
Các cấu dạng bền hơn đều có nhiều nhóm thế ở vị trí liên kết biên (equatorial)
hơn.
b) Cấu trúc dạng furanose của D-sorobose

1018 | Câu hỏi lí thuyết Hóa học hữu cơ OlympiaVN


c) Cấu dạng -anomer của D-idose có thể vòng hoá nội phân tử, tạo thành dẫn
xuất anhydro. Để thực hiện phản ứng này thì ở C-6 phải có nhóm thế liên kết trục
(axial). Cấu dạng này của glucose rất kém bền, nên nó không có phản ứng tương
tự.

1019 | Câu hỏi lí thuyết Hóa học hữu cơ OlympiaVN


Bài 19A
Đun nóng D-altrose với acid loãng tạo thành một đường anhydro (khan) không
khử (C6H10O5). Methyl hóa đường anhydro này sau đó xử lí với acid tạo thành
2,3,4-tri-O-methyl-D-altrose. Sự tạo thành đường anhydro diễn ra qua một cấu
dạng ghế của β-D-altropyranose trong đó nhóm -CH2OH ở vị trí trục.
1) Xác định cấu trúc đường anhydro và nó được tạo thành như thế nào?
2) D-glucose cũng tạo thành một đường anhydro nhưng các điều kiện cần thiết
khắc nghiệt hơn so với phản ứng tương ứng của D-altrose. Giải thích tại sao.
Hướng dẫn
1) Đường anhydro được tạo thành khi nhóm CH2OH trục phản ứng với C1 tạo
thành acetal vòng.

Do đường anhydro là acetal (nghĩa là một glycoside nội) nên nó là đường không
khử. Khi methyl hóa, sau đó thủy phân acid sẽ chuyển đường anhydro thành 2,3,4-
tri-O-methyl-D-altrose:

2) Sự tạo thành một đường anhydro đòi hỏi monosaccharide đáp ứng cấu djang
ghế với nhóm CH2OH ở vị trí trục. Với β-D-altropyranose quá trình này đòi
hỏi hai nhóm OH cũng ở vị trí OH. Tuy nhiên, với β-D-glucopyranose thì nó
đòi hỏi bốn nhóm OH phải có vị trí trục và do đó phân tử đáp ứng một cấu
dạng rất kém bền:

1020 | Câu hỏi lí thuyết Hóa học hữu cơ OlympiaVN


1021 | Câu hỏi lí thuyết Hóa học hữu cơ OlympiaVN
Bài 20
Xác định cấu trúc các chất chưa biết trong các dãy chuyển hóa sau:
1. Từ axit glucuronic chuyển thành monosaccarit 6-deoxyhexopyranozơ
(C7H14O5) theo sơ đồ:

2. Cũng có thể chuyển axit glucuronic thành một aminosaccarit theo sơ đồ sau:

Hướng dẫn
1)

1022 | Câu hỏi lí thuyết Hóa học hữu cơ OlympiaVN


2)

1023 | Câu hỏi lí thuyết Hóa học hữu cơ OlympiaVN


Bài 21
Cho α-D-glucopyranozơ phản ứng với hỗn hợp Ac2O/HBr thu được chất A. Chất
này tiếp tục phản ứng với trietylamin cho chất B. B dưới ảnh hưởng của dung
dịch nước Ba(OH)2 thu được chất C (C6H10O5). Chất C không có khả năng phản
ứng với thuốc thử Fehling và không tham gia vào quá trình lên men rượu. 1 mol
B phản ứng được với 2 mol HIO4 cho ra G và 1 mol HCOOH. Oxy hóa G bằng
Br2 đầu tiên cho ra sản phẩm D, oxy hóa tiếp chất này thu được hỗn hợp đẳng số
mol của H2C2O4 và axit D – CHOH(COOH)2. Hãy xác định cấu trúc các chất chưa
biết.
Hướng dẫn
Dựa vào công thức phân tử chất C có thể thấy rằng nó là sản phẩm dehydrat hóa
của glucozơ. Rất có khả năng đây là sự tạo thành một liên kết ete từ hai nhóm
ancol. Việc chất C không có khả năng lên men cũng như cho kết quả âm tính với
thuốc thử Fehling cho thấy nhóm hydroxyl ở C1 đã tham gia phản ứng loại nước
này. Các dữ kiện oxy hóa tiếp theo bằng HIO4 và dung dịch Br2 cho thấy rằng
chất C là 1,2,3-triol, tức nhóm OH tham gia phản ứng ete hóa với OH ở C1 là OH
ở C6. Vậy C chính là 1,6-anhydro-β-D-pyranozơ. Như vậy sơ đồ phản ứng sẽ như
sau:

1024 | Câu hỏi lí thuyết Hóa học hữu cơ OlympiaVN


Bài 22
Hợp chất C (C6H8O6) phản ứng được với 2 mol CH2N2 ở 0 oC và 1 mol CH2N2 ở
-78 oC. C có khả năng làm mất màu nước brom và tạo dị vòng 1,3-dioxolan với
axeton. Sản phẩm phản ứng giữa C và 2 mol CH2N2 được metyl hóa hoàn toàn
rồi ozon phân sẽ thu được este D. Chất này phản ứng với NH3 cho oxamit
(CONH2)2 và amit của axit 3,4-dimetyl-L-threonic. Mặt khác oxy hóa C rồi cắt
mạch bằng NaOI thu được axit E (C2H2O2) và axit F (C4H8O5). Oxy hóa F bằng
HNO3 thu được axit L-(+)-tactric. Lập luận xác định cấu trúc các chất chưa biết
với lập thể chính xác. Biết rằng khi cho D phản ứng với HIO4 thu được hỗn hợp
sản phẩm, trong số đó có HCHO.
Hướng dẫn
C phản ứng được với 2 mol CH2N2 ở 0 oC nhưng chỉ có 1 mol CH2N2 ở -78 oC,
chứng tỏ trong C có ít nhất 2 nhóm OH với khả năng phản ứng khác nhau.
C làm mất màu nước brom và tạo dị vòng 1,3-dioxolan với axeton  Trong C có
ít nhất một liên kết đôi và 2 nhóm OH ở cạnh nhau.
Este D phản ứng với NH3 cho oxamit (CONH2)2, chứng tỏ D phải có đơn vị
RCOO-COOH. D được tạo thành từ quá trình ozon phân, có nghĩa D phải là một
enol.
Việc tạo thành hỗn hợp gồm axit 3,4-dimetyl-L-threonic + oxamit (6C) chứng tỏ
ozon phân không làm gẫy mạch phân tử. Có nghĩa là phân tử ban đầu phải có cấu
trúc vòng, và vị trí endiol sẽ bị cắt mạch bằng ozon. Bên cạnh đó cắt mạch D bằng
HIO4 thu được HCHO chứng tỏ D phải có đoạn mạch R-CHOH-CH2OH.
Tổng hơp các thông tin trên có thể xác định được C là vitamin C (axit ascorbic).
Như vậy cấu trúc lập thể của các chất như sau:

1025 | Câu hỏi lí thuyết Hóa học hữu cơ OlympiaVN


Bài 23
Trong khi các monosaccharide chịu sự đồng phân hóa phức tạp trong base thì các
aldonic acid bị epimer hóa đặc thù ở C2 khi đun nóng với pyridine. Bạn có thể sử
dụng phản ứng này như thế nào để tổng hợp D-mannose từ D-glucose.
Hướng dẫn

1026 | Câu hỏi lí thuyết Hóa học hữu cơ OlympiaVN


Bài 23A
Chuỗi phản ứng dưới đây biểu diễn một phương pháp thanh lịch để tổng hợp 2-
deoxy-D-ribose, IV, được D. C. C. Smith công bố năm 1955:

1) Xác định cấu trúc II và III.


2) Đề xuất cơ chế cho chuyển hóa III thành IV.

1027 | Câu hỏi lí thuyết Hóa học hữu cơ OlympiaVN


Hướng dẫn

1028 | Câu hỏi lí thuyết Hóa học hữu cơ OlympiaVN


Bài 23B
Khi xử lí D-glucose với sodium hydroxide, thì tạo thành một hỗn hợp phức tạp
của các carbohydrate, bao gồm D-mannose và D-fructose. Theo thời gian, gần
như toàn bộ các chất hiện diện trong hỗn hợp là aldohexose. Thậm chí có thể phát
hiện L-glucose, mặc dù với nồng độ cực kì nhỏ. Trình bày một cơ chế chấp nhận
được, với ít giai đoạn nhất, biểu diễn sự tạo thành L-glucose từ D-glucose.

1029 | Câu hỏi lí thuyết Hóa học hữu cơ OlympiaVN


Hướng dẫn

1030 | Câu hỏi lí thuyết Hóa học hữu cơ OlympiaVN


Bài 24
1) Khi xử lí α-D-galactopyranose với ethanol có mặt xúc tác acid như HCl thì
tạo thành hai sản phẩm. Xác định cấu trúc hai sản phẩm và vẽ cơ chế giải
thích sự tạo thành chúng.
2) Methyl α-D-glucopyranoside là một hợp chất kém bền và không trải qua sự
nghịch đảo đường trong các môi trường trung tính hoặc base. Tuy nhiên, khi
đưa vào môi trường acid thì một cân bằng được thiết lập giữa methyl α-D-
glucopyranoside và methyl β-D-glucopyranoside. Trình bày cơ chế giải thích
cho hiện tượng này.
Hướng dẫn
1) Trong điều kiện acid, nhóm OH anomer có thể bị proton hóa, tạo thành một
nhóm rời đi tốt (H2O). Việc mất nhóm rời đi tạo thành một trung gian cation
được bền hóa cộng hưởng.

Sau đó trung gian này bị tác kích bởi ethanol, tạo thành một oxonium ion, tiểu
phân này sau đó bị deproton hóa tạo thành acetal:

Chú ý rằng như biểu diễn ở trên thì ethanol bị tác kích từ phía trên, tuy nhiên nó
cũng có thể bị tác kích từ phía dưới, tạo thành acetal sau đây:

2) Trong môi trường acid, nhóm anomer methoxy có thể bị proton hóa, tạo thành
một nhóm rời đi tốt (methanol). Sự mất nhóm rời đi tạo thành một trung gian
cation được bền hóa cộng hưởng (các cấu trúc cộng hưởng không được biểu diễn
dưới đây), có thể bị tác kích bởi methanol. Sự tác kích này có thể diễn ra từ phía

1031 | Câu hỏi lí thuyết Hóa học hữu cơ OlympiaVN


trên hoặc phía dưới tạo thành hỗn hợp hai anomer. Trong giai đoạn cuối của cơ
chế, oxonium ion bị deproton hóa:

1032 | Câu hỏi lí thuyết Hóa học hữu cơ OlympiaVN


Bài 25
1) Trình bày cơ chế cho chuyển hóa qua lại xúc tác acid của hai glucose anomer
bởi sự nghịch đảo đường:

2) Trình bày cơ chế cho phản ứng sau:

3) Trình bày cơ chế phản ứng thủy phân:

4) Trong quá trình oxid hóa D-allose thành D-allonic acid, phân lập được một
lactone có cấu trúc tổng quát A. Trình bày cơ chế giải thích sự tạo thành A.
Chỉ rõ hóa lập thể của A.

5) Phản ứng đồng phân hóa sau diễn ra với tất cả các aldohexose khi có mặt
base. Trình bày cơ chế biểu diễn cách các hợp chất được tạo thành.

Hướng dẫn

1033 | Câu hỏi lí thuyết Hóa học hữu cơ OlympiaVN


1)

2)

1034 | Câu hỏi lí thuyết Hóa học hữu cơ OlympiaVN


3)

4)

1035 | Câu hỏi lí thuyết Hóa học hữu cơ OlympiaVN


5)

1036 | Câu hỏi lí thuyết Hóa học hữu cơ OlympiaVN


Bài 26
Trình bày cơ chế cho phản ứng sau:

Hướng dẫn

1037 | Câu hỏi lí thuyết Hóa học hữu cơ OlympiaVN


Bài 27
1) Khi xử lí D-altrose hoặc D-talose với sodium borohydride khi có nước thì đều
tạo thành cùng sản phẩm. Giải thích hiện tượng này.
2) Khi xử lí D-allose hoặc L-allose với sodium borohydride khi có nước thì đều
tạo thành cùng sản phẩm. Giải thích hiện tượng này.
3) Trong số tám D-aldohexose, chỉ có hai chất tạo thành alditol không quang
hoạt khi xử lí với sodium borohydride khi có nước. Xác định hai aldohexose
này và giải thích tại sao alditol của chúng không quang hoạt.
4) Khi xử lí với sodium borohydride, D-glucose bị chuyển thành một alditol.
a) Xác định cấu trúc alditol.
b) L-aldohexose nào tạo thành cùng alditol khi xử lí với sodium
borohydride?
5) Hợp chất A là một D-aldopentose bị chuyển hóa thành một alditol quang hoạt
khi xử lí với sodium borohydride. Vẽ hai cấu trúc có thể có của hợp chất A.
Hướng dẫn
1) Trong hai trường hợp, sự khử nhóm carbonyl đều tạo thành cùng sản phẩm. Có
thể thấy được điều này bằng cách xoáy các sản phẩm 180o như dưới đây:

2)

1038 | Câu hỏi lí thuyết Hóa học hữu cơ OlympiaVN


3) Sự khử D-allose hoặc D-galactose sẽ tạo thành meso alditol. Các hợp chất meso
không quang hoạt.

5)

1039 | Câu hỏi lí thuyết Hóa học hữu cơ OlympiaVN


5)

1040 | Câu hỏi lí thuyết Hóa học hữu cơ OlympiaVN


Bài 28
1) Vẽ và gọi tên các cặp epimer được tạo thành khi tiến hành quá trình nối mạch
Kiliani-Fischer với các aldopentose sau:

2) Xác định các tác nhân có thể sử dụng để chuyển D-erythrose thành D-ribose.
3) Xác định hai aldohexose tạo thành khi D-arabinose trải qua tổng hợp Kiliani-
Fischer.
Hướng dẫn
Trong tổng hợp Kiliani-Fischer, mạch carbon được kéo dài, với C1 trở thành C2
trong sản phẩm. Ở vị trí C2 có thể nhận được hai cấu hình có thể có, tương ứng
với cặp epimer dưới đây:
a)

1041 | Câu hỏi lí thuyết Hóa học hữu cơ OlympiaVN


b)

c)

2) Chuyển hóa D-erythrose (có bốn carbon) thành D-ribose (có năm carbon) đòi
hỏi một quá trình kéo dài mạch. Quá trình này sẽ tạo thành D-ribose cùng với C2
epimer của nó, là D-arabinose, như dưới đây:

1042 | Câu hỏi lí thuyết Hóa học hữu cơ OlympiaVN


3)

1043 | Câu hỏi lí thuyết Hóa học hữu cơ OlympiaVN


Bài 29
1) Vẽ và gọi tên hai aldohexose có thể chuyển hóa thành D-ribose sử dụng thoái
phân Wohl.
2) Xác định các tác nhân bạn có thể sử dụng để chuyển D-ribose thành D-
erythrose.
3) Khi D-glucose trải qua thoái phân Wohl, sau đó kéo dài mạch Kiliani-Fischer,
thì tạo thành một hỗn hợp epimer. Xác định hai epimer này.
4) Xác định hai aldohexose khi trải qua thoái phân Wohl đều tạo thành D-ribose.
Vẽ công thức chiếu Fischer cho dạng mạch hở của mỗi aldohexose.
Hướng dẫn
1)

2)

3)

1044 | Câu hỏi lí thuyết Hóa học hữu cơ OlympiaVN


4)

1045 | Câu hỏi lí thuyết Hóa học hữu cơ OlympiaVN


Bài 30
1) Vẽ cấu trúc được dự đoán tạo thành khi xử lí dạng β-pyranose của hợp chất
A với lượng dư ethyl iodide khi có mặt silver oxide. Thông tin sau có thể
được dùng để xác định hợp chất A:
- Khối lượng mol của hợp chất A là C6H12O6.
- Hợp chất A là đường khử.
- Khi hợp chất A tham gia vào thoái phân Wohl hai lần liên tiếp thì tạo thành
D-erythrose.
- Hợp chất A là epimer với D-glucose ở C3.
- Cấu hình ở C2 là R.
2) Hợp chất X là một D-aldohexose có thể tồn tại dạng β-pyranose với chỉ một
nhóm thế trục (axial). Hợp chất X trải qua thoái phân Wohl tạo thành một
aldopentose, chất này khi xử lí với sodium borohydrie tạo thành một alditol
quang hoạt. Từ dữ liệu được cho, chỉ có hai cấu trúc khả dĩ với hợp chất X.
Xác định hai cấu trúc này và đề xuất một phép thử hóa học cho phép bạn phân
biệt hai chất này, và từ đó xác định được cấu trúc của hợp chất X.
3) Một aldohexose bị methyl hóa bởi dimethyl sulfate và sau đó xử lí với acid
êm dịu. Sản phẩm tạo thành, khi tiếp xúc với một chất oxid hóa mạnh, tạo
thành một dicarboxylic acid không quang hoạt chứa năm nguyên tử carbon.
Xác định cấu trúc có thể có của aldohexose.
Hướng dẫn
1) Công thức phân tử C6H12O6 cho thấy A là hexose. A là đường khử, nên nó
phải là aldohexose. Thoái phân Wohl liên tiếp hai lần hợp chất A tạo thành
D-erythrose, cho thấy cấu hình ở C4 và C5 của aldohexose đều là R (giống
như D-erythrose). Hợp chất A là epimer với glucose ở C3, cho thấy cấu hình
C3 là R (D-glucose có cấu hình S ở C3). Cuối cùng, C2 có cấu hình R. Từ
các dữ kiện trên xác định được A là D-allose (cấu trúc bên dưới). Dạng β-
pyranose của D-allose cũng được biểu diễn dưới đây. Khi xử lí với ethyl
iodide dư, có mặt silver oxide thì tất cả các nhóm OH trong dạng β-pyranose
của A bị alkyl hóa, chuyển thành các nhóm ethoxy:

2) Hợp chất X là D-aldohexose có dạng β-pyranose với chỉ một nhóm thế ở vị
trí trục. Hãy nhớ lại rằng, D-glucose có tất cả các nhóm thế ở vị trí biên

1046 | Câu hỏi lí thuyết Hóa học hữu cơ OlympiaVN


(equatorial), do đó hợp chất X phải là epimer với D-glucoser hoặc là ở C2
(D-mannose) hoặc C3 (D-allose) hoặc C4 (D-galactose). Hợp chất X trải qua
thoái phân Wohl tạo thành một aldopentose, chất này chuyển hóa thành
alditol quang hoạt khi xử lí với sodium borohydride. Do đó, hợp chất X không
thể là D-allose, do thoái phân Wohl của D-allose, sau đó khử tạo thành một
alditol không quang hoạt. Chúng ta kết luận rằng hợp chất X phải là D-
mannose hoặc D-galactose. Có thể xác định hợp chất X bằng cách xử lí X với
soium borohydride. Sự khử D-mannose sẽ tạo thành một alditol quang hoạt,
trong khi sự khử D-galactose tạo thành một alditol không quang hoạt.

1047 | Câu hỏi lí thuyết Hóa học hữu cơ OlympiaVN


3)

1048 | Câu hỏi lí thuyết Hóa học hữu cơ OlympiaVN


Bài 31
1) D-aldopentose nào bị oxid hóa thành aldaric acid quang hoạt và trải qua thoái
phân Wohl tạo thành một D-aldotetrose mà khi oxid hóa cũng tạo thành
aldaric acid quang hoạt?
2) D-aldopentose nào khác tạo thành cùng alditol như D-arabinose khi bị khử
bởi NaBH4 trong CH3OH?
3) D-aldopentose A bị oixd hóa bởi HNO3 tạo thành aldaric acid không quang
hoạt B. A trải qua tổng hợp Kiliani-Fischer tạo thành C và D. C bị oxid hóa
thành aldaric acid quang hoạt. Còn D bị oxid hóa thành aldaric acid không
quang hoạt. Xác định các hợp chất A-D.
4) D-aldopentose A bị khử thành một alditol quang hoạt. Sau tổng hợp Kiliani-
Fischer, A chuyển thành hai D-aldohexose B và C. B bị oxid hóa thành aldaric
acid không quang hoạt. C bị oxid hóa thành aldaric acid quang hoạt. Xác định
cấu trúc A-C.
5) D-aldohexose A bị khử thành alditol B quang hoạt, sử dụng NaBH4 trong
CH3OH. A bị chuyển hóa bởi thoái phân Wohl tạo thành aldopentose C, chất
này bị khử thành alditol không quang hoạt D. C bị chuyển hóa bởi thoái phân
Wohl tạo thành aldotetrose E, chất này bị oxid hóa thành aldaric acid quang
hoạt F. Khi chuyển đổi qua lại hai đầu của aldohexose A thì tạo thành một
aldohexose G khác. Xác định cấu trúc A-G.

1049 | Câu hỏi lí thuyết Hóa học hữu cơ OlympiaVN


Hướng dẫn
1)

Chỉ có A’’ trải qua thoái phân Wohl tạo thành aldotetrose mà bị oxid hóa thành
aldaric acid quang hoạt. Do đó A’’ là cấu trúc của D-aldopentose.
2)

3) Chỉ có hai D-aldopentose (A’ và A’’) tạo thành các aldaric acid không quang
hoạt (B’ và B’’).

1050 | Câu hỏi lí thuyết Hóa học hữu cơ OlympiaVN


Chỉ có A’ thỏa mãn các dữ kiện. Tổng hợp Kiliani-Fischer của A’ tạo thành C’
và D’ mà khi bị oxid hóa thành một aldaric acid quang hoạt và một acid không
quang hoạt. Quy trình tương tự với A’’ thì tạo thành hai acid đều quang hoạt. Do
đó, cấu trúc A-D tương ứng với A’-D’.

1051 | Câu hỏi lí thuyết Hóa học hữu cơ OlympiaVN


4)

5)

1052 | Câu hỏi lí thuyết Hóa học hữu cơ OlympiaVN


Bài 32
D-aldotetrose A phản ứng nitric acid, tạo thành một hợp chất không quang hoạt.
Cũng aldotetrose này phản ứng với HCN, sau đó xử lý với dung dịch Ba(OH)2
tạo thành hai aldonic acid B và C, là đồng phân epimer. Các aldonic acid này tồn
tại trong một cân bằng với các -aldolactone tương ứng, D và E. Cho hỗn hợp này
phản ứng với Na-Hg và nước ở pH 3-5, tạo thành các sản phẩm tương ứng là F
và G. Oxid hoá F bằng nitric acid, tạo thành một aldaric acid H không quang
hoạt, trong khi phản ứng tương tự của E lại tạo thành aldaric acid I quang hoạt.
Xác định công thức cấu tạo các chất A-I.
Hướng dẫn
D-aldotetrose duy nhất khi oxid hoá bằng HNO3 tạo thành sản phẩm không quang
hoạt là:

Khi xử lý với HCN, sau đó là dung dịch Ba(OH)2, A tạo thành B và C

Các -aldolactone D và E tương ứng là

1053 | Câu hỏi lí thuyết Hóa học hữu cơ OlympiaVN


Cho hỗn hợp cân bằng trên phản ứng với Na-Hg và nước ở pH 3-5, tạo thành F
và G. Oxid hoá F, G bằng nitric acid lần lượt tạo thành các aldaric acid H (không
quang hoạt) và I (quang hoạt).

1054 | Câu hỏi lí thuyết Hóa học hữu cơ OlympiaVN


Bài 33
Tiến hành phân cắt một monosaccarit X bằng HIO4 thu được hỗn hợp gồm
malondiandehit, fomandehit và axit formic theo tỉ lệ 1:1. X tham gia vào quá trình
chuyển hóa như sau:

a) Xác định cấu trúc các chất chưa biết.


b) Viết bốn công thức cấu tạo có thể có của E. Biết E tồn tại ở dạng vòng
sáu cạnh.
Hướng dẫn
Từ sản phẩm cắt mạch X suy ra cấu trúc mạch hở của A sẽ là HOC – CH2
– CHOH – CHOH – CH2OH và như vậy cấu trúc mạch vòng của X và của các
chất A, B, C, D sẽ như sau.

Bốn công thức cấu tạo có thể có của E:

1055 | Câu hỏi lí thuyết Hóa học hữu cơ OlympiaVN


Bài 34
Đường phân (glycolysis) là con đường sinh tổng hợp, phân cắt các chất đường
thành ATP, nguồn năng lượng của các tế bào sống. Một trong các giai đoạn then
chốt trong quá trình đường phân là sự chuyển hóa α-D-glucose-6-phosphate thành
β-D-fructose-6-phosphate. Mặc dù trong các tế bào, quá trình này được xúc tác
bởi enzyme glucose-6-phosphate isomerase nhưng chuyển hóa này cũng có thể
diễn ra ngoại bào (với tốc độ chậm hơn nhiều) trong các môi trường hơi có tính
acid. Hãy trình bày cơ chế quá trình ngoại bào.

1056 | Câu hỏi lí thuyết Hóa học hữu cơ OlympiaVN


Hướng dẫn

1057 | Câu hỏi lí thuyết Hóa học hữu cơ OlympiaVN


Di-, tri- và polysaccharide
Bài 1
1) Xác định các disaccharide sau có phải đường khử hay không:

2) Isomaltose có cấu trúc tương tự với maltose, ngoại trừ nó có liên kết 1→6 α-
glycoside, thay vì 1→4 α-glycoside. Vẽ cấu trúc của isomaltose.
3) Vẽ cấu trúc sản phẩm tạo thành khi xử lí disaccharide sau với NaBH4 trong
methanol:

4) Dự đoán sản phẩm tạo thành khi xử lí cellobiose với các tác nhân sau:
(a) NaBH4, H2O
(b) Br2, H2O (pH=6)
(c) CH3OH, HCl
(d) Ac2O, pyridine

5) Xác định các tác nhân bạn có thể sử dụng để chuyển β-D-glucopyranose thành
các hợp chất sau:

1058 | Câu hỏi lí thuyết Hóa học hữu cơ OlympiaVN


6) Trình bày cơ chế phản ứng thủy phân xúc tác acid của maltose thành hai phân
tử glucose.
Hướng dẫn
1) (a) Một trong các vị trí anomer (phía dưới bên phải) có một nhóm OH. Do đó,
disaccharide này là đường khử.
(b) Cả hai vị trí anomer đều có nhóm acetal, do đó disaccharide này không phải
đường khử.
(c) Tương tự ý b, không phải đường khử.
2)

3) Một trong các vòng (phía dưới bên phải) có một nhóm OH.

1059 | Câu hỏi lí thuyết Hóa học hữu cơ OlympiaVN


4) Một trong các vòng (phía dưới bên phải) có một nhóm OH anomer.
a)

b)

c)

d)

5)
(a) CH3OH, HCl
(b) CH3OH, HCl
(c) HNO3, H2O, heat
(d) CH3I, Ag2O dư, sau đó bởi H3O+

1060 | Câu hỏi lí thuyết Hóa học hữu cơ OlympiaVN


6)

1061 | Câu hỏi lí thuyết Hóa học hữu cơ OlympiaVN


Bài 2
Xác định các hợp chất kí hiệu bởi chữ cái trong các phản ứng sau:

Hướng dẫn

1062 | Câu hỏi lí thuyết Hóa học hữu cơ OlympiaVN


Bài 3
1) Đề xuất (các) cấu trúc vòng có thể có, chỉ rõ hóa lập thể, của (D)-tagalose
trong dung dịch theo công thức chiếu Haworth.

2) Hai sản phẩm có cùng công thức phân tử C6H10O6 được tạo thành khi xử lí
(D)-arabinose với sodium cyanide trong môi trường acid, sau đó thủy phân
acid. Viết các cấu trúc có thể có, chỉ rõ hóa lập thể, của hai hợp chất này và
cho biết chúng được tạo thành như thế nào.

3) Khi một disaccharide khử, turanose, tham gia vào quá trình thủy phân thù thu
được D-glucose và D-fructose với lượng đẳng mol với saccharide đã dùng.
Methyl hóa turanose với methyl iodide khi có mặt silver oxide, sau đó thủy
phân, thu được 2,3,4,6-tetraO-methyl-D-fructose. Đề xuất cấu trúc có thể có
của turanose, không cần quan tâm đến hóa lập thể ở (các) vị trí anomeric.

1063 | Câu hỏi lí thuyết Hóa học hữu cơ OlympiaVN


Hướng dẫn
1)

2)

3)

1064 | Câu hỏi lí thuyết Hóa học hữu cơ OlympiaVN


Bài 4
Ở Hà Lan, lactose (đường sữa) được sản xuất trên diện tương đối rộng từ váng
sữa (một sản phẩm phụ trong công nghiệp sản xuất phomat). Lactose được dùng
trong thực phẩm trẻ em và cả trong thuốc viên. Nó là hợp chất dissaccharide của
các monosaccharide D-galactose và D-glucose. Cấu trúc của nó (công thức chiếu
Haworth) được cho dưới đây. Đơn vị monosaccharide bên trải là D-galactose.

1) Vẽ công thức chiếu Fischer của D-galactose và D-glucose.


Phản ứng thủy phân xúc tác acid của lactose tạo thành D-galactose và D-glucose.
2) Sử dụng mũi tên, chỉ ra trong công thức lactose: (a) nguyên tử oxygen nào sẽ
bị proton hóa để thủy phân hiệu quả; (b) liên kết carbon-oxygen nào sẽ bị
phân cắt trong phản ứng thủy phân; (c) nguyên tử carbon nào sẽ tham gia vào
phản ứng với tác nhân Fehling (tác nhân này được sử dụng để nhận biết đường
khử).
Quá trình thủy phân lactose có thể được ghép cặp với quá trình hydrogen hóa, sử
dụng xúc tác kim loại, dẫn đến sự tạo thành các polyalcohol sorbitol và galactitol,
còn được biết đến là glucitol và dulcitol.
3) Vẽ công thức chiếu Fischer của sorbitol và galactitol. Cho biết các hợp chất
này có quang hoạt hay không.
Trong các quá trình công nghiệp, lactose bị đồng phân hóa thành lactulose, là một
dược phẩm đường ruột. Phản ứng hydrogen hóa lactose tạo thành một C12-polyol
là lactitol, đây là một chất làm ngọt ít calor. Cả hai quá trình này đều được tiến
hành ở Hà Lan.
4) a) Vẽ công thức Haworth của lactulose. (Gợi ý: Phần glucose của lactose bị
đồng phân hóa thành đường keto fructose); b) Vẽ công thức Haworth của
lactitol.
Hướng dẫn
1)

1065 | Câu hỏi lí thuyết Hóa học hữu cơ OlympiaVN


CHO CHO
H OH H OH
HO H HO H
H OH HO H
H OH H OH
CH 2OH CH 2OH

D-glucose D-galactose
2)

a
c

CH 2OH OH
OH O
O
OH
OH OH
O
b
OH OH

3)
CH 2OH CH 2OH
H OH H OH
HO H HO H
H OH HO H
H OH H OH
CH 2OH CH 2OH

Sorbitol (quang hoạt) Galactitol (không quang hoạt)


4)
CH 2OH OH CH 2 OH
OH O OH O O OH
O
OH
OH CH 2OH OH OH CH 2OH
OH O
OH OH OH CH 2 OH

Lactitol Lactolose (f uranose)


CH 2OH OH
OH O CH2 OH
O OH
OH OH O

OH
Lactolose (pyranose)

1066 | Câu hỏi lí thuyết Hóa học hữu cơ OlympiaVN


Bài 5
Disaccharide A (C12H22O12) có phản ứng âm tính với dung dịch thuốc thử
Benedict và không bị chuyển quay. A bị thuỷ phân bởi enzyme α-glucosidases
nhưng không bị thuỷ phân bởi β-glucosidases. Methyl hoá rồi thuỷ phân, tạo ra
hai đương lượng 2,3,4,6-tetra-O-methyl-D-glucose.
a) Xác định công thức cấu tạo của A.
b) 1 mol A phản ứng được với bao nhiêu mol periodic acid?
c) Có bao nhiêu mol formaldehyde và bao nhiêu mol formic acid tạo thành trong
phản ứng của 1 mol A với periodic acid?
Hướng dẫn
a) Từ các dữ kiện, ta thấy:
- Hợp chất A là một đường không khử với cầu  ở nguyên tử carbon anomer.
- Thuỷ phân A chỉ tạo thành 2,3,4,6-tetra-O-methyl-D-glucose.
do vậy cấu trúc duy nhất của A là gồm hai đơn vị glucose nối với nhau ở vị trí
carbon anomer tương ứng.

b, c) Cần 10 mol HI để oxid hoá-cắt mạch hoàn toàn A, tạo thành 10 mol formic
acid và 2 mol formaldehyde.

1067 | Câu hỏi lí thuyết Hóa học hữu cơ OlympiaVN


Bài 6
1) Suy ra cấu trúc của disaccharide isomaltose từ dữ kiện sau:
- Thủy phân chỉ thu được D-glucose.
- Isomaltose bị phân cắt bởi các enzyme α-glycosidase.
- Isomaltose là đường khử.
- Methyl hóa với CH3I dư, Ag2I sau đó thủy phân với H3O+ tạo thành
hai sản phẩm:

(mỗi sản phẩm có cả hai anomer)


2) Suy ra cấu trúc của disaccharide trehalose từ dữ kiện sau. Trehalose là “đường
huyết” của thế giới côn trùng. Nó được tìm thấy trong bào tử vi khuẩn, nấm
và nhiều loại côn trùng có môi trường tự nhiên vơi sự biến đổi nhiều về nhiệt
độ.
- Thủy phân chỉ tạo thành D-glucose.
- Trehalose bị thủy phân bởi các enzyme α-glycosidase.
- Trehalose là đường không khử.
- Methyl hóa với lượng dư CH3I, Ag2O, sau đó thủy phân với H3O+ chỉ
tạo thành một sản phẩm:

3) Xác định cấu trúc trisaccharide X từ các thông tin dưới đây:
- Methyl hóa với CH3I dư, Ag2O sau đó thủy phân với H3O+ tạo thành
ba sản phẩm. Mỗi chất đều tương ứng với hai epimer.

- X bị phân cắt bởi enzyme β-glycosidase tạo thành một disaccharide


và D-galactose.
- X bị phân cắt bởi enzyme α-glycosidase tạo thành một disaccharide
và D-fructose.
4) Thủy phân disaccharide primeverose tạo thành D-xylose và D-glucose.
Methyl hóa primeverose bởi dimethyl sulfate sau đó thủy phân acid êm dịu

1068 | Câu hỏi lí thuyết Hóa học hữu cơ OlympiaVN


tạo thành 2,3,4-tri-O-methyl-D-xylose và 2,3,4-tri-O-methyl-D-glucose. Xác
định cấu trúc primeverose dựa vào những dữ kiện trên.
Hướng dẫn
1)

2)

1069 | Câu hỏi lí thuyết Hóa học hữu cơ OlympiaVN


3)

4)

1070 | Câu hỏi lí thuyết Hóa học hữu cơ OlympiaVN


Bài 6A
1) Suy ra cấu trúc của dissaccharide melibiose từ các dữ kiện sau:
- Melibiose là một đường khử bị nghịch đảo đường và tạo thành một
phenylosazone.
- Thủy phân melibiose với acid hoặc với α-galactosidase tạo thành D-
galactose và D-glucose.
- Oxid hóa melibiose bởi nước bromine tạo thành melibionic acid. Methyl
hóa melibionic acid sau đó thủy phân tạo thành 2,3,4,6-tetra-O-methyl-
D-galactose và 2,3,4,5-tetra-O-methyl-D-gluconic acid.
- Methyl hóa melibiose sau đó thủy phân tạo thành2,3,4,6-tetra-O-
methyl-D-galactose và 2,3,4-tri-O-methyl-D-glucose.
2) Stachyose tồn tại trong rễ nhiều loại thực vật. Hãy suy luận cấu trúc của
stachyose từ các dữ kiện sau:
- Thủy phân acid 1 mol stachyose tạo thành 2 D-galactose, 1 mol D-
glucose, và 1 mol D-fructose.
- Stachyose là đường không khử.
- Xử lí stachyose với α-galactosidase tạo thành hỗn hợp gồm D-galactose,
sucrose, và một trisaccharide không khử gọi là raffinose.
- Thủy phân acid raffinose tạo thành D-glucose, D-fructose, và D-
galactose. Xử lí raffinose với α-galactosidase tạo thành D-galactose và
sucrose. Xử lí raffinose với invertase (một enzyme thủy phân sucrose)
tạo thành fructose và melibiose (xem ý 1).
- Methyl hóa stachyose, sau đó thủy phân tạo thành 2,3,4,6-tetra-O-
methyl-D-galactose, 2,3,4-tri-O-methyl-D-galactose, 2,3,4-tri-O-methyl-
D-glucose, và 1,3,4,6-tetra-O-methyl-D-fructose.
Hướng dẫn
1)
- Melibiose là một đường khử và bị nghịch đảo đường và tạo thành một
phenylosazone - những dữ kiện này cho thấy một monosaccharide hiện
diện ở dạng hemiacetal vòng.
- Thủy phân acid tạo thành D-galactose và D-glucose cho thấy melibiose
là một disaccharide tạo thành từ một đơn vị D-galactose và một đơn vị
D-glucose. Melibiose bị thủy phân bởi α-galactosidase cho thấy nó là một
α-D-galactosyl-D-glucose.
- Oxid hóa melibiose thành melibionic acid và sau đó thủy phân tạo thành
D-galactose và D-gluconic acid cho thấy rằng đơn vị glucose hiện diện ở
dạng hemiacetal vòng và đơn vị galactose ở dạng glycoside (Nếu điều
ngược lại là đúng thì thí nghiệm này sẽ tạo thành D-glucose và D-
galactonic acid.)
Sự methyl hóa và thủy phân melibionic acid tạo thành 2,3,4,6-tetra-O-
methyl-D-galactose và 2,3,4,5-tetra-O-methyl-D-gluconic acid. Sự tạo
thành sản phẩm đầu tiên - một dẫn xuất galactose với nhóm OH tự do ở

1071 | Câu hỏi lí thuyết Hóa học hữu cơ OlympiaVN


C5 - chứng minh rằng vòng galactose có sáu cạnh; sự tạo thành sản phảm
thứ hai - dẫn xuất gluconic acid với nhóm OH tự do ở C6 - chứng minh
rằng oxygen ở C6 của đơn vị glucose tham gia vào một cầu glycoside với
đơn vị galactose.
- Sự methyl hóa và thủy phân melibiose tạo thành một dẫn xuất glucose
(2,3,4-tri-O-methyl-D-glucose) với các nhóm OH tự do ở C5 và C6 cho
thấy vòng glucose cũng có sáu cạnh. Do đó, melibiose là 6-O-(α-D-
galactopyranosyl-D-glucopyranose.)

6-O-(α-D-Galactopyranosyl)-D-glucopyranose
2) Stachyose có cấu trúc như sau:

Raffinose có cấu trúc như sau:

1072 | Câu hỏi lí thuyết Hóa học hữu cơ OlympiaVN


Các quá trình thủy phân enzyme (như đã được chỉ ra ở trên) cho biết cấu trúc cơ
bản của stachyose và raffinose. Câu hỏi duy nhất còn lại là kích thước vòng của
đơn vị galactose đầu tiên trong stachyose. Sự methyl hóa stachyose sau đó thủy
phân tạo thành 2,3,4,6-tetra-O-methyl-D-galactose xác thực đây là vòng
pyranoside.

1073 | Câu hỏi lí thuyết Hóa học hữu cơ OlympiaVN


Bài 7
Disaccharide A bị thuỷ phân bởi acid loãng, tạo thành hỗn hợp D-glucose và D-
galactose. Hợp chất A là đường khử, bị oxid hoá bởi nước bromine tạo thành acid
B, chất này bị methylate hoá bởi sodium hydroxide và dimethylsulfate, tạo thành
một hợp chất octa-O-methylate. Thuỷ phân chất này, thu được tetra-O-
methylgluconic acid (C) và tetra-O-methylgalactose (D). Oxid hoá C bằng nitric
acid, tạo thành tetra-O-methylglucaric acid. C cũng được tạo thành từ phản ứng
thuỷ phân (xúc tác acid) methyl 2,3,4,6-tetra-O-methylgalactopyranoside. Hợp
chất A bị thuỷ phân bởi enzym α-galactosidase (được chiết tách từ hạnh nhân).
Xác định cấu trúc các chất A, B, C, D.
Hướng dẫn

A là 6-O-α-D-galactopyranosyl-D-glucopyranose
(Cầu nối α bị phá huỷ bởi α-galactosidase, tạo thành glucose và galatose. Trong
cấu trúc có nhóm hemiacetal nên là đường khử)

1074 | Câu hỏi lí thuyết Hóa học hữu cơ OlympiaVN


1075 | Câu hỏi lí thuyết Hóa học hữu cơ OlympiaVN
Bài 8
Khi thủy phân tinh bột bằng axit thường chỉ tạo thành các sản phẩm có vị ngọt
(glucozơ, mantozơ...). Khi thủy phân tinh bột trong công nghiệp (30 – 40% huyền
phù tinh bột trong dung dịch HCl loãng, nhiệt độ 130 – 140oC) một vài hợp chất
đắng được tạo thành. Người ta đã nghĩ rằng chất “đường đắng“ X này được tạo
thành do phản ứng thủy phân thuận nghịch. X đã được tách ra và tinh chế. Để xác
định cấu tạo của nó người ta đã tiến hành các thí nghiệm sau:
Thí nghiệm 1: Thủy phân trong axit chỉ thu được glucozơ
Thí nghiệm 2: 24,795 mg X được hòa tan trong 25 cm3 dung dịch kali
hexaxianoferat (III) K3[Fe(CN)6] 0,01 M. Sau đó thêm 5 cm3 dung dịch KOH 0,1
M và đưa hỗn hợp lên 100oC trong 15 phút. Lượng dưu K3[Fe(CN)6] được xác
định bằng phương pháp chuẩn độ iot. Thêm một lượng dư KI và hỗn hợp được
axit hóa bằng CH3COOH. Chuẩn độ bằng Na2S2O3 0,01 M thấy tốn hết 10,5 cm3
Thí nghiệm 3: X được metyl hóa bằng một lượng dư metyl iodua có mặt bạc (I)
oxit. Sau đó sản phẩm metyl hóa được đun nóng với HNO3 30%. Hai axit được
tách ra từ hỗn hợp là axit 2,3,4-trimetoxy-5-hydroxiadipic HOOC – CHOH –
[CH(OCH3)]3 – COOH và axit 2,3,4,6-tetrametoxy-5-hydroxicaproic CH3OCH2
– CHOH - [CH(OCH3)]3 – COOH.
1. Xác định cấu tạo của X
2. Viết các phương trình phản ứng trong thí nghiệm nhận biết X
Hướng dẫn
1. Cấu tạo của X: 6-O--D-glucopyranosyl-D-glucozơ (gentiobiozơ)

2. Các phản ứng xảy ra:


Thí nghiệm 1:

Thí nghiệm 2:

1076 | Câu hỏi lí thuyết Hóa học hữu cơ OlympiaVN


2[Fe(CN)6]3- + 2I-  2[Fe(CN)6]4- + I2
I2 + 2Na2S2O3  Na2S4O6 + 2NaI
n(Na2S2O3) ban đầu = 0,0105.0,0100 = 0,0105 mmol
n(K3[Fe(CN)6] ban đầu = 0,250 mmol
Vậy số mol K3[Fe(CN)6] tham gia vào phản ứng oxy hóa = 0,145 mmol  MX =
342 g/mol
Như vậy X rõ ràng là một disaccarit do M(Glu2) = 180.2 – 18 = 342 g/mol
Thí nghiệm 3:

Sự hiện diện của axit 2,3,4-trimetoxy-5-hydroxiadipic HOOC – CHOH –


[CH(OCH3)]3 – COOH trong hỗn hợp sản phẩm chứng tỏ trong disaccarit tồn tại
liên kết 1  6 glycozit.

1077 | Câu hỏi lí thuyết Hóa học hữu cơ OlympiaVN


Bài 9
Trong truyền máu thường quy, việc xác định nhóm máu là rất cần thiết. Việc trộn
lẫn các loại máu không tương thích có thể gây ra sự ngưng kết các tế bào hồng
cầu và dẫn đến tử vong. Có bốn nhóm máu đã biết: A, B, AB và O. Phân loại
nhóm máu phù thuộc vào thành phần của các oligosaccharide liên kết với bề mặt
protein của các tế bào hồng cầu.
Phần trung tâm của saccharide luôn là galactose được liên kêt với protein:

Galactose cũng liên kết với:


- Đường C bởi liên kết α-1,2’ trong máu nhóm O.
- Đường C bởi liên kết α-1,2’ và đường A bởi liên kết α-1,3’ trong máu
nhóm A.
- Đường C bởi liên kết α-1,2’ và đường B bởi liên kết α-1,3’ trong máu
nhóm B.
Xác định đường A: Đường A phản ứng với thuốc thử Tollens (thí nghiệm tráng
bạc) tạo thành hợp chất dưới đây:

Xác định đường B: Đường B phản ứng với hydroxylamine tạo thành hợp chất X,
chất này được đun nóng tiếp với acetyl anhydride khi có mặt sodium acetate tạo
thành hợp chất Y. Phản ứng thủy phân base hợp chất Y tạo thành đường Z có ít
hơn một nguyên tử carbon so với đường gốc B. Đường Z được xử lí với các tác
nhân như đường B tạo thành D-threose.
Các chuyển hóa như trên được mô tả theo sơ đồ dưới đây:

1078 | Câu hỏi lí thuyết Hóa học hữu cơ OlympiaVN


Hàm lượng các nguyên tố trong hợp chất Y là 49.6% C, 41.3% O, 3.6% N và
phần còn lại là hydrogen. Biết rằng khi oxid hóa đường B với HNO3 tạo thành
sản phẩm không quang hoạt
Xác định đường C: Đường C thuộc dãy L. Cấu hình tuyệt đối của nguyên tử
carbon thứ ba (C-3) là R. Khi đường C bị khử trong các điều kiện để chuyển hóa
nhóm CHO thành methyl thì tạo thành hợp chất meso. Phản ứng giữa 1 mol C và
HIO4 tạo thành 1 mol CH3CHO và 4 mol HCOOH.
Câu hỏi:
A. Vẽ công thức chiếu Fischer của:
1) Đường A.
2) Đường B và các dẫn xuất X, Y, Z.
3) Đường C.
Giải thích cấu trúc của các hợp chất B, Y, C.
B. Các monosaccharide tạo thành vòng pyranose 6 cạnh. Vẽ công thức chiếu
Haworth của:
1) disaccharide từ máu nhóm O.
2) trisaccharide từ máu nhóm A.
3) trisaccharide từ máu nhóm B.

1079 | Câu hỏi lí thuyết Hóa học hữu cơ OlympiaVN


Hướng dẫn
A.1) Trong phản ứng Tollens, nhóm aldehyde bị oxid hóa thành nhóm carboxylic.
Cấu trúc của sản phẩm trong thí nghiệm tráng gương đã được biết, do đó cấu trúc
của A phải là:

2) Sản phẩm của phản ứng thoái phân hai lần của đường B là D-threose, do đó
cấu trúc mảnh cuối là:

Đường B bị oxid hóa thành aldaric acid không quang hoạt khi có mặt nitric acid.
Một aldaric acid không quang hoạt khi nó có một mặt phẳng đối xứng. Chỉ có
duy nhất một cấu trúc aldaric acid với mặt phẳng đối xứng với mảnh cuối phân
tử đã cho:

Do cấu trúc của aldaric acid đã biết nên có thể xác định cấu trúc của đường B:

1080 | Câu hỏi lí thuyết Hóa học hữu cơ OlympiaVN


Sản phẩm phản ứng của đường B với hydroxylamine là một oxime X:

Công thức phân tử của hợp chất Y là:

Số nguyên tử carbon và oxygen thể hiện rằng tất cả các nhóm hydroxyl đã bị
acetyl hóa trong phản ứng với acetyl anhydride. Cũng quan sát thấy phản ứng
tách nước nhóm oxime thành nitrile. Biết tất cả các dữ kiện này, chúng ta cố thể
xác định cấu trúc hợp chất Y:

Đường Z có ít hơn 1 carbon so với đường gốc B, do đó cấu trúc của nó phải là:

1081 | Câu hỏi lí thuyết Hóa học hữu cơ OlympiaVN


3) Trong phản ứng khử, nhóm CHO bị chuyển thành CH3 và tạo thành hợp chất
meso. Điều này có nghĩa rằng nguyên tử carbon cuối cùng trong đường gốc C
được gắn vào nhóm CH3. Điều này được chứng minh bởi phản ứng với HIO4 bởi
CH3CHO đã được xác định là một trong các sản phẩm. Cấu hình tuyệt đối của
nguyên tử carbon thứ ba (C-3) là R và đường thuộc dãy , do đó hợp chất phải chứ
các mảnh được biểu diễn dưới đây:

Có duy nhất một cấu trúc của sản phẩm phản ứng khử với mặt phẳng đối xứng:

Vậy cấu trúc của đường C như sau:

1082 | Câu hỏi lí thuyết Hóa học hữu cơ OlympiaVN


B. Sơ đồ chung về sự gắn kết saccharide:

Các saccharide trong máu nhóm O:

Các saccharide trong máu nhóm A và B:

1083 | Câu hỏi lí thuyết Hóa học hữu cơ OlympiaVN


Máu nhóm A: R = NHCOCH3; máu nhóm B: R= OH

1084 | Câu hỏi lí thuyết Hóa học hữu cơ OlympiaVN


Bài 10
Hợp chất X là một trisaccharide tồn tại chủ yếu trong bột hạt bông. X không phản
ứng với các dung dịch Benedict hoặc Fehling, nó cũng không bị chuyển quay.
Thủy phân X xúc tác acid tạo thành 3 D-hexose khác nhau: A, B, C. Các hợp chất
A và B, cũng như hợp chất 1 (dưới đây), đều tạo thành osazone giống nhau khi
phản ứng với lượng dư phenylhydrazine. Hợp chất C phản ứng với nitric acid tạo
thành hợp chất D không quang hoạt. Phương pháp Kiliani-Fischer được sử dụng
để thiết lập tương quan về cấu hình giữa D-glyceraldehyde và C. Aldotetrose
trung gian tạo thành C khi bị oxid hóa bởi nitric acid không tạo thành hợp chất
meso. Khi xử lí A với nitric acid thì dicarboxylic acid (aldaric acid) tạo thành có
tính quang hoạt. Cả A và B đều phản ứng với 5 mol HIO4, 1 mol A tạo thành 5
mol methanoic (formic) acid và 1 mol methanal (formaldehyde), trong khi đó 1
mol B tạo thành 4 mol methanoic (formic) acid và 1 mol methanal (formaldehyde)
và 1 mol carbon dioxide. Cả A và B đều có liên quan với cùng một aldotetrose là
đồng phân dia của một trong các chất mà C có liên quan. Khi methyl hóa X, sau
đó thủy phân sẽ tạo thành một 2,3,4-tri-O-methyl-D-hexose (E) (có nguồn gốc từ
A), một 1,3,4,6-tetra-O-methyl-D-hexose (F) (có nguồn gốc từ B), và một 2,3,4,6-
tetra-O-methyl-D-hexose (G) (có nguồn gốc từ C).

1) Vẽ công thức chiếu Fischer của A, B, C, D.


2) Hãy hoàn thành công thức chiếu Haworth để chỉ rõ kích thước vòng và hóa
lập thể [cấu hình] tuyệt đối của E, F, G.
3) Xác định trình tự các monosaccharide trong trisaccharide X.

1085 | Câu hỏi lí thuyết Hóa học hữu cơ OlympiaVN


Hướng dẫn
1) Hợp chất X là một trisaccharide không phản ứng với dung dịch Benedict, và
cũng không chuyển quay. Thông tin này cho thấy X là một đường không-
khử, do đó chỉ có các cầu acetal hoặc ketal tồn tại ở tất cả các nguyên tử
anomeric carbon. Trong số 3 monosaccharide thì A và B tạo thành osazone
giống nhau, do đó có hóa lập thể ở C3, C4, C5 (và C6) giống nhau. A và B
cũng khác với hợp chất 1 (chính là D-mannose) - chất tạo thành cùng osazone,
do đó 1 trong số chúng phải là đồng phân epimer C2 của D-mannose (nghĩa
là D-glucose) và chất còn lại là đường keto tương ứng ở C2 (nghĩa là D-
fructose). (Kết luận này được xác thực sau ở các phản ứng oxid hóa cắt mạch.)
Hợp chất C, sau phản ứng với nitric acid, tạo thành một aldaric acid D không
quang hoạt. Hai aldaric acid có thể là D là:

Aldotetrose là tiền chất của C (và cũng là của D) không tạo thành hợp chất meso
sau phản ứng với nitric acid, do đó phải là D-threose:

Theo đó thì aldaric acid D tạo thành từ C ở trên là AA1 và do đó C phải là D-


galactose. Hợp chất A phản ứng với 5 mol HI tạo thành 5 mol methanoic (formic)
acid và 1 mol methanal (formaldehyde), cho thấy rằng nó là aldohexose, trong
khi đó B phản ứng với 5 mol HI tạo thành 4 mol methanoic (formic) acid, 1 mol
methanal (formaldehyde) và 1 mol CO2 cho thấy rằng nó là ketohexose.

1086 | Câu hỏi lí thuyết Hóa học hữu cơ OlympiaVN


Hợp chất A và B đều liên quan đến tetrose giống nhau, chất này khác với chất
liên quan đến C (nghĩa là liên quan đến D-erythrose). Tetrose liên quan đến A và
B phải có cấu trúc như sau và theo đó thì A là D-glucose và B là D-fructose.

2) Methyl hóa X, sau đó thủy phân, tạo thành các chất E, F, G sau đây:

3) Trong quá trình methyl hóa, chỉ có các nhóm hydroxyl không tham gia tạo
thành acetal/ketal (nội hoặc liên phân tử) thì mới bị ester hóa. Từ dữ kiện
methyl hóa, chỉ có E có 2 nhóm hydroxyl liên kết với các carbohydrate khác.

1087 | Câu hỏi lí thuyết Hóa học hữu cơ OlympiaVN


Do đó, A phải là carbohydrate trung tâm. Các kết quả này chỉ ra rằng, chuỗi
sắp xếp các monosaccharide trong X là C-A-B (hoặc B-A-C).
Nếu: A5 biểu diễn furanose (vòng 5 cạnh) tạo thành carbohydrate A. A6 biểu diễn
pyranose (vòng 6 cạnh) tạo thành carbohydrate A. B5 biểu diễn furanose (vòng 5
cạnh) tạo thành carbohydrate B, … thì trisaccharide X có thể được biểu diễn theo
kiểu như là: C6-A6-B5. Có 4 biến thể như vậy và dưới đây là một trong số đó.
Chú ý: Bản chất của các liên kết anomeric không được xác định trong bài này. Sự
sắp xếp liên kết của A với B và C cũng có thể bi nghịch đảo (nghĩa là liên kết 1,1’
giữa C và A và liên kết 1,6 giữa A và B.)

1088 | Câu hỏi lí thuyết Hóa học hữu cơ OlympiaVN


Bài 11
Thủy phân -chaconin (C45H73NO14) (thành phần chính trong cây họ cà và khoai
tây) bằng enzym β-glicoziđaza thu được A (C18H32O14) và alkaloid solanidin
C27H43NO). Xử lý A bằng AcOH/DMF thu được A1 chỉ chứa vòng sáu cạnh, A1
không phản ứng với (Ag2O/NH3). Metyl hóa hoàn toàn A1 bằng MeI/Ag2O thu
được A2. Thủy phân A2 bằng xúc tác axit thu được B (C8H16O6), C1 (C9H18O5)
và C2 (C8H16O5). Oxy hóa B bằng HIO4 thu được D. Tiến hành cắt mạch D bằng
dung dịch NaOH thu được HCOONa, metoxietanal, (R)-2-hydroxi-3-
metoxipropanal. Oxy hóa C1 bằng HNO3 thu được chủ yếu axit (2R, 4R)-2,3,4-
trimetoxipentadioic. Oxy hóa C2 bằng HIO4, sau đó bằng HNO3 cho chủ yếu là
axit (2R, 3R)-2,3-dimetoxibutandioic. Biết rằng B là một dẫn xuất của glucozơ,
C1 và C2 là hai dẫn xuất của cùng một đường có cấu hình L. Hãy vẽ công thức
Haworth của tất cả các chất chưa biết có trong bài. Biết alkaloid solanidin có cấu
trúc như sau:

Hướng dẫn
Vì G là dẫn xuất của glucozơ, C1 và C2 là hai dẫn xuất của cùng một đường có
cấu hình L và S có công thức phân tử C18H32O14 có thể dự đoán S được tạo thành
từ hai liên kết glycozit (tách hai phân tử nước) từ một phân tử glucozơ (C6H12O6)
và hai phân tử đường có cấu hình L có công thức C6H12O5.
B là dẫn xuất của D-glucozơ, oxy hóa G bằng HIO4 thu được D, cắt mạch D bằng
NaOH thu được HCOONa, metoxyetanal, (R)-2-hydroxi-3-metoxypropanal theo
phản ứng retro aldol và thủy phân este. Như vậy có thể xác định được cấu trúc B.

Oxy hóa C1 bằng HNO3 thu được chủ yếu axit (2R, 4R)-2,3,4-
trimetoxypentandioic, axit meso-2,3-dimetoxybutandioic và axit axetic. Do C1 và
C2 là hai dẫn xuất của cùng một đường có cấu hình L nên cấu hình của nhóm C3
– OH của C1 tương đồng với cấu hình nhóm C3 – OH trong C2. Mặt khác oxy
hóa C2 bằng HIO4, sau đó bằng HNO3 thu được axit (2R, 3R)-2,3-
dimetoxybutandioic nên có thể xác định được lập thể các nhóm OH và bộ khung

1089 | Câu hỏi lí thuyết Hóa học hữu cơ OlympiaVN


carbon. Từ đó có thể xác định hai đường C1 và C2 đều có chung bộ khung
rhamanozơ. Như vậy cấu trúc của C1 và C2 như sau:

Từ cấu trúc của B, C1, C2 cùng với A (C18H32O14) có thể thấy rằng trong ba liên
kết glycozit chỉ có một liên kết β-glycozit giữa A và solanidin và hai liên kết -
glycozit giữa các đường. Từ vị trí các nhóm OH tự do của B, C1 và C2, cũng như
A1 không cho phản ứng với Ag2O/NH3 nên nhóm OH anome của các hợp phần
A và B sẽ tạo liên kết glycozit vòng 6 khi xử lý với AcOH/DMF. Từ đó có thể
xác định được cấu trúc các hợp phần còn lại như sau:

1090 | Câu hỏi lí thuyết Hóa học hữu cơ OlympiaVN


Bài 12
Một trisaccarit thiên nhiên X được tạo thành từ ba monosaccarit A, B, C. Khối
lượng phân tử của X là 504 g/mol. Metyl hóa X bằng metyl iodua rồi thủy phân
trisaccarit đã được metyl hóa này trong điều kiện khắc nghiệt (1M H2SO4, 100oC)
và trong điều kiện êm dịu hơn (0,01M H2SO4) thì thu được các sản phẩm sau
Điều kiện khắc nghiệt: Một hỗn hợp gồm các dẫn xuất 2,3,4,6-tetrametyl; 2,3,4-
trimetyl và 1,3,4,6-tetrametyl monosaccarit
Điều kiện êm dịu: Dẫn xuất 1,3,4,6-tetrametyl của C và disaccarit Y đã bị metyl
hóa
Disaccarit Y được xử lý tiếp với metyl iodua và thủy phân trong điều kiện khắc
nghiệt cho sản phẩm 2,3,4,6-tetrametyl và 1,2,3,4-tetrametyl monosaccarit.
Thủy phân hoàn toàn chất X rồi cô lập từng sản phẩm thu được đem phân tích
cho kết quả monosaccarit C đồng nhất với sản phẩm thủy phân saccarozơ. Bên
cạnh đó B và C có thể chuyển hóa qua lại trong môi trường kiềm.
Để nhận diện B thì người ta tiến hành quá trình tổng hợp cyanohydrin. Sản phẩm
thu được từ quá trình này bị oxy hóa bởi axit nitric đặc để cho một axit
dicarboxylic D quang hoạt và một axit dicarboxylic E không quang hoạt.
Sự oxy hóa A bằng axit nitric đặc cho axit dicarboxylic không quang hoạt F. A
có thể được tổng hợp từ D-threose sau hai quá trình tổng hợp cyanohydrin liên
tiếp.
Trong trisaccarit X thì B liên kết với C, còn A được liên kết với B bằng một liên
kết -glycosid. Thứ tự liên kết đi từ trái sang phải được xác định là A – B – C.
Xác định A, B, C, X và Y bằng cách vẽ công thức Haworth của chúng

1091 | Câu hỏi lí thuyết Hóa học hữu cơ OlympiaVN


Hướng dẫn
C đồng nhất với sản phẩm thủy phân D-saccarozơ nên C phải là một hexozơ. Do
C và B có thể chuyển hóa qua lại cho nhau được nên B cũng phải là một hexozơ.
Do khối lượng phân tử của một mảnh CH2O là 30 thì số nguyên tử carbon của X
sẽ là (504 + 2.18) / 30 = 18 nguyên tử C.
Do B và C đã là hexozơ nên A cũng phải có 6 nguyên tử carbon.
Xử lý X bằng MeI rồi sau đó thủy phân trong điều kiện êm dịu cho sản phẩm
1,3,4,6-tetrametyl của chất C. Như vậy nhóm hydroxi ở vị trí C2 của C đã liên
kết với monosaccarit B, tức nhiều khả năng C là một xetozơ. Do C đồng nhất với
sản phẩm thủy phân saccarozơ nên C chỉ có thể là D-fructozơ.
Do B và C có thể chuyển hóa qua lại lẫn nhau trong môi trường kiềm nên B có
thể là D-glucozơ hay D-mannozơ. Tuy nhiên nếu tiến hành quá trình tổng hợp
cyanohydrin thì cả hai sản phẩm từ D-mannozơ đều quang hoạt (trái giả thiết) nên
B chỉ có thể là D-glucozơ. Cấu trúc hai chất D và E sẽ là:

Tiến hành hai quá trình cyanohydrin liên tiếp D-threozơ sẽ thu được bốn hexozơ,
trong đó chỉ có duy nhất D-galatozơ khi oxy hóa bằng axit nitric cho sản phẩm
không quang hoạt.
Dựa trên cấu trúc các sản phẩm metyl hóa thì C sẽ liên kết với B ở nguyên tử
carbon thứ hai, còn A liên kết với B qua nguyên tử carbon thứ nhất. B nối với C
qua nguyên tử carbon thứ nhất, và B nối với A qua nguyên tử carbon thứ sáu. Tức
cấu trúc hai đường X và Y sẽ là:

1092 | Câu hỏi lí thuyết Hóa học hữu cơ OlympiaVN


1093 | Câu hỏi lí thuyết Hóa học hữu cơ OlympiaVN
Bài 13
Inulin, chiết xuất từ rễ cây diếp xoăn ở Bỉ và Hà Lan,
được sử dụng làm phụ gia thực phẩm do nó có tác động
hữu ích đến hệ vi sinh đường ruột. Nó cũng được sử dụng
làm nguồn fructose - chất có độ ngọt gấp 1.9 lần sucrose,
và để sản xuất mannitol - chất được sử dụng trong kẹo
cao su. Inulin là một polymer mạch thẳng chứa các đơn
vị fructose với một đơn vị glucose ở đầu mạch. Công
thức chiếu Haworth của nó được cho ở hình bên. Trong
bài này, inulin được xét có 10 đơn vị fructose (n = 9).
1) Inulin có thể bị thủy phân trong môi trường H+. Hãy
cho biết liên kết C-O nào trong các phương án sau
đây dễ bị phân cắt nhất.

Quá trình thủy phân với nước đánh dấu đồng vị có thể cung cấp thông tin về cơ
chế thủy phân nhờ vào NMR - kĩ thuật hiện đại có thể “thấy” được deuterium
(2H) và đồng vị oxygen 17O.
2) Xác định phân tử nước đánh dấu nào là lựa chọn tốt nhất cho mục đích này.
□ 2H2O
□ H217O
□ 2H217O
□ Không chất nào trong số chúng.
Khi hydrogen hóa xúc tác, glucose tạo thành sorbitol (S), trong khi đó fructose
(F) tạo thành mannitol (M) và sorbitol (S).
3) Vẽ công thức chiếu Fischer của fructose (F), sorbitol (S) và mannitol (M).

1094 | Câu hỏi lí thuyết Hóa học hữu cơ OlympiaVN


Hướng dẫn
1) B
2) H217O
3)

1095 | Câu hỏi lí thuyết Hóa học hữu cơ OlympiaVN


Bài 14
1) Vẽ cấu trúc của:
(a) Polysaccharide được tạo thành bằng cách nối các đơn vị D-mannose qua
các cầu 1→4-β-glycoside;
(b) Polysaccharide được tạo thành bằng cách nối các đơn vị D-glucose qua
các cầu in 1→6-α-glycoside.
Polysaccharide ở ý (b) là dextran, một thành phần của mảng bám răng.
2) Xử lí chitin với H2O, OH- để thủy phân các cầu amide của nó, tạo thành hợp
chất gọi là chitosan. Xác định cấu trúc chitosan. Chitosan được dùng trong
các loại dầu gội, sợi để khâu và băng vết thương.
3) Vẽ cấu trúc các hợp chất sau:
a) Một polysaccharide được tạo thành bằng cách gắn các đơn vị D-
glucosamine bởi các cầu 1→6-α-glycoside.
b) Một disaccharide được tạo thành bằng cách gắn D-mannose và D-glucose
bởi cầu 1→4-β-glycoside.
c) Một α-N-glycoside được tạo thành từ D-arabinose và C6H5CH2NH2.
d) Một ribonucleoside tạo thành từ D-ribose và thymine.
Hướng dẫn
1)

1096 | Câu hỏi lí thuyết Hóa học hữu cơ OlympiaVN


2)

3)

1097 | Câu hỏi lí thuyết Hóa học hữu cơ OlympiaVN


Bài 15
1. Một nhóm các oligosaccarit có tính chất rất thú vị có thể được cô lập từ loài
vi khuẩn Bacillus macerans khi nuôi cấy chủng vi khuẩn này trong môi
trường giàu tinh bột. Tất cả các oligosaccarit này đều không khử. Thủy phân
hoàn toàn trong axit cho 6 – 8 phân tử glucozơ. Metyl hóa hoàn toàn rồi thủy
phân chỉ thu được duy nhất 2,3,6-tri-O-metyl-D-glucozơ. Hãy đề nghị một
cấu trúc tổng quát cho oliosaccarit này. Biết nó chỉ có thể phân cắt bằng
enzym α – glucozidaza.
2. Xác định cấu trúc của một pentose nếu biết rằng nó phản ứng với Ac2O cho
tetraaxetat, phản ứng được với thuốc thử Tollens và tác dụng với
phenylhydrazin cho sản phẩm không quang hoạt
Hướng dẫn
1. Việc oligosaccarit không có tính khử chứng tỏ nhiều khả năng nó có cấu trúc
vòng. Dựa trên sản phẩm metyl hóa cho thấy liên kết glycozit tạo thành giữa
C1 và C4. Mặt khác do chỉ bị thủy phân bởi α – glycozidaza chứng tỏ liên kết
là α – glycozit. Vậy cấu trúc chung của nhóm oligosaccarit này có dạng (n: 3
– 5).

2. Khi tác dụng với phenylhydrazin cho sản phẩm không quang hoạt tức sản
phẩm cuối không có C*. Điều này chỉ xảy ra khi hai nhóm CH2OH cùng gắn
trên một cacbon. Như vậy cấu trúc của pentose sẽ như sau:

1098 | Câu hỏi lí thuyết Hóa học hữu cơ OlympiaVN


Glycoside
Bài 1
1) Salicin là một chất giảm đau tự nhiên, có trong vỏ cây liễu và được sử dụng
từ hàng nghìn năm nay để xử lí các vết thương và hạ sốt.

a) Salicin có phải là đường khử không?


b) Xác định các sản phẩm tạo thành khi thủy phân salicin khi có mặt acid.
c) Salicin là α-glycoside hay β-glycoside?
d) Vẽ sản phẩm chính được tạo thành khi xử lí salicin với lượng dư acetic
acid khi có mặt pyridine.
e) Theo bạn thì salicin có thể hiện tính chất nghịch đảo đường khi hòa tan
trong nước trung tính không?
2) Trình bày cơ chế phản ứng sau:

3) Vẽ cấu trúc α-N-glycoside và the β-N-glycoside tạo thành khi xử lí D-glucose


với aniline (C6H5NH2).
4) Vẽ cấu trúc nucleoside tạo thành từ mỗi cặp hợp chất sau. Gọi tên nucleoside:
(a) 2-Deoxy-d-ribose và adenine
(b) d-Ribose và guanine
5) (a) Xác định tất cả các nguyên tử O thuộc phần glycoside trong rebaudioside
A. Rebaudioside A, được bán dưới tên thương mại Truvia, là một glycoside
ngọt, thu được từ cây stevia, loại cây đã được người Paraguay sử dụng trong
nhiều thế kỉ với mục đích làm ngọt thực phẩm.

1099 | Câu hỏi lí thuyết Hóa học hữu cơ OlympiaVN


(b) Alcohol được tạo thành từ sự thủy phân của một glycoside gọi là aglycon.
Aglycol và các monosaccharide nào được tạo thành bởi sự thủy phân
rebaudioside A?

1100 | Câu hỏi lí thuyết Hóa học hữu cơ OlympiaVN


Hướng dẫn
1)
a) Không phải đường khử do vị trí anomer có một nhóm acetal.
b) Nhóm acetal bị thủy phân, tạo thành hai anomer của hemiacetal vòng.

c) Salicine là β-glycoside.
d)

e) Không. Khi không có xúc tác acid thì nhóm acetal không dễ bị thủy phân.

1101 | Câu hỏi lí thuyết Hóa học hữu cơ OlympiaVN


2)

3)

4)

1102 | Câu hỏi lí thuyết Hóa học hữu cơ OlympiaVN


5)

1103 | Câu hỏi lí thuyết Hóa học hữu cơ OlympiaVN


Bài 2
1) Xác định sản phẩm của các phản ứng sau:

2) Trình bày cơ chế chuyển hóa β-D-glucose thành cả hai anomer của N-ethyl
glucopyranoside.

3) Aglycon và các monosaccharide nào được tạo thành khi thủy phân mỗi chất:
sacilin và solanine trong dung dịch acid?

4) Linamarin được tìm thấy trong sắn. Giải thích tại sao hợp chất này có độc
tính?

1104 | Câu hỏi lí thuyết Hóa học hữu cơ OlympiaVN


5) Vanillin được tìm thấy trong một β-glycoside của D-glucosoe. Vẽ cấu trúc
của glycoside.

6) Các phenyl glycoside được tạo thành từ phenol (trong tự nhiên hoặc trong
phòng thí nghiệm) như thế nào? Tại sao cấu hình của glycoside không liên
quan đến đường ban đầu?

1105 | Câu hỏi lí thuyết Hóa học hữu cơ OlympiaVN


Hướng dẫn
1)

2)

3)

1106 | Câu hỏi lí thuyết Hóa học hữu cơ OlympiaVN


4) Aglycone của linamarin là cyanohydrin của acetone. Cyanohydrin kém bền,
nó bị thủy phân thuận nghịch giải phóng HCN, là chất có độc tính cực mạnh.
5)

6) Hemiacetal tạo thành một oxonium ion phẳng có thể cộng hợp phenol từ mặt
trên hoặc mặt dưới. Mặt dưới được ưu tiên hơn bởi hiệu ứng anomer và sự tạo
thành acetal chiu sự khống chế nhiệt động học.

1107 | Câu hỏi lí thuyết Hóa học hữu cơ OlympiaVN


Bài 3
Giải thích tại sao ở cùng pH thì methyl β-D-2-deoxyglucopyranoside (I) bị thủy
phân nhanh hơn methyl β-D-glucopyranoside (II)?

Hướng dẫn
Cả hai hợp chất đều tạo thành oxocarbocation trong phản ứng thủy phân. Nguyên
tử anomer carbon mang điện tích trong hợp chất I (một acetal) có một nguyên tử
hydrogen liên kết với nó, trong khi đó nguyên tử carbon tương ứng trong hợp chất
II (một ketal) lại gắn với một carbon. Do đó, carbocation của hợp chất II bền hơn
và trạng thái chuyển tiếp dẫn đến sự tạo thành nó có năng lượng thấp hơn hợp
chất I.

1108 | Câu hỏi lí thuyết Hóa học hữu cơ OlympiaVN


Bài 4
Rutinose là một loại đường có trong
thành phần một số bioflavonoid
(flavonoid sinh học), các hợp chất được
tìm thấy trong những loại thực vật có giá
trị y học với việc duy trì sức khỏe tim
mạch nói chung và cụ thể là với sức
mạnh của thành mạch máu. Rutin là một
bioflavonoid chứa rutinose được tìm
thấy trong kiều mạch và bạch đàn.
Hesperidin là một chất khác, có nguồn
gốc từ vỏ chanh và cam. Mỗi chất chứa
rutinose liên kết với aglycon ba vòng.

Sử dụng các thông tin sau để suy ra cấu trúc của đường rutinose.
a) Rutinose là mộ đường khử mà khi thủy phân acid tạo thành một đương lượng
của mỗi chất D-glucose và đường A, có công thức C6H12O5. Đường A phản
ứng với bốn đương lượng HIO4 tạo thành bốn đương lượng formic acid và
một đương lượng acetaldehyde. Ở giai đoạn này, chúng ta có thể kết luận gì
về đường A?
b) Đường A có thể được tổng hợp từ L-(-)-mannose theo sơ đồ dưới. Giai đoạn
3 (được kí hiệu bởi dấu hoa thị) là một phản ứng đặc biệt chuyển hóa alcohol
bậc một đầu mạch thành carboxylic acid. Kết quả này thể hiện điều gì về các
tâm lập thể trong đường A?

1109 | Câu hỏi lí thuyết Hóa học hữu cơ OlympiaVN


c) Methyl hóa hoàn toàn rutinose với lượng dư dimethyl sulfate tạo thành dẫn
xuất heptamethyl hóa. Thủy phân acid êm dịu tiếp thì thu được một đương
lượng của 2,3,4-tri-O-methyld-glucose và một đương lượng của dẫn xuất
2,3,4-tri-O-methyl của đường A. Xác định các cấu trúc có thể có của rutinose
phù hợp với những dữ kiện này.
Hướng dẫn
a) Kết quả của thoái phân HIO4 cho ta biết điều gì? Mỗi đương lượng HIO4 phân
cắt một liên kết giữa hai nguyên tử carbon nối với nhau bởi cầu oxygen.
Formic acid tạo thành hoặc từ nhóm formyl đầu mạch, hoặc bởi nhóm
hydroxy bậc hai bên trong. Acetaldehyde là một sản phẩm thoái phân bất
thường. Sự tạo thành nó cho thấy có nhóm thế methyl đầu mạch, gắn vào
nguyên tử carbon chứa hydroxy bậc hai. Cấu trúc của A được sắp xếp logic
lại như dưới đây. Cấu trúc này quả thực có công thức phân tử C6H12O5.

2)

1110 | Câu hỏi lí thuyết Hóa học hữu cơ OlympiaVN


A là 6-deoxy-L-mannose.
3)

1111 | Câu hỏi lí thuyết Hóa học hữu cơ OlympiaVN


Bài 5
Glycoside A (C20H27NO11), được tìm thấy trong hạt Rosaceae (họ Hoa hồng) có
kết quả âm tính với các dung dịch Benedict hoặc Fehling. Thủy phân xúc tác
enzyme A tạo thành (-)B, C8H7NO và C, C12H22O11 nhưng nếu thủy phân hoàn
toàn với xúc tác acid thì tạo thành các sản phẩm hữu cơ, (+)D, C6H12O6 và (-)E,
C8H8O3. C có một liên kết β-glycosidic và có kết quả dương tính với dung dịch
Benedict hoặc Fehling. Methyl hóa C với MeI/Ag2O tạo thành C20H38O11, chất
này khi thủy phân xúc tác acid tạo thành 2,3,4-tri-O-methyl-D-glucopyranose và
2,3,4,6-tetra-O-methyl-D-glucopyranose. (±)B có thể được điều chế từ
benzaldehyde và NaHSO3, sau đó xử lí với NaCN. Thủy phân xúc tác acid (±)B
tạo thành (±)E.
1) Xác định cấu trúc các chất A-D với hóa lập thể phù hợp trong công thức chiếu
Haworth, ngoại trừ B.
Glycoside A được phát hiện thấy có độc tính và nguyên nhân được cho là bởi chất
F kịch độc, được tạo trong các môi trường thủy phân. Sự khử độc hợp chất F
trong thực vật có thể đi kèm với các phản ứng sau (không biểu diễn hóa lập thể):

Một lượng nhỏ hợp chất F trong cơ thể người được cho là bi khử độc bởi phản
ứng trực tiếp với cystine tạo thành L-cysteine và hợp chất I, C4H6N2O2S - chất
này bị bài tiết trong nước tiểu (không biểu diễn hóa lập thể).

Hợp chất I không có hấp thụ ở 2150-2250 cm-1 trong phổ hồng ngoại, nhưng có
một dải ở 1640 cm-1. Ngoài ra cũng quan sát được các dải tương ứng của nhóm
carboxyl.

1112 | Câu hỏi lí thuyết Hóa học hữu cơ OlympiaVN


2) Xác định công thức phân tử của các chất F, G và công thức cấu tạo của các
chất H, I. Chỉ rõ hóa lập thể của H.

(Stretching Vibration Region = Vùng dao động kéo căng)


Hướng dẫn
1)

2) F = HCN, G = H2S

1113 | Câu hỏi lí thuyết Hóa học hữu cơ OlympiaVN


1114 | Câu hỏi lí thuyết Hóa học hữu cơ OlympiaVN
Bài 6
Hương liệu được chiết xuất từ rễ cây cam thảo ngọt hơn đường kính 50-150 lần.
Hợp chất quan trọng nhất và hàm lượng lớn nhất chịu trách nhiệm cho độ ngọt và
các tác động dược lí của cây cam thảo là glycyrrhizin (C42H62O16). Glycyrrhizin
cần 3 đương lượng NaOH để trung hòa. Khi glycyrrhizin bị thủy phân acid thì
thu được Glycyrrhizinic acid (A (C30H46O4)) và B (C6H10O7) theo tỉ lệ mol 1:2
(Sơ đồ 1).

Khi methyl hóa glycyrrhizin bởi methyl iodide (MeI) tại mọi vị trí có thể trước
khi thủy phân thì quá trình thủy phân tạo thành A’ (methyl glycyrrhizinate), C và
D (Sơ đồ 2). B, C, D tồn tại ở dạng hỗn hợp của các anomer.

Methyl hóa C và D với MeI tạo thành cùng một hỗn hợp đồng phân, J (Sơ đồ 3).

C bị khử bởi LiAlH4 tạo thành K, khử hóa tiếp chất này thu được L. Phản ứng
oxid hóa cắt mạch vicinal diol L với NaIO4 tạo thành M và 2 đương lượng
formaldehyde. Khử hóa M tạo thành N. Cấu trúc và hóa lập thể của N được xác
nhận bởi quá trình tổng hợp N từ D-(-)-tartaric acid qua quá trình methyl hóa, sau
đó khử hóa (Sơ đồ 4). Phổ 1H NMR của L cho thấy 2 đỉnh riêng biệt ứng với các
nhóm methyl. (Trong cấu trúc của L không có đối xứng.)

1115 | Câu hỏi lí thuyết Hóa học hữu cơ OlympiaVN


1) Xác định cấu trúc của L, M, N.
2) C có thể có bao nhiêu cấu trúc? Hãy biểu diễn chúng.
Chuỗi phản ứng sau đã được tiến hành để xác định cấu trúc chính xác của C. J bị
khử thành E, và thủy phân acid E tạo thành F. Sự khử F tạo thành G, khi oxid
hóa chất này bởi NaIO4 tạo thành H, kèm theo đó là sự tạo thành 1 đương lượng
formadehyde. Trong số các hợp chất từ A - I, chỉ có I không quang hoạt (Sơ đồ
5).

3) Xác định cấu trúc của G và I.


4) Xác định cấu trúc chính xác của C trong số các cấu trúc đã biểu diễn trong ý
2.
5) Xác định cấu trúc của B, D, J.
6) Xác định cấu trúc Glycyrrhizin.
Hướng dẫn
1)

1116 | Câu hỏi lí thuyết Hóa học hữu cơ OlympiaVN


2) Có thể có 2 cấu trúc:

3)

4) Cấu trúc 1 đúng.

1117 | Câu hỏi lí thuyết Hóa học hữu cơ OlympiaVN


5)

6)

1118 | Câu hỏi lí thuyết Hóa học hữu cơ OlympiaVN


Bài 7
Đảo Crete ở Hi Lạp là một trong những khu vực địa lí mà thói quen dinh dưỡng
của người dân đã thu hút sự chú ý như “hình mẫu về dinh dưỡng”. Trong một
nghiên cứu tiến hành ở 5 quốc gia, chế độ ăn kiêng của người Crete có liên quan
đến tỉ lệ bệnh tim mạch vành (CHD, coronary heart disease) thấp. Tỉ lệ tử vong
từ CHD là 7 trên 10000 đối tượng nghiên cứu, trong khi đó tỉ lệ tương ứng ở Phần
Lan, Mỹ, Hà Lan và Ý lần lượt là 566, 424, 317 và 200. Nguyên nhân chủ yếu là
do sự tiêu thụ nhiều dầu olive, có chứa nhiều oleuropein, một chất chống oxid
hóa mạnh mẽ. Oleuropein (A) có công thức cấu tạo như sau với R là nhóm
alkylpolyphenolic:

1) Thuỷ phân có xúc tác acid oleuropin tạo thành glucose và hai hợp chất khác
gồm: polyphenolic (A1) và một monoterpenoid (A2). Hãy sử dụng mũi tên
để chỉ vào công thức của oleuropin.
a) Nguyên tử oxygen sẽ bị proton hóa trong phản ứng thuỷ phân xúc tác
acid để tạo ra polyphenolic A1.
b) Liên kết C–O bị phân cắt để tạo ra glucose.
Trong phổ khối của A1, peak tương ứng với ion phân tử có m/z = 154. Phổ 1H-
NMR của (A1) được cho dưới đây. Nhóm hydroxylic proton đã tham gia phản
ứng trao đổi nên không có mặt trong các peak dưới đây.

1119 | Câu hỏi lí thuyết Hóa học hữu cơ OlympiaVN


2) Với những thông tin trên hãy chỉ ra A1 là chất nào trong ba chất sau?

3) Từ công thức cấu tạo của A1 hãy chỉ ra proton nào của A1 phù hợp với các
tín hiệu trên phổ 1H–NMR.

1120 | Câu hỏi lí thuyết Hóa học hữu cơ OlympiaVN


Hướng dẫn

Công thức cấu tạo đúng là C

1121 | Câu hỏi lí thuyết Hóa học hữu cơ OlympiaVN


Bài 8
Axit turgorinic (X) là một hormon thực vật đóng vai trò điều khiển sự cử động
của lá cây. Chẳng hạn nó điều chỉnh sự cụp lại của lá cây xấu hổ khi ta chạm vào.
Hợp chất X không phản ứng với thuốc thử Fehling. Khi thủy phân X bằng enzym
β-glucozidaza thu được chất A (C6H12SO9) có tính axit và phản ứng được với
thuốc thử Fehling cùng với hợp chất B. Đun nóng B quan sát thấy sự thoát ra CO2
và thu được pyrogalol. Metyl hóa hoàn toàn X thu được C. Thủy phân hoàn toàn
C bằng axit vô cơ thu được metyl 3,5-dimetoxy-4-hydroxibenzoat và hợp chất D
(C9H18SO9). Oxy hóa D bằng axit nitric thu được sản phẩm chính là axit 2,3,4-
trimetoxypentadioic. Hãy xác định cấu trúc X dựa vào các dữ kiện trên.
Hướng dẫn
X tham gia phản ứng thủy phân xúc tác enzym β-glucozidaza chứng tỏ X có tồn
tại liên kết β-glycozit.
Đun nóng chất B thấy tách ra CO2, thu được pyrogalol (1,2,3-trihydroxibenzen)
và khi thủy phân hoàn toàn C bằng axit vô cơ thu được metyl 3,5-dimetoxy-4-
hydroxibenzoat chứng tỏ B là axit 3,4,5-trihydroxibenzoic.
Tiến hành oxy hóa D (C9H18SO9) bằng axit nitric thu được axit 2,3,4-
trimetoxypentadioic. Như vậy công thức của D sẽ là.

Từ các dữ kiện trên có thể suy ra axit turgorinic như sau:

1122 | Câu hỏi lí thuyết Hóa học hữu cơ OlympiaVN


Bài 9
Photphoramidon là những chất ức chế tự nhiên của enzym thermolysin, một
enzym thuộc nhóm proteaza có trong các sinh vật đơn bào sống trong vùng nước
biển lạnh. Hãy đưa ra cấu trúc của hợp chất này dựa trên những dữ kiện cho dưới
đây:
- Hợp chất này là một muối dinatri có công thức phân tử C23H32N3Na2O10P.
- Tiến hành phân cắt hợp chất này thu được dipeptit leuxyltryptopan,
đường L-rhamanozơ và muối NaH2PO4.
- Đầu N của dipeptit được nối với phân đoạn photphat qua liên kết amit.
- Nhóm photphat liên kết với monosaccarit qua liên kết este.
- Photphoamidon là đường không khử.
- Metyl hóa hoàn toàn và thủy phân hoàn toàn photphoamidon thu được
2,3,4-tri-O-metyl L-rhamanozơ (và các sản phẩm không đường khác).
- Oxy hóa monosaccarit này bằng axit nitric loãng thu được một axit
monocacboxylic có mạch carbon không phân nhánh có công thức phân
tử C6H12O6
- Đơn vị L-rhamanozơ trong photphoamidon có cấu hình tuyệt đối của các
nguyên tử carbon bất đối giống như L-mannozơ. Cấu hình của C1 trong
rhamanozơ là 1S.

1123 | Câu hỏi lí thuyết Hóa học hữu cơ OlympiaVN


Hướng dẫn
Dựa vào các thông tin đã cho có thể xác định được số nguyên tử C và O trong L-
rhamanozơ như sau. Số C = 23 – 11 (Tryptopan) – 6 (leuxin) = 6. Số O = 10 – O2
(tryptopan) – 1 (leuxin) – 2 (photphat) = 5. Bằng con đường tương tự có thể xác
định đường này không chứa nitơ và có công thức phân tử C6H12O5.
Dựa trên các dữ kiện 6 và 7 cho thấy rằng nó là một hexozơ và nhóm CH2OH
thường thấy được thay bằng nhóm CH3 (không tạo dẫn xuất metyl hóa ở C6 cũng
như không thể tạo ra được axit dicaboxylic dưới tác dụng của HNO3). Do đã biết
cấu trúc của L-mannozơ nên cấu trúc của L-rhamanozơ sẽ là:

Bên cạnh đó dữ kiện 6 cho thấy rhamanozơ tồn tại ở dạng vòng pyranozơ (vắng
mặt C5 trong sản phẩm metyl hóa). Cùng với việc cấu hình của rhamanozơ của
C1 là 1S chứng tỏ nhóm OH ở C1 mạch vòng sẽ là OH β. Dựa vào những dữ kiện
cắt mạch có thể xác định được cấu trúc của photphoamidon sẽ là:

1124 | Câu hỏi lí thuyết Hóa học hữu cơ OlympiaVN


Bài 10
Hợp chất hữu cơ A (C17H18O9) là một chất độc được tạo ra trong một số loài cây
do cơ chế tự vệ chống lại sâu bọ và động vật có vú. Thủy phân hợp chất A bằng
enzym emulsin thu được hợp chất B (C11H6O3) và D-Glucozơ. Hợp chất B có thể
được tổng hợp theo sơ đồ sau:

Đun nóng trong nước thì A bị đồng phân hóa thành A’. A’ cũng bị thủy phân bởi
enzym emulsin thu được hợp chất B’ (C11H8O4).Đun nóng B’ với axit lại thu được
B.Xác định cấu trúc của A.
Hướng dẫn
Xác định B:

Cấu trúc của A là:

1125 | Câu hỏi lí thuyết Hóa học hữu cơ OlympiaVN


1126 | Câu hỏi lí thuyết Hóa học hữu cơ OlympiaVN
Bài 11
Axit turgorinic (PLMF1) là một hormon thực vật đóng vai trò điều khiển sự cử
động của lá cây. Chẳng hạn nó điều chỉnh sự cụp lại của lá cây xấu hổ khi ta chạm
vào. Hợp chất PLMF1 có tính axit, không phản ứng với thuốc thử Felinh. Khi
thủy phân PLMF1 bằng enzym β-glycozidaza thu được hợp chất A (C6H12SO9,
có tính axit, phản ứng được với thuốc thử Felinh) và hợp chất B. Khi đun nóng B
thấy CO2 tách ra và thu được pirogalol. Khi metyl hóa hoàn toàn PLMF1 thu được
hợp chất C. Thủy phân hoàn toàn C bằng axit vô cơ thu được metyl 3,5-dimetoxi-
4-hydroxibenzoat và hợp chất D (C9H18SO9). Oxy hóa D bằng axit nitric thu được
sản phẩm chính là axit 2,3,4-trimetoxypentandioic.
a) Hãy xác định công thức của các hợp chất A, B, D và PLMF1.
b) Trong tự nhiên, nhóm cacboxyl của PLMF1 bị este hóa bằng nhóm OH ở vị trí
số 3 của B tạo ra hợp chất E thuộc nhóm depsides (tiếng Hy Lạp có nghĩa là có
tính thuộc da). Xác định công thức của E.
Hướng dẫn
Khi thủy phân PLMF1 bằng β-glycozidaza chứng tỏ trong PLMF1 có liên kết β-
glucozit. Từ dữ kiện khi đun nóng chất B thấy tách ra CO2 cho pirogalol (1,2,3-
trihydroxibenzen) và khi thủy phân hoàn toàn chất C bằng axit vô cơ thu được
metyl 3,5-dimetoxy-4-hydroxibenzoat chứng tỏ B là axit 3,4,5-trihydroxibenzoic.

Khi oxy hóa D (C9H18SO9) bằng axit nitric thu được axit 2,3,4-
trimetoxypentadioic. Vậy D có công thức như sau:

Từ dữ kiện trên suy ra công thức của PLMF1 là:

1127 | Câu hỏi lí thuyết Hóa học hữu cơ OlympiaVN


b) Công thức của E là

1128 | Câu hỏi lí thuyết Hóa học hữu cơ OlympiaVN


Bài 12
Avenacin A-1 là glycoside
thuộc nhóm Saponin được tìm
thấy trong yến mạch với tác
dụng ức chế sự phát triển của
một số mầm bệnh nấm. Thủy
phân Avenacin A-1 bởi enzyme
α-glycosidase tạo thành X1
(C17H30O15) và Y (cấu trúc như hình bên). Thủy phân X1 bởi enzyme β-
glycosidase thu được 2 monosaccharide quen thuộc X2 và X3 (C6H12O6, là
aldohexose thuộc dãy D).
a. Xác định công thức phân tử của X2.
Thoái phân Ruff X3 rồi oxi hóa sản phẩm bằng HNO3 cho X4 không
quang hoạt, là dẫn xuất của pentandioic acid. Methyl hóa hoàn toàn X1 bằng MeI
dư/Ag2O và thủy phân sản phẩm bằng HCl thu được X5 và X6 (C6H12O5). Oxi
hóa cắt mạch X5 bằng HNO3 cho 2,3,4-trimethoxypentandioic acid (X7) không
quang hoạt, (2R,3R)-2,3-dimethoxybutandioic acid (X8) và methoxyaxetic acid.
b. Xác định công thức chiếu Fischer của X5, X7 và X8.
X6 tồn tại ở dạng vòng pyranose (đây cũng là dạng tồn tại của hợp phần
đường tương ứng trong các chất đã nêu), khi xử lí với MeI dư/Ag2O rồi đun nóng
với dung dịch HNO3 tạo thành X8 và X9 là một epimer của X7. Nếu xử lí X6 với
NaBH4 cho X10. Nếu thay đổi cấu hình tại một nguyên tử carbon bất đối trong
X10 sẽ thu được X11 không quang hoạt.
c. Xác định công thức chiếu Fischer của X6, X9, X10 và X11.
d. Vẽ công thức Haworth của hợp phần glycone trong Avenacin A-1.
Hướng dẫn

a. X1 (C17H30O15) X2 + X3 (C6H12O6)
Do X2, X3 là hai đường đơn, mặt khác X1 chứa số lẻ carbon nên X2 có số lẻ
carbon, quá trình thủy phân 1 phân tử X1 sẽ tạo thành 2 phân tử X3 và 1 phân tử
X2 với sự thủy phân 2 liên kết β-glycoside. Quá trình này được mô tả đầy đủ như
sau:

X1 (C17H30O15) + 2H2O X2 + 2X3 (C6H12O6)


Do đó, trong X2 có số C bằng 17 – 6.2 = 5; số H bằng 30 + 2.2 – 12.2 = 10;
số O bằng 15 + 2 – 6.2 = 5.
Vậy công thức phân tử của X2 là C5H10O5.

1129 | Câu hỏi lí thuyết Hóa học hữu cơ OlympiaVN


b. + X6 chứa 5O, 6C nên là dẫn xuất mono-O-methyl của X2 còn X5 là dẫn xuất
của aldohexose X3. X3 thuộc dãy D nên C5 của X3 có cấu hình R. Sơ đồ của quá
trình hình thành X4:

X4 quang hoạt, chứng tỏ C2 của X4 và C3 của X3 đều có cấu hình S, cùng


cấu hình với C4 của X4 để khi đó X4 không có mặt phẳng đối xứng.
+ HNO3 phân cắt mạch ở vị trí nguyên tử carbon gắn với nhóm OH, mặt khác từ
X5 còn tạo ra X7 chứa ba nhóm methoxy kế nhau nên nhóm OH tự do trong X5
gắn với C5. Từ sản phẩm phân cắt mạch X7, X8 và cấu hình (3S, 5R) suy ra công
thức chiếu Fischer của X5:

c. + X6 tồn tại ở dạng pyranose nên X2 phải là aldopentose, vòng pyranose tạo
bởi nhóm OH ở C5 với nhóm aldehyde ở C1. Sau quá trình methyl hóa các nhóm
OH tự do trong X6 bằng MeI/Ag2O, khi xử lí với HNO3 ban đầu cho sản phẩm
thủy phân chứa OH-hemiacetal. Sự oxi hóa sản phẩm này tạo X8 và một epimer
của X7, do đó cấu trúc của sản phẩm ở dạng mạch hở như sau:

+ X6 tác dụng với NaBH4 cho X10 chứa 4 nhóm OH và 1 nhóm OMe. Do thay
đổi cấu hình của 1 nguyên tử carbon bất đối trong X10 tạo ra X11 không quang

1130 | Câu hỏi lí thuyết Hóa học hữu cơ OlympiaVN


hoạt nên nhóm methoxy phải gắn với nguyên tử C3 trong mạch carbon của X6 và
X10. Vậy cấu trúc của X6, X10 và X11 là:

d. Từ cấu trúc của X5 và X6 suy ra 2 hợp phần glucose đều tồn tại ở dạng pyranose
và tạo liên kết β-glycoside với các nhóm OH ở C2, C4 của hợp phần X2. Liên kết
của X2 với Y là liên kết α-glycoside. Vậy công thức Haworth của hợp phần
glycone trong Avenacin A-1 như sau:

1131 | Câu hỏi lí thuyết Hóa học hữu cơ OlympiaVN


Bài 13
Albutin có công thức C12H16O7. Khi xử lý chất này
với axit hay với enzym α-glycozidaza thu được D-
glucozơ và hợp chất X có công thức C6H6O2.
Metyl hóa hoàn toàn albutin rồi thủy phân trong
môi trường axit thu được2,3,4,6-tetra-O-metyl-D-
glucozơ và hợp chất Y (C7H8O2). Xử lý Y bằng
NaOH / (CH3)2SO4 thu được Z (C8H10O2). Biết
rằng khi oxy hóa X rồi chiếu sáng thu được sản
phẩm có cấu trúc như hình bên. Hãy xác định công thức của X, Y, Z và albutin
Hướng dẫn
Sản phẩm được tạo thành khi chiếu sáng chính là dime của quinon, như vậy X
phải là dihydroquinon, Y là monometylete và Z là dimetylete của hydroxiquinon.
Metyl hóa hoàn toàn albutin rồi thủy phân thu được được2,3,4,6-tetra-O-
metyl-D-glucozơ, tức nhóm OH C1 đã bị thế.
Albutin chỉ bị thủy phân bằng α-glycozidaza tức liên kết là α-glycozit.
Như vậy cấu trúc các chất như sau:

1132 | Câu hỏi lí thuyết Hóa học hữu cơ OlympiaVN


Bài 14
Trong những năm gần đây, phương pháp chữa trị ung thư bằng những “viên đạn
thuốc” đang rất được quan tâm. Chúng gồm 2 phần chính là phần truyền dẫn và
phần dược tính. Trong đó phần truyền dẫn có ái lực đặc biệt với tế bào ung thư
sẽ nhận trách nhiệm “mang thuốc” đến vị trí ung thư. Phương pháp này không chỉ
giúp tăng khả năng khỏi bệnh mà còn giảm các phản ứng phụ.
Để kiểm tra khả năng truyền dẫn thì một nhóm nghiên cứu đã tiến hành tổng hợp
dẫn xuất A theo sơ đồ sau:

1. Vẽ cấu trúc các chất B, C, D ở dạng vòng phẳng


2. Cho biết tác nhân nào đã được sử dụng trong bước cuối cùng?
3. Dự đoán kết quả thu được khi thủy phân A trong kiềm loãng
Hướng dẫn
1. Cấu trúc các chất B, C, D

2. Pirole
3. Khi thủy phân trong kiềm sẽ tạo 16 nhóm OH nên tăng mạnh tính tan của
sản phẩm thủy phân.

1133 | Câu hỏi lí thuyết Hóa học hữu cơ OlympiaVN


Bài 15
Một số giống cá nóc, Fugu trong tiếng Nhật, là loại thực phẩm được đánh giá cao
ở Nhật Bản. Do nội tạng (đặc biệt là buồng trứng và gan) của loài cá này có chứa
độc tố cực mạnh (tetrodotoxin), nên các vụ ngộ độc thực phẩm liên quan đến loài
cá này thường xuyên xảy ra. Các nghiên cứu về tetrodotoxin (1) đã được thực
hiện từ đầu thế kỉ 20 và vào năm 1964, cấu trúc của nó đã được xác định rõ:

Nhóm guanidine trong tetrodotoxin thể hiện tính base mạnh. Guanidinium ion tạo
thành từ sự proton hóa nhóm guanidine được bền hóa bởi sự tồn tại của các cấu
trúc cộng hưởng sau.

1) Vẽ các cấu trúc cộng hưởng B, C.


Nhiều phản ứng dẫn xuất hóa đã được tiến hành trong nghiên cứu cấu trúc
tetrodotoxin. Xử lí tetrodotoxin (1) (xem sơ đồ 1) với ethanolic potassium
hydroxide, đun nóng có thể tạo thành dẫn xuất quinazoline 2, chất này giúp mang
tới cái nhìn về bản chất bộ khung cơ bản của tetrodotoxin. Cơ chế phản ứng có
thể được mô tả như sau đây. Trước tiên, tetrodotoxin bị thủy phân thành
carboxylate 3. Sau đó, nhóm hydroxyl trong khung vuông bị tách loại bởi base,
tạo thành trung gian D. Phản ứng retro-aldol của D phân cắt liên kết carbon-
carbon, tạo thành các trung gian E và F. Cuối cùng, phản ứng dehydrogen hóa và
thơm hóa E tạo thành dẫn xuất quinazoline 2.
2) Vẽ cấu trúc các trung gian giả thiết D, E, F.

1134 | Câu hỏi lí thuyết Hóa học hữu cơ OlympiaVN


Mặc dù quá trình sinh tổng hợp tetrodotoxin vẫn còn phải làm sáng tỏ hơn, nhưng
có thể nó được sinh tổng hợp từ L-arginine và isopentenyl diphosphate.

3) Khoanh tròn các nguyên tử carbon trong tetrodotoxin không phải có nguồn
gốc từ L-arginine.
Trong thập niên 1990, một hướng sinh tổng hợp thay thế tetrodotoxin đã được đề
xuất. Phản ứng ngưng tụ 2-deoxy-3-oxo-D-pentose và guanidine tạo thành trung
gian G với hợp phần guanidine vòng (C6H11N3O3). Tetrodotoxin có thể sinh tổng
hợp từ trung gian G và isopentenyl diphosphate.

4) Vẽ cấu trúc của trung gian giả thiết G, chỉ rõ hóa lập thể.

1135 | Câu hỏi lí thuyết Hóa học hữu cơ OlympiaVN


Hướng dẫn
1)

2)

3)

4)

Các phương án được chấp nhận:

1136 | Câu hỏi lí thuyết Hóa học hữu cơ OlympiaVN


Mỗi cấu trúc ion lưỡng cực như sau cũng được chấp nhận:

1137 | Câu hỏi lí thuyết Hóa học hữu cơ OlympiaVN


Bài 16
Các carbohydrate là những hợp phần thiết yếu trong các tế bào sống và là nguồn
năng lượng cho động vật. Chúng có thể là các chất đường đơn giản với phân tử
nhỏ hoặc cũng có thể là những chất đại phân tử. Khi nguyên tử oxygen trong vòng
trong các chất đường bị thay thế bởi một nhóm methylene thì hợp chất tạo thành
được gọi là chất giả-đường hoặc carbasugar. Do các carbasugar bền (khó bị thủy
phân) với acid và enzyme, nên nhiều chất được ứng dụng trong việc ứng chế
glycosidase.
Các quy trình tổng hợp toàn phần của 2 carbasugar đồng phân có bộ khung 1 được
mô tả dưới đây.

Quy trình tổng hợp 1 bắt đầu với sự khử benzene bởi sodium trong ammonia lỏng,
tạo thành A. Phổ 13C NMR của A có 2 tín hiệu 124.0 và 26.0 ppm. Tiểu phân hoạt
động S (xem Sơ đồ 1) được tạo thành từ trichloroacetyl chloride khi có mặt kẽm.
1 đương lượng S tham gia phản ứng cộng vòng [2+2] với A tạo thành sản phẩm
racemic B. Phản ứng của B với Zn trong acetic acid tạo thành C. Hợp chất C chỉ
chứa carbon, hydrogen và oxygen. Phổ 13C NMR của C có 3 tín hiệu sp2 carbon
ở 210.0, 126.5 và 125.3 ppm. Phản ứng của C với 1 đương lượng m-
chloroperbenzoic acid (m-CPBA) trong methylene chloride tạo thành sản phẩm
chính là D. Phổ 13C NMR của D có 3 tín hiệu trong vùng sp2 ở 177.0, 125.8, 124.0
ppm.

Các vùng chuyển dịch hóa học 13C-NMR của các nhóm chức điển hình.

1138 | Câu hỏi lí thuyết Hóa học hữu cơ OlympiaVN


Sơ đồ 1

1) Vẽ các cấu trúc A, B, C, D và trung gian S. Sự khử của D với LiAlH4 tạo
thành E, chất này phản ứng với acetyl chloride dư trong pyridine tạo thành F
(Sơ đồ 2).

2) Vẽ cấu trúc (một đối quang) của E và F, sử dụng kí hiệu đường kẻ vạch-in
đậm. Gắn các kí hiệu cấu hình tuyệt đối (R/S) cho các carbon bất đối trong
E.
Hợp chất F (sử dụng đối quang được vẽ ở trên) phản ứng với bromine tạo thành
các đối quang G1, G2.
3) Vẽ các cấu trúc của G1, G2 sử dụng kí hiệu đường kẻ vạch-in đậm.
Hỗn hợp G1, G2 phản ứng với 2 đương lượng 1,8-diazabicyclo- [5.4.0]undec7-
ene (DBU) - một amine base mạnh - tạo thành H.
4) Vẽ cấu trúc của H sử dụng kí hiệu đường kẻ vạch-in đậm.
Phản ứng của H với singlet oxygen (tạo ra tại chỗ [in situ]) tạo thành I. Mặc dù
về mặt lí thuyết có thể có 2 đồng phân nhưng thực tế thì I được tạo thành ở dạng
đồng phân tinh khiết do sự án ngữ không gian và sự đẩy điện tử. Phản ứng của I
với LiAlH4 (dư) dẫn tới sự tạo thành J (xem Sơ đồ 3). Phổ 13C NMR của J có 8
tín hiệu, 2 trong số đó thuộc vùng sp2.
Phản ứng của J với acetyl chloride dư khi có mặt pyridine tạo thành K. Phản ứng
tiếp theo của K với OsO4 khi có mặt 4-methylmorpholine 4-oxide (NMO) tạo
thành các đồng phân lập thể L và M. Khử hóa L và M với LiAlH4 thì lần lượt tạo
thành 1a và 1b.

1139 | Câu hỏi lí thuyết Hóa học hữu cơ OlympiaVN


5) Vẽ cấu trúc của I, J, K, L, M, 1a, và 1b sử dụng kí hiệu đường kẻ vạch-in
đậm.
Hướng dẫn
1)

1140 | Câu hỏi lí thuyết Hóa học hữu cơ OlympiaVN


2)

3)

4)

5)

1141 | Câu hỏi lí thuyết Hóa học hữu cơ OlympiaVN


6)

1142 | Câu hỏi lí thuyết Hóa học hữu cơ OlympiaVN


Amino acid
Bài 1
1) Vẽ công thức chiếu Fischer của các amino acid sau:
(a) L-Threonine
(b) L-Serine
(c) L-Phenylalanine
(d) L-Asparagine
2) Vẽ tất cả các đồng phân lập thể của L-isoleucine. Với mỗi đồng phân, hãy chỉ
rõ cấu hình R/S của các tâm thủ tính.
3) Arginine có tính base mạnh nhất trong số 20 amino acid tồn tại tự nhiên. Ở
pH sinh lí học, nhánh phụ của arginine bị proton hóa. Xác định nguyên tử
nitrogen nào ở nhánh phụ bị proton hóa.
4) Histidine có nhánh phụ có tính base bị proton hóa ở pH sinh lí học. Xác định
nguyên tử nitrogen nào ở nhánh phụ bị proton hóa.
Hướng dẫn
1)
a) b) c) d)

2)

3) Sự proton hóa ở nguyên tử nitrogen được bôi màu sẽ tạo thành acid liên hợp
được bền hóa mạnh bởi cộng hưởng (điện tích dương được giải tỏa mạnh).

1143 | Câu hỏi lí thuyết Hóa học hữu cơ OlympiaVN


4) Dạng proton hóa dưới đây có tính thơm. Ngược lại, sự proton hóa ở nguyên tử
nitrogen khác trong vòng sẽ làm mất bền hóa thơm.

1144 | Câu hỏi lí thuyết Hóa học hữu cơ OlympiaVN


Bài 2
Phản ứng ngưng tụ một carboxylic acid với một amine tạo thành sản phẩm amide.
Ví dụ, sự ngưng tụ formic acid với dimethylamine tạo thành N,N-
dimethylformamide (DMF), chất này có thể mô tả bởi các cấu trúc cộng hưởng
sau đây:

1) Dự đoán trật tự giảm dần nhiệt độ nóng chảy của N,N-dimethylformamide


(hợp chất A), N-methylacetamide (CH3CONHCH3, hợp chất B), và
propionamide (hợp chất C, CH3CH2CONH2).
2) Các nhóm carbonyl thường được xác định bởi các hấp thụ mạnh đặc trưng
của chúng trong phổ hồng ngoại. Vị trí của hấp thụ phụ thuộc vào độ bền của
liên kết C=O, điều này cũng phản ánh độ dài liên kết. Trong các amide, độ
bền của các nhóm carbonyl có thể được thể hiện bởi các cấu trúc cộng hưởng
cho ở trên. Ví dụ, cyclohexanone thể hiện một hấp thụ ở 1715 cm-1 cho các
nhóm carbonyl (C=O). Dự đoán dải hấp thụ của nhóm carbonyl trong
propionamide so với cyclohexanone. Chọn một trong các phương án sau:
(a) 1660 cm-1 bởi độ dài liên kết carbonyl ngắn hơn.
(b) 1660 cm-1 bởi độ dài liên kết carbonyl dài hơn.
(c) 1740 cm-1 bởi độ dài liên kết carbonyl ngắn hơn.
(d) 1740 cm-1 bởi độ dài liên kết carbonyl dài hơn.
3) Glycine (H2N-CH2-COOH) là một α-amino acid. 3 phân tử glycine có thể tạo
thành tripeptide Gly-Gly-Gly qua các cầu amide, kèm theo sự tách 2 phân tử
nước. Vẽ cấu trúc của tripeptide này.
4) Khi một α-amino acid chứa một nhóm thế thì có thể có các đồng phân quang
học. Ví dụ, L-alanine và D-alanine là hai đối quang. Tính số lượng khả dĩ của
các tripeptide thẳng có thể được tạo thành từ 3 amino acid: glycine, L-alanine
và D-alanine từ phản ứng ngưng tụ.

5) Trong số các tripeptide được tổng hợp trong ý 4, có bao nhiêu chất có tính
quang hoạt?
Hướng dẫn
1) Trật tự giảm dần nhiệt độ nóng chảy: C > B > A.

1145 | Câu hỏi lí thuyết Hóa học hữu cơ OlympiaVN


Cấu trúc cộng hưởng của amide cho thấy có điện tích âm riêng phần trên oxygen
và điện tích dương riêng phần trên nitrogen. Các amide bậc một và bậc hai cũng
tham gia tạo thành liên kết hydrogen mạnh, nhưng amide bậc ba thì không. Thực
tế thì nhiệt độ nóng chảy (m.p.) của các amide là: Propionamide, m.p. = 79 °C;
N-methylacetamide, m.p. = 28 °C; N,N-dimethylformamide, m.p. = -61 °C.
2) b) 1660 cm-1 bởi độ dài liên kết carbonyl dài hơn.
3)

4) Có thể có 27 tripeptide.
5) Có 26 tripeptide có tính quang hoạt. Trong đó H2N-GGG-OH không quang
hoạt, còn lại các tripeptide như H2N-G-GLA-OH, H2N-GGDA-OH, H2N-
GLAG-OH, H2N-LAGDA-OH, H2N-LALADA-OH, … đều quang hoạt.

1146 | Câu hỏi lí thuyết Hóa học hữu cơ OlympiaVN


Bài 3
Bàng giá trị pKa (ghi chú: Side chan là nhánh phụ)

1) Vẽ dạng tồn tại của amino acid chiếm ưu thế ở pH được chỉ định.
(a) Alanine ở pH 10.
(b) Proline ở pH 10.
(c) Tyrosine ở pH 9.
(d) Asparagine ở pH sinh lí học.
(e) Histidine ở pH sinh lí học
(f) Glutamic acid ở pH 3.
2) Giải thích tại sao ở pH 11 thì arginine là chất nhường proton hiệu quả hơn
asparagine?
3) Sử dụng dữ kiện ở bảng pKa ở trên để tính điểm đẳng điện pI của các amino
acid sau:
(a) Aspartic acid
(b) Leucine
(c) Lysine
(d) Proline

1147 | Câu hỏi lí thuyết Hóa học hữu cơ OlympiaVN


4) Tiến hành điện di một hỗn hợp chứa phenylalanine, tryptophan, và leucine.
Xác định xem amino acid nào di quyển xa nhất, giả sử rằng thí nghiệm được
thực hiện ở pH: a) 6.0; b) 5.0.
5) Các amino acid quang hoạt bị racemic hóa ở vị trí α khi xử lí với base mạnh.
Đề xuất cơ chế của quan sát này.

1148 | Câu hỏi lí thuyết Hóa học hữu cơ OlympiaVN


Hướng dẫn
1)

2) Arginine có nhánh phụ có tính base, trong khi đó asparagine thì không. Ở pH
11, arginine tồn tại chủ yếu ở dạng trong đó nhánh phụ bị proton hóa. Do đó, nó
đóng vai trò như một chất nhường proton.
3)
Aspartic acid: Leucine:

Lysine: Proline:

4) pI của Phe = 5.48, pI của Trp = 6.11 và Leu = 6.00. Sử dụng các giá trị này,
chúng ta thực hiện các dự đoán sau: (a) pH = 6.0 Phe sẽ di chuyển xa nhất; (b) ở
pH = 5.0, Trp sẽ di chuyển xa nhất.

1149 | Câu hỏi lí thuyết Hóa học hữu cơ OlympiaVN


5)

1150 | Câu hỏi lí thuyết Hóa học hữu cơ OlympiaVN


Bài 4
1) Dự đoán sản phẩm chính giữa L-valine với:
(a) MeOH, H+
(b) Di-tert-butyl-dicarbonate
(c) NaOH, H2O
(d) HCl
2) Trình bày cơ chế của phản ứng sau:

3) Khi xử lí hợp chất sau với HCl đặc ở 100 oC trong nhiều giờ thì xảy ra sự
thủy phân, tạo thành một trong 20 amino acid tồn tại trong tự nhiên. Xác định
amino acid đó.

1151 | Câu hỏi lí thuyết Hóa học hữu cơ OlympiaVN


Hướng dẫn
1)
a) b)

c) d)

2)

1152 | Câu hỏi lí thuyết Hóa học hữu cơ OlympiaVN


3)

1153 | Câu hỏi lí thuyết Hóa học hữu cơ OlympiaVN


Bài 5
Các L-α-amino acid đóng vai trò quan trọng trong tất
cả các sinh vật. Histidine (dưới đây) có giá trị pKa2
là 6.00. Nó là amino acid duy nhất góp phần vào khả
năng đệm của máu (pH = 7.40). Glutamic acid (2-
aminopentane diacid) và aspartic acid (2-aminobutan
diacid) là các chất dẫn truyền thần kinh quan trọng.
Cysteine (2-amino-3-mercapto propanoic acid,
mercapto = thiol) đóng vai trò quan trọng trong cấu
trúc protein bởi nó có thể tạo thành các cầu disulfide
bền. Hai phân tử cysteine có thể liên kết với nhau qua
một cầu disulfide tạo thành dicysteine (cystine).
a) Tiểu phân nào của histidine tồn tại trong cân bằng ở các giá trị pH 1.82 và
9.17 (đó cũng là các giá trị pKa khác của histidine)? Chỉ có duy nhất một
trong hai nguyên tử nitrogen trong vòng có thể bị proton hóa, đó là nguyên
tử nào?
b) Vẽ cấu trúc L-glutamic acid theo công thức chiếu Fischer.
c) Vẽ cấu trúc 3D của S-aspartic acid và R-cysteine.
Trong các protein và peptide, các amino acid liên kết với nhau bởi các liên kết
peptide - hợp chất tạo thành được gọi là carboxylic acid amide. Trong một số giáo
trình, phản ứng được viết như sau:

Phản ứng chỉ thực sự diễn ra theo cách này trong những điều kiện khắc nghiệt bởi
carboxylic acid không đủ hoạt tính với các tác nhân nucleophile (hoạt tính
carbonyl thấp).
d) Hiệu ứng điện tử nào chịu trách nhiệm cho hoạt tính carbonyl thấp của các
carboxylic acid.

1154 | Câu hỏi lí thuyết Hóa học hữu cơ OlympiaVN


Để làm tăng hoạt tính của các carboxylic acid, chúng có thể được chuyển thành
các halide hoặc anhydride.
e) Trình bày cơ chế tạo thành amide giữa tác nhân hoạt động RCOX và amine
R’NH2.
f) Viết phương trình phản ứng tạo thành dicysteine từ cysteine (không cần xét
đến hóa lập thể). Phản ứng thuộc loại nào?
Hướng dẫn
a) Chỉ có nguyên tử nitrogen có liên kết đôi với nguyên tử carbon được proton
hóa. Cặp electron tự do của nguyên tử N còn lại thuộc hệ thơm và do đó chỉ
có tính base yếu.

b)

c)

d) Hợp phần OH của nhóm carboxyl gây ra hiệu ứng +M vào nguyên tử carbonyl
carbon và làm giảm tính electrophile của nó.
e)

1155 | Câu hỏi lí thuyết Hóa học hữu cơ OlympiaVN


f) Đây là loại phản ứng oxid hóa-khử (chính xác là: ghép cặp oxid hóa):

1156 | Câu hỏi lí thuyết Hóa học hữu cơ OlympiaVN


Bài 6
Amino acid là các khối cấu trúc của protein. Sự có mặt của các nhóm -NH2 và -
COOH tạo nên bản chất lưỡng tính của amino acid. Một số nhánh amino acid
trong các protein cực kì quan trọng với hoạt tính và vai trò xúc tác của chúng.
Glutamic acid (dưới đây) là một trong các amino acid như vậy.

a) Tại sao pKa của nhóm α-COOH thấp hơn nhóm γ-COOH?
b) Tính % nhóm γ-COOH vẫn ở dạng không bị ion hóa ở pH 6.3.
c) Glutamic acid được tiêm vào giấy điện di ở pH = 3.25. Nó sẽ di chuyển về
andoe (+) hay cathode (-)? Tại sao?
Sự thủy phân các polysaccharide như chitin, cellulose và peptidoglycan là một
quá trình hóa sinh phổ biến. Quá trình này liên quan đến sự thủy phân các liên kết
glycosidic như cầu β-1,4 dưới đây.

Một phản ứng thủy phân như vậy được xúc bởi lysozyme.
d) Giả sử rằng phản ứng xúc tác bởi lysozyme được tiến hành trong nước được
làm giàu với 18O, hãy dự đoán 18O có được đưa vào sản phẩm không? Nếu có
thì ở đâu?
Đồ thị pH-hoạt tính của lysozyme được cho trong hình dưới đây.

1157 | Câu hỏi lí thuyết Hóa học hữu cơ OlympiaVN


e) Giải thích đồ thị này theo 2 carboxylate (Asp-52 và Glu-35) có ở tâm hoạt
động lysozyme (Chú ý: các nhóm có thể ion hóa trên chất nền không tham
gia). Viết trạng thái lí tưởng của quá trình ion hóa ở tâm hoạt hóa lysozme tại
pH 5.0.
f) pKa của Glu-35 trong tâm hoạt động lysozyme là 6.0 chứ không phải 4.3 như
được tìm thấy ở amino acid tự do. Những hiệu ứng cục bộ nào sau đây có liên
quan?
1. Điện tích âm được tăng cường.
2. Điện tích dương được tăng cường.
3. Độ phân cực được tăng cường.
4. Độ phân cực suy giảm.
Các phản ứng hữu cơ kiểu mẫu đã giúp hiểu được nhiều đặc tính của các cơ chế
xúc tác enzyme. Với một phản ứng nội phân tử (giống như các xúc tác enzyme
thực hiện!) thì sự gia tăng tốc độ diễn ra khi nồng độ khả kiến của tác nhân ở tâm
tăng lên rất nhiều. Nhóm carboxylate hỗ trợ cho phản ứng thủy phân của 3
phenylacetate và các hằng số tốc độ (k) được cho dưới đây

(pseudo first order = giả bậc một)

1158 | Câu hỏi lí thuyết Hóa học hữu cơ OlympiaVN


g) Tính nồng độ cục bộ hiệu dụng của nhóm COO- trong (2) và (3) ở trên.
h) Tại sao hằng số tốc độ của (3) cao hơn (2)?
Hướng dẫn
a) Nhóm amino bị proton hóa gây ra hiệu ứng hút electron. Hiệu ứng này làm
tăng khả năng giải phóng proton từ nhóm -COOH gần đó, bởi nó giúp bền
hóa base liên hợp -COO-. Hiệu ứng này càng lớn khi -COO- càng gần -NH3+.
Do nhóm -NH3+ có mặt ở α-carbon nên hiệu ứng tác động lên α-COOH hơn
là γ-COOH. Do đó pKa của α-COOH thấp hơn γ-COOH.
b) Tỉ lệ nhóm γ-COOH bị ion hóa/chưa bị ion hóa nhận được bằng cách sử dụng
phương trình Henderson-Hasselbalch.

pH = 6.3 và pKa của nhóm γ-COOH là 4.3. Thế các giá trị này vào phương trình
trên, ta có:

c) Glutamic acid có hai giá trị pKa thấp hơn 7.0 và một giá trị pKa cao hơn 7.0.
Do đó, điểm đẳng điện (pI) của glutamic acid sẽ nằm giữa 2 giá trị pKa có
tính acid: pI = (2.2 + 4.3)/ 2 = 3.25. Ở pH 3.25, điện tích toàn phần của
glutamic acid sẽ bằng 0 do pH này trùng khớp với pI của glutamic acid. Do
đó, glutamic acid sẽ đứng yên ở pH 3.25.

1159 | Câu hỏi lí thuyết Hóa học hữu cơ OlympiaVN


d) Trong quá trình thủy phân liên kết glycosidic, oxygen cầu glycosidic tách ra
cùng C4 trên đường B. Sau khi phân cắt, 18O từ nước sẽ được tìm thấy trên
C1 của đường A.

Chú ý: Phản ứng diễn ra với một carbonium ion được bền hóa trên C1 của đường
A.
e) Đa số các glycosidase chứa 2 nhóm carboxylate ở tâm hoạt động của đều rất
quan trọng về mặt xúc tác. Lysozyme hoạt động chỉ khi 1 nhóm carboxylate
được proton hóa và nhóm khác bị deproton hóa. Bờ giảm dần ở phía kiềm
trên đồ thị pH là do sự ion hóa -COOH. Bờ tăng dần ở phía acid là do sự
proton hóa -COO-. Do đó, hoạt tính enzyme giảm mạnh ở hai phía của pH tối
ưu. Trạng thái lí tưởng của sự ion hóa ở pH = 5 sẽ là

f) 2 và 4 đều đúng. Sự ion hóa của -COOH dẫn đến sự tạo thành tiểu phân mang
điện tích âm -COO-. Tiểu phân mang điện này bị kém bền hóa bởi sự phân
cực suy giảm và điện tích âm được tăng cường. Do đó, sự ion hóa nhóm -
COOH bị ức chế và giá trị pKa được nâng lên.

1160 | Câu hỏi lí thuyết Hóa học hữu cơ OlympiaVN


g) Tỉ lệ của hằng số giả-bậc 1 (ở CH3COO- 1 M) trong (a) với các hằng số bậc
1 trong (b) và (c) cho biết các nồng độ cục bộ hiệu dụng. Ví dụ:
(2) (0.4) / (0.002) = 200, nghĩa là nồng độ hiệu dụng = 200 M
(3) (20) / (0.002) = 10000 nghĩa là nồng độ = 10000 M
h) Ngoài hiệu ứng nồng độ cục bộ được tăng cường, nhóm -COO- trong (3) còn
được định hướng tốt hơn để tác động trong xúc tác. Liên kết đôi ngăn trở
chuyển động của -COO- và do đó làm giảm số lượng định hướng không phù
hợp của -COO-, dẫn đến sự tăng tốc độ phản ứng.

1161 | Câu hỏi lí thuyết Hóa học hữu cơ OlympiaVN


Bài 7
2) Xác định các tác nhân cần thiết để tổng hợp các amino acid sau bởi phản ứng
Hell-Volhard-Zelinsky:
(a) Leucine
(b) Alanine
(c) Valine
3) Xác định các tác nhân cần thiết để tạo nên các amino acid qua tổng hợp
amidomalonate:
(a) Isoleucine
(b) Alanine
(c) Valine
4) Xác định các tác nhân cần thiết để tạo nên các amino acid sau sử dụng phương
pháp tổng hợp Strecker:
(a) Methionine
(b) Histidine
(c) Phenylalanine
(d) Leucine
5) Mỗi aldehyde sau được chuyển hóa thành α-amino nitrile sau đó thủy phân
tạo thành amino acid. Trong mỗi trường hợp, hãy vẽ công thức và gọi tên
amino acid được tạo thành.
(a) Acetaldehyde
(b) 3-Methylbutanal
(c) 2-Methylpropanal
Hướng dẫn
1)
(a) (b)

(c)

1162 | Câu hỏi lí thuyết Hóa học hữu cơ OlympiaVN


2)

3)
a) b)

c) d)

1163 | Câu hỏi lí thuyết Hóa học hữu cơ OlympiaVN


4)
a) b)

c)

1164 | Câu hỏi lí thuyết Hóa học hữu cơ OlympiaVN


Bài 8
Xác định các sản phẩm hữu cơ được tạo thành trong mỗi phản ứng sau:

Hướng dẫn

1165 | Câu hỏi lí thuyết Hóa học hữu cơ OlympiaVN


Bài 9
1) Khi điều chế leucine bởi tổng hợp amidomalonate thì thu được sản phẩm khí
phụ là isobutylene (còn gọi là 2-methylpropene). Trình bày cơ chế phản ứng
tạo thành sản phẩm phụ này.
2) Xác định các trung gian được kí hiệu bằng chữ cái sau đây. Đây là một
phương pháp thay thế để tổng hợp các amino acid, dựa vào tổng hợp Gabriel
của amine bậc 1.

3) Glutamic acid được tổng hợp theo chuỗi phản ứng sau. Trình bày cơ chế cho
các bước [1]-[3].

Hướng dẫn
1) Enolate ion (tạo thành ở giai đoạn đầu tiên) có thể đóng vai trò như một base,
thay vì nucleophile, dẫn đến phản ứng tách E2:

2)

1166 | Câu hỏi lí thuyết Hóa học hữu cơ OlympiaVN


3)

1167 | Câu hỏi lí thuyết Hóa học hữu cơ OlympiaVN


Bài 10
1) Viết sơ đồ phân giải hai đối quang của racemic lacic acid
[CH3CH(OH)COOH] sử dụng tác nhân phân giải (R)-α-methylbenzylamine.
2) Một chiến lược khác được sử dụng để phân giải các amino acid bao gồm việc
chuyển nhóm carboxy thành ester, sau đó sử dụng carboxylic acid thủ tính để
thực hiện phản ứng acid-base ở nhóm amino tự do. Sơ đồ tổng quát dưới đây
sử dụng (R)-mandelic acid làm tác nhân phân giải. Sử dụng một hỗn hợp đối
quang alanine và (R)-mandelic acid làm tác nhân phân giải, hãy cho biết quá
trình phân giải được thực hiện như thế nào.

3) Brucine là một alkaloid độc tính được phân lập từ Strychnos nux vomica, một
loại cây sinh trưởng ở Ấn Độ, Ski Lanka và Bắc Úc. Trình bày sơ đồ phân
giải brucine bởi hỗn hợp racemic phenylalanine.

1168 | Câu hỏi lí thuyết Hóa học hữu cơ OlympiaVN


Hướng dẫn
1)

1169 | Câu hỏi lí thuyết Hóa học hữu cơ OlympiaVN


2)

1170 | Câu hỏi lí thuyết Hóa học hữu cơ OlympiaVN


3)

1171 | Câu hỏi lí thuyết Hóa học hữu cơ OlympiaVN


Bài 10A
Amide của alanine có thể được phân giải bởi acylase trong thận lợn. Đối quang
nào của alanine bị acyl hóa với acetic anhydride nhanh hơn? Trong phản ứng thủy
phân xúc tác enzyme, đối quang nào thủy phân nhanh hơn? Trong quá trình tách
này, tại sao hỗn hợp được đun nóng trong hỗn hợp acid và cái gì được lọc? Làm
thế nào mà quá trình tách alanine tự do bằng cách hòa tan trong ethanol diễn ra
được?

Ghi chú: pig kidney = thận lợn; filter hot = lọc nóng; cool and filter = để nguội
và lọc; solid material = vật chất rắn; heat = đun nóng; filter = lọc; in solution =
trong dung dịch; crystallizes = kết tinh
Nếu quá trình acyl hóa diễn ra thiếu cẩn thận, đặc biệt nếu đun nóng quá lâu hoặc
quá mạnh thì tạo thành một sản phẩm phụ không bị thủy phân bởi enzyme. Quá
trình này diễn ra như thế nào?

(overheating = đun quá nhiệt)


Hướng dẫn
Phản ứng acyl hóa diễn ra theo cơ chế thông thường để tạo thành amide từ
anhydride, đó là tác kích nucleophile trên nhóm carbonyl và tách anion bền nhất
(acetate) từ trung gian tứ diện. Hai đồng phân alanine là đối quang và các đối
quang cần phải phản ứng với tốc độ như nhau với các tác nhân phi thủ tính.

1172 | Câu hỏi lí thuyết Hóa học hữu cơ OlympiaVN


Trong phản ứng xúc tác enzyme, acylase thủy phân amide của một đối quang
nhưng với đối quang còn lại thì không. Lúc này, hai đối quang không phản ứng
với tốc độ như nhau khi tác nhân (hoặc xúc tác) là đối quang tinh khiết của peptide
lớn. Không có gì bất ngờ, enzyme phân cắt amide của alanine tự nhiên và để lại
đối quang còn lại:

Quá trình tinh chế và lọc trước tiên đòi hỏi việc loại bỏ enzyme. Nó tan ở hệ đệm
pH 8 nhưng sự acid hóa và đun nóng làm biến tính enzyme (khá giống những gì
xảy ra với lòng trắng trứng khi đun nóng) và phá hủy cấu trúc của nó. Chất rắn
được lọc tách là enzyme đã biến tính này. Quá trình tách với ethanol diễn ra bởi
các amino acid phân cực chỉ tan trong nước trong khi đó amine lại tan trong
ethanol.

Sự đun nóng quá mức dung dịch acid gây ra sự vòng hóa của nguyên tử amide
oxygen vào carboxylic acid. Phản ứng diễn ra bởi sự tạo thành vòng năm cạnh,
một “azalactone”.

1173 | Câu hỏi lí thuyết Hóa học hữu cơ OlympiaVN


Bài 11
1) Xác định các alkene ban đầu cần thiết để tổng hợp các amino acid sau, sử
dụng phương pháp hydrogen hóa xúc tác bất đối:
(a) L-Alanine
(b) L-Valine
(c) L-Leucine
(d) L-Tyrosine
Ví dụ về tổng hợp L-phenylalanine theo phương pháp này:

2) Giải thích tại sao việc sử dụng xúc tác bất đối không phù hợp cho tổng hợp
glycine?
3) Xác định sản phẩm của các phản ứng sau:

1174 | Câu hỏi lí thuyết Hóa học hữu cơ OlympiaVN


Hướng dẫn
1)
a) b)

c) d)

2) Glycin không có tâm thủ tính, do đó việc sử dụng xúc tác bất đối là không cần
thiết. Ngoài ra, không có alkene nào có thể tạo thành glycine bằng cách hydrogen
hóa.
3)

1175 | Câu hỏi lí thuyết Hóa học hữu cơ OlympiaVN


Bài 12
Sử dụng công thức chiếu Fischer hãy hoàn thành chuỗi phản ứng sau :

Hướng dẫn

1176 | Câu hỏi lí thuyết Hóa học hữu cơ OlympiaVN


Bài 13
Alanin được tổng hợp theo sơ đồ sau đây. Hãy xác định cấu trúc các chất chưa
biết:

Hướng dẫn

1177 | Câu hỏi lí thuyết Hóa học hữu cơ OlympiaVN


Peptide
Bài 1
1) Vẽ cấu trúc các peptide sau:
(a) Leu-Ala-Gly
(b) Cys-Asp-Ala-Gly
(c) Met-Lys-His-Tyr-Ser-Phe-Val
2) Vẽ cấu dạng s-trans của dipeptide Phe-Leu và chỉ rõ đầu N, đầu C.
3) Vẽ cấu dạng s-cis của dipeptide Phe-Phe và chỉ rõ nguồn gốc tương tác không
gian lớn liên quan đến cấu dạng này.
4) Biểu diễn cấu trúc không gian của tetrapeptide tạo thành khi nối hai phân tử
Cys-Phe bởi một cầu disulfide.
5) Biểu diễn cấu trúc peptide tương ứng với trình tự các gốc amino acid sau và
chỉ rõ các đầu N, C: Trp-Val-Ser-Met-Gly-Glu
6) Methionine enkephalin là một pentapeptide được tạo thành bởi cơ thể để chế
ngự cơn đau. Từ trình tự các gốc amino acid, vẽ cấu trúc methionine
enkephalin.

7) Từ trình tự chuỗi amino acid, hãy dự đoán dạng nào của aspartame tồn tại
chủ yếu ở pH sinh lí:

1178 | Câu hỏi lí thuyết Hóa học hữu cơ OlympiaVN


Hướng dẫn
1)

2)

3) Trong cấu dạng s-cis, các nhóm phenyl sẽ chịu tương tác không gian mạnh, do
đó khiến cho cấu dạng s-cis có năng lượng cực kì cao.

1179 | Câu hỏi lí thuyết Hóa học hữu cơ OlympiaVN


4)

5)

6)

7) Ở pH sinh lí học, nhóm carboxylic acid bị deproton hóa (và sẽ tồn tại chủ yếu
ở dạng carboxylate ion), trong khi đó nhóm amino bị proton hóa (và sẽ tồn tại chủ
yếu ở dạng ammonium ion):

1180 | Câu hỏi lí thuyết Hóa học hữu cơ OlympiaVN


Bài 2
1) Cystine (C6H12N2O4S2) là một diamino-dicarboxylic acid và là dimer của L-
cysteine. Dimer này có thể bị phân cắt bằng cách xử lí với một thiol như
mercaptoethanol (HOCH2CH2SH), tạo thành L-cysteine (C3H7NO2S).
a) Xác định cấu trúc cystine. Chỉ rõ cấu hình tuyệt đối.
b) Vai trò của mercaptoethanol trong phản ứng này là gì?
Cysteine (1 mol) cũng có thể bị phân cắt khi xử lí với performic acid (HCOO2H),
tạo thành cysteic acid, C3H7NO5S (2 mol) - một acid mạnh.
c) Xác định cấu trúc của cysteic acid ở điểm đẳng điện.
d) Khi xử lí một peptide - chứa 2 chuỗi, A và B, liên kết với nhau bởi một
liên kết đơn disulfide giữa hai gốc cysteine trong mỗi chuỗi - với
perfomic acid thì thu được hai peptide biến tính ở pH 7.0, A’ và B’, có
điện tích toàn phần lần lượt là +5 và -3. Tính điện tích toàn phần của
peptide ban đầu ở pH như trên.
2) Khi peptide C (khối lượng phân tử 464.5) bị thủy phân hoàn toàn trong dung
dịch HCl thì thu được các lượng đẳng mol của glycine (Gly), phenylalanine
(Phe), aspartic acid (Asp), glutamic acid (Glu) và 1 đương lượng ammonia
(NH3). Khi xử lí C với enzyme carboxypeptidase thì thu được glutamic acid
và một tripeptide. Thủy phân acid không hoàn toàn tripeptide này thu được
một hỗn hợp sản phẩm, hai trong số đó được xác định là glycylaspartic acid
(Gly-Asp) và aspartylphenylalanine (Asp-Phe).
a) Từ thông tin ở trên, hãy suy ra trật tự chuỗi hoàn chỉnh của peptide C.
b) So sánh điểm đẳng điện gần đúng của peptide C với pH 7.0. ươ

1181 | Câu hỏi lí thuyết Hóa học hữu cơ OlympiaVN


Hướng dẫn
1) a)

b) Chất khử.
c)

d) +4.
2) a) Gly-Asp-Phe-Glu
b) pI < 7.0

1182 | Câu hỏi lí thuyết Hóa học hữu cơ OlympiaVN


Bài 3
1) Xác định các gốc amino acid tạo nên peptide dưới đây:

2) Bacitracin A được tạo thành bởi vi khuẩn và do đó có chứa một số gốc không
thuộc về nhóm 20 amino acid tồn tại tự nhiên.
a) Xác định các gốc amino acid trong peptide này.
b) Xác định các amino acid nào có cấu hình D thay vì L.
c) Xác định tất cả các gốc không có trong nhóm 20 amino acid tồn tại tự
nhiên.

3) Các nhà nghiên cứu tin rằng các kháng sinh penicillin được tổng hợp từ các
tiền chất amino acid. Xác định hai amino acid nhiều khả năng đã tham gia
vào tổng hợp các kháng sinh penicillin:

1183 | Câu hỏi lí thuyết Hóa học hữu cơ OlympiaVN


4) Protein huỳnh quang xanh lục (GPP), được cô lập lần đầu tiên từ sứa phát
quang sinh học, là một protein chứa 238 gốc amino acid. Sự khám phá GFP
đã cách mạnh hóa lĩnh vực hiển vi huỳnh quang, cho phép các nhà hóa sinh
theo dõi quá trình sinh tổng hợp các protein. Giải Nobel Hóa học năm 2008
được rao cho Martin Chalfie, Osamu Shimomura, và Roger Tsien vì đã khám
phá và phát triển GFP. Khối con cấu trúc (subunit) của GFP chịu trách nhiệm
cho sự phát huỳnh quang, gọi là fluorophore - được tạo thành khi ba gốc
amino acid trải qua sự vòng hóa. Xác định ba amino acid cần cho quá trình
sinh tổng hợp fluorophore.

1184 | Câu hỏi lí thuyết Hóa học hữu cơ OlympiaVN


Hướng dẫn
1)

2)

3)

1185 | Câu hỏi lí thuyết Hóa học hữu cơ OlympiaVN


4)

1186 | Câu hỏi lí thuyết Hóa học hữu cơ OlympiaVN


Bài 4
Sự racemic hóa các α-amino acid và peptide có thể xảy ra bởi cơ chế α-enol hóa
và tác động đun nóng hay có mặt base mạnh đều thúc đẩy cho quá trình này:

1) Vẽ công thức hóa lập thể I và II (với các liên kết đậm và kẻ vạch) cho các
hợp phần amino acid của hỗn hợp đạt tới cân bằng qua cơ chế α-enol hóa
được mô tả ở trên khi tiến hành với mỗi hydroxyamino acidA và B sau:

2) Xác định mối liên hệ giữa các cấu trúc mà bạn vẽ trong mỗi trường hợp A, B
ở trên.
AI, II □ đối quang □ đồng phân dia
BI ,II □ đối quang □ đồng phân dia
Trong tổng hợp peptide, để tạo thành liên kết peptide mới, nhóm carboxyl cần
phải được hoạt hóa - nó cần phải mang một nhóm rời đi tốt - được biểu diễn như
trong sơ đồ đơn giản hóa sau:

Chính ở giai đoạn này của tổng hợp, cơ chế racemic hóa thứ hai có thể diễn ra.
Nguyên tử oxygen của nhóm amidic carbonyl nằm cách xa 5 nguyên tử so với
nhóm carboxyl hoạt hóa và có thể tấn công nội phân tử vào nhóm hoạt hóa này,

1187 | Câu hỏi lí thuyết Hóa học hữu cơ OlympiaVN


tạo thành trung gian vòng 5 cạnh (một azalactone), chất này nhanh chóng đạt tới
cân bằng diễn ra tại tâm lập thể như biểu diễn trong sơ đồ đơn giản hóa dưới đây:

3) Xác định công thức cấu tạo của trung gian C trong chuyển hóa qua lại giữa
hai azalactone và giải thích sự xáo trộn ở tâm lập thể.
Các azalactone là những hợp chất rất hoạt động, có thể phản ứng với nhóm amino
của một amino acid. Do đó, phản ứng ghép cặp có thể diễn tới hoàn toàn dẫu cho
có các sản phẩm racemic hóa hoặc epimer hóa.
4) Nếu N-benzoyl glycine, C9H9NO3, được đun tới 40 oC với acetic anhydride
thì nó có thể chuyển thành hợp chất hoạt động mạnh, C9H7NO2 (P1).
a) Đề xuất cấu trúc của P1.
b) Viết các sản phẩm tạo thành từ phản ứng của P1 với S-alanine ethyl ester
(P2) (mạch nhánh R của amino acid alinine là nhóm methyl) sử dụng các
công thức hóa lập thể (với các liên kết đậm và kẻ vạch) cho cả chất phản
ứng và sản phẩm.

1188 | Câu hỏi lí thuyết Hóa học hữu cơ OlympiaVN


Hướng dẫn
1)

2) AI, II là đối quang; BI, II là đồng phân dia.


3) Trung gian C

4)

1189 | Câu hỏi lí thuyết Hóa học hữu cơ OlympiaVN


Bài 5
Protein hiện diện trong tất các các tế bào sống và thực hiện vô số các chức năng
khác nhau trong hóa học của sự sống. Chúng được tạo bởi các α-aminocarboxylic
acid. Peptide là các protein “nhỏ” với lượng tương đối ít các amino acid. Liên kết
peptide là liên kết amide tạo bởi tương tác của nhóm amine trong amino acid. với
nhóm carboxylic acid của phân tử kế cận.
1) Có những peptide nào được hình thành từ phenylalanine F và alanine A? Viết
cấu trúc của chúng.

Trong phân tích cấu trúc của peptide các gốc ở đầu N và đầu C có vai trò quan
trọng.
Phương pháp Sanger để xác định gốc ở đầu N (là gốc amino acid trong peptide
với nhóm NH2 tự do) bao gồm xử lí với 2,4-dinitrofluorobenzene trong môi
trường kiềm, tiếp theo là sự thủy phân hoàn toàn các liên kết peptide trong môi
trường acid. Amino acid đầu N khi ấy có màu vàng có thể xác định dễ dàng trong
phương pháp phân tích sác kí trên giấy. Sanger nhận giải Nobel năm 1958 và
1980.
2) Phản ứng nào diễn ra với thuốc thử Sanger (để đơn giản, có thể viết phía đầu
N của peptide là H2NR). Viết phương trình phản ứng.
Gốc ở đầu C, có chứa nhóm COOH trong peptide, được xác định bằng sự thủy
phân chọn lọc xúc tác enzyme gốc amino acid đầu C bằng carboxypeptidase (có
trong tuyến tụy). Với một tetrapeptide tạo bởi các amino acid F, A, Glyxine G và
Leucine L, gốc ở đầu C được xác định là F. Phương pháp Sanger xác định gốc ở
đầu N là G.
3) Suy ra các cấu trúc có thể có của tetrapeptide này. Viết các cấu trúc tương
ứng.

1190 | Câu hỏi lí thuyết Hóa học hữu cơ OlympiaVN


Hướng dẫn
1) Công thức của các peptide nhận được:
Ph Ph
O O O O
H H H H
N N N N
H2 N OH H 2N OH H 2N OH H 2N OH
O O O O
Ph Ph
FA AF FF AA

2) Phản ứng xảy ra:


NO 2 NO 2

O 2N F + 2H 2NR O 2N NHR + RNH 3+F-

3) Không thể biết được thứ tự hai amino acid giữa là AL hay LA nên peptide ban
đầu có có thể có cấu tạo như sau: GALF hoặc GLAF.

1191 | Câu hỏi lí thuyết Hóa học hữu cơ OlympiaVN


Bài 6
Giải thích một số vấn đề sau đây
1. Peptit Gly-Ala-Arg-Ala-Glu dễ dàng bị cắt mạch bằng dung dịch nước chứa
enzym trypsin ở pH = 8, nhưng lại hoàn toàn trơ với dung dịch trypsin trong
ure ở cùng pH
2. Một đoạn peptit chứa cystin (có cầu nối disunfua) sau khi xử lý với
HSCH2CH2OH rồi xử lý tiếp với aziridin lại có thể bị cắt mạch bằng enzym
trypsin
3. Oxy hóa L-methionin bằng H2O2 thu được hai sản phẩm
4. Benzamidine clorua có thể làm mất hoạt tính của enzym trypsin (Cho rằng
chất này không làm biến tính peptit ở điều kiện đang xét)
5. Axit Polyglutamic tồn tại ở dạng xoắn ở pH < 3. Tuy nhiên khi tăng dần pH
đến pH = 5 thì xảy ra sự tháo xoắn.
Hướng dẫn
1. Trong môi trường ure ở cùng pH thì đầu guanidin của araginin đã phản ứng
với ure cho sản phẩm biến tính. Do cấu trúc chuỗi peptit thay đổi nên sự cắt
mạch bằng trypsin không xảy ra

2. Phản ứng xảy ra với chuỗi peptit như sau:

Cấu trúc sản phẩm cuối cũng tương tự như cấu trúc của phần mạch nhánh lysine
nên có thể bị cắt mạch bởi trypsin
3. Thu được sunfoxit và có khả năng có cả sunfon.
4. Do benzamidin clorua có nhóm chức guanidin nên có khả năng cạnh tranh
tâm hoạt động với trypsin. Tâm hoạt động của trypsin đã bị chiếm nên enzym
mất đi hoạt tính.
5. Trong axit polyglutamic vẫn còn một nhóm COOH tự do. Khi tăng pH thì
nhóm COOH này sẽ chuyển thành dạng COO-. Các nhóm này ở gần nhau
trong dạng xoắn sẽ đẩy lẫn nhau gây ra sự tháo xoắn.
Bài 7
Glutathione, kí hiệu GSH, là một peptide nhỏ, hiện diện trong hầu hết mô của các
loài động vật. GSH đáp ứng các chức năng sinh học quan trọng, như giải độc các
hóa chất có ái lực điện tử (electrophilic) và khử các peroxide (hữu cơ) trong máu.

1192 | Câu hỏi lí thuyết Hóa học hữu cơ OlympiaVN


Một hợp chất ái điện tử có phản ứng bất thuận nghịch với GSH, đặc biệt là trong
gan, tạo thành sản phẩm chính được biến đổi bởi một loạt sinh chuyển hóa, tạo
thành một mercapturic acid - chất này được bài tiết qua nước tiểu. Các chất oxid
hóa phản ứng với GSH tạo thành disulfide GSSG, chất này có thể được hoàn
nguyên enzyme về GSH bởi các men khử. Tỉ lệ GSH/GSSG trong hầu hết các tế
bào là ≥ 500.

1)
a) Có bao nhiêu gốc amino acid trong GSH?
b) Vẽ cấu trúc của các amino acid tương ứng và đánh dấu các tâm chiral bởi
dấu hoa thị.
Mercapturic acid A được phân lập từ nước tiểu của một người đã tiếp xúc với
acrylonitrile (H2C=CH-CN) có công thức phân tử là C8H12N2O3S. Phổ 1H NMR
của A trong (CD3)2SO được cho trong hình 1. Khi sản phẩm này được xử lí trước
với D2O thì các tín hiệu ở δ 12.8 và δ 6.8 không xuất hiện nữa, còn tín hiệu 3 thì
được đơn giản hóa.

1193 | Câu hỏi lí thuyết Hóa học hữu cơ OlympiaVN


2)
a) Các tín hiệu NMR tương ứng với các proton trong các nhóm sau: CH,
CH2, CH3, OH và NH. Hãy chỉ ra nhóm proton thích hợp với các tín hiệu
1-7.

b) Có bao nhiêu nguyên tử carbon trong hợp chất A mà không liên kết với
bất kì proton nào?
c) Vẽ cấu trúc hợp chất A.
Vitamin C (ascorbic acid) phản ứng với các chất oxid hóa tạo thành
dehydroascorbic acid D

3) Việc ăn rau và hoa quả tươi rất tốt cho sức khỏe, bởi vì
□ Vitamin C tạo thành phức chất với GSH.
□ Vitamin C phản ứng với các hợp chất ái điện tử.
□ Vitamin C loại bỏ các chất oxid hóa và ngăn sự suy giảm không mong
muốn của GSH.
□ Nhiều lý do, nhưng không liên quan đến GSH.

1194 | Câu hỏi lí thuyết Hóa học hữu cơ OlympiaVN


Hướng dẫn
1) a) 3 gốc amino acid.
b)

2) a)

b) 3
c)

3) Vitamin C loại bỏ các chất oxid hóa và ngăn sự suy giảm không mong muốn
của GSH.

1195 | Câu hỏi lí thuyết Hóa học hữu cơ OlympiaVN


Bài 8
1) Thủy phân không hoàn toàn octapeptide angiotensin II tạo thành produces
Pro-Phe, Val-Tyr-Ile, Asp-Arg-Val, và Ile-His-Pro. Xác định trình tự peptide.
2) Xử lí somatostatin với tác nhân Edman tạo thành một dẫn xuất của alanine.
Thủy phân không hoàn toàn polypeptide này tạo thành các oligopeptide sau
đây. Xác định cấu trúc polypeptide.
I: Phe-Trp II: Lys-Thr III: Thr-Ser-Cys
IV: Thr-Phe-Thr-Ser-Cys V: Asn-Phe-Phe-Trp-Lys
VI: Ala-Gly-Cys-Lys-Asn-Phe
3) Thành phần amino acid của một peptide là (Arg2, Gly, Phe2, Pro3, Ser). Xử lí
peptide này với tác nhân Edman tạo thành dẫn xuất PTC của arginine. Thủy
phân không hoàn toàn tạo thành nhiều mảnh peptide gồm các oligopeptide
sau. Xác định trình tự amino acid của peptide này.
I: Gly-Phe-Ser II: Arg-Pro-Pro-Gly III: Phe-Arg-Ser-Pro-Phe
4) Các giai đoạn cuối trong thoái phân Edman dẫn đến sự chuyển vị thiazolinone
thành mộ N-phenylthiohydantoin. Trình bày cơ chế từng bước cho phản ứng
xúc tác acid này.

1196 | Câu hỏi lí thuyết Hóa học hữu cơ OlympiaVN


Hướng dẫn
1)

2)

3) Xử lí với tác nhân Edman giúp xác định arginine là amino acid đầu N.

1197 | Câu hỏi lí thuyết Hóa học hữu cơ OlympiaVN


4)
c

1198 | Câu hỏi lí thuyết Hóa học hữu cơ OlympiaVN


Bài 9
1) Pentapeptide met-enkephalin bị thủy phân bởi chymotripsin tạo thành Met,
Tyr, và Gly-Gly-Gly. Thông tin này đã đủ giúp thiết lập trình tự các amino
acid trong met-enkephalin chưa?
2) Tetrapeptide tuftsin bị thủy phân bởi trypsin tạo thành Pro-Arg và Thr-Lys.
Thông tin này đã đủ giúp thiết lập trình tự các amino acid trong tuftsin chưa?
Hướng dẫn
1) Chưa đủ. Chymotrypsin phân cắt các peptide ở đầu C của amoni acid thơm
phenylalanine, tyrosine và trytophan. Sản phẩm thủy phân có hai amino acid thơm
và chúng có thể dẫn đến một trong hai cấu trúc sau. Các tâm bị phân cắt được chỉ
rõ bằng cách in đậm.
Tyr-Gly-Gly-Phe-Met
Gly-Gly-Phe-Tyr-Met
2) Chưa đủ. Trypsin phân cắt các peptide ở đầu C của các amino acid có tính base
lysine và arginine. Cả hai dipeptide đều có các amino acid có tính base đầu C.
Tetrapeptide có thể là một trong hai cấu trúc sau. Các tâm bị phân cắt được chỉ rõ
bằng cách in đậm.
Pro-Arg-Thr-Lys
Thr-Lys-Pro-Arg

1199 | Câu hỏi lí thuyết Hóa học hữu cơ OlympiaVN


Bài 10
1) Xác định các amino acid và các mảnh peptide tạo thành khi xử lí decapeptide
A-P-F-L-K-W-S-G-R-G với mỗi tác nhân hoặc enzyme sau: (a)
chymotrypsin; (b) trypsin; (c) carboxypeptidase; (d) C6H5N=C=S.
2) Angiotensin là một octapeptide làm thu hẹp các mạch máu, do đó làm tăng
huyết áp. Các chất ức chế ACE là một nhóm thuốc được sử dụng để điều trị
cao huyết áp bằng cách ngăn quá trình tổng hợp angiotensin trong cơ thể.
Angiotensin chứa các amino acid Arg (2 đương lượng), His, Ile, Phe, Pro,
Tyr, và Val. Xác định trình tự angiotensin biết các mảnh sau được tạo thành
bởi quá trình thủy phân không hoàn toàn với acid: Ile-His-Pro-Phe, Arg-Arg-
Val, Tyr-Ile-His, and Val-Tyr
3) Sử dụng các dữ kiện thực nghiệm dưới đây để suy ra trình tự octapeptide chứa
các amino acid sau: Ala, Gly (2 đương lượng), His (2 đương lượng), Ile, Leu,
và Phe. Thoái phân Edman phân cắt Gly từ octapeptide, còn carboxypeptidase
tạo thành Leu và một heptapeptide. Thủy phân không hoàn toàn thu được các
mảnh sau: Ile-His-Leu, Gly, Gly-Ala-Phe-His, và Phe-His-Ile.
4) Một octapeptide chứa các amino acid sau: : Arg, Glu, His, Ile, Leu, Phe, Tyr,
và Val. Xử lí octapeptide trên với carboxypeptidase tạo thành Phe và một
heptapeptide. Xử lí octapeptide với chymotrypsin tạo thành hai tetrapeptide,
A và B. Xử lí A với trypsin tạo thành hai dipeptide, C và D. Thoái phân
Edman phân cắt các amno acid từ mỗi peptide: : Glu (octapeptide), Glu (A),
Ile (B), Glu (C), và Val (D). Thủy phân không hoàn toàn tetrapeptide B tạo
thành Ile-Leu cùng với các sản phẩm khác. Suy ra cấu trúc của octapeptide
và các mảnh A-D.
Hướng dẫn
1)

2)

3)

1200 | Câu hỏi lí thuyết Hóa học hữu cơ OlympiaVN


4)

1201 | Câu hỏi lí thuyết Hóa học hữu cơ OlympiaVN


Bài 11
Một trong số những neuropeptit (những peptit ảnh hưởng trực tiếp đến hệ thần
kinh) chính là β-neoendorphin phân lập từ não lợn. Kết quả phân tích sơ bộ cho
thấy nó gồm 9 aminoaxit liên kết trực tiếp với nhau. Để xác định trình tự sắp xếp
các aminoaxit này thì đầu tiên người ta tiến hành cắt mạch bằng enzym
dipeptidiaminopeptidaza (cắt mạch từ aminoaxit thứ hai tính từ đầu N). Tiếp theo
cắt mạch nonapeptit này bằng enzym dipeptidicacboxipeptidaza (cắt mạch từ
aminoaxit thứ 2 tính từ đuôi C) thu được kết quả như sau:
- Dipeptidiaminopeptidaza: Gly-Phe; Leu-Arg ; Tyr-Gly; Lys-Tyr; Pro
- Dipeptidicacboxipeptidaza: Phe-Leu; Gly-Gly; Arg-Lys; Tyr-Pro; Tyr
1. Xác định trật tự sắp xếp các aminoaxit trong β-neoendorphin
2. Trong dãy các dipeptit trên thì dipeptit nào
a) Không quang hoạt?
b) Có điểm đẳng điện > 7
c) Khó bị thủy phân để tạo aminoaxit tự do nhất
Giải thích
Hướng dẫn
1. Tyr – Gly – Gly – Phe – Leu – Arg – Lys – Tyr – Pro
2. a) Gly – Gly. Vì Gly là aminoaxit duy nhất không có tính quang hoạt
b) Leu – Arg; Lys – Tyr; Arg – Lys. Vì tất cả chúng đều còn một nhóm
NH2 tự do
c) Tyr – Pro. Do liên kết amit trong dipeptit này được tạo thành từ amin
bậc 2 nên khó bị thủy phân hơn nhiều.

1202 | Câu hỏi lí thuyết Hóa học hữu cơ OlympiaVN


Bài 12
Thủy phân toàn phần một hexapeptit X có hoạt tính sinh học thu được hỗn hợp 3
aminoaxit Arg, Trp và Phe với tỉ lệ tương ứng 3:2:1. Tiến hành thủy phân X bằng
trypsin rồi xác định bằng DNFB thu được DNB-Trp, Phe, Trp, DNB-Arg và Arg
với tỉ lệ tương ứng 1:1:1:2:1. Còn nếu tiến hành tương tự với chymotrypsin thu
được DNB-Arg, Arg, DNB-Phe và Trp với tỉ lệ tương ứng 1:2:1:2.
Vẽ công thức cấu tạo của X nếu biết rằng MX = 987 g/mol
Hướng dẫn
Tổng 3Arg + 2 Trp + 1Phe = 1095, chênh 108 đơn vị so với giá trị MX thực tế,
ứng với 6 phân tử nước. Hay nói cách khác một hexapeptit tạo được 6 liên kết
peptit chứng tỏ nó có cấu trúc vòng.
Dựa trên cấu trúc sản phẩm thủy phân từng phần có thể kết luận hexapeptit X có
cấu trúc: xiclo – (Trp – Arg – Arg – Arg – Trp – Phe).
X không phản ứng với DNFB nên X có công thức cấu tạo như sau:

1203 | Câu hỏi lí thuyết Hóa học hữu cơ OlympiaVN


Bài 13
1) Bradykinin có trình tự như sau:
(đầu N) Arg-Pro-Pro-Gly-Phe-Ser-Pro-Phe-Arg (đầu C)
Xác định tất cả các mảnh sẽ được tạo thành khi xử lí bradykinin với:
a) Trypsin
b) Chymotrypsin
2) Xác định gốc đầu N của peptide tạo thành dẫn xuất PTH
(phenylthiohydantoin) khi thoái phân Edman:

3) Xử lí một tripeptide với phenyl isothiocyanate tạo thành hợp chất A và một
dipeptide. Xử lí dipeptide với phenyl isothiocyanate tạo thành hợp chất B và
glycine. Xác định cấu trúc tripeptide ban đầu.

Hướng dẫn
1) (a) Trypsin xúc tác cho sự thủy phân liên kết peptide ở phía carboxyl của
arginine, tạo thành hai mảnh sau:
Arg + Pro-Pro-Gly-Phe-Ser-Pro-Phe-Arg
(b) Chymotrypsin xúc tác cho sự thủy phân các liên kết peptide ở phía carboxyl
của phenylalanine, tạo thành ba mảnh sau:
Arg-Pro-Pro-Gly-Phe + Ser-Pro-Phe + Arg
2) Nhóm R (bôi màu) trong dẫn xuất PTH giúp xác định gốc đầu N. Do gốc R là
nhóm benzylic (-CH2Ph) nên đó phải là phenylalanine.

1204 | Câu hỏi lí thuyết Hóa học hữu cơ OlympiaVN


3)

1205 | Câu hỏi lí thuyết Hóa học hữu cơ OlympiaVN


Bài 14
1) Đề xuất hai cấu trúc có thể có của một tripeptide chứa glycine, L-alanine, and
L-phenylalanine nhưng không phản ứng với phenyl isothiocyanate.
2) Vẽ cấu trúc dẫn xuất PTH (phenylthiohydantoin) đầu tiên được tạo thành khi
tripeptide Ala-Phe-Val trải qua thoái phân Edman.
3) Một peptide với 22 gốc amino acid được xử lí với trypsin tạo thành bốn mảnh
(fragment), trong khi đó xử lí với chymotrypsin thì tạo thành sáu mảnh. Xác
định chuỗi 22 gốc amino acid trong peptide ban đầu.

4) Xét cấu trúc octapeptide vòng sau. Phân cắt peptide này bởi trypsin có tạo
thành các mảnh khác với phân cắt bởi chymotrypsin không? Giải thích.

5) Glucagon là một peptide hormone được tạo thành bởi tuyến tụy mà nó cùng
với insulin giúp điều hòa mức độ đường huyết. Glucagon được tạo thành bởi
29 gốc amino acid. Xử lí với trypsin tạo thành bốn mảnh, trong khi đó xử lí
với chymotrypsin tạo thành sáu mảnh. Xác định trình tự của các gốc amino
acid trong glucagon và chỉ rõ bất kì cầu disulfide nào nếu có.
Các mảnh trypsin:
His-Ser-Gln-Gly-Thr-Phe-Thr-Ser-Asp-Tyr-Ser-Lys
Ala-Gln-Asp-Phe-Val-Gln-Trp-Leu-Met-Asn-Thr
Tyr-Leu-Asp-Ser-Arg
Arg
Các mảnh chymotrypsin:
His-Ser-Gln-Gly-Thr-Phe
Thr-Ser-Asp-Tyr
Leu-Met-Asn-Thr
Ser-Lys-Tyr

1206 | Câu hỏi lí thuyết Hóa học hữu cơ OlympiaVN


Leu-Asp-Ser-Arg-Arg-Ala-Gln-Asp-Phe
Val-Gln-Trp
Hướng dẫn
1) Nếu một tripeptide không phản ứng với phenyl isothiocyanate thì nó phải có
đầu N tự do. Nó phải là một tripeptide vòng. Dưới đây là hai cấu trúc có thể
có của tripeptide vòng này:

2) Thoái phân Edman sẽ loại bỏ gốc amino acid đầu N và trong tripeptide Ala-
Phe-Val thì đó là Ala. Do đó, aline bị loại bỏ, tạo thành dẫn xuất PTH sau:

3) Chỉ có một trong các mảnh trypsin có đầu C không phải là arginine hoặc
lysine. Mảnh này, kết thúc với valine, phải là mảnh cuối cùng trong chuỗi
peptide. Ba mảnh trypsin còn lại có thể được đặt vào vị trí phù hợp bằng cách
phân tích các mảnh chymotrypsin. Trình tự peptide đúng là:
Ala-Val-Met-Phe-Val-Ala-Tyr-Lys-Pro-Val-Ile-Leu-Arg-Trp-His-Phe-Met-
Cys-Arg-Gly-Pro-Phe-Ala-Val
4) Phân cắt với trypsin sẽ tạo thành Phe-Arg, trong khi phân cắt với chymotrypsin
sẽ tạo thành Arg-Phe. Hai peptide này không giống nhau, chúng là đồng phân cấu
tạo của nhau.
5) His-Ser-Gln-Gly-Thr-Phe-Thr-Ser-Asp-Tyr-Ser-LysTyr-Leu-Asp-Ser-Arg-
Arg-Ala-Gln-Asp-Phe-Val-GlnTrp-Leu-Met-Asn-Thr
(không có cầu disulfide)

1207 | Câu hỏi lí thuyết Hóa học hữu cơ OlympiaVN


Bài 15
Quy trình xác lập trật tự các aminoaxit trong một heptapeptit Y (Met2 ,
Asp, Lys, Arg, Phe, Gly) có hoạt tính sinh học được tiến hành như sau:
Phản ứng với DNFB cho DNP – Met.
Cắt mạch bằng cacboxipeptidaza cũng thu được Met.
Cắt mạch heptapeptit bằng BrCN giải phóng 1 mol homoserin lacton
Cắt mạch bằng chymotrypsin thu được một pentapeptit và một dipeptit.
Khi cho pentapeptit phản ứng với 5-(dimetylamino)naphtalene-1-sunfonyl clorua
rồi cắt mạch thu được dẫn xuất của methionin.
Cắt mạch bằng trypsin thu được hai tripeptit có chứa Met và Arg tự do.
Cắt mạch bằng pepsin cho tetrapeptit (Met, Arg, Lys và Asp) và tripeptit
(Met, Phe và Gly).
Từ những dữ kiện đã cho xác định trật tự sắp xếp các aminoaxit trong
peptit.
Hướng dẫn
Aminoaxit đầu N lẫn đầu C đều là Met.
BrCN đóng vai trò cắt mạch peptit sau Met, và biến đoạn Met bị cắt ra
thành homoserin lacton theo cơ chế:

Chymotrypsin cắt mạch sau aminoaxit thơm nên cấu trúc pentapeptit phải
là (Met - ? -? - ? – Phe).
Việc cắt bằng trypsin cho Arg tự do và hai tripeptit có chứa Met chứng
tỏ Arg phải là aminoaxit thứ 4.
Dựa trên dữ liệu cắt mạch bằng pepsin có thể suy ra được peptit phải có
trật tự sắp xếp như sau: Met – Asp – Lys – Arg – Phe – Gly – Met.

1208 | Câu hỏi lí thuyết Hóa học hữu cơ OlympiaVN


Bài 16
1) Feline gastrin, một loại hormone kích thích tiết dịch dạ dày ở mèo, có thành
phần amino acid là Ala2, Asp, Gly2, Glu5, Leu, Met, Phe, Pro, Trp2, Tyr. Kết
quả phân tích nhóm đầu cho thấy các amino acid đầu C và N lần lượt là Phe
và Glu. Thủy phân với chymotrypsin tạo thành bốn peptide sau. Viết hai trình
tự amino acid có thể có của feline gastrin.
I: Gly-Trp
II: Met-Asp-Phe
III: Glu-Gly-Pro-Trp
IV: Leu-Glu-Glu-Glu-Glu-Ala-Ala-Tyr
2) Corticotropin, một hormone tuyến yên, kích thích vỏ thượng thạn. Khi thủy
phân chymotrypsin tạo thành sáu peptide sau:

Thủy phân không hoàn toàn bởi trypsine tạo thành lysine, arginine và năm
peptide:

Xác định trình tự amino acid của corticotropin.


Hướng dẫn
1)

2) Cả peptide IV và V từ sự thủy phân chymotrypsin đều chứa peptide V từ sự


thỷ phân trypsin. Cả hai peptide II và III từ sự thủy phân trypsin đều có trong
peptide VI từ thủy phân chymotrypsin. Cả hai peptide III và II từ sự thủy phân
chymotrypsin đều có trong peptide IV từ sự thủy phân trypsin. Do đó, ba peptide
sau chứa phần lớn các amino acid:

Tryptophan chỉ có trong hai mảnh. Do đó, dipeptide I từ sự thủy phân bởi
chymotrypsin và peptide I từ sự thủy phân trypsin có thể được kết hợp thành một

1209 | Câu hỏi lí thuyết Hóa học hữu cơ OlympiaVN


chuỗi amino acid Arg-Trp-Gly-Lys. Một phần của chuỗi này được nhân đôi theo
hai amino acid ở các vị trí đầu N của peptide B. Xếp chồng phù hợp thu được:
Arg-Trp-Gly-Lys-Pro-Val-Gly-Lys-Lys-Arg-Pro-Lys-Val-Tyr
Amino acid đầu N của mảnh này chung với arginine của peptide C, không khớp
với arginine của peptide B. Peptide tạo thành với sự xếp chồng phù hợp là:
Ser-Tyr-Ser-Met-Glu-His-Phe-Arg-Trp-Gly-Lys-Pro-Val-Gly-Lys-Lys-Arg-
Pro-Lys-Val-Tyr
Trình tự amino acid của peptide ban đầu là:
Ser-Tyr-Ser-Met-Glu-His-Phe-Arg-Trp-Gly-Lys-Pro-Val-Gly-Lys-Lys-Arg-
Pro-Lys-Val-Tyr-Pro-Asp-Ala-Gly-Glu-Asp-Gln-Ser-Ala-Glu-Ala-Phe-Pro-
Leu-Glu-Phe

1210 | Câu hỏi lí thuyết Hóa học hữu cơ OlympiaVN


Bài 17
Khi thủy phân hoàn toàn một peptit X thu được: Arg, Asp, Cys2 , Gly,
Ile, Leu, Lys, Met, Phe, Pro, Ser. Kết quả phân tích theo phương pháp Edman cho
Leu và Ser, trong khi đó thủy phân bằng enzym cacboxipeptidaza cho Asp và Ile.
Xử lý X với HOCH2CH2SH sau đó cho phản ứng với ICH2COOH rồi
phân cắt mạch bằng trypsin thu được ba dipeptit (Arg, Ser), (Asp, Met), (Cys,
Lys) và một Hexapeptit (Cys, Gly, Ile, Leu, Phe, Pro).
Xử lý X với HOCH2CH2SH sau đó cho phản ứng với BrCH2CH2NH3+Br-
rồi phân cắt mạch bằng trypsin thu được hai dipeptit (Arg, Ser), (Asp, Met), hai
tripeptit (Cys, Gly, Leu) và (Ile, Phe, Pro) cùng với hai aminoaxit tự do là Cys và
Lys.
Cắt mạch bằng pepsin thu được nonapeptit (Arg, Asp, Cys2 , Gly, Leu,
Lys, Met, Ser) và một tripeptit (Ile, Phe, Pro).
Từ các thông tin trên hãy xác định trật tự sắp xếp các aminoaxit trong
peptit X.
Hướng dẫn
Việc cắt mạch bằng Edman thu được hai aminoaxit đầu N là Leu và Ser chứng tỏ
chuỗi peptit có khả năng gồm hai chuỗi peptit nhỏ riêng rẽ được nối lại với nhau
bằng liên kết disunfua của hai phân tử Cys. Trong đó một chuỗi peptit nhỏ có Asp
là aminoaxit đầu C, chuỗi còn lại không có nhóm COOH tự do.
Hợp chất HOCH2CH2SH cắt liên kết disunfua thành hai nhóm SH riêng
rẽ, sau đó xử lý tiếp với ICH2COOH rồi cắt bằng trypsin thu được một hexapeptit
và các dipeptit riêng rẽ. Trypsin cắt sau Arg và Lys nên từ đó suy ra trật tự ba
dipeptit sẽ là Ser – Arg, Cys – Lys và Met – Asp. Như vậy trật tự sắp xếp của một
chuỗi peptit nhỏ sẽ là Ser – Arg – Cys – Lys – Met – Asp.
Việc xử lý bằng BrCH2CH2NH3+Br- khiến cho mạch alkyl nối với Cys
tích điện dương làm cho peptit trở nên nhạy hơn với trypsin nên sự cắt mạch xảy
ra dữ dội hơn. Lúc này đoạn hexapeptit phía trên đã bị cắt thành hai tripeptit nhỏ.
Do Leu được xác định là aminoaxit đầu N nên một tripeptit sẽ có thứ tự Leu –
Gly – Cys.
Trypsin cắt mạch peptit ở đầu NH của aminoaxit thơm nên thứ tự các
aminoaxit trong tripeptit còn lại phải là Phe – Pro – Ile.
Như vậy cấu trúc của chuỗi peptit ban đầu sẽ là:

1211 | Câu hỏi lí thuyết Hóa học hữu cơ OlympiaVN


1212 | Câu hỏi lí thuyết Hóa học hữu cơ OlympiaVN
Bài 18
Peptide A có công thức phân tử 1007. Thủy phân xúc tác acid tạo thành các amino
acid sau với lượng đẳng mol: Asp, Cystine, Glu, Gly, Ile, Leu, Pro, và Tyr (xem
Bảng 1). Oxid hóa A với HCO2OH chỉ tạo thành B, chất này có 2 gốc cysteic acid
(Cya là dẫn xuất cysteine với nhóm thiol bị oxid hóa thành sulfonic acid).
1) Có bao nhiêu nhóm sulfonic acid được tạo thành từ phản ứng oxid hóa một
liên kết disulfide?
Thủy phân không hoàn toàn B tạo thành một số di- và tripeptide (B1-B6). Trình
tự chuỗi của mỗi sản phẩm thủy phân được xác định theo các cách sau đây. Amino
acid đầu N được xác định bằng cách xử lí peptide với 2,4- dinitrofluorobenzene
(DNFB), tạo thành DNP-peptide. Sau khi thủy phân acid hoàn toàn DNP-peptide
thì một DNP-amino acid sẽ được tạo thành, chất này có thể được xác định nhanh
chóng bằng cách so sánh với các DNP-amino acid chuẩn.
2) B1, khi xử lí với DNFB, sau đó thủy phân acid tạo thành một sản phẩm là
DNP-Asp. Thông tin này gợi ý rằng B1 có một gốc aspartic acid ở đầu N.
Biểu diễn cấu trúc của DNP-Asp tại điểm đẳng điện (không yêu cầu hóa lập
thể).
Tiếp theo, amino acid đầu C được xác định bằng cách đun nóng peptide ở 100 oC
với hydrazine, làm phân cắt toàn bộ các liên kết peptide và chuyển tất cả, ngoại
trừ amino acid đầu C, thành các amino acid hydrazide, còn nhóm carboxyl đầu C
giữ nguyên. Theo cách này, các amino acid đầu N và C được xác định và chuỗi
hoàn chỉnh của B1-B6 được cho dưới đây:
B1 Asp-Cya B4 Ile-Glu
B2 Cya-Tyr B5 Cya-Pro-Leu
B3 Leu-Gly B6 Tyr-Ile-Glu
Thủy phân B bởi enzyme từ trực khuẩn Bacillus tạo thành B7-B9 với công thức
như sau:
B7 Gly-NH2 (Glycinamide)
B8 Cya, Glu, Ile, Tyr
B9 Asp, Cya, Leu, Pro
3) Xác định trình tự chuỗi của B8, biết DNP-Cya được tạo thành khi xử lí B8
với DNFB, sau đó thủy phân acid hoàn toàn.
4) Biết các amino acid đầu N và C của B9 được xác định lần lượt là Asp và Leu,
hãy viết trình tự chuỗi của B9.
5) Xác định cấu trúc hoàn chỉnh của A, sử dụng các kí hiệu viết tắt trong Bảng
1, chỉ rõ vị trí của liên kết disulfide.

1213 | Câu hỏi lí thuyết Hóa học hữu cơ OlympiaVN


Tuy nhiên, khối lượng mol tính được của A dựa vào chuỗi ở trên lại lớn hơn 2
đơn vị so với giá trị thực nghiệm. Khi quan sát cẩn thận hỗn hợp từ quá trình thủy
phân acid hoàn toàn A thì thấy 3 đương lượng mol ammonia cũng được tạo thành
ngoài các amino acid được xác định ban đầu.
6) Hãy đề xuất cấu trúc được sửa đổi của A và khoanh tròn (các) tâm trong cấu
trúc là nguồn tạo ra ammonia.
7) Sử dụng thông tin trong Bảng 2, hãy tính điểm đẳng điện của A.
Bảng 1: Kí hiệu và công thức của một số amino acid phổ biến tại điểm đẳng điện.

1214 | Câu hỏi lí thuyết Hóa học hữu cơ OlympiaVN


Bảng 2: pKa của một số nhóm quan trọng trong các amino acid

1215 | Câu hỏi lí thuyết Hóa học hữu cơ OlympiaVN


Hướng dẫn
1) 2 nhóm sulfonic acid được tạo thành khi oxid hóa một liên kết disulfide.
2) Cấu trúc hoàn chỉnh ở DNP-Asp ở điểm đẳng điện là

3) Trình tự chuỗi B8 là: Cya-Tyr-Ile-Glu


4) Trình tự chuỗi B9 là: Asp-Cya-Pro-Leu
5) Cấu trúc hoàn chỉnh của A là:

6) Cấu trúc hoàn chỉnh của A đã hiệu chỉnh là:

7) Điểm đẳng điện của A là pI = 9.

1216 | Câu hỏi lí thuyết Hóa học hữu cơ OlympiaVN


Bài 19
Depsipeptit là những peptit mà một hoặc nhiều liên kết amit (CO-NH) được thay
thế bằng liên kết este (CO-OR). Do có cấu tạo khác lạ nên chúng có hoạt tính sinh
học lý thú. Một số depsipeptit tách được từ nấm đã được dùng để chữa trị ung thư
và HIV.
Khi cho depsipeptit P (C24H36N4O6S2) phản ứng với 2-mercaptoetanol
(HOCH2CH2SH) dư, thu được (HOCH2CH2S)2 và Q (C24H38N4O6S2). Q phản ứng
với MeI trong dung dịch K2CO3 tạo thành R. Cho R phản ứng với dung dịch nước
của imidazole (1,3-diazole, C3H4N2) thu được T (C26H44N4O7S2).
Thủy phân T bằng enzym cacboxipeptidaza lần lượt thu được L-valin (L–
H2NCH(CHMe2)COOH), cis – dehydrobutyrin (axit (E)-2-aminobut-2-enoic), L-
metylcystein (L – H2NCH(CH2SMe)COOH) và U. Thủy phân U bằng axit thì tạo
ra D-valin và hợp chất V. Ozon hóa V rồi chế hóa với Zn/HCl thu được axit 3-
hydroxi-4-oxobutanoic, 3-(metylsunfinyl)propanal (MeS(O)CH2CH2CHO) và 3-
(metylsunfonyl)propanal (MeS(O2)CH2CH2 CHO).
Hãy xác định cấu tạo của Q, R, T, U, V và depsipeptit P.
Hướng dẫn
Hãy xác định cấu tạo của Q, R, T, U, V và depsipeptit P
Các dữ kiện đầu bài được tóm tắt từng bước và phân tích như sau:
 P (C24H36N4O6S2) + HOCH2CH2SH dư  Q (C24H38N4O6S2) +
(HOCH2CH2S)2 (1)
Phản ứng (1) cho thấy cầu disunfua (-S-S-) ở P đã bị khử thành hai nhóm
-SH ở Q (phân tử tăng thêm 2H).
 Q (C24H38N4O6S2) 
MeI / K 2CO3
(2)
 R 
H 2O /Im idazole
(3)
 T (C26H44N4O7S2)

Phản ứng (2) và (3) cho thấy 2 nhóm -SH ở Q đã chuyển thành 2 nhóm -
SMe ở R rồi cộng thêm 1 H2O thành T (phân tử tăng thêm 2C, 6H và 1O).
 Thủy phân T nhờ enzym cacboxipeptidaza thì lần lượt thu được L-valin
(L– H2NCH(CHMe2)COOH), cis – dehydrobutyrin (axit (E)-2-aminobut-
2-enoic), L-metylcystein (L – H2NCH(CH2SMe)COOH) và U.
Dữ kiện này cho thấy thứ tự các hợp phần chính trong T như sau:
U – HNCH(CH2SMe)CO-HNC(=CHMe)CO-HNCH(CHMe2)COOH.
 Thủy phân U nhờ xúc tác axit sinh ra D-valin và hợp chất V. Ozon hóa
V rồi chế hóa với Zn/HCl thu được axit 3-hydroxi-4-oxobutanoic, 3-
(metylsunfinyl)propanal (MeS(O)CH2CH2CHO) và 3-
(metylsunfonyl)propanal (MeS(O2)CH2CH2CHO).

1217 | Câu hỏi lí thuyết Hóa học hữu cơ OlympiaVN


Dữ kiện này cho thấy V liên kết với D-valin bởi liên kết amit CO-NH.
Ở T có 26C, tổng số C ở bốn hợp phần tách ra là 18  Ở V có 8C. Do đó V có
cấu tạo là:
MeS – CH2CH2CH=CHCH(OH)CH2COOH.
Cấu tạo của U như sau:
MeS – CH2CH2CH=CHCH(OH)CH2CO-NHCH(CHMe2)COOH
Suy ra T có cấu tạo như sau:

Viết gọn là:


MeS – CH2CH2CH=CHCH(OH)CH2CO(D-Val)-(metylCys)-(cis-
dehydrobutyrin)-(Val)
 R (C26H42N4O6S2) + H2O/Imidazole  T (C26H44N4O7S2) (3)
So sánh công thức của R và T thì thấy ở phản ứng (3) R đã cộng thêm
một phân tử H2O để tạo ra T. Theo đầu bài ở depsipeptit phải có liên kết este nên
phản ứng (3) là sự thủy phân một este vòng nhờ xúc tác imidazole (vì không tạo
ra ancol và axit riêng rẽ và thành phần tăng thêm 2H và 1O).
Đối với T (C26H44N4O7S2) độ không no ( + v) = 7 phù hợp với 7 liên kết
 của 5 nhóm C=O và 2 nhóm C=C. Đối với R (C26H42N4O6S2) độ không no ( +
v) = 8, phù hợp với 7 liên kết  của 5 nhóm C=O, 2 nhóm C=C và 1 vòng este.
Liên kết este ở R chỉ có thể được tạo thành từ nhóm COOH của L-valin và nhóm
OH ancol của axit V.
Như vậy R có cấu tạo như sau:

Q có cấu tạo như sau:

P có cấu tạo như sau:

1218 | Câu hỏi lí thuyết Hóa học hữu cơ OlympiaVN


1219 | Câu hỏi lí thuyết Hóa học hữu cơ OlympiaVN
Bài 20
*opioid peptide là các peptide liên kết với các thụ thể opioid trong não.
Các hợp chất A, B, C – những chất tương tự với các peptide giảm đau nội sinh –
đã được tiến hành phân tích. Biết rằng tất cả các hợp chất này đều là những peptide
ngắn (chúng có cùng số gốc amino acid) và được tạo thành chỉ bởi các amino acid
thơm và iminoacid. Trong chuỗi của hai peptide, amino acid tạo protein đã bị thay
thế bởi amino acid phi tự nhiên.
Bằng phổ khối lượng, xác định được rằng khối lượng mol của các peptide A và
B lớn hơn 14 g/mol so với khối lượng mol của peptide C. Tất cả các peptide này
đều dương tính với phép thử ninhydrin.
Kết quả của thủy phân chymotrypsin là nhận được ba mảnh cấu trúc từ các
peptide A và C, và chỉ có hai mảnh từ peptide B. Trong trường hợp của hai trong
số các peptide được phân tích, khối lượng mol của một mảnh bị phân cắt là 262
g/mol, ngoài ra một trong các peptide này cũng nhận được một mảnh với khối
lượng mol 165 g/mol.
Thoái phân Edman (một chu trình) của mỗi peptide tạo thành dẫn xuất giống
nhau với khối lượng mol 298 g/mol. Các peptide bị rút ngắn (từ thoái phân
Edman) không tạo màu tím trong phép thử ninhydrine.
Amino acid phi tự nhiên có thể được tổng hợp theo chuyển hóa sau:

a) Trình bày sơ đồ tổng quát của thoái phân Edman và vẽ cấu trúc của dẫn xuất
tạo thành.
b) Vẽ cấu trúc các hợp chất X và Z. Sản phẩm của các chuyển hóa này có tính
quang hoạt không?
c) Xác định chuỗi các peptide A, B, C và giải thích ngắn gọn lựa chọn của bạn
(sử dụng các kí hiệu ba-chữ cái, với amino acid phi tự nhiên thì kí hiệu là U).
d) Mô tả ngắn gọn tác động của amino acid phi tự nhiên trong các chuỗi peptide
đến tính bền của chúng trong các điều kiện sinh lí.

1220 | Câu hỏi lí thuyết Hóa học hữu cơ OlympiaVN


Hướng dẫn
a) Thoái phân Edman là phương pháp xác định trình tự các amino acid trong một
peptide. Nó cũng cho phép xác định amino acid đầu N. Sơ đồ phản ứng diễn ra
như sau:

Hoặc ngắn gọn hơn là:


R S

NCS peptide H+
+ H2N N NH
- peptide
O
O R

Biết rằng khối lượng mol của dẫn xuất tạo thành là 298 g/mol. Mảnh
isothiocyanate có khối lượng mol là 135 g/mol. Có thể dễ dàng tính được khối
lượng mol của gốc amino acid:
298 -135= 163,
163+18(H2O) = 181 g/mol – tyrosine (Tyr).
Cấu trúc của dẫn xuất tạo thành là:
S

N NH

HO

b) Sơ đồ tổng hợp amino acid phi tự nhiên:

1221 | Câu hỏi lí thuyết Hóa học hữu cơ OlympiaVN


Hợp chất X là nitrile không no, được tạo thành từ phản ứng Knoevenagel. Sự khử
với NaBH4 khi có mặt CoCl2 của nitrile như vậy tạo thành ester của amino acid
phi tự nhiên (Z). Phản ứng tạo thành hỗn hợp đẳng mol của các đối quang
(racemic), có nghĩa là sản phẩm tổng hợp không có tính quang hoạt.
c) Biết rằng trong trình tự tất cả các peptide chỉ có các amino acid thơm và
iminoacid. Tất cả các peptide được phân tích đều tạo màu tím với ninhydrin
nhưng các peptide được rút ngắn sau một chu trình thoái phân Edman thì không.
Điều này có nghĩa là gốc thứ hai là iminoacid. Proline (Pro) là imioacid tự nhiên
duy nhất được biết tới. Do đó, đầu N của các peptide là Tyr-Pro.
Khối lượng mol của các peptide A và B lớn hơn C 14 g/mol - gợi ý về sự hiện
diện của thêm một nhóm CH2 nữa. Amino acid phi tự nhiên là β-hPhe, vậy nó có
thể được tìm thấy trong chuỗi các peptide A và B.
Chymotrypsin là enzyme phân cắt các peptide ở phía carboxyl của các amino
acid thơm. Sau thủy phân, nhận được ba mảnh từ các peptide A và C và chỉ có
haai mảnh từ peptide B. Một trong các mảnh được phân cắt là tyrosine đầu N.
Trong trường hợp hai peptide được phân tích thì mảnh 262 g/mol cũng bị phân
cắt. Nó có thể chứa Pro, vậy: 262 – 115(Pro) + 18(H2O) =165 g/mol – khối lượng
mol của phenylalanine (Phe). 165 g/mol cũng là khối lượng mol của mảnh khác
được tạo thành từ thủy phân enzyme của một peptide.
Như đã đề cập ở trên, C được tạo thành chỉ từ các amino acid thơm tạo protein
và iminoacid. Chuỗi peptide của C là:

Trong thủy phân chymotrypsin của các peptide A và C, nhận được ba mảnh, sự
khác nhau giữa hai peptide này phải là phần đầu C. Do đó chuỗi peptide A là:

Sự khác nhau giữa các peptide A và B phải là vị trí thứ ba (thủy phân enzyme).
Do đó, chuỗi peptide B là:

1222 | Câu hỏi lí thuyết Hóa học hữu cơ OlympiaVN


d) Các peptide với amino acid phi tự nhiên trong chuỗi thường chống chịu tối hơn
trước tác động thủy phân của enzyme. Liên kết peptide trên phía carboxylic của
amino acid phi tự nhiên không bị thủy phân.

1223 | Câu hỏi lí thuyết Hóa học hữu cơ OlympiaVN


Bài 21
Việc xác định trình tự của một protein (polypeptide) gồm các giai đoạn sau: a)
tinh chế; b) xác định amino acid đầu N; c) phân cắt chuỗi polypeptide bằng
phương pháp hóa học hoặc enzyme; d) cô lập các mảnh peptide và (e) xác định
trình tự của chúng bằng một máy giải trình tự tự động (sequenator). Cũng có thể
xác định trình tự hỗn hợp các mảnh peptide mà không cần phân giải. Trình tự
chuỗi cuối cùng có thể được xác định bằng cách xếp chồng các chuỗi sau khi phân
tích thông tin về vị trí của các amino acid trong các mảnh khác nhau.
Một protein nhỏ, tạo nên từ 40 gốc amino acid được xác định trình tự chuỗi như
sau:
- Tiến hành thoái phân Edman bằng cách xử lí với phenyl isothiocyanate, sau
đó thủy phân và xác định bằng quang phổ amino acid biến tính. Quá trình này
xác định được aspartic acid (Asp) là gốc đầu N.
- Protein được xử lí với CNBr (cyanogen bromide) - chất này phân cắt liên kết
peptide giữa methionine và bất kì amino acid nào khác ở đầu C của nó. Mảnh
peptide thu được không được tách riêng. Hỗn hợp các peptide được phân tích
trên máy giải trình tự protein. Do đó, máy giải này có thể phát hiện nhiều
amino acid ở vị trí xác định. Kết quả được cho ở Bảng 1(a).
- Protein được xử lí với một enzyme trypsin phân giải protein. Enzyme này
phân cắt liên kết peptide giữa amino acid có tính base (Arg hoặc Lys) và gốc
đầu C tiếp theo. Hỗn hợp peptide thu được cũng được tiến hành phân tích như
trên. Kết quả được cho ở Bảng 1(b).

1224 | Câu hỏi lí thuyết Hóa học hữu cơ OlympiaVN


Bảng 1: Dữ liệu từ máy giải trình tự protein.

Dựa vào các thông tin trên:


a) Suy ra trình tự chuỗi amino acid chung với mảnh đầu tiên (đầu N) nhận được
bởi các xử lí CNBr và trypsin.
b) Suy ra trình tự chuỗi của mảnh đầu tiên được tạo ra bởi xử lí CNBr.
c) Suy ra trình tự toàn chuỗi của polypeptide ban đầu. Chỉ rõ các tâm bị phá hủy
bởi CNBr và trypsin ở chuỗi này.
d) Tính % các gốc là amino acid có tính base.
e) Biết polypeptide tồn tại ở dạng chuỗi xoắn α, tính độ dài cấu trúc chuỗi xoắn
α này.
f) Tính kích thước của doạn DNA (exon) mã hóa cho polypeptide 40 amino acid
này. Xác định kích thước theo các cặp base, cũng như theo Dalton. (Xét khối
lượng phân tử trung bình của một nucleotide trong DNA là 330.)
g) Giả sử rằng DNA tương ứng với exon chứa cùng số Adenine và Cytosine,
hãy tính số liên kết hydrogen để giữ chuỗi xoắn kép này.

1225 | Câu hỏi lí thuyết Hóa học hữu cơ OlympiaVN


Hướng dẫn
Trình tự các amino acid trong một protein hoặc polypeptide được biểu diễn từ
amino acid đầu N. Từ phương pháp thoái phân Edman, amino acid đầu N là Asp.
Do đó, trong mảnh đầu N được tạo ra bởi trypsin hoặc CNBr, amino acid này phải
ở vị trí 1. Tất cả các peptide khác được tạo thành bởi phân cắt CNBr sẽ được đặt
trước bởi Met trên phía đầu N. Tương tự như vậy, tất cả các peptide được tạo
thành bởi trypsine sẽ được đặt trước bởi Arg hoặc Lys. Khi chúng ta tiến hành từ
amino acid đầu N đến đầu C, chúng ta xem xét cẩn thận các amino acid ở mỗi vị
trí được đưa ra trong Bảng 1(a) và 1(b).
Từ mảnh đầu tiên bắt đầu từ Asp đầu N ở vị trí 1, chúng ta tìm các gốc chung ở
mỗi vị trí với các peptide bị phân cắt CNBr và trypsin. Thu được:

Ở vị trí 6, Arg sẽ khiến polypeptide nhạy cảm với trypsin. Do đó, gốc thứ 7 của
mảnh CNBr này (Bảng 1(a)) sẽ giống như gốc 1 trong mảnh peptide khác được
tạo ra bởi trypsin và gốc thứ 8 của mảnh CNBr này sẽ giống như gốc 2 trong Bảng
1(b). Do đó ta có

Do 8 sẽ là Tyr, Pro sẽ được gắn với vị trí 2 của polypeptide


…..(3)
Gốc 9 trong polypeptide sẽ ở vị trí thứ 3 trong Bảng 1(b) và các gốc 10, 11, 12,
13 và 14 sẽ phải lần lượt ở các vị trí 4, 5, 6, 7 và 8 trong Bảng 1(b). Các gốc tương
tự phải ở các vị trí từ 1 trở đi trong Bảng 1(a). Không gốc nào trong các gốc ở vị
trí 3 (Bảng 1(b)) giống như vị trí 1 trong Bảng 1(a). Tuy nhiên, các vị trí 4 đến 8
trong Bảng 1(b) có chung gốc với các vị trí 1 đến 5 trong Bảng 1(a). Hơn nữa,
Glu ở vị trí 1 (Bảng 1(a)) sẽ được đặt trước bởi Met (do nó là một phần của peptide
phân cắt CNBr). Và vị trí 3 trong Bảng 1(b) có Met. Do đó, ta có:

Vị trí 5 trong polypeptide bây giờ có thể được gán chắc chắn cho Ilu …..(5)
Các vị trí 15 và 16 trong polypeptide sẽ vượt quá gốc 8 trong peptide phân cắt
trypsin (không đưa ra ở đây). Bây giờ, chúng ta cố gắng xây dựng các mảnh
trypsin và CNBr còn lại.

1226 | Câu hỏi lí thuyết Hóa học hữu cơ OlympiaVN


Bảng 1(a) cho thấy Arg ở vị trí 1. Gốc này được đặt trước bởi một Met. Kết hợp
các gốc chưa được gán ở vị trí 2 trong Bảng 1(a) với các gốc ở vị trí 1 trong Bảng
1(b) và với các vị trí tiếp sau theo cách đã trình bày ở trước, ta có:

(2 gốc cuối là các gốc chưa được gán ở vị trí 1 và 2 trong Bảng 1(b)).
Khi xét (2), (5), (6) cùng nhau thì có thể chắc chắn gán vị trí 7 của polypeptide
với Gly …..(7)
a)

b)

Để hoàn thành chuỗi polypeptide chúng ta cần xây dựng trình tự chuỗi của mảnh
trypsin khác. Bắt đầu từ vị trí 4-(Arg) trong Bảng 1(a), ta có trình tự chuỗi:

Ở bước này, ta lại xem xét các gốc chưa được gán. Arg ở (8) sẽ phải được đặt
trước bởi Asn, Gln, Gly và Met (chúng là các gốc chưa được gán ở các vị trí tương
ứng trong Bảng 1(a)). Sau đó, ta có được trình tự chuỗi

Và sau Ala ở (9)

Từ (9) và (10), ta có trình tự chuỗi

Do mảnh nhỏ nhất là một dipeptide (Bảng 1(b)) và (6) cho thấy rằng nó theo sau
Lys nên nó sẽ ở đầu C. Do đó, chuỗi cục bộ ở (6) sẽ ở sau chuỗi cục bộ ở (11).
Ta có:

Đã có một Met ở vị trí 9 trong polypeptide. Met tiếp theo chỉ có thể đến trước tiên
ở vị trí 17 do mảnh CNBr có ít nhất 8 amino acid. Do đó, gốc bắt đầu của (12) có

1227 | Câu hỏi lí thuyết Hóa học hữu cơ OlympiaVN


thể được gán cho vị trí 17 - để lại các vị trí 15 và 16 sẽ được điền bởi các gốc
chưa được gán Val và Al trong mảnh CNBr ở các vị trí 6 và 7 (Bảng 1(a)).
c) Do đó, chuỗi cuối là:

Mũi tên (↓) chỉ các tâm bị phá hủy bởi CNBr và trypsin.
d) Có 6 gốc amino acid có tính base trong peptide. 6/40 = 15%.
e) Một chuỗi xoắn α helix có 3.6 gốc amino acid trên mỗi vòng 5.4Å. Do đó,
chiều dài của polypeptide trong cấu dạng xoắn α sẽ là:
40/3.6 × 5.4 = 59.4 Å.
f) Polypeptide có 40 amino acid. Do mỗi amino acid được mã hóa bởi một bộ
ba nucleotide nên tổng số cặp nucleotide trong DNA mạch kép của exon sẽ
là: 40 x 3 = 120 cặp base.
Khối lượng phân tử DNA tạo thành exon là = 330 x 2 x 120 = 79200 Da
g) Nếu exon chứa 120 cặp base và A và C có số lượng bằng nhau thì sẽ có 60
cặp A-T và 60 cặp G-C. Mỗi cặp A-T được giữ bởi 2 liên kết hydrogen và
mỗi cặp G-X được giữ bởi 3 liên kết hydrogen. Do đó tổng số liên kết
hydrogen để giữ chuỗi xoắn kép này là:
(60 x 2) + (60 x 3) = 300

1228 | Câu hỏi lí thuyết Hóa học hữu cơ OlympiaVN


Bài 22
a) Dự đoán sản phẩm tạo thành khi butanoic acid phản ứng với methyl alcohol
khi có mặt hydrochloric acid? Viết phương trình phản ứng và gọi tên sản
phẩm.
b) Trình bày sự tấn công của alcohol trong giai đoạn tốc định. Đó là sự tấn công
nucleophile hay electrophile? Vẽ trạng thái trung gian.
c) Sản phẩm chính nào được tạo thành khi acid hóa 5-hydroxypentanoic acid?
Chuyển hóa của các amine RNH2 thành amide RCONH2 có thể được tiến hành
với acetic anhydride:

Trong tổng hợp peptide, thường sử dụng di-tert-butyl dicarbonate, tạo thành tert-
butoxycarbonylamid, hoặc BOC-Amid.
d) Biểu diễn cấu trúc của BOC-Ala.

Nhóm BOC có thể bị tách loại bằng cách xử lí với acid hữu cơ như CF3COOH.
e) Trình bày quy trình tổng hợp peptide H-Leu-Ala-
OH. Mô tả các giai đoạn - chưa cần viết phương trình
phản ứng. Cho biết chỉ khi tiến hành bảo vệ các
nhóm chức hoạt động của amino acid thì mới có thể
đạt hiệu suất cao.
f) Viết các phương trình phản ứng.

1229 | Câu hỏi lí thuyết Hóa học hữu cơ OlympiaVN


Hướng dẫn
a)

b) Tấn công nucleophile: Trung gian:

c) Phản ứng nội phân tử:

d)

e) Quy trình tổng hợp H-Leu-Ala-OH:


1. Đưa nhóm bảo vệ vào leucine (nhóm NH2) và alanine (nhóm COOH).
2. Ghép cặp hai amino acid đã được bảo vệ.
3. Loại nhóm bảo vệ khỏi peptide tạo thành.
f)
1. Đưa các nhóm bảo vệ vào

1230 | Câu hỏi lí thuyết Hóa học hữu cơ OlympiaVN


2. Ghép cặp các amino acid đã được bảo vệ:

1231 | Câu hỏi lí thuyết Hóa học hữu cơ OlympiaVN


3. Loại nhóm bảo vệ
Thủy phân acid

Thủy phân base

1232 | Câu hỏi lí thuyết Hóa học hữu cơ OlympiaVN


Bài 23
1) Trình bày tất cả các giai đoạn và tác nhân cần thiết để tổng hợp mỗi dipeptide
sau từ các amino acid tương ứng của chúng:
(a) Trp-Met
(b) Ala- le
(c) Leu-Val
2) Trình bày tất cả các giai đoạn và tác nhân cần thiết để tổng hợp tripeptide với
trình tự Ile-Phe-Gly.
3) Trình bày tất cả các giai đoạn và tác nhân cần thiết để tổng hợp pentapeptide
với trình tự Leu-Val-Phe- le-Ala.
Hướng dẫn
1) a)

b)

c)

2) Hai gốc amino acid đầu tiên là Ile và Phe. Do đó, chúng ta cần bắt đầu với các
amino acid này. Trước tiên, chúng ta phải thiết lập các nhóm bảo vệ phù hợp. Sau
đó, xử lí với DCC, các amino acid được bảo vệ sẽ được ghép cặp. Nhóm bảo vệ
ở đầu C sau đó được loại bỏ và đầu C loại nhóm bảo vệ tạo thành được ghép cặp
với amino acid được bảo vệ phù hợp (glycine, được bảo vệ đầu C) bởi DCC. Và
cuối cùng, loại các nhóm bảo vệ:

1233 | Câu hỏi lí thuyết Hóa học hữu cơ OlympiaVN


3)

1234 | Câu hỏi lí thuyết Hóa học hữu cơ OlympiaVN


1235 | Câu hỏi lí thuyết Hóa học hữu cơ OlympiaVN
Bài 24
Ngoài cách tạo thành amino acid được bảo vệ Fmoc sử dụng Fmoc-Cl, nhóm bảo
vệ Fmoc cũng có thể được thêm vào nhóm amino bằng cách sử dụng tác nhân A.
a) Trình bày cơ chế phản ứng thêm nhóm bảo vệ Fmoc vào amino acid.

b) Trình bày cơ chế phản ứng loại nhóm Fmoc khỏi amino acid sử dụng các điều
kiện sau:

1236 | Câu hỏi lí thuyết Hóa học hữu cơ OlympiaVN


Hướng dẫn
a)

b)

1237 | Câu hỏi lí thuyết Hóa học hữu cơ OlympiaVN


Bài 25
Peptide là các polyamide mạch thẳng, được tạo thành bởi các liên kết đầu-đến-
đuôi của các α-amino acid, chủ yếu là ở cấu hình L (hay S).
1) Các dipeptide nào có thể được tạo thành bởi sự ngưng tụ L- alanine and L-
phenylalanine? Chỉ rõ hóa lập thể.
2) Sự kéo dài từng phần mạch peptide hầu hết luôn bắt đầu từ nguyên tử C của
các amino acidbậc ba (sử dụng ở dạng ester) liên kết với mỗi đơn vị amino
acidkế tiếp (sử dụng ở dạng dẫn xuất thế ở nitrogen) dẫn đến sự thay thế
nguyên tử N - thế trước khi đơn vị kế tiếp được gắn vào. Dẫn xuất thế thường
được sử dụng là các nhóm alkoxycarbonyl ROCO - và dẫn xuất carbamatecủa
nó. Hãy giải thích lý do tại sao sự hiện diện của nhóm thế (nhóm bảo vệ) của
nguyên tử nitrogen amine làm trở ngại việc tạo liên kết amide với nhóm
cacboxyl.
a) Vì nitơ bây giờ chỉ còn có 1H.
b) Vì nhóm bảo vệ có mật độ electron ít hơn nguyên tử nitrogen.
c) Vì nhóm bảo vệ chắn sự tấn công của nhóm carbonyl.
d) Vì sự kháng tĩnh điện.
e) Vì nó vốn đã là một amide.
3) Vẽ các công thức cộng hưởng của một nửa nhóm amide. Sử dụng các ký hiệu
lập thể và các mũi tên để chỉ rõ sự chuyển dịch electron.
4) Tác nhân nào dưới đây sẽ được sử dụng để gắn nhóm benzyl carbamate vào
một amine (nhóm Bergmann-Zervas). Viết phản ứng.
a) C6H5CH2OCONH2
b) C6H5CH2OCO2CH3
c) C6H5CH2OCO2C(CH3)3
d) C6H5CH2OCOCl
e) C6H5OCOCl
5) Việc loại nhóm bảo vệ alkoxycarbonyl thường kèm theo phản ứng cắt mạch
dưới tác dụng của các acid theo sơ đồ:

Xếp theo chiều tăng dần tính hoạt động của các carbamate sau đây dưới tác dụng
của acid:

1238 | Câu hỏi lí thuyết Hóa học hữu cơ OlympiaVN


Hướng dẫn
1) Công thức cấu tạo các peptide có thể có:

Các dipeptide vòng (diketo piperazine) cũng được chấp nhận:

2) Câu trả lời tốt nhất là 5 và 2


3)

1239 | Câu hỏi lí thuyết Hóa học hữu cơ OlympiaVN


4) Tác nhân 4 (Bbenzyl chloroformate) sẽ phản ứng với amine theo sơ đồ sau:
C6H5CH2OCOCl + H2NR  base
 C6H5CH2OCONHR + HCl
5) Nếu chúng ta giả thiết trạng thái chuyển tiếp có tạo thành carbonium ion thì
chất nào tạo thành carbonium ion dễ dàng nhất thì tính bền cũng tỉ lệ thuận
với khả năng đó. Trong chất D thì có sự giải toả electron mạnh nhất:

và khó nhất ở A:

Giải thích tương tự ta thấy cation tạo thành từ B bền hơn C. Như vậy thứ tự sẽ là:
D>B>C>A

1240 | Câu hỏi lí thuyết Hóa học hữu cơ OlympiaVN


Bài 26
Dẫn ra quy trình tổng hợp từng bước cho Val-Leu-Val từ 3-methylbutanal
[(CH3)2CHCH2CHO] là nguồn nguyên liệu đầu hữu cơ duy nhất. Bạn có thể dùng
thêm bất kì tác nhân vô cơ, hữu cơ nào khác.
Hướng dẫn

1241 | Câu hỏi lí thuyết Hóa học hữu cơ OlympiaVN


Bài 27
1) Xác định cấu trúc amino acid được bảo vệ phải neo vào chất trợ rắn để dùng
cho tổng hợp Merrifield để điều chế leucine enkephalin.
(Đầu N) Try-Gly-Gly-Phe-Leu (Đầu C)
2) Xác định trình tự tripeptide có thể được tạo thành theo trật tự sau của các tác
nhân. Chỉ rõ đầu C và đầu N của tripeptide:

3) Xác định tất cả các giai đoạn cần thiết để tổng hợp mỗi peptide sau theo
phương pháp Merrifield:
(a) Phe-Leu-Val-Phe
(b) Ala-Val-Leu- le

1242 | Câu hỏi lí thuyết Hóa học hữu cơ OlympiaVN


Hướng dẫn
1) Leucine được bảo vệ (bởi nhóm Boc):

2) Trước tiên, gốc valine được bảo vệ được gắn vào polymer. Sau khi loại nhóm
bảo vệ, gốc alanine đã bảo vệ được gắn vào. Sau đó, lại loại nhóm bảo vệ rồi gắn
gốc phenylalanine đã bảo vệ vào. Loại nhóm bảo vệ, sau đó tách khỏi polymer,
tạo thành tripeptide sau:
(Đầu N) Val-Ala-Phe (Đầu C)

3) a) Trước tiên, chúng ta gắn gốc phù hợp, được bảo vệ bởi Boc vào polymer:

Sau đó, nhóm bảo vệ Boc được loại bỏ và một liên kết peptide mới được tạo thành
với amino acid được bảo vệ bởi Boc, sử dụng DCC. Quá trình hai giai đoạn này
(loại bỏ nhóm bảo vệ Boc, sau đó tạo liên kết peptide mới) được lặp lại để bổ
sung thêm mỗi gốc, cho đến khi sắp xếp được trình tự mong muốn. Cuối cùng,
loại bỏ nhóm bảo vệ Boc và peptide mong muốn được tách khỏi polymer:

1243 | Câu hỏi lí thuyết Hóa học hữu cơ OlympiaVN


b) Tương tự như trên:

1244 | Câu hỏi lí thuyết Hóa học hữu cơ OlympiaVN


Bài 28
Quá trình sinh tổng hợp peptide của kháng sinh vancomycin trong các vi sinh vật
bắt đầu với việc xây dựng một chuỗi peptide bởi các enzyme có khả năng sử dụng
các amino acid phi-tiêu chuẩn [không chính tắc] và có thể đồng phân hóa chúng
trong quá trình tổng hợp peptide. Các enzyme này gồm các module [khối] - mà
mỗi module chịu trách nhiệm cho việc cộng thêm một amino acid. Module gồm
một chuỗi các domain với chức năng cụ thể: Domain A - chọn lọc amino acid, T
- gắn kết amino acid hoặc chuỗi peptide với enzyme, C - tạo thành liên kết amino
acid tiếp theo và phát triển mạnh, E - thay đổi cấu hình R/S trên nguyên tử α-
carbon của một amino acid, Te - tách chuỗi peptide khỏi enzyme.
a) Mỗi amino acid sau có cấu hình R hay S?

Hpg, Dpg, và β-OH-Cl-Tyr là các amino acid phi-tiêu chuẩn. Hpg có cấu trúc
tương tự như Tyr, nhưng M(Tyr) - M(Hpg) = 14 g/mol. Dpg có cấu trúc tương tự
Dpg với khối lượng lớn hơn 16 g/mol. Tất cả các nhóm thế của Dpg trên vòng
phenyl đều có khoảng cách với nhau bằng nhau. Cấu trúc của β-OH-Cl-Tyr dựa
trên cấu trúc Tyr, với nhóm hydroxyl gắn vào nguyên tử β-carbon và vòng phenyl
bị chlorine hóa ở vị trí ortho tương ứng với nhóm OH.
b) Vẽ cấu trúc của Hpg, Dpg, và β-OH-Cl-Tyr.
Hình dưới đây biểu diễn chuỗi các module của enzyme tham gia vào quy trình
tổng hợp chuỗi peptide vancomycin. Chuỗi amino acid của peptide được tổng hợp
theo các A domain. Quy trình tổng hợp peptide diễn ra theo chiều từ N → C
(nhóm Leu amino không tham gia tạo thành liên kết peptide). Nếu module có E-
domain thì amino acid được thêm vào theo cấu hình R. Còn nếu không có thì theo
cấu hình S. (Trong hình: domeen = domain.)

c) Vẽ cấu trúc phẳng của peptide tạo thành và chỉ rõ cấu hình với mỗi amino
acid.

1245 | Câu hỏi lí thuyết Hóa học hữu cơ OlympiaVN


1246 | Câu hỏi lí thuyết Hóa học hữu cơ OlympiaVN
Hướng dẫn
a) Asn - R; Leu - R; Thr - 2S, 3R và Tyr - S
b)

c)

1247 | Câu hỏi lí thuyết Hóa học hữu cơ OlympiaVN


Bài 28A
Quy trình tổng hợp peptide phase rắn, do Robert Bruce Merrifield (Nobel Hóa
học 1984) tiên phong, đã cho phép tổng hợp tự động hóa các peptide với độ dài
lên tới ~ 50 amino acid. Sau khi nạp trước amino acid đầu teien lên nhựa polymer,
máy tổng hợp sẽ xoay vòng qua các giai đoạn loại nhóm bảo vệ và ghép lặp liên
tiếp cho đến khi tạo thành polypeptide được bảo vệ. Chiến lược tổng hợp gắn kết
với polymer (hoặc “nhựa”) cho phép thực hiện các giai đoạn loại nhóm bảo vệ và
ghép cặp liên tục mà không cần cô lập sản phẩm. Các sản phẩm tan được được
loại bỏ dễ dàng bởi các bước “rửa”, bằng cách dẫn dung môi qua nhựa được giữ
lại bởi bộ lọc. Nhựa Merrifield ban đầu (chấ nền polymer) đã được phát triển
thành các loại nhựa cao cấp hơn. Các quy trình thực tế hiện nay sử dụng nhựa
Wang, sẵn có thương mại, như minh họa dưới đây.
a) Ở giai đoạn đầu tiên, nhựa Wang và amino acid đầu tiên được ghép cặp. Hãy
đề xuất các tác nhân bổ sung để tạo ra synthesize N-Fmoc-Gly-Polymer.

b) Fmoc-Gly-Polymer, các giai đoạn loại nhóm bảo vệ và ghép cặp được quay
vòng kết hợp với quá trình rửa. Một tác nhân ghép cặp phổ biến trong tổng
hợp peptide phase rắn là tácnhaan có cấu trúc phức tạp 1-
[Bis(dimethylamino)methylene]-1H-1,2,3-triazolo[4,5-b]pyridinium-3-
oxide hexafluorophosphate (HATU), có cấu trúc như dưới đây. Hãy trình bày
cơ chế phản ứng loại nhóm bảo vệ của N-Fmoc-Gly-Polymer bởi piperidine,

1248 | Câu hỏi lí thuyết Hóa học hữu cơ OlympiaVN


sau đó ghép cặp với N-Fmoc-Ala-OH được sắp xếp bởi HATU và N-
methylmorpholine. Trung gian trong phản ứng ghép cặp amide là ester được
hoạt hóa A.

c) Tại sao ester được hoạt hóa A thúc đẩy sự tạo thành liên kết peptide thuận lợi
hơn ester thay thế B được tạo thành từ tác nhân cũ hơn (HBTU).

d) Sau chu kì loại nhóm bảo vệ và ghép cặp khác với N-Fmoc-L-phenylalanine,
đầu N được loại nhóm bảo vệ bởi piperidine và polypeptide gắn với nhựa
được tạo thành. Hãy trình bày cơ chế phản ứng loại nhóm bảo vệ-nhựa để
nhận được H2N-Phe-Ala-Gly-OH.

1249 | Câu hỏi lí thuyết Hóa học hữu cơ OlympiaVN


1250 | Câu hỏi lí thuyết Hóa học hữu cơ OlympiaVN
Hướng dẫn
a)

b) Cơ chế phản ứng:

1251 | Câu hỏi lí thuyết Hóa học hữu cơ OlympiaVN


c)

7-aza-pyridine nitrogen tham gia vào liên kết hydrogen với phần [partner] ghép
cặp amine trước quá trình tấn công nucleophile của amine vào nhóm ester. Điều
này dẫn đến sự tăng cường tính nucleophile của nhóm amine và do đó tăng tốc
độ phản ứng.
d)

1252 | Câu hỏi lí thuyết Hóa học hữu cơ OlympiaVN


1253 | Câu hỏi lí thuyết Hóa học hữu cơ OlympiaVN
Bài 29
1. Một peptit P có hoạt tính sinh học được cô lập từ một loài cóc ở Nam Mỹ.
Thứ tự sắp xếp các aminoaxit trong peptit P có thể được xác định dựa
trên các thông tin sau
- Heptapeptit P có thành phần gồm 6 aminoaxit thiên nhiên và một
aminoaxit ở dạng D. Peptit P chứa đầu C amit.
- Phản ứng giữa P và 2,4-DNFB rồi thủy phân trong axit sinh ra sản phẩm
có khối lượng phân tử 347.
- Cắt mạch peptit P bằng chymotrypsin thu được các phân đoạn có khối
lượng phân tử lần lượt là R 181, S 201, T 236 và U 238 g/mol. Phân tách
các peptit nhỏ hơn này rồi cho phản ứng với DNFB sau đó thủy phân
trong axit thu được DNP – Pro (S); DNP – Ala (T) và DNP – Gly (U).
- Cắt mạch peptit P bằng enzym thermolysin thu được hai mảnh W (252
g/mol) và X (568 g/mol). Biết rằng thermolysin là enzym cắt mạch peptit
ở đầu N của một aminoaxit kị nước (mạch ankyl phân nhánh hay chứa
vòng thơm).
- Peptit P không phản ứng với enzym dipeptidylpeptidaza. Biết rằng enzym
dipeptidylpeptidaza cắt mạch peptit ở nhóm carbonyl của aminoaxit thứ
hai.
Hãy xác định cấu trúc các mảnh chưa biết, từ đó suy ra cấu trúc peptit P.

2. Một heptapeptit (2 Met, Asp, Arg, Lys, Phe và Gly) được phân lập từ một
loài sinh vật được cho là có hoạt tính sinh học. Các kết quả xác định trật
tự sắp xếp của aminoaxit trong peptit này được cho như sau. Từ đó hãy
cho biết trật tự sắp xếp của các aminoaxit trong peptit này.
- Thủy phân peptit bằng enzym cacboxipeptidaza cho Met.
- Tiến hành phân cắt heptapeptit này bằng CNBr thu được 1 mol homoserin
lacton. Còn khi phân cắt bằng chymotrypsin thu được một pentapeptit và
một dipeptit.
- Phân cắt bằng trypsin cho hai aminoaxit đều có chứa Met và Arg tự do.
- Khi phân cắt bằng pepsin cho một tetrapeptit (Met, Asp, Lys, Arg) và
một tripeptit.

3. Các nhà nghiên cứu vừa mới phân lập một peptit thiên nhiên có cấu trúc
lý thú. Xử lý peptit này với axit mạnh thu được nhiều sản phẩm, trong đó
có dẫn xuất có cấu trúc vòng như hình dưới. Dẫn xuất này được cho tham
gia phản ứng Edman. Đề nghị sản phẩm của phản ứng này và giải thích
bằng cơ chế phản ứng (tạo ra dẫn xuất thiazoline).

1254 | Câu hỏi lí thuyết Hóa học hữu cơ OlympiaVN


4. Các nhà khoa học đã cố gắng để xác định được enzym  - lactamaza từ
Staphylococcus aureus. Khi enzym tinh khiết được đánh dấu bằng đồng
vị 32P thì ta chỉ đánh dấu một mình Serin. Trong phân tích thì serin chỉ
chiếm 0,35% về khối lượng của enzym  - lactamaza. Biết MSerin = 105
g/mol.
a) Ước lượng Menzym.
b) Để đánh dấu đầu hoạt động,  - lactamaza được phân cắt bởi tripsin.
Hexapeptit ban đầu (P1) chứa các aminoaxit sau: Glu, Leu, Lys, Met, Phe
và Ser
 Sử dụng phương pháp Edman ta biết được aminoaxit “đầu N” là Phe và
dãy peptit P2.
 Sử dụng BrCN để cắt mạch P1 thu được dipeptit P4 và tetrapepetit P3
 Sử dụng 1 – flo – 2,4 – dinitrobenzen sau đó thuỷ phân hoàn toàn peptit
P2 ta thu được N–2,4–dinitrophenyl–Glu .
Biết P1; P2; P3 đều chứa Ser. Hãy xác định vị trí các aminoaxit trong P1;
P2; P3; P4.
5. Hoàn chỉnh sơ đồ phản ứng sau đây:

1255 | Câu hỏi lí thuyết Hóa học hữu cơ OlympiaVN


Hướng dẫn
1. Các mảnh chưa xác định:
S: H – Pro – Ser – NH2 T: H – Ala – Phe – OH
U: H – Gly – Tyr – OH X: H – Phe – Gly – Tyr – Pro –
Ser – NH2
Dựa trên các kết quả phân tích khối lượng và sản phẩm phân cắt từ các
enzym trypsin và chymotrypsin cũng như việc peptit P không bị cắt mạch bởi
enzym dipeptidyl cho thấy Aminoaxit có cấu hình D là Ala
Như vậy cấu trúc của peptit cần xác định sẽ là:

2. Dựa trên kết quả phân cắt bằng enzym cacboxipeptidaza thì Met là
aminoaxit đuôi C.
Việc cắt mạch bằng CNBr thu được homoserin cho thấy aminoaxit đầu
N cũng là Met (CNBr cắt mạch sau Methionin, và bản thân nó chuyển thành
homoserin lacton theo cơ chế như sau)

Chymotrypsin cắt mạch sau aminoaxit thơm, như vậy aminoaxit thứ năm
của dãy (vì chia thành pentapeptit và dipeptit) sẽ là Phe.
Do aminoaxit cần phân tích có đầu N và C đều là Met, thế nên việc cắt
bằng trypsin cho ra Arg tự do cho thấy nó chính là aminoaxt thứ tư trong thứ tự.
Điều này dẫn đến kết quả Lys là aminoaxit thứ ba (Trypsin cắt mạch sau Lysin
và araginin).
Khi tiến hành cắt mạch bằng pepsin thu được tetrapeptit Met, Asp, Lys,
Arg tức thứ tự của nó sẽ là Met – Asp – Lys – Arg. Tripeptit còn lại gồm ba
aminoaxit Phe, Gly, Met sẽ có thứ tự Phe – Gly – Met.
Như vậy trật tự sắp xếp của các aminoaxit trong heptapeptit sẽ là Met –
Asp – Lys – Arg – Phe – Gly – Met.
3. Cơ chế phản ứng:

1256 | Câu hỏi lí thuyết Hóa học hữu cơ OlympiaVN


4. Do Mserin = 105 Da và Serin chiếm 0,35% khối lượng peptit nên có thể
dự đoán gần đúng Mpeptit = 105.100/0,35 = 30000 Da.
Có thể xác định được Phe là aminoaxit đầu N bằng phương pháp Edman
BrCN chỉ cắt sau Met nên với việc thu được một dipeptit và một
tetrapeptit có thể xác định được Met là aminoaxit thứ tư.
Khi cho P2 phản ứng với DNFB thu được dẫn xuất của Glu chứng tỏ nó
là aminoaxit đầu mạch P2, tức nó nằm ngay sau Phe trong hexapeptit.
Peptit có thể bị phân cắt bởi trypsin tức Lys không thể là aminoaxit cuối
mạch.
Do tất cả các peptit P1, P2 và P3 đều chứa Ser nên thứ tự sắp xếp của các
aminoaxit trong peptit sẽ như sau:
P1: Phe – Glu – Ser – Met – Lys - Leu
P2: Gly – Ser – Met – Lys - Leu
P3: Phe – Glu – Ser – Met.
P4: Lys – Leu

5. Sơ đồ phản ứng tổng hợp dipeptit Gly – Ala (PS là gốc polysyren)

1257 | Câu hỏi lí thuyết Hóa học hữu cơ OlympiaVN


1258 | Câu hỏi lí thuyết Hóa học hữu cơ OlympiaVN
Phụ lục 1: Bảng thuật ngữ
A
Acetylene zipper: Chuyển vị nối ba ra ngoài mạch. Dưới tác dụng của một base
mạnh, nối ba ở bất kỳ vị trí nào bên trong mạch sẽ di chuyển ra ngoài do tạo thành
acetylide bền không có khả năng chuyển vị tiếp

Adams catalyst: Xúc tác Adams. Xúc tác PtO2 đặc hiệu dùng cho phản ứng
hydrogen hóa.
Aldol addition: Phản ứng aldol (hay ngưng tụ aldol). Phản ứng giữa một enolate
với một aldehyde và một ketone để tạo thành sản phẩm -hydroxycarbonyl. Nếu
quá trình tiếp diễn sâu hơn sẽ dẫn đến sự tách loại nước tạo thành sản phẩm ,-
carbonyl bất bão hòa, giai đoạn này gọi là aldol – croton. Tùy điều kiện phản ứng
có thể thu được syn hay anti aldol.

Allyl boron hóa: Phản ứng giữa một dẫn xuất allyl boran với aldehyde và ketone.
Phản ứng xảy ra tạo thành một dẫn xuất homoallylic alcohol. Boron trong dẫn
xuất allyl boron ban đầu đóng vai trò phối trí với nhóm carbonyl để hoạt hóa phản
ứng này.

Anchimeric assistance: Hiệu ứng nhóm kề. Sự có mặt của một nhóm kề gây ổn
định carbocation hay tương tác với carbocation trong tiến trình phản ứng. Điều
này dẫn đến sự tăng tốc độ phản ứng và giữ nguyên cấu hình của nguyên tử carbon
gắn nhóm đi ra.

1259 | Câu hỏi lí thuyết Hóa học hữu cơ OlympiaVN


Anomeric effect: Hiệu ứng anomer. Cặp e của dị tố gắn với nguyên tử carbon
acetal trong hệ vòng pyran sẽ dẫn đến việc nhóm thế trên carbon này chiếm vị trí
axial thay vì equatorial thường gặp đối với dẫn xuất cyclohexane.

Arbuzov rearrangement: Phản ứng Arbuzov. Phản ứng giữa một trialkyl
phosphite và một dẫn xuất halogen dẫn đến sự tạo thành dialkyl
alkylphosphonate.

Nếu dùng -haloketone sẽ tạo thành enol phosphate (phản ứng Perkov)

B
Babler oxidation: Phản ứng oxid hóa Babler. Phản ứng oxy hóa một allylic
alcohol bậc ba dưới tác dụng của Cr(VI) (thường là PCC) để tạo thành dẫn xuất
,-carbonyl bất bão hòa đi qua chuyển vị Claisen [3.3]

Baeyer – Villiger rearrangement: Chuyển vị Bayer – Villiger. Phản ứng chuyển


vị ketone thành ester dưới tác dụng của một peracid.

Baker – Nathan effect: Hiệu ứng Baker – Nathan. Hiệu ứng này được biết đến
với cái tên quen thuộc hơn là hiệu ứng siêu liên hợp (hyperconjugation). Nguyên
lý chủ đạo là càng có nhiều liên kết C-H ở vị trí  so với orbital có khả năng nhận

1260 | Câu hỏi lí thuyết Hóa học hữu cơ OlympiaVN


electron (AO p trống của carbocation hay MO * của alkene) thì hệ thống càng
ổn định.
Bamford – Stevens reaction: Phản ứng Bamford – Stevens. Nhiệt phân
Tosylhydrazon trong sự có mặt của base thu được alkene. Phản ứng được cho là
đi qua trung gian carbene, bởi trong số sản phẩm phụ của phản ứng có dẫn xuất
cyclopropane.

Một hướng khác của phản ứng này là Shapiro (Shapiro reaction), khi dùng dư
base rất mạnh (chẳng hạn PhLi) thì chỉ thu được duy nhất sản phẩm alkene ít lần
thế hơn.

Barton ester: Ester Barton. Ester của N-hydroxy-2-pyridinethione, là một chất


tạo gốc tự do alkyl.

Barton-McCombie deoxygenation: Phản ứng Barton-McCombie


deoxygenation. Phản ứng chuyển một nhóm C-OH alcohol về alkane C-H sử dụng
cơ thiếc.

Barton reaction: Phản ứng Barton. Hoạt hóa một vị trí ở xa nhóm chức hoạt
động bằng cách phân cắt đồng ly dẫn xuất nitrite rồi tương tác với hydrogen ở vị
trí C6 thông qua trung gian vòng 6 cạnh thu được -nitrosoalcohol sẽ bị tautomer
hóa thành oxime

1261 | Câu hỏi lí thuyết Hóa học hữu cơ OlympiaVN


Beckmann rearrangement: Chuyển vị Beckmann: Chuyển vị oxime thành
amide

Benzoin condensation: Ngưng tụ benzoin. Hai phân tử aldehyde ngưng tụ lại với
nhau tạo thành -hydroxyketone (thường xảy ra dưới xúc tác CN-). Một biến thể
thường gặp của phản ứng này là phản ứng Stetter (Stetter reaction) cũng đi qua
cơ chế tương tự.

Bergmann cyclysation: Vòng hoa Bergmann: Sự vòng hóa một endiyne liên hợp
tạo thành p-benzyne (gốc tự do kép). Sản phẩm này có khả năng phá hủy DNA,
và cũng là cơ chế thường gặp của một số hợp chất tự nhiên có hoạt tính sinh học
giá trị calcheamicyne trị ung thư cũng tồn tại khung endiyne trong phân tử.

Bestmann reagent: Tác nhân Bestmann (hay Ohira-Bestmann). Được sử dụng


trong phản ứng Gilbert–Seyferth để tạo alkyne từ hợp chất carbonyl.

Birch reduction: Khử Birch, phản ứng phá hủy hệ thơm benzene dưới tác dụng
của kim loại kiềm trong ammonia lỏng, thường cần thêm alcohol làm nguồn cung
cấp proton.

Nếu sử dụng Ca trong dung môi amine (do có nhiệt độ sôi thấp) thì phản ứng này
gọi là Benkeser

1262 | Câu hỏi lí thuyết Hóa học hữu cơ OlympiaVN


Bouveaut-Blanc reduction: Khử Bouveaut – Blanc: Khử ester về alcohol bằng
Na trong ethanol sôi. Phản ứng này rất thông dụng trước khi LiAlH4 ra đời vào
những năm 1940.

Buchward-Hartwig reaction: Phản ứng Buchward – Hartwig: Phản ứng thường


xảy ra giữa một dẫn xuất halide thơm với aryl amine hay phenol dưới sự có mặt
của Pd(0) tạo thành sản phẩm ghép mạch trực tiếp arylamine hay aryl ether.

Bürgi – Dunitz trajectory: Góc tấn công Bürgi – Dunitz: Tác nhân nucleophile
tấn công vào nhóm carbonyl C=O theo góc 109o. Điều này có thể giải thích dễ
dàng bằng lý thuyết MO hữu cơ.
C
Cahn-Ingold-Prelog system: Hệ thống Cahn – Ingol – Prelog: Danh pháp đọc
R/S, E/Z…
Cannizaro reaction: Phản ứng Cannizaro: Phản ứng khử các aldehyde không
có H dưới tác dụng của base, thường hiệu quả với aldehyde thơm. Phản ứng
Cannizaro chéo giữa HCHO và một aldehyde khác trong sự có mặt của base sẽ
dẫn đến sự khử aldehyde đó.

Carbene: Carben. Một tiểu phân không mang điện, có tính electrophile trong đó
nguyên tử carbon có 6e và chỉ có hai liên kết. Trung gian có thể tồn tại ở dạng
singlet hay triplet. Phức chất của carbene với một cơ kim được gọi là carbenoid.
Carbocation: Carbocation. Carbon tích điện dương và có ba liên kết, dạng R3C+.
Trong trường hợp carbon có 5 liên kết thì đó là carbonium ion như CH5+.
Cascade reaction: Phản ứng liên hoàn. Sản phẩm của bước đầu tiên trở thành
chất đầu của phản ứng kế tiếp ngay trong cùng một bình phản ứng. Còn có tên
gọi khác là tandem hay domino reaction.

1263 | Câu hỏi lí thuyết Hóa học hữu cơ OlympiaVN


Chemoselectivity: Chọn lọc nhóm chức. Tùy điều kiện mà phản ứng ưu tiên một
nhóm chức hơn các nhóm chức khác.
Claisen condensation: Ngưng tụ Claisen. Phản ứng xảy ra giữa hai ester trong
sự có mặt của base để tạo thành -ketoester. Nếu hai ester ban đầu khác nhau sẽ
có tối đa bốn sản phẩm tạo thành.

Claisen rearrangement: Chuyển vị Claisen: Phản ứng chuyển vị [3.3] của allyl
vinyl ether dưới tác dụng của nhiệt để tạo thành , - carbonyl bất bão hòa.

Có một số phiên bản khác nhau của phản ứng này, sau đây là một số phản ứng
hay gặp:
Phản ứng Carroll:

Phản ứng Eschenmoser:

Phản ứng Ireland

Phản ứng Johnson:

1264 | Câu hỏi lí thuyết Hóa học hữu cơ OlympiaVN


Clemensen reduction: Phản ứng khử Clemensen: Chuyển hóa nhóm C=O về
CH2 dưới tác dụng của hỗn hống kẽm trong acid. Cơ chế của phản ứng hiện vẫn
chưa rõ ràng.

Collins reagent: Tác nhân Collins. CrO3.2Pyridine trong CH2Cl2, chuyên dùng
để oxid hóa alcohol về carbonyl tương ứng.
Concerted: Đồng bộ. Giai đoạn hình thành và cắt đứt liên kết xảy ra trong đúng
một bước duy nhất.
Corey – Chaycovsky reaction: Phản ứng Corey – Chaycovsky. Phản ứng này
tạo thành dẫn xuất cyclopropane sử dụng một ylide lưu huỳnh dạng R3S+X- hay
[R3S(=O)]+X-, với X là một halogen (thường là iod)

Corey-Fuchs reaction: Phản ứng Corey-Fuchs. Phản ứng tạo alkyne đầu mạch
từ aldehyde thông qua hai bước Wittig và tách loại halogen bằng một base mạnh
(thường là BuLi)

Corey-Kim oxidation: Phản ứng oxid hóa Corey – Kim. Phản ứng này dùng để
chuyển alcohol về carbonyl tương ứng bằng cách dùng Me2S và NCS trong sự có
mặt của một base (thường là Et3N) ở nhiệt độ thấp.

Corey-Winter elimination: Phản ứng Corey-Winter. Cho đến nay phản ứng này
vẫn là một trong số rất ít các phản ứng chuyển trực tiếp 1,2-diol về alkene dưới
tác dụng của một trialkyl phosphine hay trialkyl phosphite.

1265 | Câu hỏi lí thuyết Hóa học hữu cơ OlympiaVN


Cram’s rule: Quy tắc Cram. Quy tắc này dự đoán sự tạo thành sản phẩm ưu thế
hơn trong phản ứng cộng AN vào một aldehyde có tâm thủ tính ở C. Ông cho
rằng sản phẩm sẽ tạo thành từ hướng tấn công ít bị cản trở không gian hơn (mũi
tên in đậm).
Sau này quy tắc Felkin-Anh được sử dụng nhiều hơn, nhưng quy tắc Cram vẫn
còn được áp dụng trong nhiều trường hợp (nhất là khi tạo chelat).

Crieege oxidation: Phản ứng oxid hóa Crieege. Sự oxid hóa cắt mạch 1,2-diol
trong sự có mặt của Pb(OAc)4. Phản ứng cắt mạch diol dùng HIO4 gọi là phản
ứng Malaprade (Malaprade reaction)

Curtius rearrangement: Chuyển vị Curtius. Phản ứng chuyển vị acyl azide về


isocyanate.

Cycloaddition: Cộng đóng vòng. Phản ứng cộng đóng vòng là một nhóm phản
ứng tiêu biểu trong tổng hợp hữu cơ. Có một số loại hay gặp sau đây
Phản ứng [2+1]: Thường gặp nhất là phản ứng Simmons-Smith cộng carbene
vào alkene để tạo sản phẩm cyclopropane:

1266 | Câu hỏi lí thuyết Hóa học hữu cơ OlympiaVN


Phản ứng [2+2]: Phản ứng chủ yếu tạo ra vòng cyclobutane giữa hai liên kết đôi
(có thể là C=O hay C=C) dưới ảnh hưởng của nhiệt hoặc ánh sáng.

Phản ứng [3+2]: Phản ứng đóng vòng giữa một lưỡng cực (zwitterion) với một
alkene và một alkyne để tạo thành sản phẩm vòng 5 cạnh

Phản ứng này sau trở thành Click chemistry do giáo sư Sharpless phát triển. Hóa
học Click cũng là phản ứng [3+2] xảy ra dưới xúc tác kim loại để tạo thành các
dị vòng 5 cạnh. Ưu điểm của hóa học Click là phản ứng nhanh, hiệu suất cao và
không đi kèm sản phẩm phụ.
Phản ứng [2+4]. Phản ứng này được biết đến dưới cái tên phổ biến hơn là Diels
– Alder, xảy ra giữa một diene và dienophile dưới tác dụng của nhiệt để cho sản
phẩm cyclohexane.

Một loại diene hay gặp trong phản ứng Diels – Alder là Danishefsky diene có cấu
trúc như sau:

D
Darzens condensation: Ngưng tụ Darzens. Phản ứng ngưng tụ của một enolate
với một -haloester tạo thành sản phẩm epoxide, hay còn biết đến dưới cái tên
glycidic ester (X = O, G = OR).

1267 | Câu hỏi lí thuyết Hóa học hữu cơ OlympiaVN


Davis oxarizidines: Tác nhân Davis. Một dị vòng ba cạnh chứa cả O và N, rất
hiệu quả để đưa nhóm OH vào vị trí  của một aldehyde và ketone có H

Dess-Martin periodinane. Tác nhân oxid hóa Dess-Martin (DMP). Tác nhân iod
hóa trị cao dùng để oxid hóa êm dịu alcohol về carbonyl tương ứng.

Dieckmann condensation: Ngưng tụ Dieckmann. Phiên bản nội phân tử của


phản ứng Claisen

Duff reaction: Phản ứng Duff. Chuyển arene thành aldehyde thơm tương ứng
dưới tác dụng của hexamethylene tetramine trong acetic acid rồi thủy phân muối
iminium tạo thành.

E
Ene-reaction: Phản ứng ene. Phản ứng còn được biết đến với cái tên Alder-ene
là một phản ứng đồng bộ (pericyclic) giữa hai alkene dẫn đến sự dịch chuyển của
một hydrogen allylic và tạo thành một liên kết C-C mới.

1268 | Câu hỏi lí thuyết Hóa học hữu cơ OlympiaVN


Eschenmoser – Tanabe fragmentation: Phân mảnh Eschenmoser-Tanabe: Đun
nóng một ,-epoxyketone trong sự có mặt của tosylhydrazone sẽ dẫn đến sự phá
vỡ phân tử tạo thành một alkyne.

Eschenweiler-Clark methylation: Phản ứng methyl hóa theo Eschenweiler-


Clarke. Chuyển một amine bậc 1, hay bậc 2 thành amine bậc 3 (N, N-
dimethylamine hay N-methylamine) dưới tác dụng của HCOOH và HCHO.

F
Felkin-Anh rule: Quy tắc Felkin – Anh. Quy tắc dự đoán sản phẩm cộng
nucleophile vào nhóm CO của aldehyde có tâm thủ tính ở C. Quy tắc có tính
chính xác cao hơn quy tắc Cram do có xét đến hiệu ứng lập thể điện tử
(stereoelectronic effect) và góc tấn công Burgi-Dunitz.

Fétizon reagent: Tác nhân Fétizon, oxid hóa alcohol bậc 1 và lactone với độ
chọn lọc cao.

Fischer complexes: Phức carbene Fischer: Carbene có nhân kim loại, nguyên
thủy phức Fischer là nhân chromium.

1269 | Câu hỏi lí thuyết Hóa học hữu cơ OlympiaVN


Friedel-Crafts reaction: Phản ứng Friedel – Crafts. Phản ứng đưa nhóm thế vào
nhân thơm hoạt hóa trong sự có mặt của Lewis acid. Có hai loại là acyl hóa và
alkyl hóa, trong đó phản ứng alkyl hóa dễ dẫn đến sự thế nhiều lần.

Frost-Musulin circle: Vòng tròn Frost-Musulin. Một hệ thống đơn giản dùng để
xác định năng lượng tương đối các orbital  trong hệ thơm Hückel mà không cần
phải giải định thức Hückel cho hệ thơm.

Frontier orbitals: Orbital biên. Có ba loại MO chính


HOMO = Highest occupied molecular orbitals: MO bị chiếm (2e) có năng lượng
cao nhất.
LUMO = Lowest Unoccupied Molecular orbitals: MO không bị chiếm (0e) có
năng lượng thấp nhất
SOMO = Singly Occupied Molecular orbitals: MO bị chiếm 1e có năng lượng
cao nhất (gốc tự do)
Tương tác giữa 3 loại orbital này ảnh hưởng nhiều đến hướng và tốc độ phản ứng.
G
Gatterman formylation: Phản ứng Gatterman. Phản ứng đưa nhóm CHO vào
nhân thơm, và không hiệu quả nếu vòng thơm có chứa nhóm rút. Có ba phiên bản
chính của phản ứng này.
Phản ứng Gatterman: Phản ứng của một arene, một cyanide, HCl dưới tác dụng
của acid Lewis

1270 | Câu hỏi lí thuyết Hóa học hữu cơ OlympiaVN


Phản ứng Gatterman-Koch: Thay vì dùng cyanide thì CO được sử dụng.

Phản ứng Gatterman-Olah: Sử dụng formyl fluoride trong sự có mặt của BF3

Gilman reagent: Tác nhân Gilman. Hợp chất cơ kim đồng-lithium với công thức
tổng quát dạng R2CuLi được tạo thành khi cho 2 mol alkyl lithium tác dụng với
muối Cu(I).

Gomberg-Bachmann reaction: Phản ứng Gomberg-Bachmann: Khử hóa 1e


muối aryl diazonium tạo sản phẩm ghép cặp biaryl.

Grieco selenylation: Phản ứng selenium hóa theo Grieco: Phản ứng chuyển một
alcohol thành một dẫn xuất seleno với sự quay cấu hình

Green chemistry: Hóa học xanh. Sử dụng các điều kiện phản ứng ít gây bất lợi
cho môi trường nhất để hạn chế ô nhiễm đến mức tối thiểu.
Grubbs catalyst: Xúc tác Grubbs. Xúc tác phức carbene ruthenium chuyên dùng
cho phản ứng hoán vị (metathesis)

1271 | Câu hỏi lí thuyết Hóa học hữu cơ OlympiaVN


H
Hantzsch ester: Hantzsch ester, dialkyl 4-aryl-2,6-dimethyl-1,4-
dihydropyridine-3,5-dicarboxylate, chuyên dùng để cho hydrogen trong các phản
ứng khử theo cơ chế ion.

Hapto number: Số vị trí trong một ligand tạo liên kết với kim loại, ký hiệu .
Nó còn thể hiện số lượng electron mà một ligand trung hòa điện đóng góp cho
kim loại trung tâm.
Heck reaction: Phản ứng Heck. Phản ứng ghép cặp palladium giữa một aryl
halide (hay vinyl halide) với một alkene trong sự có mặt của Lewis base.

Hell-Volhard-Zelinskii reaction. Phản ứng Hell-Volhard-Zelinskii. Phản ứng


này đưa halogen vào vị trí C của nhóm acid dưới tác dụng của Br2/P đi qua trung
gian enol của acyl bromide trung gian.

1272 | Câu hỏi lí thuyết Hóa học hữu cơ OlympiaVN


Henry reaction Phản ứng Henry. Một biến thể của phản ứng aldol giữa một dẫn
xuất nitro và một aldehyde, trong đó dẫn xuất nitro sẽ tạo enolate dưới tác dụng
của base.

Hiyama coupling: Phản ứng ghép Hiyama. Phản ứng ghép cặp giữa một silane
và một dẫn xuất halide trong sự có mặt của Pd(0) và anion F-.

Trong ghép Hiyama-Denmark (Hiyama-Denmark coupling) thì phản ứng giữa


silanol và dẫn xuất halide không cần đến ion F-

Hofmann elimination: Tách Hofmann. Là phản ứng tách theo cơ chế E2 tạo
thành alkene có ít nhóm thế hơn. Thường gặp với các dẫn xuất muối ammonium
bậc 4, muối sulfonium và dẫn xuất fluoride.
Hofmann-Löffler-Freytag reaction: Phản ứng Hofmann-Löffler-Freytag. Phản
ứng hoạt hóa liên kết C-H ở một ví trí xa nhóm chức chính (tương tự Barton).
Phản ứng xảy ra khi nhiệt phân N-haloammonium ion sinh ra gốc tự do rồi tương
tác với H ở vị trí 6 thông qua trạng thái chuyển vòng 6 cạnh. Sản phẩm thu được
sau cùng là -haloamine.

Hofmann rearrangement Phản ứng chuyển vị Hofmann. Phản ứng chuyển vị N-


haloamine thành isocyanate. Trung gian kém bền có thể chuyển thành amine hay
carboxamate tùy tương tác với nucleophile tiếp sau:

1273 | Câu hỏi lí thuyết Hóa học hữu cơ OlympiaVN


Horner-Wadsworth-Emmons (HWE) reaction: Phản ứng HWE. Là một phiên
bản của phản ứng Wittig, trong đó phosphonium ylide được thay thế bằng base
liên hợp của một phosphonate ester, sản phẩm chủ yếu là E alkene.

Hosomi – Sakurai reaction: Phản ứng Hosomi – Sakurai. Xử lý một allyl silane
hay allyl stanane với một tác nhân carbon electrophile (carbocation hay carbonyl)
sẽ tạo liên kết C-C mới và có sự dịch chuyển nối đôi allyl.

Houben – Hoesch acylation: Phản ứng acyl hóa Houben-Hoesch. Phản ứng điều
chế aryl ketone giữa một arene và một nitrile trong sự có mặt của Lewis acid

Huang-Minlon modification Phản ứng Huang-Minlon (hay Hoàng Minh Long).


Sử dụng KOH cùng dung môi có nhiệt độ sôi cao (glycol solvent) để khử Wolff-
Kishner aldehyde, ketone.

Hückel aromaticity Lý thuyết tính thơm Hückel. Những hệ thống liên hợp vòng
kín có số electron  thỏa mãn quy tắc N= 4n + 2 là những hệ ổn định đặc biệt gọi
là hệ thơm.
I
Ionic hydrogenation: Phản ứng hydrogen hóa qua cơ chế ion. Phản ứng được
tiến hành bằng cách cho liên kết  nhận proton và ion hydride (H-). Chẳng hạn
khử C=O thành CHOH bằng Et3SiH.
Ionic solvent: Chất lỏng ion. Là các hợp chất ion có điểm chảy thấp nên ở nhiệt
độ thường tồn tại ở dạng lỏng. Có thể xem chúng như các dung môi xanh do ít
độc hại, ít có khả năng gây cháy nổ. Thường dùng các dẫn xuất imidazolium, ví
dụ [bmim][BF4].

1274 | Câu hỏi lí thuyết Hóa học hữu cơ OlympiaVN


J
Jacobsen – Katsuki epoxidation: Phản ứng epoxy hóa Jacobsen-Katsuki. Phản
ứng giữa một alkene đơn giản và hypochlorite trong sự có mặt của xúc tác
SALEN.

Jones reagent: Tác nhân Jones. Hỗn hợp gồm K2Cr2O7, H2SO4, H2O và acetone
có thể oxid hóa được alcohol bậc 1 về acid.
Julia-Lythgoe olefination: Phản ứng tổng hợp olefin theo Julia-Lythgoe. Sự tạo
thành alkene giữa một aldehyde hay ketone với một anion tạo thành từ dẫn xuất
sulfone. Để tạo được alkene thì trung gian tạo thành cần phải đi qua quá trình tách
khử.

Trong biến thể Julia-Kocieńsky thì giai đoạn tách khử không còn cần thiết nữa (X
= N).

K
Kharash reaction: Phản ứng Kharash. Sự cộng vào alkene trái Markovnikov.
Knoevenagel condensation: Phản ứng Knoevenagel. Phản ứng giữa hợp chất có
nhóm methylene linh động (thường có dạng RCO-CH2COR’) với một ketone để
tạo sản phẩm liên hợp.

1275 | Câu hỏi lí thuyết Hóa học hữu cơ OlympiaVN


Nếu xúc tác cho phản ứng là một amine bậc 2 trong sự có mặt của carboxylic acid
thì đó là sự bổ sung Doebner

Kolbe electrolysis. Điện phân Kolbe. Điện phân muối carboxylate để tạo gốc tự
do alkyl.

Krapcho decarboxylation: Phản ứng decarboxylation Krapcho. Phản ứng nhiệt


phân -ketoester trong sự có mặt của LiCl/DMF sẽ dẫn đến sự tách CO2. Thường
dùng đối với diester của malonic acid.

Kraus-Pinnick oxidation. Phản ứng oxid hóa Kraus-Pinnick (thường gọi là


Pinnick) là sự oxid hóa aldehyde về acid trong sự có mặt của NaClO2 và 2-methyl-
2-butene để xử lý sản phẩm phụ chlorine (chlorine scavenger).

Kumada coupling: Phản ứng ghép Kumada. Phản ứng xảy ra giữa tác nhân
Grignard với một alkyl halide trong sự có mặt của Pd(0).

1276 | Câu hỏi lí thuyết Hóa học hữu cơ OlympiaVN


L
Lemieux – Johnson cleavage: Phản ứng cắt mạch Lemieux-Johnson. Oxid hóa
alkene bằng OsO4 rồi xử lý tiếp diol sinh ra bằng periodate. Osmate ester sinh ra
sẽ được cắt thành carbonyl và hoàn trả lại OsO4.

Leuckart reaction: Phản ứng Leuckart. Khử hóa iminium trung gian trong phản
ứng giữa carbonyl và NH3 bằng HCOOH để thu được amine bậc 1. Thường không
dùng hỗn hợp NH3/HCOOH mà muối ammonium formate sẽ được sử dụng. Khi
đun nóng muối sẽ phân hủy cho NH3 và HCOOH.

LICKOR. Schlosser base. Hỗn hợp 1:1 giữa t-BuOK và BuLi.


Lindlar hydrogenation: Hydro hóa Lindlar. Phản ứng hydrogen hóa alkyne trên
xúc tác Pd đã được đầu độc một phần bằng muối chì/quinoline để phản ứng dừng
lại ở alkene.

Lipshutz cuprate: Cơ đồng Lipshutz . Là một cơ đồng có tính nucleophile cao


được tạo thành khi cho 2 đương lượng RLi tác dụng với CuCN.

Lossen rearrangement: Chuyển vị Lossen. Phản ứng chuyển vị N-


acylhydroxamate tạo thành isocyanate.

1277 | Câu hỏi lí thuyết Hóa học hữu cơ OlympiaVN


M
Mannich reaction: Phản ứng Mannich. Phản ứng giữa một enol và iminium để
tạo thành -dialkylamino ketone. Enol và iminium ion đều có thể được tạo thành
in situ (bên trong phản ứng)

Markovnikov’s rule: Quy tắc Markovnikov. Phản ứng cộng AE vào alkene xảy
ra đi qua trung gian carbocation bền vững nhất.
McMurry reaction: Phản ứng ghép cặp McMurry. Phản ứng dime hóa giữa hai
aldehyde hay hai ketone bằng Titanium kim loại hay muối Ti(III). Thường chỉ
xảy ra sự tạo thành alkene với ketone, với aldehyde chủ yếu dừng ở giai đoạn tạo
diol.

Meervain-Pondorf-Verley reduction: Phản ứng khử Meervain-Pondorf-


Verley. Khử ketone thành alcohol dưới tác dụng của isopropoxide nhôm trong
isopropanol. Do phản ứng là một cân bằng nên chiều ngược lại có thể xảy ra được
(sự oxid hóa Oppenauer).

Metathesis: Phản ứng hoán vị. Phản ứng giữa hai olefin (thường đầu mạch) dẫn
đến sự loại bỏ một alkene (thường là ethylene) hay ghép giữa alkene và alkyne
tạo thành diene. Hai alkyne cũng có thể hoán vị, nhưng ít gặp hơn.

Michael addition Phản ứng cộng Michael. Sự cộng nucleophile 1,4 vào hệ ,-
carbonyl bất bão hòa. (G = COR, COOR, CN…)

1278 | Câu hỏi lí thuyết Hóa học hữu cơ OlympiaVN


Mitsunobu reaction: Phản ứng Mitsunobu. Sự xoay chuyển cấu hình SN2 khi
cho một nucleophile tác dụng với muối alkyl phosphonium sinh ra từ phản ứng
của alcohol với trung gian tạo thành từ diethyl diazocarboxylate và triphenyl
phosphine.

Moffat-Pfitzner oxidation Phản ứng oxid hóa Moffat-Pfitzner. Oxid hóa alcohol
về carbonyl tương ứng bằng DMSO và DCC.

Morita-Baylis-Hilman reaction: Phản ứng Morita-Baylis-Hilman: Phản ứng


giữa một dẫn xuất acrylic acid với một aldehyde trong sự có mặt của một amine
bậc ba hay một phosphine tạo thành -hydroxyalkylacrylic.

Mukaiyama aldol reaction: Phản ứng aldol hóa Mukaiyama. Phản ứng giữa một
enol silane và một aldehyde trong sự có mặt của acid Lewis.

Nếu thay aldehyde bằng dẫn xuất halide sẽ thành alkyl hóa Mukaiyama.

N
Negishi coupling: Ghép Negishi. Phản ứng ghép xúc tác palladium giữa một
alkyl halide và cơ kẽm hay cơ zirconium

1279 | Câu hỏi lí thuyết Hóa học hữu cơ OlympiaVN


Norrish reaction Phản ứng Norrish. Sự quang hóa ketone ở trạng thái kích thích.
Có hai kiểu phản ứng
Cắt mạch Norrish kiểu I. Tạo gốc tự do alkyl và gốc acyl. Gốc acyl nhanh chóng
loại CO để tạo gốc alkyl thứ hai.

Phản ứng Norrish kiểu II. Chuyển dịch hydrogen nội phân tử từ carbon qua
oxygen đi qua trung gian vòng 6 cạnh.

Non-classics carbocation: Carbocation không cổ điển. Carbocation được bền


hóa bằng tương tác với nhóm ổn định ở vị trí không kề với tâm carbocation bằng
cầu  hay . Thường gặp trong hệ norbonan.
O
Oxyanion Cope rearrangement: Chuyển vị oxygen-anion Cope. Chuyển vị [3.3]
của 1,5-diene được tăng tốc khi có mặt nhóm alkoxide ở vị trí 3.

Ozonolysis: Ozone phân. Phản ứng ozone phân alkene tạo thành nhiều sản phẩm
tùy thuộc vào cách xử lý sau phản ứng, chẳng hạn với Me2S, (MeO)3P, thu được
dicarbony, còn nếu là H2O2 sẽ thu được acid.

P
Parikh-Doering oxidation: Phản ứng oxid hóa Parikh-Doering. Phản ứng oxid
hóa alcohol về carbonyl tương ứng sử dụng phức pyridine-sulfur trioxide trong
sự có mặt của base.

1280 | Câu hỏi lí thuyết Hóa học hữu cơ OlympiaVN


Pericyclic reaction: Phản ứng pericyclic. Phản ứng đi qua cơ chế đồng bộ thông
qua một trạng thái chuyển tiếp vòng, trong đó mọi liên kết hình thành và mất đi
xảy ra gần như cùng lúc.
Perkin condensation: Phản ứng Perkin. Phản ứng giữa acetic anhydride với một
aldehyde thơm trong sự có mặt của một muối acetate kim loại kiềm sẽ tạo thành
dẫn xuất của cinnamic acid.

Peterson reaction: Phản ứng Peterson: Sự tạo thành alkene khi cho aldehyde
hay ketone phản ứng với trialkylsilyl anion trong sự có mặt của acid hay base.

Pinacol reaction: Phản ứng pinacol. Sự dime hóa khử hóa hợp chất carbonyl
bằng hỗn hống Mg trong sự vắng mặt của nguồn cấp proton sẽ tạo thành 1,2-diol.

Polonovski reaction: Phản ứng Polonovski: Chuyển một amine bậc ba thành
iminium ion để cuối cùng tạo carbonyl đi qua trung gian N-oxide.

Prilezhaev reaction: Phản ứng Prilezhaev: Sự tạo thành epoxide khi cho alkene
phản ứng với peracid.

1281 | Câu hỏi lí thuyết Hóa học hữu cơ OlympiaVN


Prins reaction: Phản ứng Prins. Phản ứng giữa một alkene và một aldehyde
trong sự có mặt của Lewis acid. Thường tạo thành dẫn xuất 1,3-dioxolane qua
trung gian 1,3-diol nếu dùng dư aldehyde.

Pyrolytic elimination: Phản ứng tách nhiệt. Tạo thành alkene từ các dẫn xuất dễ
tách, thường là N-oxide (tách Cope) hay tách từ ester, thoester (Chugaev). Trong
tổng hợp hữu cơ sự tách sulfoxide và selenoxide tạo thành alkene tương đối phổ
biến.

R
Ramberg-Bäcklund reaction: Phản ứng Ramberg-Bäcklund. Sự tạo thành
alkene bởi sự ghép hai gốc alkyl quanh nhóm sulfone khi xử lý sulfone với
halogen trong sự có mặt của base. Trung gian sinh ra sẽ tách loại SO2 tạo alkene.

Raney nickel: Nickel Raney. Là loại nickel đặc biệt được tạo thành khi loại nhôm
ra khỏi hợp kim Ni-Al bằng NaOH.
Reformatskii reaction: Phản ứng Reformatskii. Phản ứng giữa cơ kẽm tạo thành
khi cho kẽm tác dụng với -haloester với hợp chất carbonyl (thường là aldehyde)
tạo thành -hydroxyester.

Reich – Gross formylation: Phản ứng Reich-Gross chuyển arene về aldehyde


tương ứng khi xử lý arene với methyl dichloromethyl ether trong sự có mặt của

1282 | Câu hỏi lí thuyết Hóa học hữu cơ OlympiaVN


Lewis acid. Aldehyde được tạo thành khi thủy phân trung gian tương ứng. Phản
ứng không hiệu quả với arene gắn nhóm thế phản hoạt hóa.

Reppe chemistry: Phản ứng Reppe. Sự oligomer hóa alkyne tạo thành các
polyene.

Retrosynthesis analysis: Phân tích tổng hợp ngược (hay hồi tổng hợp). Quá trình
xây dựng sơ đồ tổng hợp bằng cách phân tích các phản ứng tạo thành chất đích
để từ đó suy ra chất đầu cần sử dụng.
Robinson annelation (annulation) Phản ứng vòng hóa Robinson. Phản ứng giữa
một carbonyl và một hợp chất dạng ,-carbonyl bất bão hòa để tạo hệ
cyclohexanone thông qua cơ chế cộng Michael-aldol.

Rosenmund reduction: Khử Rosenmund. Khử hóa acyl chloride bằng hydrogen
với hệ xúc tác Pd/BaSO4 thu được aldehyde.

S
Saegusa oxidation: Phản ứng oxid hóa Saegusa. Sự oxid hóa enol silane trong
sự có mặt của Pd(OAc)2 và một chất đồng oxid hóa (sẽ bị khử) như benzoquinone.

Sarret reagent: Tác nhân Sarret. Phức CrO3.Pyridine, dùng để oxid hóa alcohol
về carbonyl tương ứng.

1283 | Câu hỏi lí thuyết Hóa học hữu cơ OlympiaVN


Schlosser modification Wittig reaction: Phản ứng Wittig được bổ khuyết bởi
Schlosser. Tiến hành phản ứng Wittig ở nhiệt độ thấp bằng việc thêm muối
lithium vào sẽ thu được chủ yếu E alkene.

Schmidt rearrangement: Chuyển vị Schmidt. Xử lý aldehyde hay ketone với


hydrazoic acid (HN3) tạo thành amide.

Schrock catalyst: Xúc tác Schrock. Xúc tác molybdenium carbene cho phản ứng
metathesis

Seyfert-Gilbert homologation: Phản ứng Seyfert – Gilbert. Sử dụng tác nhân


Bestmann (hay Ohira Bestmann) để chuyển hóa aldehyde hay ketone về alkyne
trong sự có mặt của base.

Sharpless asymmetric reaction: Phản ứng Sharpless. Có hai phản ứng chính
Sharpless epoxidation. Sự epoxide hóa theo Sharpless dùng để epoxide hóa allyl
alcohol với tert-butylperoxide, titanium isopropoxide và ligand tartrate để thu
được sản phẩm epoxide có cấu hình dự đoán được.

Sharpless dihydroxylation: Dihydroxyl hóa alkene với ligand dihydroquinine


quang hoạt và potassium osmate thu được diol với cấu hình có thể dự đoán được.

1284 | Câu hỏi lí thuyết Hóa học hữu cơ OlympiaVN


Shi oxidation: Phản ứng Shi. Sự epoxide hóa alkene đầu mạch bằng dẫn xuất
dioxirane có nguồn gốc từ đường.

Sigmatropic rearrangement: Chuyển vị sigmatropic. Phản ứng đồng bộ trong


đó một nguyên tố ở vị trí bất kỳ trong chất phản ứng dịch chuyển tới một vị trí
khác kéo theo sự di chuyển của các liên kết . Tiêu biểu có chuyển vị Claisen và
Cope

Sommelet-Hauser rearrangement: Phản ứng chuyển vị Sommelet – Hauser:


Chuyển vị benzyl trialkylammonium tạo thành o-alkylbenzylamine trong sự có
mặt của base mạnh.

Sonogashira coupling: Phản ứng ghép Sonogashira. Phản ứng ghép chéo dùng
xúc tác Pd(0) giữa alkyl đầu mạch và một dẫn xuất halide trong sự có mặt của
Cu(I) cùng với base.

1285 | Câu hỏi lí thuyết Hóa học hữu cơ OlympiaVN


Stille coupling: Ghép Stille: Phản ứng ghép xúc tác Pd(0) giữa cơ thiếc và một
dẫn xuất halide.

Stobbe condensation: Ngưng tụ Stobbe: Phản ứng giữa một succinate diester và
một aldehyde trong sự có mặt của base tạo thành dẫn xuất alkylidene succinic
acid.

Suzuki-Miyaura coupling: Phản ứng Suzuki (hay Suzuki – Miyaura): Phản ứng
ghép xúc tác Pd giữa một dẫn xuất boronic acid và một halide. Có thể sử dụng
potassium alkyltriifluoroborate.

Swern oxidation: Sự oxid hóa Swern: Phản ứng xảy ra ở nhiệt độ thấp với sự có
mặt của hỗn hợp gồm oxalyl chloride, DMSO và triethylamine.

T
Tebbe olefination: Phản ứng Tebbe. Phản ứng sử dụng tác nhân Tebbe để chuyển
hóa hợp chất carbonyl thành alkene có nhóm methylene. Phản ứng không thể tạo
thành các alkene nhiều nhóm thế hơn.

Nếu dùng hỗn hợp kẽm, dibromomethylene và TiCl4 thì phản ứng có tên là
Lombardo

1286 | Câu hỏi lí thuyết Hóa học hữu cơ OlympiaVN


Thorpe-Ziegler condensation: Phản ứng ngưng tụ Thorpe-Ziegler: Phản ứng
ngưng tụ nội phân tử dinitrile để tạo thành -aminonitrile trong sự có mặt của
base

Tischenko reaction: Phản ứng Tischenko: Phản ứng xúc tác alkoxide nhôm sẽ
chuyển aldehyde thành ester của alcohol bậc 1 tương ứng.

Tollens reagent: Thuốc thử Tollens. Hỗn hợp gồm Ag2O, NH3 và H2O sẽ oxid
hóa aldehyde thành carboxylic acid.
Tsuji – Trost reaction: Phản ứng Tsuji – Trost: Phản ứng xúc tác palladium hay
gặp trong sự thay thế allylic ester bằng carbanion nucleophile.

U
Umpolung: Chuyển hóa lưỡng cực. Thường ám chỉ sự thay đổi tính
nucleophilic/electrophilic của một vị trí trong phân tử. Ví dụ nguyên tử C của
nhóm carbonyl thường có tính electrophile (cộng AN), nhưng khi chịu ảnh hưởng
của CN- thì cũng chính C đó trở thành nucleophile (phản ứng benzoin).
V
Vilsmeyer-Haack formylation: Phản ứng Vilsmeyer-Haack. Chuyển arene
thành aldehyde tương ứng dưới tác dụng của dimethyl formamide và POCl3 (có
thể dùng oxalyl chloride). Phản ứng không hiệu quả với arene có nhóm phản hoạt
hóa.

1287 | Câu hỏi lí thuyết Hóa học hữu cơ OlympiaVN


W
Wacker oxidation: Phản ứng Wacker. Oxid hóa alkene thành ketone (riêng
ethylene thành CH3CHO), phản ứng đi qua trung gian enol.

Wagner-Meervain rearrangement: Chuyển vị Wagner-Meervain. Thường gặp


sự chuyển dịch 1,2-hydride hay 1,2-alkyl dẫn đến sự tạo thành carbocation bền
hơn.

Wharton rearrangement: Chuyển vị Wharton. Khi xử lý epoxyketone trong


điều kiện của phản ứng Wolff-Kishner (Bổ sung Huang Minlon) sẽ dẫn đến sự
tạo thành allyl alcohol.

Wilkinson catalyst: Xúc tác Wilkinson. ClRh(PPh3)3, thường dùng để xúc tác
phản ứng hydrogen hóa.
Wittig reaction: Phản ứng Wittig: Tổng hợp alkene từ phosphorus ylide và
carbonyl trong sự có mặt của base mạnh.

Wolff-Kishner reduction: Phản ứng khử Wolff-Kishner: Loại bỏ oxygen trong


aldehyde hay keton bằng hydrazine trong base.

Wurtz coupling: Ghép Wurtz. Ghép mạch hai dẫn xuất alkyl halide dưới tác dụng
của kim loại hoạt động mạnh (Na, K…)

1288 | Câu hỏi lí thuyết Hóa học hữu cơ OlympiaVN


Z
Zaitsev rule: Quy tắc Zaitsev: Phản ứng tách E2 của alkyl halide (trừ fluoride)
hay sulfonate dẫn đến sự tạo thành alkene có nhiều nhóm thế hơn.

1289 | Câu hỏi lí thuyết Hóa học hữu cơ OlympiaVN


Phụ lục 2: Các phản ứng mang
tên người
1. Phản ứng acyl hóa Friedel-Crafts

Phản ứng acyl hóa Friedel-Crafts dùng để chuyển một hợp chất aryl và một acyl
halide (hoặc anhydride) thành aryl ketone với xúc tác Lewis acid (như AlCl3).

2. Phản ứng alkyl hóa Friedel-Crafts

Phản ứng alkyl hóa Friedel-Crafts dùng để gắn thêm một nhóm thế alkyl vào một
hợp chất aryl sẵn có, bằng tác nhân alkyl halide và xúc tác Lewis acid (như AlCl3).

3. Phản ứng amine hóa Buchwald-Hartwig

Amine hóa Buchwald-Hartwig là phản ứng hữu cơ được dùng để tạo liên kết
carbon-nitrogen. Đây là một phản ứng ghép cặp chéo (cross-coupling) quan trọng
của một aryl halide với một amine, sử dụng xúc tác palladium và base mạnh (như
NaOtBu).

4. Phản ứng Appel

1290 | Câu hỏi lí thuyết Hóa học hữu cơ OlympiaVN


Phản ứng Appel được dùng để chuyển một alcohol thành alkyl halide bởi tác nhân
tetrahalomethane và triphenylphosphine.

5. Phản ứng Baylis-Hillman

Baylis-Hillman là phản ứng hữu cơ để tạo liên kết C-C mới từ một hợp chất
carbonyl không no α, β và một aldehyde, hoặc ketone hoạt động, hoặc một hợp
chất chứa nguyên tử carbon có tính electrophile. Phản ứng thường dùng xúc tác
DABCO7, nhưng các phosphine hoặc amine bậc ba khác cũng có thể được lựa
chọn.

6. Phản ứng Biginelli

Biginelli là phản ứng tổng hợp ‘một bình’ (one-pot)8 từ ba chất hữu cơ - một β-
keto ester, một aryl aldehyde và urea, trong môi trường acid - để tạo thành dị vòng
pyrimidone.
7. Phản ứng ngưng tụ Cannizzaro

7 1,4-diazabicyclo[2.2.2]octane: N2(C2H4)3
8 Các phản ứng xảy ra trong cùng một bình phản ứng mà không cần tinh lọc chất tạo ra
trung gian.

1291 | Câu hỏi lí thuyết Hóa học hữu cơ OlympiaVN


Cannizzaro là phản ứng oxid hóa-khử (với xúc tác hydroxide base) giữa hai phân
tử aldehyde, tạo thành alcohol bậc I và carboxylic acid.

8. Phản ứng chuyển vị Ireland-Claisen

Chuyển vị Ireland-Claisen là phản ứng hữu cơ dùng để chuyển một allyl ester
thành carboxylic acid không no γ,δ bởi các tác nhân: LDA9, TMSCl10 và
NaOH/H2O.

9. Phản ứng Beckmann

Chuyển vị Beckmann là phản ứng hữu cơ để chuyển oxime thành amide với xúc
tác acid.

9 Lithium diisopropylamide: [(CH3)2CH]2NLi


10 Trimethylsilyl chloride: (CH3)3SiCl

1292 | Câu hỏi lí thuyết Hóa học hữu cơ OlympiaVN


10. Phản ứng chuyển vị Claisen

Chuyển vị Claisen là phản ứng hữu cơ để chuyển allyl vinyl ether thành hợp chất
carbonyl không no γ,δ dưới tác dụng nhiệt (đun nóng) hoặc có xúc tác Lewis acid.
Phản ứng này thuộc họ phản ứng sigmatropic - trong đó, các liên kết được tạo
thành và phá vỡ cùng lúc.
Nếu allyl vinyl ether ban đầu có các nhóm thế thì hóa lập thể của phản ứng có thể
được dự đoán bằng cách vẽ phân tử ở cấu dạng ghế và đặt các nhóm thế ở vị trí
ưu tiên là equatorial (liên kết biên) để giảm tương tác không gian xuống cực tiểu.

11. Phản ứng chuyển vị Cope

Chuyển vị Cope là phản ứng hữu cơ dưới tác dụng nhiệt (đun nóng) để chuyển
1,5-diene thành một đồng phân cấu tạo 1,5-diene khác. Phản ứng này thuộc họ
sigmatropic (với các giai đoạn hình thành và phá vỡ liên kết xảy ra đồng thời).
Nếu diene ban đầu có các nhóm thế thì hóa lập thể của phản ứng có thể được dự
đoán bằng cách vẽ phân tử ở cấu dạng ghế và đặt các nhóm thế ở vị trí ưu tiên là
equatorial (liên kết biên) để giảm tương tác không gian xuống cực tiểu. Chuyển
v Cope là một quá trình cân bằng và chiều chuyển dịch của cân bằng phụ thuộc
vào độ bền (tính ổn định) của chất đầu và sản phẩm.

1293 | Câu hỏi lí thuyết Hóa học hữu cơ OlympiaVN


12. Phản ứng chuyển vị Curtius

Chuyển vị Curtius là một phản ứng hữu cơ để chuyển acyl azide thành isocyanate
khi đun nóng. Cơ chế phản ứng gồm một giai đoạn dịch chuyển nhóm R từ vị trí
liên kết với nhóm carbonyl tới nguyên tử nitrogen gần nhất và giải phóng khí N2
(đây là động lực của phản ứng) và tạo thành sản phẩm isocyanate. Sản phẩm này
có hoạt tính hóa học mạnh và có thể phản ứng tiếp với các tác nhân nucleophile
trong dung dịch (nếu có).

13. Phản ứng chuyển vị Eschenmoser-Claisen

Chuyển vị Eschenmoser-Claisen là phản ứng hữu cơ chuyển allylic alcohol thành


amide không no γ,δ khi đun nóng với N,N-dimethylacetamide.

14. Phản ứng chuyển vị Fries

Chuyển vị Fries là phản ứng hữu cơ để chuyển phenyl ester thành ortho- và para-
hydroxy aryl ketone với xúc tác Lewis acid, sau đó xử lí với Bronsted acid.

1294 | Câu hỏi lí thuyết Hóa học hữu cơ OlympiaVN


15. Phản ứng chuyển vị Hofmann

Chuyển vị Hofmann là phản ứng hữu cơ để chuyển amide bậc I thành amine bậc
I, sử dụng các tác nhân: halogen, base, nước và nhiệt (đun nóng).

16. Phản ứng chuyển vị Jonhson-Claisen

Chuyển vị Johnson-Claisen là phản ứng hữu cơ để chuyển allylic alcohol thành


ester không no γ,δ khi đun nóng với trialkyl orthoacetate.

17. Phản ứng chuyển vị Wagner-Meerwein

Chuyển vị Wagner-Meerwein là phản ứng hữu cơ để chuyển alcohol thành


olefin, sử dụng xúc tác acid.

1295 | Câu hỏi lí thuyết Hóa học hữu cơ OlympiaVN


18. Phản ứng ngưng tụ Claisen

Ngưng tụ Claisen là phản ứng hữu cơ để tạo liên kết carbon-carbon mới. Phản
ứng này đi từ hai phân tử ester với xúc tác alkoxide base trong alcohol tương ứng,
tạo ra β-keto ester.

19. Phản ứng cộng aldol Mukaiyama

Phản ứng cộng aldol Mukaiyama xảy ra giữa aldehyde và silyl enol ether, với
xúc tác Lewis acid (như TiCl4), sau đó xử lý với nước, thu được 1,3 ketol.

1296 | Câu hỏi lí thuyết Hóa học hữu cơ OlympiaVN


20. Phản ứng cộng Michael

Phản ứng cộng Michael xảy ra giữa một hợp chất chứa nhóm methylene hoạt
động và một hợp chất carbonyl không no, liên hợp. Phản ứng sử dụng xúc tác
base, sau đó xử lí với acid.

21. Phản ứng Dakin-West

Phản ứng Dakin-West được dùng để chuyển một amino acid và một anhydride
thành acylamino ketone, với xúc tác base (đun nóng).

22. Phản ứng Diels-Alder

Diels-Alder là phản ứng hữu cơ để chuyển một diene liên hợp và một tác nhân
dienophile thành một olefin vòng khi đun nóng.

23. Phản ứng oxid hoá Sharpless

1297 | Câu hỏi lí thuyết Hóa học hữu cơ OlympiaVN


Oxid hóa Sharpless là phản ứng hữu cơ chọn lọc lập thể để chuyển một allylic
alcohol thành epoxy alcohol, sử dụng xúc tác titanium isopropoxide (Ti(OiPr)4)
và t-butyl hydroperoxide (TBHP) và diethyl tartrate quang hoạt (DET).

24. Phản ứng Eshweiler-Clarke

Eshweiler-Clarke là phản ứng hữu cơ để chuyển amine bậc I hoặc II thành amin
bậc III, sử dụng formaldehyde và formic acid.

1298 | Câu hỏi lí thuyết Hóa học hữu cơ OlympiaVN


25. Phản ứng Finkelstein

Finkelstein là phản ứng hữu cơ để chuyển alkyl halide RX thành alkyl halide RY
khác bằng tác nhân muối halide.

26. Phản ứng ghép cặp chéo Kumada

Ghép cặp chéo Kumada là phản ứng của một dẫn xuất halide với một hợp chất cơ
magnesium (tác nhân Grignard), sử dụng xúc tác palladium hoặc nickel, tạo thành
hydrocarbon.

27. Phản ứng ghép cặp chéo Negishi

Ghép cặp chéo Negishi là phản ứng hữu cơ giữa một dẫn xuất halide với một hợp
chất cơ kẽm, sử dụng xúc tác palladium hoặc nickel, tạo thành hydrocarbon. Phản
ứng này cũng tương tự phản ứng ghép cặp chéo Kumada.

1299 | Câu hỏi lí thuyết Hóa học hữu cơ OlympiaVN


28. Phản ứng ghép cặp chéo Sonogashira

Ghép cặp chéo Sonogashira là phản ứng hữu cơ giữa một dẫn xuất halide với một
alkyne đầu mạch, tạo thành hydrocarbon, sử dụng xúc tác palladium, muối Cu(I)
và base.

29. Phản ứng ghép cặp chéo Stille

Ghép cặp chéo Stille là phản ứng hữu cơ của một dẫn xuất halide với một hợp
chất cơ thiếc, sử dụng xúc tác palladium.

30. Phản ứng ghép cặp chéo Suzuki

Ghép cặp chéo Suzuki là phản ứng hữu cơ giữa một dẫn xuất halide với một hợp
chất cơ borane, sử dụng xúc tác palladium và base.

1300 | Câu hỏi lí thuyết Hóa học hữu cơ OlympiaVN


31. Phản ứng Grignard

Phản ứng (của tác nhân) Grignard là phản ứng hữu cơ được ứng dụng để tạo ra
nhiều sản phẩm khác nhau, đi từ hợp chất carbonyl và hợp chất cơ magnesium
(tác nhân Grignard).

32. Phản ứng Heck

Phản ứng Heck là phản ứng ghép cặp chéo của một dẫn xuất halide với một alkene
(có nguyên tử hydrogen gắn với nối đôi), tạo thành một alkene có nhiều nhóm thế
hơn. Phản ứng sử dụng xúc tác palladium và base.

1301 | Câu hỏi lí thuyết Hóa học hữu cơ OlympiaVN


33. Phản ứng Hell-Volhard-Zelinsky

Phản ứng Hell-Volhard-Zelinsky là phản ứng hữu cơ giữa một carboxylic acid có
nguyên tử α-hydrogen và một halogen, tạo thành α-halo carboxylic acid. Phản
ứng sử dụng xúc tác phosphorous và nước.

34. Phản ứng Henry

Phản ứng Henry là phản ứng hữu cơ giữa một nitroalkene (có chứa nguyên tử α-
hydrogen) và aldehyde (hoặc ketone), thành β-nitro alcohol. Phản ứng sử dụng
xúc tác base, như NaOH.

35. Phản ứng khử Clemmensen

Phản ứng khử Clemmensen là phản ứng hữu cơ để khử một aldehyde hoặc
ketone thành alkane, sử dụng hỗn hống kẽm và hydrochloric acid.

1302 | Câu hỏi lí thuyết Hóa học hữu cơ OlympiaVN


36. Phản ứng khử Birch

Phản ứng khử Birch là phản ứng khử dạng 1,4, được dùng để chuyển vòng thơm
benzene thành cyclohexadiene không liên hợp. Phản ứng được thực hiện với tác
nhân Na hoặc Li trong ammonia lỏng, có mặt alcohol.

37. Phản ứng khử Luche

Phản ứng khử Luche được dùng để chuyển ketone không no α,β thành allylic
alcohol, sử dụng cerium trichloride, sodium borohydride và dung môi alcohol.

38. Phản ứng khử Wolff-Kishner

Phản ứng khử Wolff-Kishner được dùng để chuyển một aldehyde hoặc ketone
thành alkane, sử dụng hydrazine và base (đun nóng).

39. Phản ứng Mannich

1303 | Câu hỏi lí thuyết Hóa học hữu cơ OlympiaVN


Phản ứng Mannich là phản ứng giữa một amine bậc I hoặc II và hai hợp chất
carbonyl (một chất có thể enol hóa, một chất không), tạo thành β-amino carbonyl
(thường gọi là Mannich base). Phản ứng có thể thực hiện với xúc tác acid hoặc
base.

40. Phản ứng Mitsunobu

Phản ứng Mitsunobu được dùng để chuyển một alcohol bậc I (hoặc II) thành nhiều
sản phẩm khác nhau, sử dụng DEAD và triphenylphosphine. Sản phẩm cuối phụ
thuộc vào tác nhân có tính acid được sử dụng. Về mặt lập thể thì đây là một phản
ứng làm nghịch đảo cấu hình.

41. Phản ứng ngưng tụ Dieckmann

Phản ứng ngưng tụ Dieckmann được dùng để tạo liên kết carbon-carbon mới, từ
hai nhóm ester cùng một hợp chất, sử dụng xúc tác alkoxide base trong alcohol.
Sản phẩm tạo thành là β-keto ester vòng. Đây là dạng ngưng tụ Claisen nội phân
tử.

1304 | Câu hỏi lí thuyết Hóa học hữu cơ OlympiaVN


42. Phản ứng ngưng tụ Knoevenagel

Ngưng tụ Knoevenagel là một phản ứng hữu cơ để chuyển aldehyde (hoặc ketone)
và một hợp chất chứa nhóm methylene hoạt động (ví dụ malonic ester) thành một
olefin có nhóm thế. Phản ứng sử dụng xúc tác base.

43. Phản ứng oxid hóa Baeyer-Villiger

Phản ứng Baeyer-Villiger là phản ứng oxid hóa để chuyển ketone thành ester, sử
dụng peroxyacid (như mCPBA).

44. Phản ứng oxid hóa Corey-Kim

Oxid hóa Corey-Kim là một phản ứng hữu cơ để chuyển alcohol thành aldehyde
hoặc ketone, sử dụng N-chlorosuccinimide (NCS), dimethylsulfide (DMS) và
triethylamine (TEA).

1305 | Câu hỏi lí thuyết Hóa học hữu cơ OlympiaVN


45. Phản ứng oxid hóa Jones

Oxid hóa Jones là phản ứng oxid hóa alcohol bằng chromic trioxide và acid trong
nước. Alcohol bậc I bị oxid hóa thành aldehyde (nếu tiếp tục oxid hóa sẽ thành
carboxylic acid), còn alcohol bậc II bị oxid hóa thành ketone.

46. Phản ứng oxid hóa Oppenauer

Oxid hóa Oppenauer là phản ứng hữu cơ dùng để chuyển alcohol bậc I hoặc II
thành ketone, sử dụng lượng dư tác nhân ketone khác (như acetone) và xúc tác
aluminium triisopropoxide.

1306 | Câu hỏi lí thuyết Hóa học hữu cơ OlympiaVN


47. Phản ứng oxid hóa Swern

Oxid hóa Swern là phản ứng hữu cơ để chuyển alcohol bậc I (hoặc bậc II) thành
aldehyde (hoặck etone), sử dụng tác nhân DMSO, oxalyl chloride và
triethylamine.

48. Phản ứng oxid hóa Pauson-Khand

Oxid hóa Pauson-Khand là phản ứng hữu cơ để chuyển một alkyne và một alkene
thành α,β-cyclopentenone có nhóm thế. Phản ứng được tiến hành trong khí quyển
carbon monoxide và có xúc tác phức chất dicobalt.

49. Phản ứng Perkin

Phản ứng Perkin là phản ứng hữu cơ để chuyển một aldehyde thơm và một
anhydride thành một carboxylic acid không no α,β, sử dụng tác nhân sodium
acetate, base, sau đó xử lý với acid.

1307 | Câu hỏi lí thuyết Hóa học hữu cơ OlympiaVN


50. Phản ứng Pictet-Spengler

Phản ứng Pictet-Spengler là phản ứng hữu cơ để chuyển α,β-arylenylamine và


một aldehyde (hoặc ketone) thành tetrahydroisoquinoline. Phản ứng sử dụng xúc
tác acid.

51. Phản ứng Reformatsky

Phản ứng Reformatsky được dùng để chuyển một α-haloester và một aldehyde
(hoặc ketone) thành một β-hydroxyester, sử dụng tác nhân kẽm, sau đó xử lí với
acid.

52. Phản ứng Reimer-Tiemann

Phản ứng Reimer-Tiemann dùng để chuyển phenol thành o-hydroxy


benzaldehyde, sử dụng tác nhân chloroform, base, sau đó xử lí với acid.

53. Phản ứng Ritter

1308 | Câu hỏi lí thuyết Hóa học hữu cơ OlympiaVN


Phản ứng Ritter được dùng để chuyển một nitrile và một tiền chất carbocation
(như olefin hoặc alcohol bậc III) thành một amide, sử dụng xúc tác acid và nước.

54. Phản ứng Sandmeyer

Phản ứng Sandmeyer được dùng để chuyển muối aryl diazonium thành aryl
halide, sử dụng xúc tác đồng(I) halide.
55. Phản ứng Schotten-Baumann

Phản ứng Schotten-Baumann được dùng để chuyển acyl halide hoặc anhydride
thành amide (nếu dùng tác nhân amine và base) hoặc thành ester (nếu dùng tác
nhân alcohol và base).

56. Phản ứng tách Hofmann

1309 | Câu hỏi lí thuyết Hóa học hữu cơ OlympiaVN


Phản ứng tách Hofmann được dùng để chuyển một amine có nguyên tử β-
hydrogen thành alkene, sử dụng tác nhân methyl iodide, bạc oxide và nước (đun
nóng).

57. Phản ứng Ullmann

Phản ứng Ullmann được dùng để ghép cặp hai phân tử aryl halide thành biaryl,
bằng tác nhân Cu (đun nóng).

58. Phản ứng Vilsmeier-Haack

EDG = electron donating group: nhóm đẩy electron


Phản ứng Vilsmeier-Haack được dùng để chuyển một hợp chất vòng thơm giàu
electron thành một aryl aldehyde bằng tác nhân DMF, acid chloride (như POCl3)
rồi xử lí với nước.

1310 | Câu hỏi lí thuyết Hóa học hữu cơ OlympiaVN


59. Phản ứng vòng hóa Robinson

Phản ứng vòng hóa Robinson dùng để chuyển một ketone chứa nguyên tử α-
hydrogen và một ketone không no α,β, cũng chứa nguyên tử α-hydrogen thành
cyclohexenone. Phản ứng dùng xúc tác base (như NaOH).

60. Phản ứng Wittig

Phản ứng Wittig dùng để chuyển một alkyl halide bậc I hoặc II và một aldehyde
hoặc ketone thành olefin, sử dụng tác nhân triphenylphosphine và base.

61. Phản ứng tổng hợp amino acid của Strecker

Phản ứng tổng hợp amino acid của Strecker được dùng để chuyển một aldehyde
(hoặc ketone) và một amine bậc I (hoặc ammonia) thành α-amino acid, sử dụng
xúc tác muối cyanide, acid và nước.

1311 | Câu hỏi lí thuyết Hóa học hữu cơ OlympiaVN


62. Phản ứng tổng hợp ether của Williamson

Phản ứng tổng hợp ether theo Williamson được dùng để chuyển một alcohol và
một alkyl halide thành ether, sử dụng xúc tác base (như NaOH).

63. Phản ứng tổng hợp Gabriel

Phản ứng tổng hợp Gabriel được dùng để chuyển một alkyl halide thành một
amine bậc I, qua phản ứng với phthalimide (xúc tác base), sau đó là phản ứng với
hydrazine.

64. Phản ứng tổng hợp indole theo Fischer

Phản ứng tổng hợp indole theo Fischer dùng để chuyển một phenylhydrazine (với
một hydrogen ở vị trí ortho) và một aldehyde (hoặc ketone) thành indole, sử dụng
xúc tác acid.

1312 | Câu hỏi lí thuyết Hóa học hữu cơ OlympiaVN


65. Phản ứng tổng hợp Kolbe-Schmitt

Phản ứng Kolbe-Schmitt được dùng để chuyển một phenol thành hydroxy
benzoic acid qua phản ứng với khí CO2 (xúc tác base), sau đó xử lí với acid.

66. Phản ứng tổng hợp pyrazole theo Knorr

Phản ứng tổng hợp pyrazole theo Knorr được dùng để chuyển hydrazine (hoặc
dẫn xuất của nó) và một hợp chất 1,3-dicarbonyl thành pyrazole. Phản ứng sử
dụng xúc tác acid.

1313 | Câu hỏi lí thuyết Hóa học hữu cơ OlympiaVN

You might also like